Download as pdf or txt
Download as pdf or txt
You are on page 1of 167

CHAPTER

02
The Straight Lines
Learning Part
Session 1
● Definition ● Angle of Inclination of a Line
● Slope or Gradient of a Line ● Angle Between Two Lines
● Lines Parallel to Coordinate Axes ● Intercepts of a Line on Axes
● Different Forms of the Equation of a Straight Line ● Reduction of General Equation to Standard Form
● The Distance Form or Symmetric Form or Parametric
Form of a Line
Session 2
● Position of Two Points Relative to a Given Line ● Position of a Point which Lies Inside a Triangle
● Equations of Lines Parallel and Perpendicular to ● Distance of a Point From a Line
a Given Line
● Distance Between Two Parallel Lines ● Area of Parallelogram
Session 3
● Points of Intersection of Two Lines ● Concurrent Lines
● Family of Lines ● How to Find Circumcentre and Orthocentre by Slopes
Session 4
● Equations of Straight Lines Passing Through a Given ● A Line Equally Inclined with Two Lines
Point and Making a Given Angle with a Given Line
● Equation of the Bisectors ● Bisector of the Angle Containing The Origin
● Equation of that Bisector of the Angle Between ● How to Distinguish the Acute (Internal) and Obtuse
Two Lines Which Contains a Given Point (External) Angle Bisectors
Session 5
● The Foot of Perpendicular Drawn from the ● Image or Reflection of a Point ( x1 , y 1 ) about a Line
Point ( x1 , y 1 ) to the Line ax + by + c = 0 Mirror
● Image or Reflection of a Point ( x1 , y 1 ) in Different Cases ● Use of Image or Reflection
Session 6
● Reflection of Light ● Refraction of Light

● Condition of Collineirty If Three Given Points are in Cyclic Order

Practice Part
● JEE Type Examples
● Chapter Exercises

Arihant on Your Mobile !


Exercises with the #L
symbol can be practised on your mobile. See inside cover page to activate for free.
Session 1
Definition, Angle of Inclination of a Line, Slope or X¢

Gradient of a Line, Angle Between Two Lines,


Lines Parallel to Coordinate Axes, Intercepts of a
Line on Axes, Different Forms of the Equation of
A Straight Line, Reduction of General Equation to
Standard Form, The Distance Form or Symmetric
Form or Parametric Form of a Line

Definition Remarks
1. The number of arbitrary constants in the equation of a
A straight line defined as the curve which is such that the straight line is two (we observe three constants a, b and c in the
line segment joining any two points on it lies wholly on it. equation ax + by + c = 0 of a straight line. The given equation
of line can be rewritten as   x +   y + 1= 0 or
a b
Theorem : Show that the general equation of the first  c  c
degree in x , y represents a straight line. a b
px + qy + 1 = 0 where p = and q = .
c c
Proof : The general equation of the first degree is
Thus, we have only two arbitrary constants p and q in the
ax + by + c = 0 ...(i) equation of a straight line.
Let P ( x 1 , y 1 ) and Q ( x 2 , y 2 ) be the coordinates of any two Hence, to completely determine the equation of a straight line,
we require two conditions to determine the two unknowns in
points on the curve given by Eq. (i), then general.
ax 1 + by 1 + c = 0 ...(ii) 2. A straight line is briefly written as a ‘line.’
ax 2 + by 2 + c = 0 ...(iii) 3. The equation of a straight line is the relation between x and y,
which is satisfied by the coordinates of each and every point
Multiplying Eq. (iii) by λ and adding to Eq. (ii), we have on the line.
a ( x 1 + λx 2 ) + b (y 1 + λy 2 ) + c (1 + λ ) = 0

or
 x + λx 2 
a 1
 y 1 + λy 2 
 +b   + c = 0 ( λ ≠ − 1)
Angle of Inclination of a Line
 1+ λ   1+ λ  The angle of inclination of a line is the measure of the
This relation shows that the point angle between the X-axis and the line measured in the
 x 1 + λx 2 y 1 + λy 2  anticlockwise direction.
 ,  lies on Eq. (i). Y
 1+ λ 1+ λ 
But from previous chapter we know that this point divides
the join of P ( x 1 , y 1 ) and Q ( x 2 , y 2 ) is the ratio λ : 1. B

Since, λ can have any value, so each point on the line PQ 150°
lies on Eq. (i) i.e. the line wholly lies on Eq. (i). Hence, by 30°
X' X
the definition of the straight line as given above we O A
conclude that Eq. (i) represents a straight line.
Y'
Hence, the general equation of first degree in x , y viz
ax + by + c = 0 represents a straight line. Here, angle of inclination of line AB = 150 °.
Chap 02 The Straight Lines 75

Remarks Y 135° Y
O
1. When two lines are parallel, they have the same inclination. X′ X
2. The inclination of a line which is parallel to X-axis or coinciding 45° 45°
with X-axis is 0°.
3. The angle of inclination of the line lies between 0° and 180° i.e. 45°
π 45°
0 < θ ≤ π and θ ≠ . X′ X
2 O

Y′ Y′
Slope or Gradient of a Line Theorem : If P ( x 1 , y 1 ) and Q ( x 2 , y 2 ) are two points on a
If inclination of a line is (θ ≠ 90 ° ), then tan θ is called the line l, then the slope m of the line l is given by
slope or gradient of the line. It is usually denoted by m. y − y1
m= 2 ,x1 ≠ x2
θ is positive or negative according as it is measured in x2 − x1
anticlockwise or clockwise direction.
If x 1 = x 2 , then m is not defined. In that case the line is
Y perpendicular to X-axis.
B
Y

(x2,y2) Q
π+ θ
θ
X′ X (x1,y1) P θ y2
O N
π– θ
A y1
Y′
θ
A
i.e. Slope of AB = m of AB =m ( AB ) X′
O L M
X
x1
= tan θ or tan [ − ( π − θ )] x2
= tan ( π + θ ) Y′
= slope of BA = m of BA
=m( BA ) Proof : Given P ( x 1 , y 1 ) and Q ( x 2 , y 2 ) are two points on
a line l, let line l makes an angle θ with positive direction
∴ m( AB ) = m( BA )
of X-axis. Draw PL, QM perpendiculars on X-axis and
Hence, we do not take into consideration the direction of a
line segment while talking of its slope. PN ⊥ QM
Then, PN = LM = OM − OL = x 2 − x 1
Remarks and QN = QM − NM = QM − PL
1. Slope of a line is not the angle but is the tangent of the
inclination of the line. = y2 − y1
2. If a line is parallel to X-axis, then its slope = tan0 ° = 0. Also, ∠QAM = ∠QPN = θ
3. Slope of a line parallel to Y-axis or perpendicular to X-axis is not Now, in ∆QPN
defined. Whenever we say that the slope of a line is not defined.
QN y 2 − y 1 Difference of ordinates
4. If a line is equally inclined to the axes, then it will make an angle tan θ = = =
of 45° or 135° with the positive direction of X-axis. Slope in this PN x 2 − x 1 Difference of abscissaes
case will be tan 45° or tan135°. i.e. ± 1
y2 − y1
. or m=
Y x2 − x1
Y
45° π
X′
O
X If x 1 = x 2 , then tan θ = ∞ or θ = i.e. m is not defined or
45° 2
45° the line is perpendicular to X-axis.
45°
45° 135° Remarks
X′ X 1. When the two lines are parallel, then their slopes are equal
O
i.e. m1 = m2.
Y′ Y′ 2. If three points A, B, C are collinear, then slope of AB = slope of
BC = slope of AC.
76 Textbook of Coordinate Geometry

y Example 1. Find the inclination of the line whose y Example 6. For what value of k the points
slope is −
1
. (k , 2 − 2k ), ( − k + 1, 2k ) and ( − 4 − k , 6 − 2k ) are
3 collinear ?
Sol. Let α be the inclination of a line then its slope = tanα Sol. Let A ≡ (k , 2 − 2k ), B ≡ ( −k + 1, 2k ) and C ≡ ( − 4 − k , 6 − 2k )
are collinear, then
1
∴ tan α = − = − tan 30 ° Slope of AB = Slope of AC
3 2k − (2 − 2k ) 6 − 2k − (2 − 2k )
⇒ =
= tan (180 ° − 30 ° ) = tan 150 ° −k +1−k − 4 −k −k
⇒ α = 150 ° 4k − 2 4  1 
⇒ = k ≠ (QDenominator ≠ 0)
− 2k + 1 − 4 − 2k  2 
y Example 2. Find the slope of the line through the
⇒ ( 4k − 2) ( − 4 − 2k ) = 4 ( − 2k + 1)
points (4, − 6 ), ( −2 , − 5) .
⇒ (2k − 1) ( − 2 − k ) − ( − 2k + 1) = 0
− 5 − ( − 6) 1
Sol. Slope of the line m = =− ⇒ (2k − 1) ( − 2 − k + 1) = 0
−2 − ( 4 ) 6 1
∴ k ≠ , ∴ k = −1
2
y Example 3. Determine λ , so that 2 is the slope of
the line through (2 , 5) and ( λ , 3) .
Sol. Slope of the line joining (2, 5) and ( λ , 3)
Angle Between Two Lines
3−5 −2 Theorem : The acute angle θ between the lines having
= = =2 (given) slopes m 1 and m 2 is given by
λ −2 λ −2
⇒ − 2 = 2λ − 4 m − m 2 
θ = tan −1  1 
⇒ 2λ = 2 1 + m 1m 2
∴ λ =1 Proof : Let l 1 and l 2 be two non-perpendicular lines,
y Example 4. Show that the line joining the points neither of which is parallel to the Y-axis.
(2 , − 3) and ( − 5 , 1) is parallel to the line joining (7, − 1) Y l2
π−θ l1
and (0, 3) .

Sol. Slope of the line joining the points (2, − 3) and ( − 5, 1) is θ


1 − ( − 3) 4
m1 = =−
− 5 −2 7
θ1 θ2
and slope of the line joining the points (7, − 1) and (0, 3) is X′ X
O
3 − ( − 1) − 4
m2 = = Y′
0−7 7
Here, m1 = m 2 Let m 1 and m 2 be the slopes of two given lines l 1 and l 2
Hence, lines are parallel. respectively. Let θ 1 and θ 2 be the inclinations of these
lines.
y Example 5. Find whether the points
∴ m 1 = tan θ 1 and m 2 = tan θ 2
( − a, − b ), [ − (s + 1) a, − (s + 1) b ] and [(t − 1) a, (t − 1) b ]  π
Let θ and π − θ be the angles between the lines  θ ≠  .
are collinear ?  2
Sol. Let A ≡ ( − a, − b ), B ≡ [ − (s + 1) a, − (s + 1) b ] and Then, θ 2 = θ + θ 1 or θ = θ 2 − θ 1
C ≡ ((t − 1) a, (t − 1) b ) ⇒ tan θ = tan (θ 2 − θ 1 )
− ( s + 1) b + b b
Then, slope of AB = =  tan θ 2 − tan θ 1   m 2 − m 1 
− ( s + 1) a + a a ⇒ tan θ =  =  ...(i)
( t − 1) b + ( s + 1) b b
 1 + tan θ 2 tan θ 1   1 + m 1m 2 
and slope of BC = =
( t − 1) a + ( s + 1) a a  m − m1 
Also, tan( π − θ ) = − tan θ = −  2  ...(ii)
Hence, given points are collinear.  1 + m 2m 1 
Chap 02 The Straight Lines 77

From Eqs. (i) and (ii) the angle between two lines of slopes π
y Example 8. The angle between two lines is and the
m 1 and m 2 is given by 4
 m − m2  1
tan θ = ±  1 
slope of one of them is . Find the slope of the other
 1 + m 1m 2  2
line.
  m − m2  
⇒ θ = tan −1  ±  1  Sol. If θ be the acute angle between the lines with slopes m1 and
  1 + m 1m 2   m 2 , then
m − m 2 
Hence, the acute angle between the lines l 1 and l 2 is given tanθ = 1 
by 1 + m1m 2
m − m 2  π 1
θ= and m1 =
θ = tan −1 1 . Let
4 2
1 + m 1m 2
1 −m 
Corollary 1 : If two lines, whose slopes are m 1 and m 2 are π 2 2

then tan =
parallel, 4 1 + 1 ⋅ m 
2
iff θ = 0 ° (or π) ⇔ tan θ = 0  2 
⇔ m1 = m2 1 − 2m 2
⇒ 1 = 
Thus, when two lines are parallel, their slopes are equal. 2 + m2 
1 − 2m 2
Corollary 2 : If two lines, whose slopes are m 1 and m 2 are ⇒ = ±1
perpendicular, 2 + m2

π π Taking positive sign then,


iff θ =  or −  ⇔ cot θ = 0 1 − 2m 2 = 2 + m 2
2  2
1
⇔ m 1m 2 = − 1 ∴ m2 = −
3
Thus, when two lines are perpendicular, the product of and taking negative sign then,
their slopes is −1. The slope of each is the negative 1 − 2m 2 = − 2 − m 2
reciprocal of the slope of other i.e. if m is the slope of a ∴ m2 = 3
1
line, then the slope of a line perpendicular to it is − . Hence, the slope of the other line is either −
1
or 3.
m 3

y Example 7. Find the angle between the lines joining y Example 9. Without using pythagoras theorem, show
the points (0, 0), (2 , 3) and (2 , − 2), ( 3 , 5). that the points A ( − 1, 3), B (0, 5) and C ( 3 , 1) are the
Sol. Let the given points be A ≡ (0, 0), B ≡ (2, 3), C ≡ (2, − 2) and vertices of a right angled triangle.
D ≡ (3, 5). Let m1 and m 2 be the slopes of the lines AB and Sol. In ∆ ABC, we have
CD respectively.
5−3
3−0 3 5 − ( − 2) Slope of side AB = = 2 = m1 (say)
∴ m1 = = and m 2 = =7 0 − ( − 1)
2−0 2 3−2
1−5 4
Let θ be the acute angle between the lines Slope of side BC = = − = m2 (say)
3−0 3
 3 
 −7  3−1 1
 m − m  and Slope of side CA = = − = m3 (say)
∴ tanθ = 1 2
= 2  −1−3 2
1 + m1m 2 1 +  3 ⋅ 7
   1
  2  Clearly, m1m 3 = 2 ×  −  = −1
 2
− 11 11
=  = ∴ AB and CA are perpendicular to each other i.e.
 23  23
∠ BAC = 90°
 11
∴ θ = tan −1   Hence, the given points are the vertices of a right angled
 23
triangle.
78 Textbook of Coordinate Geometry

y Example 10. A line passes through the points Y l


A (2 , − 3) and B (6, 3) . Find the slopes of the lines
N P(x,y)
which are a >0
(i) parallel to AB (ii) perpendicular to AB a x= |a|= a

3 − ( − 3) 6 3 X′ X
Sol. Let m be the slope of AB. Then m = = = O a
6−2 4 2
(i) Let m1 be the slope of a line parallel to AB, then Y′
3 Here, |a| = a
m1 = m =
2
l Y
(ii) The slope of a line perpendicular to AB is a<0
1 1 2
− =− =− P(x,y) N
m 3 3
2 x= |a|= –a a

X′ X
y Example 11. Show that the triangle which has one of –a O
the angles as 60°, can not have all vertices with
integral coordinates. Y′
Here, |a| = –a
Sol. Let ABC be a triangle whose vertices are A ( x 1, y1 ), B ( x 2 , y 2 )
and C ( x 3 , y 3 ). Assume x 1, x 2 , x 3 , y1, y 2 , y 3 all are integers. Let P ( x , y ) be any point on the line l, then
Let ∠BAC = 60°
x = a is the required equation. (Here, | a | = a )
y − y1
Slope of AC = 3 = m1 (say)
x 3 − x1 Remarks
y 2 − y1 1. In particular equation of Y-axis is x = 0 (Qa=0 )
and Slope of AB = = m2 (say) 2. A line is parallel to Y-axis, at a distance from it and is on the
x 2 − x1
negative side of Y-axis, then its equation is x = − a .
C (x3,y3)
(ii) Equation of a line parallel to X-axis : Let l be a
straight line parallel to X-axis and at a distance b from it, b
being the directed distance of the line from the X-axis.
Therefore, the line lies above the X-axis, if b > 0 and if
b < 0, then the line would lie below the X-axis.
60°
(x1,y1) A B (x2,y2) Y b>0
Here, m1 and m 2 are rational numbers
(Q x 1, x 2 , x 3 , y1, y 2 , y 3 are integers) b y=|b|=b P(x,y)
l
m − m2
∴ tan ( ∠BAC ) = 1
1 + m1m 2 b
= Rational (Qm1 and m 2 are rational)
X′ X
But tan ( ∠BAC ) = tan 60° = 3 = Irrational O M

Q Rational number ≠ Irrational number Y′


Here, |b|= b
Which is contradiction so our assumption that the vertices
are integers is wrong. Hence, the triangle having one angle Y b<0
of 60° can not have all vertices with integral coordinates.
M
Lines Parallel to Coordinate Axes X¢
O
X

(i) Equation of a line parallel to Y-axis : Let l be a b


straight line parallel to Y-axis and at a distance a from it, a
l
being the directed distance of the line from the Y-axis . –b y=|b|= –b P(x,y)
Therefore, the line lies on the right of Y-axis if a > 0 and if Y¢
a < 0, then the line would lies on the left of Y-axis. Here, |b|= –b
Chap 02 The Straight Lines 79

Let P ( x , y ) be any point on the line l, then y = b is the Sol. Since, the given (both) lines are parallel to Y-axis and the
required equation (Here, | b | = b ). required line is equidistant from these lines, so it is also
parallel to Y-axis. Let equation of any line parallel to Y-axis is
Remarks x =a
1. In particular equation of X-axis is y = 0 (Qb = 0 )  7 15
− + 
2. A line parallel to X-axis at a distance b from it and is on the  2 2 8
negative side of X-axis, then its equation is y = − b. Here, a = = = 2 units
2 4
y Example 12. Find the equation of the straight line Hence, its equation is x = 2 .
parallel to Y-axis and at a distance (i) 3 units to the
right (ii) 2 units to the left. Intercepts of a Line on Axes
Sol. (i) Equation of straight line parallel to Y-axis at a distance a If a line cuts X-axis at A (a , 0 ) and the Y-axis at B (0, b )
units to the right is x = a .
then OA and OB are known as the intercepts of the line on
∴ Required equation is x = 3
X-axis and Y-axis respectively. | a | is called the length of
(ii) Equation of straight line parallel to Y-axis at a distance a
intercept of the line on X-axis. Intercept of a line on X-axis
units to the left is x = − a.
may be positive or negative and | b| is called the length of
∴ Required equation is x = − 2. intercept of the line on Y-axis. Intercept of a line on Y-axis
may be positive or negative.
y Example 13. Find the equation of the straight line
parallel to X-axis and at a distance Y
B(0,b)
(i) 5 units above the X-axis
(ii) 9 units below the X-axis.

y intercept
Sol. (i) Equation of a straight line parallel to X -axis at a distance
b units above the X -axis is y = b.
∴ Required equation is y = 5
X′ X
(ii) Equation of a straight line parallel to X -axis at a O x intercept A(a,0)
distance b units below the X -axis is y = − b. Y′
∴ Required equation is y = − 9
Remark
y Example 14. Find the equation of the straight line If a line parallel to Y-axis, then its intercept on Y-axis is not
which passes through the point (2 , − 3) and is defined and if a line parallel to X-axis, then its intercept on
X-axis is not defined.
(i) parallel to the X-axis
Intercepts in II quadrant Intercepts in I quadrant
(ii) perpendicular to the X-axis Y
Y
B (0,b)
Sol. (i) Let equation of any line parallel to X -axis is B (0,b)

y =b ....(i) b b
Since, it passes through the point (2, − 3) . X′ X X′
A(a,0)
X
A a O O a
Putting y = − 3 in Eq. (i), then (–a,0)
Y′ Y′
b= −3 Intercept on X-axis = –a, length of Intercept on X-axis = a, length of
Hence, required equation of the line is y = − 3 . intercept on X-axis = |a| intercept on X-axis = |a|
Intercept on Y-axis = b, length of Intercept on Y-axis = b, length of
Eq. (i) Let equation of any line perpendicular to X -axis intercept on Y-axis = |b| intercept on Y-axis = |b|
= Equation of any line parallel to Y-axis is
Intercepts in III quadrant Intercepts in IV quadrant
x =a ...(ii) Y Y
Since, it passes through the point (2, − 3) putting x = 2 in A(–a,0) a A(a,0)
X′ X X′ X
Eq. (ii) a O O
Then, 2=a ⇒ a=2 b b
Hence, required equation of the line x = 2 . B (0,–b)
B (0,–b)
Y′ Y′
y Example 15. Find the equation of a line which is Intercept on X-axis = –a, length of Intercept on X-axis = a, length of
7 15
equidistant from the lines x = − and x = .
intercept on X-axis = |a| intercept on X-axis = |a|
Intercept on Y-axis = –b, length of Intercept on Y-axis = –b, length of
2 2 intercept on Y-axis = |b| intercept on Y-axis = |b|
80 Textbook of Coordinate Geometry

Different Forms of the Equation of a y Example 17. What are the inclination to the X-axis
and intercept on Y -axis of the line
Straight Line 3y = 3 x + 6 ?
(i) Slope-Intercept Form Sol. The given equation can be written as
Theorem : The equation of the straight line whose slope x
y= +2 ...(i)
is m and which cuts an intercept c on the Y-axis is 3
y = mx + c Now, comparing Eq. (i) with y = mx + c , then we get
1
Y m=
3
B
P (x,y) Let θ be the inclination to the X -axis, then
θ
tan θ = tan 30°
Q M ∴ θ = 30° and c = 2.
c
θ
X′
R O L
X y Example 18. Find the equation of the straight line
A cutting off an intercept of 3 units on negative direction
 3
Y′ of Y -axis and inclined at an angle tan −1   to the
 5
Proof : Let AB be a line whose slope is m and which cuts
axis of x.
an intercept c on Y-axis. Let P ( x , y ) be any point on the
line. Draw PL ⊥ to X-axis and QM ⊥ to PL.  3
Sol. Here, c = − 3 and θ = tan −1  
 5
Then, from figure,
∠ PRL = ∠ PQM = θ, OQ = c 3
or tanθ = =m
5
∴ PM = PL − ML = PL − OQ = y − c
Hence, the equation of the line
and QM = OL = x y = mx + c
PM 3
Now in ∆PQM, tan θ = i.e. y = x −3
QM 5
y −c or 3x − 5y − 15 = 0
⇒ m= ⇒ y = mx + c
x y Example 19. Find the equation to the straight line
which is the required equation of the line. cutting off an intercept of 5 units on negative
direction of Y -axis and being equally inclined to the
Remarks axes.
1. If the line passes through the origin, then c = 0 (Q 0 = m.0 + c
⇒ c = 0) and hence equation of the line will become y = mx. Sol. Here, c = − 5
2. Equation of any line may be taken as y = mx + c.
m = tan 45° or tan135°
3. If the line is parallel to X-axis, then θ = 0°
i.e. m = tan0 ° = 0. Hence, equation of the line parallel to X-axis i.e. m = ±1
is y = c. Y
135°
45°
y Example 16. If the straight line y = mx + c passes X′
O
X
45° 45°
through the points (2 , 4 ) and ( −3 , 6 ) , find the values of
m and c . 45° 45°
Sol. Since, (2, 4 ) lies on y = mx + c (0,–5)
∴ 4 = 2m + c ...(i)
Again, ( −3, 6) lies on y = mx + c Y′
∴ 6 = − 3m + c ...(ii)
Hence, required equation is
2 24
On solving Eqs. (i) and (ii), we get m = − , c =
5 5 y = ( ± 1) x − 5
or y = ± x −5
Chap 02 The Straight Lines 81

y Example 20. Find the equations of the bisectors of Now, in triangle PQN ,
the angle between the coordinate axes. PN y − y 1
tan θ = =
Sol. Let L1 and L 2 be the straight lines bisecting the co-ordinate QN x − x 1
axes. y − y1
Both L1 and L 2 pass through origin ∴ m=
x − x1
∴ Equation of line through origin is y = mx
for L1, m = tan 45° = 1 or y − y 1 = m (x − x 1 )
Y which is the required equation of the line.
L2 L1 Aliter : Let the equation of the required line be
y = mx + c ...(i)

13

45° where, m is the slope of the line.


X′ X
O Since line Eq. (i) passes through the point ( x 1 , y 1 ), therefore
y 1 = mx 1 + c ...(ii)
Subtracting (ii) form (i), we get
Y′
y − y 1 = m (x − x 1 )
∴ Equation of line L1 is y = x
i.e. x −y =0
which is the required equation of the line.
For L 2 , m = tan 135° = − 1 Corollary : If the line passes through the origin, then
∴ Equation of line L 2 is y = − x putting x 1 = 0 and y 1 = 0 in y − y 1 = m ( x − x 1 ).
i.e. x +y =0 It becomes y = mx , which is the equation of the line
Hence, equations of the bisectors of the angle between the passing through the origin and having slope m.
coordinate axes are x ± y = 0.
Remark
(ii) The Point–Slope Form of a Line The equation y − y1 = m ( x − x1 ) is called point-slope form or
Theorem : The equation of the straight line which one point form of the equation.
passes through the point ( x 1 , y 1 ) and has the slope ‘m’ is y Example 21. Find the equation of a line which
y − y 1 = m (x − x 1 ) makes an angle of 135° with the positive direction
Proof : Let AB be a straight line whose slope is m and of X-axis and passes through the point ( 3 , 5) .
which pass through the point Q ( x 1 , y 1 ). Let the line AB Sol. The slope of the line = m = tan135° = − 1
cuts X-axis at R and ∠BRX = θ, then
Here x 1 = 3, y1 = 5 .
tan θ =m ∴ The required equation of the line is
Y B y − 5 = − 1 ( x − 3)
y 1) P(x,y) or x +y −8=0
x 1,
Q( θ
N
y Example 22. Find the equation of the straight line
bisecting the segment joining the points ( 5 , 3) and (4 , 4 )
and making an angle of 45° with the positive direction
θ of X-axis.
X′ X
R O M L
A Sol. Here, m = slope of the line = tan 45° = 1.
Y′
Let A be the mid-point of (5, 3) and ( 4 , 4 ) . Then, the
coordinates of A are
Let P ( x , y ) be any point on the line AB. Draws PL and QM
5 + 4 3 + 4 9 7
perpendiculars from P and Q on X-axis respectively. Also  ,  i.e.  ,  .
 2 2   2 2
draw QN perpendicular from Q on PL, then from figure
Hence, the required equation of the line is
∠PRL = ∠PQN = θ, OL = x , OM = x 1 , PL = y , QM = y 1 7  9
Then, QN = ML = OL − OM = x − x 1 y − = 1 x − 
2  2
and PN = PL − NL = PL − QM = y − y 1 or x −y −1=0
82 Textbook of Coordinate Geometry

y Example 23. Find the equation of the right bisector Now, triangles PHQ and QKR are similar, then
of the line joining (1, 1) and ( 3 , 5) . PH QH
=
Sol. Let m be the slope of the line joining (1, 1) and (3, 5). QK RK
5−1 4 y − y1 x − x1
Then, m= = =2 ⇒ =
3−1 2 y1 − y2 x1 − x2
∴ Slope (M) of right bisector of the join of (1, 1)
 y − y1 
( 3, 5) = −
1 or y − y1 =  2  (x − x 1 )
and
m  x2 − x1 
1 which is the required equation of the line.
∴ M=−
2 Aliter I : Let the equation of the required line be
 1 + 3 1 + 5
Mid-point of the join of (1, 1) and (3, 5) is  ,  y = mx + c ...(i)
 2 2 
where m is the slope of the line.
i.e. (2, 3) .
Hence, equation of the right bisector passing through (2, 3) Since, line Eq. (i) passes through the points ( x 1 , y 1 ) and
1 ( x 2 , y 2 ) therefore
and having slope M = − is
2 y 1 = mx 1 + c ...(ii)
1 y 2 = mx 2 + c
y − 3 = − ( x − 2) and ...(iii)
2
Now, subtracting Eqs. (ii) from (i), we get
or x + 2y − 8 = 0
y − y 1 = m (x − x 1 ) ...(iv)
(iii) The Two-Point Form of a Line and subtracting Eqs. (ii) from (iii), we get
Theorem : The equation of a line passing through two y 2 − y 1 = m (x 2 − x 1 ) ...(v)
given points ( x 1 , y 1 ) and ( x 2 , y 2 ) is given by Dividing Eqs. (iv) by (v) then, we get
 y − y1  y − y1 x − x1
y − y1 =  2  (x − x 1 ) =
 x2 − x1  y2 − y1 x2 − x1
Proof :
 y − y1 
Let AB be a line which passes through two points or y − y1 =  2  (x − x 1 )
 x2 − x1 
Q ( x 1 , y 1 ) and R ( x 2 , y 2 ). Let P ( x , y ) be any point on
the line AB. which is the required equation of the line.
Y B Aliter II : Since points, P ( x , y ), Q ( x 1 , y 1 ) and R ( x 2 , y 2 )
y 1)
Q(
x 1, P(x,y) are collinear then area of ∆ PQR = 0
) H
,y 2 x y 1 x y 1
R(x 2 1
K
i.e. | x 1 y 1 1 | = 0 or x 1 y 1 1 = 0
2
x2 y2 1 x2 y2 1
X′ X
O N M L which is the required equation of the line.
A
Y′
y Example 24. Find the equation to the straight line
Draws PL, QM and RN are perpendiculars from P , Q and R  a  a
joining the points  at 1 ,  and  at 2 ,  .
on X-axis respectively. Also draws QH and RK are  t1   t2 
perpendiculars on PL and QM respectively. Then from
figure  a
Sol. The equation of the line joining the points at 1,  and
ON = x 2 , OM = x 1 , OL = x , RN = y 2 ,  t1
 a
QM = y 1 and PL = y at 2 ,  is
 t2
then, RK = NM = OM − ON = x 1 − x 2
 a a 

QH = ML = OL − OM = x − x 1 a  t 2 t1 
y− =  ( x − at 1 )
QK = QM − KM = QM − RN = y 1 − y 2 t 1  at 2 − at 1 
 
and PH = PL − HL = PL − QM = y − y 1  
Chap 02 The Straight Lines 83

a a (t 1 − t 2 ) Hence, equation of altitude AD which passes through (0, 4 )


⇒ y− =− ( x − at 1 )
t1 at 1t 2 (t 1 − t 2 ) 1
and having slope is
a 1 5
or y− =− ( x − at 1 ) 1
t1 t 1t 2 y − 4 = ( x − 10)
5
or t 1t 2y − at 2 = − x + at 1
or x − 5y + 10 = 0
or x + t 1t 2y = a (t 1 + t 2 )
which is the required equation of the line.
y Example 27. Find the equations of the medians of
y Example 25. Let ABC be a triangle with A ( −1, − 5) , a triangle, the coordinates of whose vertices are
( −1, 6 ) , ( −3 , − 9 ) and ( 5 , − 8 ) .
B (0, 0) and C (2 , 2) and let D be the middle point of BC.
Find the equation of the perpendicular drawn from B to Sol. Let A ( −1, 6), B ( −3, − 9 ) and C (5, − 8) be the vertices of
AD. ∆ ABC. Let D , E and F be the mid-points of the sides BC , CA
Sol. Q D is the middle point of BC. and AB respectively.
 0 + 2 0 + 2  − 3 + 5 − 9 − 8
∴ Coordinates of D are  Coordinates of D ≡  , 
,   2 2 
 2 2 
 17 
Y i.e. 1, − 
C(2,2)  2
D  5 − 1 − 8 + 6
Coordinates of E ≡  , 
X′
(0,0)B
X
 2 2 
M
i.e. (2, − 1)
Y
(–1,6)A
(–1,–5)A
Y′ X′ X
O E
i.e. D(1, 1) F
1+5
Slope of median AD = =3
1+1 C(5,–8)
1 (–3,–9)B D
∴ Slope of BM which is perpendicular to AD = − .
3 Y′
Hence, equation of the line BM is
 −1 − 3 6 − 9 
1 and coordinates of F ≡  , 
y − 0 = − ( x − 0) ⇒ x + 3y = 0  2 2 
3
which is the required equation of the line. i.e. ( −2, − 3 / 2)
∴ Equation of the median AD = Equation of line through
y Example 26. The vertices of a triangle are  17 
( −1, 6) and 1, −  is
A (10, 4 ), B ( −4, 9 ) and C ( −2 , − 1). Find the equation of  2
the altitude through A. 17
− −6
−1 − 9 −10 y −6= 2 ( x + 1)
Sol. Q Slope of BC = = = −5
−2 + 4 2 1+1
1 1 29
∴ Slope of altitude AD = − = ⇒ y −6= − ( x + 1) or 29 x + 4y + 5 = 0
−5 5 4
Y Equation of median BE is
(–4,9) B −1 + 9
y +9 = ( x + 3) or 8x − 5y − 21 = 0
2+3
A(10,4) and equation of median CF is
3
D − +8
X′ X y +8= 2 ( x − 5)
O −2 − 5
(–2,–1) C
or 13x + 14y + 47 = 0
Y′
84 Textbook of Coordinate Geometry

y Example 28. Find the ratio in which the line segment 1 1 1


⇒ ⋅ OA ⋅ OB = ⋅ OA ⋅ PL + ⋅ OB ⋅ PM
joining the points (2 , 3) and (4 , 5) is divided by the line 2 2 2
joining (6 , 8 ) and ( −3 , − 2) . 1 1 1
⇒ ⋅ OA ⋅ OB = ⋅ OA ⋅ PL + ⋅ OB ⋅ OL
Sol. The equation of line passing through (6, 8) and ( −3, − 2) is 2 2 2
−2 − 8 1 1 1
y −8= ( x − 6) or ab = ay + bx
−3 − 6
2 2 2
⇒ 9y − 72 = 10x − 60
⇒ ab = ay + bx
or 10x − 9y + 12 = 0 ...(i)
Let the required ratio be λ : 1.
x y
or + =1
Now, the coordinates of the point P which divide the line a b
segment joining the points (2, 3) and ( 4, 5) in the ratio λ : 1 is which is the required equation of the line.
 2 + 4 λ 3 + 5λ  Aliter I : Equation of the line through A (a, 0 ) and B (0, b )
P , 
 1+ λ 1+ λ  is
Clearly P lies on Eq. (i), then b −0
y −0 = (x − a )
2 + 4λ   3 + 5λ  0 −a
10   −9   + 12 = 0
 1+ λ  1+ λ 
or − ay = bx − ab
or 20 + 40λ − 27 − 45λ + 12 + 12λ = 0
5
or bx + ay = ab
or 7 λ + 5 = 0 or λ = −
7 x y
or + =1
5 a b
∴ The required ratio = λ : 1 = − : 1 = − 5 : 7
7 which is the required equation of the line.
Hence, the required ratio is 5 : 7 (externally).
Aliter II : Points A (a, 0 ), P ( x , y ) and B (0, b ) are collinear,
(iv) The Intercept Form of a Line : we have
slope of AB = slope of AP
Theorem : The equation of the straight line which cuts
off intercepts of lengths of a and b on X-axis and Y-axis b −0 y −0
⇒ = ⇒ bx − ab = − ay
respectively, is 0 −a x −a
x y x y
+ =1 ⇒ bx + ay = ab ⇒ + =1
a b a b
Proof : Let QR be a line which cuts off intercepts OA = a or
and OB = b on the X-axis and Y-axis respectively, where a 0 1
a ≠ 0. The line is non vertical, because b is finite. Let
x y 1 =0
P ( x , y ) be a general point on the line.
0 b 1
Draws PL and PM perpendiculars on X-axis and Y-axis
respectively. Then PL = y and OL = x also join OP. Clearly, ⇒ a (y − b ) − 0 + 1 ⋅ (bx ) = 0
Y or bx + ay = ab
R B(0,b) x y
or + =1
a b
M x P(x,y)
which is the required equation of the line.
y y Example 29. Find the equation of the line through
A(a,0)
X′ X (2 , 3) so that the segment of the line intercepted
O L Q
between the axes is bisected at this point.
Y′
Sol. Let the required line segment be AB.
Area of ∆OAB = Area of ∆OAP + Area of ∆OPB Let O be the origin and OA = a and OB = b.
Then the coordinates of A and B are (a , 0) and (0, b )
respectively.
Chap 02 The Straight Lines 85

a+0
∴ =2 ⇒ a= 4 Hence, the required equations are
2 x y x y
0+b + = 1 and + =1
and =3 ⇒ b=6 6 8 7 7
2
i.e. 4 x + 3y = 24 and x + y = 7
Y
B(0,b) y Example 31. Find the equation of the straight line
x y
through the point P (a, b ) parallel to the line + = 1.
a b
(2,3)
Also find the intercepts made by it on the axes.
x y
A(a,0) Sol. Let the line + =1
X′ X a b
O
meets the axes in A and B respectively. So that
Y′ OA = a , OB = b
Hence, the equation of the required line is
Y
x y
+ =1 B'(0,b' )
4 6
i.e. 3x + 2y = 12
P(a,b)
y Example 30. Find the equation to the straight line (0,b)B
which passes through the points ( 3 , 4 ) and have A'(a',0)
intercepts on the axes : X′ X
O (a,0)A
(a) equal in magnitude but opposite in sign
Y′
(b) such that their sum is 14
Let the required parallel line meet in A ′ and B′ respectively,
Sol. (a) Let intercepts on the axes be a and −a respectively. so that
∴ The equation of the line in intercept form is OA ′ = a ′ (say)
x y and OB ′ = b ′ (say)
+ = 1 or x − y = a ...(i)
a −a ∴ Equation of required line is
Since, Eq. (i) passes through (3, 4 ) , then x y
+ =1 ...(i)
3− 4 =a a′ b′
∴ a = −1 Since, ∆′s OAB and OA ′ B ′ are similar, then
From Eq. (i), x − y + 1 = 0 OA ′ OB ′
which is the required equation of the line. =
OA OB
x y a′ b′
(b) Let the equation of the line be + = 1 i.e. = =λ (say)
a b a b
This passes through (3, 4 ). ⇒ a ′ = aλ , b ′ = bλ
3 4
Therefore + =1 ...(ii) Substituting these values in Eq. (i), then
a b x y
It is given that a + b = 14 + =1 ...(ii)
aλ bλ
∴ b = 14 − a It passes through (a, b ), then
Putting b = 14 − a in Eq. (ii), we get a b
3 4 + =1
+ =1 aλ bλ
a 14 − a 2
⇒ = 1 or λ = 2
⇒ 42 − 3a + 4a = 14a − a 2 λ
From Eq. (ii) required equation is
⇒ a 2 − 13a + 42 = 0
x y
⇒ ( a − 7 ) ( a − 6) = 0 + =1
2a 2b
∴ a = 6, 7 Evidently intercepts on the axes are 2a and 2b.
Then, b = 8, 7 (Qb = 14 − a )
86 Textbook of Coordinate Geometry

(v) The Normal Form or Perpendicular Form OR


⇒ cosec α =
of a Line p
Theorem : The equation of the straight line upon which or OR = p cosec α
the length of perpendicular from the origin is p and this Thus, AB makes intercepts p sec α and p cosec α on X-axis
normal makes an angle α with the positive direction of and Y-axis respectively.
X-axis is x y
∴ Equation of AB is + =1
x cos α + y sin α = p. p sec α p cosec α
Proof : Let AB be a line such that the length of or x cos α + y sin α = p
perpendicular from O to the line be p
which is the required equation of the line AB.
i.e. ON = p
Aliter II : The points Q ( p sec α, 0 ), P ( x , y ) and
and ∠ NOX = α R (0, p cosec α ) are collinear, then
Let P ( x , y ) be any point on the line. Draw PL p secα 0 1
perpendicular from P on X-axis.
x y 1 =0
Y
0 p cosec α 1
B R
N
or p sec α (y − p cosec α ) − 0 + 1 ( px cosec α ) = 0
or p (y sin α − p ) + px cos α = 0
p P(x,y)
or x cos α + y sin α = p
y α
α which is the required equation of the line AB.
X′ X
O L Q
x A
Y′ Remarks
1. Here, p is always taken as positive and α is measured from
Let line AB cuts X and Y-axes at Q and R respectively. positive direction of X-axis in anticlockwise direction between 0
and 2 π ( i.e. 0 ≤ α < 2 π ).
Now, ∠ NQO = 90 ° − α 2. (Coefficient of x) 2+ (Coefficient of y) 2 = cos 2 α + sin2 α = 1
∴ ∠ LPQ = 90 ° − (90 ° − α ) = α 3. cosα and cos ( 90° − α ) are the direction cosines of ON.
LQ LQ 4.
In ∆PLQ, tan α = =
PL y II quadrant I quadrant
Y Y
∴ LQ = y tan α ...(i)
B B
ON
Also, in ∆ONQ, cos α =
OQ
N N
p p p
⇒ cos α =
OL + LQ α α
X′ X X′ X
⇒ OL cos α + LQ cos α = p A
O O
A
⇒ x cos α + y tan α cos α = p Y′ Y′
cos α < 0, sin α > 0, p > 0 cos α > 0, sin α > 0, p > 0
(Q OL = x and LQ = y tan α )
∴ x cos α + y sin α = p III quadrant IV quadrant
Y Y
which is the required equation of the line AB. B
α α B
Aliter I : Q ∠ NOQ = α X′
O
X X′
O
X

then ∠ NOR = 90 ° − α p p
OQ OQ N N
Now, in ∆ONQ, sec α = =
ON p
A A
or OQ = p sec α
Y′ Y′
OR
Also in ∆ONR, sec (90° − α ) =
ON cos α < 0, sin α < 0, p > 0 cos α > 0, sin α < 0, p > 0
Chap 02 The Straight Lines 87

Corollary 1 : If α = 0 °, then equation x cos α + y sin α = p In ∆ONA, cos30° =


ON p
=
becomes x cos 0 ° + y sin 0 ° = p OA a
i.e. x = p (Equation of line parallel to Y-axis) Y
π B
Corollary 2 : If α = , then equation x cos α + y sin α = p
2
π π
becomes x cos   + y sin   = p b
2 2 N
60° p
i.e. y = p (Equation of line parallel to X-axis). X′
O
30°
a X
A
Corollary 3 : If α = 0 °, p = 0 then equation
Y′
x cos α + y sin α = p becomes x cos 0 ° + y sin 0 ° = 0
i.e. x = 0 (Equation of Y-axis) 3 p 2p
⇒ = or a =
π 2 a 3
Corollary 4 : If α = , p = 0 then equation
2 ON p
and in ∆ONB, cos60° = =
π π OB b
x cos α + y sin α = p becomes x cos   + y sin   = 0
2 2 ⇒
1 p
= or b = 2p
i.e. y =0 (Eq. (i) of X-axis). 2 b
1 1  2p 
Q Area of ∆OAB = ab =  
y Example 32. The length of perpendicular from the 2 2  3
origin to a line is 9 and the line makes an angle of 2p 2
120° with the positive direction of Y -axis. Find the ( 2p ) =
3
equation of the line.
2p 2 50
Sol. Here, α = 60° and p = 9. ∴ = (given)
3 3
∴ Equation of the required line is
⇒ p 2 = 25
x cos 60 ° + y sin 60 ° = 9
or p =5 (Qp > 0)
Y ∴ Using x cos α + y sin α = p , the equation of the line AB is
120°
x cos 30° + y sin 30° = 5
B
60° N
or x 3 + y = 10

30° Reduction of General Equation to


X′
O
60° A
X Standard Form
Let Ax + By + C = 0 be the general equation of a straight
Y′ line where A and B are not both zero.
 1  3 (i) Reduction of ‘Slope-Intercept’ Form
⇒ x   +y   =9
 2  2
Given equation is Ax + By + C = 0
or x + y 3 = 18 ⇒ By = − Ax − C
y Example 33. Find the equation of the straight line on  A  C
⇒ y = −  x + −  (Assuming B ≠ 0)
which the perpendicular from origin makes an angle of  B  B
30° with X-axis and which forms a triangle of area Comparing it with y = mx + c
 50 
  sq units with the coordinates axes. A
slope (m ) = − = −
coefficient of x
 3 we get
B coefficient of y
Sol. Let ∠ NOA = 30° C constant term
Let ON = p > 0, OA = a , OB = b and y intercept (c ) = − =−
B coefficient of y
88 Textbook of Coordinate Geometry

Corollary 1 : Find angle between the lines (ii) Reduction to ‘Intercept’ Form
A1 x + B 1 y + C 1 = 0 and A2 x + B 2 y + C 2 = 0.
Given equation is Ax + By + C = 0
Slope of the line
A ⇒ Ax + By = − C
A1 x + B 1 y + C 1 = 0 is − 1 = m 1 (say) A B
B1 ⇒ x+ y =1 (Assuming C ≠ 0)
−C −C
and slope of the line
A2 x y
A2 x + B 2 y + C 2 = 0 is − = m2 (say) ⇒ + = 1 (Assuming A ≠ 0, B ≠ 0)
B2 (− C / A) (− C / B )
If θ is the angle between the two lines, then x y
Comparing with + =1
a b
  A1   A2 
m − m 2    − B  −  − B  C constant term
we get, x-intercept (a) = − =−
tan θ = 1  = 1 2
A coefficient of x
1 + m 1 2 
m  A  A 
1 +  − 1   − 2 
constant term
  B1   B2 
C
and y-intercept (b) = − =−
B coefficient of y
A B − A2 B 1
= 1 2 
A1 A2 + B 1 B 2 (iii) Reduction to ‘Normal’ Form
A B − A2 B 1 Given equation is Ax + By + C = 0 . Let its normal form
∴ θ = tan −1  1 2 
be x cos α + y sin α = p.
A1 A2 + B 1 B 2
Clearly, equations Ax + By + C = 0 and
Corollary 2 : Find the condition of (i) parallelism (ii)
x cos α + y sin α = p represent the same line.
perpendicularity of the lines
A B C
A1 x + B 1 y + C 1 = 0 Therefore, = =
cos α sin α −p
and A2 x + B 2 y + C 2 = 0
Ap
(i) If the two lines are parallel, θ = 0 ° ⇒ cos α = −
C
∴ tan θ = tan 0 ° = 0
Bp
sin α = −
A1 B 2 − A2 B 1
 and ...(i)
⇒ = 0 C
A1 A2 + B 1 B 2
cos 2 α + sin2 α = 1
⇒ A1 B 2 − A2 B 1 = 0 2 2
 Ap   Bp 
A1 B 1
= −  + −  = 1
or (Remember)  C   C 
A2 B 2
which is required condition of parallelism. C2
or p2 =
(ii) If the two lines are perpendicular, θ = 90 ° A2 + B 2
∴ tan θ = tan 90 ° = ∞ | C|
⇒ p=
⇒ A1 B 2 − A2 B 1
 = ∞ ( A2 + B 2 )
A1 A2 + B 1 B 2 | C| A
From Eq. (i), cos α = − . ,
⇒ A1 A2 + B 1 B 2 = 0 (Remember) C ( A2 + B 2 )
which is required condition of perpendicularity. | C| B
sin α = − . .
Remark C ( A2 + B 2 )
If two lines are coincident, then
Putting the values of cos α, sin α and p in
A1 B 1 C 1
= = (Remember) x cos α + y sin α = p, we get
A2 B 2 C 2
Chap 02 The Straight Lines 89

 − | C|    1
x ⋅
A  x + y  − | C| . B  where, cos α = −
2
= − cos 60° = cos (180° − 60° )
 C ( A 2 + B 2 )   C ( A 2 + B 2 )  cos (180° + 60° )
  or
∴ α = 120° or 240°
| C|
= 3
(A + B 2 )
2 and sin α = − = − sin 60° = sin (180° + 60° )
2
    or sin (360° − 60° )
− A  x + − B y = C ∴ α = 240° or 300°

 2 2 
(A + B )   2 
(A + B ) 
2
(A + B 2 )
2
α = 240° , p = 2
  Hence,
∴ Required normal form is
This is the normal form of the line Ax + By + C = 0.
x cos 240° + y sin 240° = 2
Rule : First shift the constant term on the RHS and make
it positive, if it is not so by multiplying the whole equation y Example 35. Find the measure of the angle of
by ‘− 1 ’ and then divide both sides by intersection of the lines whose equations are
3x + 4 y + 7 = 0 and 4 x − 3y + 5 = 0.
(coefficient of x ) 2 + (coefficient of y ) 2
Sol. Given lines are 3x + 4y + 7 = 0, 4 x − 3y + 5 = 0. Comparing
y Example 34. Reduce x + 3y + 4 = 0 to the : the given lines with A1x + B1y + C 1 = 0, A 2 x + B 2y + C 2 = 0
respectively, we get
(i) Slope-intercept form and find its slope and y-intercept A1 = 3, B1 = 4 and A 2 = 4, B 2 = − 3
(ii) Intercept form and find its intercepts on the axes Q A1A 2 + B1B 2 = 3 × 4 + 4 ( −3) = 0
Hence, the given lines are perpendicular.
(iii) Normal form and find the values of p and α
Sol. (i) Given equation is x + 3 y + 4 = 0 y Example 36. Find the angle between the lines
⇒ 3y = − x − 4 (a 2 − ab) y = (ab + b 2 ) x + b 3
 1  4 and (ab + a 2 ) y = (ab − b 2 ) x + a 3
⇒ y = −  x + − 
 3   3 where a > b > 0.
which is in the slope-intercept form y = mx + c Sol. The given equations of lines can be written as
1 4
Where slope (m ) = − and y-intercept (c ) = − (ab + b 2 ) x − (a 2 − ab ) y + b 3 = 0 ...(i)
3 3
and (ab − b ) x − (ab + a ) y + a = 0
2 2 3
...(ii)
(ii) Given equation is
x + 3y + 4 = 0 Comparing the given lines (i) and (ii) with the lines
A1x + B1y + C 1 = 0 and A 2 x + B 2y + C 2 = 0
⇒ x + 3y = − 4
respectively, we get
x 3y
⇒ + =1 A1 = ab + b 2 , B1 = − (a 2 − ab )
−4 −4
x y and A 2 = ab − b 2 , B 2 = − (ab + a 2 )
⇒ + =1
−4 − 4 / 3 Let θ be the acute angle between the lines, then
x y A B − A 2 B1
which is in the intercept form + = 1 tanθ = 1 2 
a b A1A 2 + B1B 2
4
where x-intercept (a ) = − 4 and y-intercept (b ) = − (ab + b 2 ) × ( − (ab + a 2 )) − (ab − b 2 ) × ( − (a 2 − ab ))
3 tan θ =
(ab + b 2 ) (ab − b 2 ) + (a 2 − ab ) (ab + a 2 )
(iii) Given equation is x + 3y + 4 = 0
− {a 2b 2 + a 3b + ab 3 + a 2b 2 − a 3b + a 2b 2 + a 2b 2 − b 3a }
⇒ x + 3y = − 4 =
(a 2b 2 − b 4 + a 4 − a 2b 2 )
⇒ − x − 3y = 4 (RHS made positive)
− 4a 2b 2 4 a 2b 2
Dividing both sides by ( −1)2 + ( − 3 )2 = 2 , we get = 4 =
a − b  a − b
4 4 4

 1  3
−  x + −  y =2  4 a 2b 2 
 2  2  ∴ θ = tan −1  4 
a − b4 
Which is the normal form x cos α + y sin α = p .
90 Textbook of Coordinate Geometry

y Example 37. Two equal sides of an isosceles triangle The Distance form or Symmetric form
are given by the equations 7 x − y + 3 = 0 and
x + y − 3 = 0 and its third side passes through the or Parametric form of a line
point (1, − 10) . Determine the equation of the third Theorem : The equation of the straight line passing
side. through ( x 1 , y 1 ) and making an angle θ with the positive
Sol. Given equations direction of X-axis is
7x − y + 3 = 0 ...(i) x − x1 y − y1
= =r
and x +y −3=0 ...(ii) cos θ sin θ
represents two equal sides AB and AC of an isosceles where, r is the directed distance between the points ( x , y )
triangle ABC. Since its third side passes through D (1, − 10)
then its equation is
and ( x 1 , y 1 ).
y + 10 = m ( x − 1) ...(iii) Proof : Let AB be a line which passes through the point
Q AB = AC Q ( x 1 , y 1 ) and meet X-axis at R and makes an angle θ with
Let ∠ ABC = ∠ ACB = θ the positive direction of X-axis.
then ∠ ACE = π − θ Y B
Y P(x,y)
r
(x1,y1)Q θ
A N

θ
X′ X
X′ X O R M L
O
x+
3=0

A
y–
3
=
7x–y+

p–q Y′
0

q
C
E Let P ( x , y ) be any point on the line at a distance r from Q.
q Draws PL and QM are perpendiculars from P and Q on
B D(1,–10) X-axis respectively and draw QN perpendicular on PL.
Y′ Then,
From Eqs. (i) and (ii), slopes of AB and AC are QN = ML = OL − OM = x − x 1
m1 = 7 and m 2 = − 1 and PN = PL − NL = PL − QM = y − y 1
respectively. from ∆PQN ,
7 −m QN x − x 1 x − x1
∴ tanθ = cos θ = = or =r ...(i)
1 + 7m PQ r cos θ
−1 − m 1 + m
and tan ( π − θ ) = = −  PN y − y 1 y − y1
1 + ( − 1) m 1 − m and sin θ = = or =r ...(ii)
PQ r sin θ
1 + m 1 + m
⇒ − tanθ = −   ⇒ tanθ =   From Eqs. (i) and (ii), we get
 1 − m  1 − m
x − x1 y − y1
7 −m 1+m = =r
∴ = cos θ sin θ
1 + 7m 1 − m
x − x1 y − y1
⇒ ( 7 − m ) ( 1 − m ) = ( 1 + 7m ) ( 1 + m ) Corollary 1 : Q = = r , then
cos θ sin θ
⇒ 6m 2 + 16m − 6 = 0
x = x 1 + r cos θ
or 3m 2 + 8m − 3 = 0 or (3m − 1) (m + 3) = 0 
y = y 1 + r sin θ 
1
⇒ m = ,−3
3 parametric equations of straight line AB.
Hence from Eq. (iii), the third side BC has two equations Corollary 2 : If P point above Q then r is positive then
1
y + 10 = ( x − 1) and y + 10 = − 3 ( x − 1) coordinates of P are ( x 1 + r cos θ, y 1 + r sin θ ) and if P
3 below Q then r is negative then coordinates of P are
or x − 3y − 31 = 0 and 3x + y + 7 = 0 ( x 1 − r cos θ, y 1 − r sin θ ).
Chap 02 The Straight Lines 91

y Example 38. The slope of a straight line through 6


3 ⇒ (cos θ + sin θ ) = 1
A ( 3 , 2) is . Find the coordinates of the points on 3
4 3 3
the line that are 5 units away from A. ⇒ cos θ + sin θ = =
6 2
Sol. Let straight line makes an angle θ with positive direction of 1 1 3
X -axis, ⇒ cos θ + sin θ =
2 2 2
3
then tanθ =  π  π
4 ⇒ cos θ −  = cos  
 4  6
3 4
∴ sinθ = and cosθ = π π
5 5 θ − = 2nπ ± ; n ∈I
4 6
5 π π
3 for n = 0, θ=± + (Q0 ≤ θ < π )
6 4
θ
4 = 15° , 75°
∴ Equation of the straight line through A (3, 2) in 3π
parametric form is y Example 40. A line through (2 , 3) makes an angle
4
x −3 y −2
= = ±5 with the negative direction of X-axis. Find the length of
cos θ sin θ the line segment cut off between (2 , 3) and the line
4
∴ x = 3 ± 5 cosθ = 3 ± 5 × = 3 ± 4 = 7 or −1 x + y − 7 = 0.
5 3π
3 Sol. Q Line makes an angle with the negative direction of
and y = 2 ± 5 sinθ = 2 ± 5 × = 2 ± 3 = 5 or −1 X -axis. 4
5 π
∴ Line makes an angle with the positive direction of
Hence, the coordinates of the points are (7, 5) and ( − 1, − 1) . 4
X -axis.
y Example 39. Find the direction in which a straight Y
line must be drawn through the point (1, 2) so that
its point of intersection with the line x + y = 4 may π/4
1 r P
be at a distance 6 from this point.
3 A(2,3)
3π/4
Sol. Let the straight line makes an angle θ with the positive π/4 π/α
direction of X -axis. X′ X
O
∴ Equation of the line through (1, 2) in parametric form is x+
y–
Y′
x −1 y −2 1 =7
0
= = 6
cos θ sin θ 3 ∴ The equation of the line through (1, 2) in parametric form is
Y x −2 y −3
= =r
 π  π
(0,4) cos   sin  
6  4  4
3
x −2 y −3
i.e. = =r ...(i)
1 1
A(1,2)
2 2
X′ X r r
O (4,0) ∴ x =2+ and y = 3 +
2 2
Y′
Let the line (i) meet the line x + y − 7 = 0 in P
6 6  r r 
or x =1+ cosθ and y = 2 + sinθ ∴ Coordinates of P 2 + ,3 +  lies on x + y − 7 = 0
3 3  2 2
 6 6  r r
Since, the point 1 + cos θ, 2 + sin θ lies on the line then 2+ +3+ −7 =0
 3 3  2 2
x +y = 4 2r
or = 2 or r = 2
6 6 2
∴ 1+ cos θ + 2 + sin θ = 4
3 3 ∴ AP = 2
92 Textbook of Coordinate Geometry

y Example 41. Find the distance of the point (2 , 3) Special Corollaries


from the line 2x − 3y + 9 = 0 measured along the line (i) Angle made by AB with positive X-axis (where
2x − 2y + 5 = 0 . A and B are given points) :be two points and let AB
makes an angle θ with the positive direction of X-axis
Sol. Since, slope of the line 2x − 2y + 5 = 0 is 1, its makes an
and let d be the distance between A and B. Then Let
π A ( x 1 , y 1 ) and B ( x 2 , y 2 )
angle with positive direction of X -axis.
4 x − x1 y − y1
The equation of the line through (2, 3) and making an angle cos θ = 2 , sin θ = 2
d d
π
in parametric form
4 Y
x −2 y −3 x −2 y −3 B'(x2',y2' )
= = r or = =r
π
  π
  1 1 B(x2,y2)
cos   sin  
 4  4 2 2
(y2–y1)
 r r  α
Coordinates of any point on this line are 2 + ,3 + . θ
 2 2 (x1,y1)A
(x2–x1)
This point lies on the line 2x − 3y + 9 = 0 O
X

 r   r 
⇒ 2 2 +  − 3 3 +  +9 =0
 2  2
where, d = ( x 2 − x 1 ) 2 + (y 2 − y 1 ) 2 = AB
r
⇒ − + 4 =0
2 and x 2 = x 1 + d cos θ, y 2 = y 1 + d sin θ
∴ r =4 2 If AB rotates an angle α about A, then new
coordinates of B are
y Example 42. If the line y − 3x + 3 = 0 cuts the
x 2′ = x 1 + d cos(θ + α ),
parabola y 2 = x + 2 at A and B, then find the value of
y ′2 = y 1 + d sin(θ + α )
PA. PB { where P ≡ ( 3, 0)}.
and here, AB = AB ′ = d .
Sol. Slope of line y − 3x + 3 = 0 is 3
(ii) Complex number as a rotating arrow in Argand
If line makes an angle θ with X-axis, then tan θ = 3 plane :
∴ θ = 60° Let z = r (cos θ + i sin θ ) = re iθ , where i = −1 ... (i)
x− 3 y −0
= =r be a complex number representing a point P in the
cos 60° sin 60°
Argand plane.
Y
Y
A
Q(zeiφ)
Imaginary axis

60°
X P(z)
O P(√3,0)

φ
B θ X
O Real axis
 r r 3 Then, OP = | z | = r and ∠ POX = θ
⇒  3+ ,  be a point on the parabola y = x + 2
2
 2 2 
Now, consider complex number z 1` = ze iφ
3 2 r
then, r = 3 + + 2 ⇒ 3r 2 − 2r − 4 (2 + 3 ) = 0 or z 1 = re iθ ⋅ e i φ = r ⋅ e i ( θ + φ ) [from Eq. (i)]
4 2
−4 (2 + 3 )
 = 4 (2 + 3 )
Clearly the complex number z 1 represents a point Q
∴ PA ⋅ PB = r1r 2 =
 3  3 in the Argand plane, when
OQ = r and ∠ QOX = θ + φ
Chap 02 The Straight Lines 93


Clearly multiplication of z with e iφ rotates the vector OP Since, AC = r = 2
through angle φ in anti-clockwise sense. Similarly → Put r = 2 in Eq. (i), then
multiplication of z with e − iφ will rotate the vector OP in
1 4+ 2
clockwise sense. x = 2+ 2⋅ =
2 2
Remark 3 6
If z1, z2 and z3 are the affixes of the three points A, B and C such and y = 2. =
that AC = AB and ∠ CAB = θ. Therefore 2 2
→ → Equation of the line AC is
AB = z2 − z1, AC = z3 − z1
x −2 1
→ → = cot 60° =
Then AC will be obtained by rotating AB through an angle θ in y 3
anticlockwise sense and therefore
or x 3 −y −2 3 =0
C(z3)
 4 + 2 6
and coordinates of C are  , .
 2 2
B(z2)
Aliter (By special corollary (ii))
Q A ≡ (2, 0), B ≡ (3, 1), let C ≡ ( x , y )
θ ∴ z A = 2,z B = 3 + i , zC = x + iy , where i = −1

zC − z A i  5π 
A(z1) =e 12
Q15° = 
zB − zA  12 
→ →
AC = ABei θ ⇒ zC − 2 = (1 + i ) (cos 15° + i sin 15° )
iθ  z − z1  iθ  3 +1  3 − 1 
or ( z3 − z1 ) = ( z2 − z1 ) e or  3  =e
 z2 − z1  or zC = 2 + ( 1 + i )  +i  
 2 2  2 2 
y Example 43. The line joining the points A (2 , 0) and  3 +1 3 − 1  3 +1 3 − 1
= 2 + −  +i  + 
B ( 3 , 1) is rotated about A in the anticlockwise direction  2 2 2 2   2 2 2 2 
through an angle of 15°. Find the equation of the line  1  3 4+ 2  6
in the new position. If B goes to C in the new position, = 2 +  +i   = +i  
 2  2 2  2
what will be the coordinates of C ?
 4 + 2 6
Sol. By special corollary (i) ∴ C ≡ , 
 2 2
Here AB = (2 − 3)2 + (0 − 1)2 = 2
and equation of AC
1−0
and slope of AB = = 1 = tan 45° y − 0 = tan 60° ( x − 2) ⇒ x 3 − y − 2 3 = 0
3−2

Y y Example 44. The centre of a square is at the origin


and one vertex is A (2 , 1). Find the coordinates of other
C vertices of the square.
B(3,1)
15°
Sol. [By special corollary (ii)]
45° Q A ≡ ( 2, 1)
X′ X
O A(2,0) ∴ z A = 2 + i , where i = −1

Y′ B
A(2,1)
∴ ∠ BAX = 45°
Now line AB is rotated through an angle of 15°
⇒ ∠ CAX = 45° + 15° = 60°
O
and AB = AC = 2
Equation of line AC in parametric form is
C
x = 2 + r cos 60°
 ...(i) D
y = 0 + r sin 60° 
94 Textbook of Coordinate Geometry

Now, in triangle AOB,  1 


then centre of square E ≡  − , 0
π  2 
OA = OB, ∠ AOB = 90° =
2 1
∴ zE = −
π 2
i
∴ zB = zA e 2 = iz A = 2i − 1 Now, in ∆ AEB, ( EA = EB )
π
∴ B ≡ ( − 1 , 2) zB − zE i
=e 2 =i
QO is the mid-point of AC and BD zA − zE
∴ C ≡ ( −2, − 1) and D ≡ (1, − 2). 1  1
⇒ zB + = i 1 + i + 
2  2
y Example 45. The extremities of the diagonal of a
square are (1, 1) , ( −2 , − 1). Obtain the other two vertices ∴
3 3
zB = − + i
and the equation of the other diagonal. 2 2
 3 3
Sol. (By special corollary (ii)) B ≡ − , 
 2 2
Q A ≡ (1, 1)
∴ z A = 1 + i , where i = −1  3 3
then D ≡  −1 + , − 
 2 2
and C ≡ ( − 2, − 1)
∴ zC = − 2 − i  1 3
or D ≡ ,− 
B
 2 2
A(1,1) Hence, equation of other diagonal BD is
3
−0
90°  1
y −0= 2 x + 
3 1  2
E − +
(–1/2,0) 2 2
⇒ 6x + 4y + 3 = 0
C
(–2,–1)
D

Exercise for Session 1


1. The distance of the point (3, 5) from the line 2x + 3y − 14 = 0 measured parallel to the line x − 2y = 1
is
7 7
(a) (b) (c) 5 (d) 13
5 13

2. The lines x cos α + y sin α = p1 and x cos β + y sin β = p 2 will be perpendicular, if


(a) α = β (b) | α − β | = π / 2
(c) α = π / 2 (d) α ± β = π / 2

3. If each of the points ( x1, 4), ( −2, y1) lies on the line joining the points (2, − 1), (5, − 3), then the point P ( x1, y1) lies
on the line
(a) 6 (x + y ) − 25 = 0 (d) 2x + 6y + 1 = 0
(c) 2x + 3y − 6 = 0 (d) 6 (x + y ) + 25 = 0

4. The equation of the straight line passing through the point (4, 3) and making intercepts on the coordinate axes
whose sum is −1 is
x y x y x y x y
(a) + = − 1 and + =−1 (b) − = − 1 and + = −1
2 3 −2 1 2 3 −2 1
x y x y x y x y
(c) + = 1 and + =1 (d) − = 1 and + =1
2 3 −2 1 2 3 −2 1
Chap 02 The Straight Lines 95

5. If the straight lines ax + by + c = 0 and x cos α + y sin α = c enclose an angle π / 4 between them and meet
the straight line x sin α − y cos α = 0 in the same point, then
(a) a 2 + b 2 = c 2 (b) a 2 + b 2 = 2 (c) a 2 + b 2 = 2c 2 (d) a 2 + b 2 = 4

6. The angle between the lines 2x − y + 3 = 0 and x + 2y + 3 = 0 is


(a) 30° (b) 45° (c) 60° (d) 90°

7. The inclination of the straight line passing through the point (–3, 6) and the mid-point of the line joining the
points (4, –5) and (–2, 9) is
(a) π / 4 (b) π / 2 (c) 3 π / 4 (d) π

8. A square of side a lies above the X-axis and has one vertex at the origin. The side passing through the origin
makes an angle π /6 with the positive direction of X-axis. The equation of its diagonal not passing through the
origin is
(a) y ( 3 − 1) − x (1 − 3 ) = 2a (b) y ( 3 + 1) + x (1 − 3 ) = 2a
(c) y ( 3 + 1) + x (1 + 3 ) = 2a (d) y ( 3 + 1) + x ( 3 − 1) = 2a

9. A (1, 3) and C (7, 5) are two opposite vertices of a square. The equation of side through A is
(a) x + 2y − 7 = 0 (b) x − 2y + 5 = 0
(c) 2x + y − 5 = 0 (d) 2x − y + 1 = 0

10. The equation of a straight line passing through the point ( −5 , 4) and which cuts off an intercept of 2 units
between the lines x + y + 1 = 0 and x + y − 1 = 0 is
(a) x − 2y + 13 = 0 (b) 2x − y + 14 = 0
(c) x − y + 9 = 0 (d) x − y + 10 = 0

11. Equation to the straight line cutting off an intercept 2 from negative direction of the axis of y and inclined at 30°
to the positive direction of axis of x is
(a) y + x − 3 = 0 (b) y − x + 2 = 0
(c) y − x 3 − 2 = 0 (d) y 3 − x + 2 3 = 0

12. What is the value of y so that the line through (3, y ) and (2, 7) is parallel to the line through (–1, 4) and (0, 6) ?

13. A straight line is drawn through the point P (2, 2) and is inclined at an angle of 30° with the X-axis. Find the
coordinates of two points on it at a distance 4 from P on either side of P.
π
14. If the straight line through the point P (3, 4) makes an angle with X-axis and meets the line 12x + 5y + 10 = 0
6
at Q, find the length of PQ.

15. Find the distance of the point (2, 3) from the line 2x − 3y + 9 = 0 measured along the line x − y + 1 = 0.

16. A line is such that its segment between the straight line 5x − y − 4 = 0 and 3x + 4y − 4 = 0 is bisected at the
point (1, 5) . Obtain the equation.

17. The side AB and AC of a ∆ ABC are respectively 2x + 3y = 29 and x + 2y = 16. If the mid-point of BC is (5, 6),
then find the equation of BC.
18. A straight line through A ( −15, − 10) meets the lines x − y − 1 = 0, x + 2y = 5 and x + 3y = 7 respectively at A, B
12 40 52
and C. If + = , prove that the line passes through the origin.
AB AC AD
Session 2
Position of Two Points Relative to a Given Line,
Position of a Point Which Lies Inside a Triangle,
Equations of Lines Parallel and Perpendicular to

a Given Line, Distance of a Point From a Line,


Distance Between Two Parallel Lines,
Area of Parallelogram,

Position of Two Points Relative ∴ λ is negative

to a Given Line ⇒
 ax + by 1 + c 
− 1  <0
 ax 2 + by 2 + c 
Theorem : The points P ( x 1 , y 1 ) and Q ( x 2 , y 2 ) lie on the
same or opposite sides of the line ax + by + c = 0 according  ax 1 + by 1 + c 
⇒   >0
as  a x 2 + by 2 + c 
ax 1 + by 1 + c
> 0 or <0 . f (x 1 , y 1 )
ax 2 + by 2 + c or >0
f (x 2 , y 2 )
Proof : Let the line PQ be divided by the line
ax + by + c = 0 in the ratio λ : 1 (internally) at the point R. where, f ( x , y ) ≡ ax + by + c .
 x + λx 2 y 1 + λy 2  Case II : Let P and Q are on opposite sides of the line
∴ The coordinates of R are  1 , . ax + by + c = 0
 1+ λ 1+ λ 
The point of R lies on the line ax + by + c = 0 ∴ R divides PQ internally.
 x + λx 2   y 1 + λy 2  ∴ λ is positive
then a 1  +b   +c =0  ax + by 1 + c 
 1+ λ   1+ λ  ⇒ − 1  >0
P R Q

⇒ λ (ax 2 + by 2 + c ) + (ax 1 + by 1 + c ) = 0  ax 2 + by 2 + c 

 ax + by 1 + c   ax 1 + by 1 + c 
⇒ λ =− 1  (Qax 2 + by 2 + c ≠ 0 ) ⇒   <0
 ax 2 + by 2 + c   ax 2 + by 2 + c 
Case I : Let P and Q are on same side of the line f (x 1 , y 1 )
or <0
ax + by + c = 0. f (x 2 , y 2 )
∴ R divides PQ externally. where, f ( x , y ) = ax + by + c

R
Remarks
1. The side of the line where origin lies is known as origin side.
Q 2. A point ( α, β ) will lie on origin side of the line ax + by + c = 0, if
P aα + bβ + c and c have same sign.
Q
P 3. A point ( α, β ) will lie on non-origin side of the line
R
ax + by + c = 0, if aα + bβ + c and c have opposite sign.
Chap 02 The Straight Lines 97

y Example 46. Are the points (2 , 1) and ( −3, 5) on the Hence, the given village V does not lie on the canal.
9
same or opposite side of the line 3 x − 2y + 1 = 0 ? Also if f ( x , y ) ≡ x + y −
2
Sol. Let f ( x , y ) ≡ 3x − 2y + 1
 9 
4 +3− 
f ( 2, 1) 3 (2) − 2(1) + 1 5 f ( 4, 3)  2  = −  7 2 − 9  <0
∴ = = − <0 ∴ =  
f ( −3, 5) 3 ( −3) − 2 (5) + 1 18 f (0, 0)  0 + 0 − 9   9 
 
Therefore, the two points are on the opposite sides of the 2
given line. Hence, the village is on that side of the canal on which
origin or the city lies.
y Example 47. Is the point (2 , − 7 ) lies on origin side of
the line 2x + y + 2 = 0 ?
Sol. Let f ( x , y ) ≡ 2x + y + 2
Position of a Point Which Lies
∴ f ( 2, − 7 ) = 2 ( 2) − 7 + 2 = − 1 Inside a Triangle
f (2, − 7 ) < 0 and constant 2 > 0
Hence, the point (2, − 7 ) lies on non-origin side. Let P ( x 1 , y 1 ) be the point and equations of the sides of a
triangle are
y Example 48. A straight canal is at a distance of BC : a 1 x + b 1 y + c 1 = 0
1
4 km from a city and the nearest path from the city CA : a 2 x + b 2 y + c 2 = 0
2
to the canal is in the north-east direction. Find and AB : a 3 x + b 3 y + c 3 = 0
whether a village which is at 3 km north and 4 km east
A (x′,y′)
from the city lies on the canal or not. If not, then on
which side of the canal is the village situated ?
Sol. Let O (0, 0) be the given city and AB be the straight canal.
9
Given, OL = km P (x1,y1)
2
North
Y
(x′′ ,y′′ )B C (x′′′,y′′′)
B
First find the coordinates of A, B and C say,
L V(4,3)
A ≡ ( x ′ , y ′ ); B ≡ ( x ′ ′ , y ′ ′ ) and C ≡ ( x ′ ′ ′ , y ′ ′ ′ )
t
as
-E
rth

and if coordinates of A, B, C are given, then find equations of


No

45° BC , CA and AB.


X′ X
O A East
If P ( x 1 , y 1 ) lies inside the triangle, then P and A must be
Y′
on the same side of BC , P and B must be on the same side
∴ Equation of AB of AC , P and C must be on the same side of AB , then
i.e. Equation of canal is a 1 x 1 + b 1y 1 + c 1
>0 ...(i)
9 a 1 x ′ + b 1y ′ + c 1
x cos 45° + y sin 45° =
2 a 2 x 1 + b 2y 1 + c 2
>0 ...(ii)
9 a 2 x ′ ′ + b 2y ′ ′ + c 2
or x +y = ...(i)
2 a 3 x 1 + b 3y 1 + c 3
Let V be the given village, then V ≡ ( 4, 3)
and >0 ...(iii)
a 3 x ′ ′ ′ + b 3y ′ ′ ′ + c 3
Putting x = 4 and y = 3 in Eq. (i),
9 9 The required values of P ( x 1 , y 1 ) must be intersection of
then 4 + 3 = , i.e. 7 = which is impossible. these inequalities Eqs. (i), (ii) and (iii).
2 2
98 Textbook of Coordinate Geometry

Aliter (Best Method) : First draw the exact diagram of or 4t − 6 < 0


the problem. If the point P ( x 1 , y 1 ) 3
∴ t< ...(ii)
move on the line y = ax + b for all x 1 , then 2
P ≡ ( x 1 , ax 1 + b ) and P , C must be on the same side of AB
value of (3x − 2y + 6) at P (t , t + 1)
and the portion DE of the line y = ax + b (Excluding D and ∴ >0
value of (3x − 2y + 6) at C (6, 1)
E) lies within the triangle. Now line y = ax + b cuts any
two sides out of three sides, then find coordinates of D and 3t − 2 ( t + 1) + 6
i.e. >0
E. 18 − 2 + 6
D ≡ (α, β) t +4
or >0
and E ≡ ( γ , δ ) (say) 22
then α < x1 < γ or t + 4>0
∴ t>−4 ...(iii)
and β < ax 1 + b < δ

y Example 49. For what values of the parameter t From Eqs. (i), (ii) and (iii), we get
does the point P (t, t + 1) lies inside the triangle ABC 6
− <t <
3
where A ≡ (0, 3), B ≡ ( − 2 , 0) and C ≡ (6 , 1) . 7 2
 6 3
Sol. Equations of sides i.e. t ∈ − , 
 7 2
BC : x − 8y + 2 = 0
CA : x + 3y − 9 = 0 Aliter : First draw the exact diagram of ∆ ABC, the point
and AB : 3x − 2y + 6 = 0 P (t , t + 1) move on the line
Since, P (t , t + 1) lies inside the triangle ABC, then P and A y = x +1
must be on the same side of BC for all t.
value of ( x − 8y + 2) at P (t , t + 1)
∴ >0 Y
value of ( x − 8y + 2) at A (0, 3) A(0,3)
t − 8 ( t + 1) + 2 E
i.e. >0
0 − 24 + 2
1
− 7t − 6
x+
P
>0 C(6,1)
y=

or
− 22 D
X′ X
or 7t + 6 > 0 B(–2,0) O
6 Y′
∴ t>− ...(i)
7 Now, D and E are the intersection of
Y
y = x + 1, x − 8y + 2 = 0
A(0,3)
and y = x + 1, x + 3y − 9 = 0
respectively.
P
 6 1
∴ D ≡ − , 
C(6,1)  7 7
X′  3 5
B(–2,0) O
X and E≡ , 
 2 2
Y′
and P , B must be on the same side of CA Thus, the points on the line y = x + 1 whose x-coordinates
6 3
value of ( x + 3y − 9 ) at P (t , t + 1) lies between − and lie within the triangle ABC.
∴ >0 7 2
value of ( x + 3y − 9 ) at B ( − 2, 0)
6 3
t + 3 ( t + 1) − 9 Hence, − <t <
i.e. >0 7 2
−2+0−9  6 3
4t − 6 i.e. t ∈ − , 
or >0  7 2
−11
Chap 02 The Straight Lines 99

y Example 50. Find λ if ( λ , 2) is an interior point of  5 7  1 1


A  ,  , B ( − 7, 5) and C  , 
 4 8 3 9
∆ ABC formed by x + y = 4, 3x − 7 y = 8 and
4 x − y = 31. Q P (α, α 2 ) lies inside the ∆ ABC, then
Sol. Let P ≡ ( λ , 2) (i) A and P must lie on the same side of BC
(ii) B and P must lie on the same side of CA
First draw the exact diagram of ∆ ABC, the point P ( λ , 2)
move on the line y = 2 for all λ . (iii) C and P must lie on the same side of AB, then
5 21
Y
+ −1
2 8 >0
A
2α + 3α 2 − 1
D E
=8 33
7y
P ⇒ >0
3x– 2α + 3α 2 − 1
B
X′ X or 3α 2 + 2α − 1 > 0
O
=31

 1
x+

⇒ ( α + 1) α −  > 0
y=

4x–y

 3
4

C 1 
⇒ α ∈ ( − ∞, − 1) ∪  , ∞ ...(i)
3 
Y′ − 35 − 30 − 1
and >0
5α − 6α 2 − 1
Now, D and E are the intersection of
⇒ 5α − 6α 2 − 1 < 0
3x − 7y = 8, y = 2
and 4 x − y = 31, y = 2 respectively.  1  1
⇒ α −  α −  > 0
 3  2
 22   33 
∴ D ≡  , 2 and E ≡  , 2 ∴ α ∈ ( − ∞, 1 / 3) ∪ (1 / 2, ∞ ) ..(ii)
3  4 
1 2
Thus, the points on the line y = 2 whose x-coordinates lies + −3
and 3 9 >0
22 33 α + 2α 2 − 3
between and lie within the ∆ ABC.
3 4
⇒ α + 2α 2 − 3 < 0
22 33
Hence, <λ< ⇒ (2α + 3) (α − 1) < 0
3 4
 22 33 ∴ α ∈ ( − 3 / 2, 1) ...(iii)
i.e. λ ∈ ,  From Eq. (i), Eq. (ii) and Eq. (iii), we get
 3 4
α ∈ ( −3 / 2, − 1) ∪ (1 / 2, 1).
y Example 51. Determine all values of α for which the Aliter : Let P (α, α 2 ) first draw the exact diagram of
point (α , α 2 ) lies inside the triangle formed by the lines ∆ ABC.
2x + 3y − 1 = 0, x + 2y − 3 = 0 and 5x − 6 y − 1 = 0. The point P (α, α 2 ) move on the curve y = x 2 for all α.
Sol. The coordinates of the vertices are Now, intersection of y = x 2
and 2x + 3y − 1 = 0
Y
B or 2x + 3x 2 − 1 = 0
1
∴ x = − 1, x =
3
Let intersection points
P
 1 1
A D ≡ ( − 1, 1) and E ≡  , 
3 9
intersection of y = x 2
C
X′ X and x + 2y − 3 = 0
O
or x + 2x 2 − 3 = 0
Y′
∴ x = 1, x = − 3 / 2
100 Textbook of Coordinate Geometry

Y
B
Dividing it by k, then
y=x2 c
G ax + by + =0
k
 c
F or ax + by + λ = 0  writing λ for 
A  k
D
Hence, any line parallel to ax + by + c = 0 is
I
C ax + by + λ = 0
X′ X
O where λ is some constant.
Y′ Aliter : The given line is
Let intersection points ax + by + c = 0 ...(i)
 3 9 a
F ≡ (1, 1) and G ≡  − ,  Its slope = −
 2 4 b
and intersection of y = x 2 and 5x − 6y − 1 = 0 Thus, any line parallel to Eq. (i) is given by
or 5x − 6x 2 − 1 = 0  a
y =  −  x + λ1
1 1  b
∴ x = ,x =
3 2 ⇒ ax + by − bλ 1 = 0
Let intersection points
⇒ ax + by + λ = 0 (writing λ for − bλ 1 )
 1 1 1 1
H ≡  ,  and I ≡  ,  . where, λ is some constant.
3 9 2 4
Thus the points on the curve y = x 2 whose x-coordinate
Corollary : The equation of the line parallel to
1
ax + by + c = 0 and passing through ( x 1 , y 1 ) is
lies between −3 / 2 & − 1 and & 1 lies within the triangle a ( x − x 1 ) + b (y − y 1 ) = 0
2
ABC. Working Rule :
3 1
Hence, − < α < − 1 or < α < 1 (i) Keep the terms containing x and y unaltered.
2 2
(ii) Change the constant.
 3  1 
i.e. α ∈  − , − 1 ∪  , 1 (iii) The constant λ is determined from an additional
 2  2 
condition given in the problem.
Theorem 2 : The equation of the line perpendicular to the
Equations of Lines Parallel and line ax + by + c = 0 is
bx − ay + λ = 0, where λ is some constant.
Perpendicular to a Given Line Proof : Let the equation of any line perpendicular to
Theorem 1: The equation of line parallel to ax + by + c = 0 ... (i)
ax + by + c = 0 is ax + by + λ = 0, where λ is some be a 1 x + b 1y + c 1 = 0 ...(ii)
constant. then aa 1 + bb 1 = 0
Proof : Let the equation of any line parallel to or aa 1 = − bb 1
ax + by + c = 0 ...(i) a1 b1
⇒ = =k (say)
be a 1 x + b 1y + c 1 = 0 ...(ii) b −a
a1 b1 ∴ a 1 = bk , b 1 = − ak
then = =k (say)
a b
Then, from Eq. (ii), bkx − aky + c 1 = 0 dividing it by k, then
∴ a 1 = ak , b 1 = bk c
Then from Eq. (ii), bx − ay + 1 = 0
k
akx + bky + c = 0
Chap 02 The Straight Lines 101

 c1  y Example 53. Find the general equation of the line


or bx − ay + λ = 0  writing λ for 
 k which is perpendicular to x + y + 4 = 0 . Also find such
line through the point (1, 2) .
Hence, any line perpendicular to ax + by + c = 0 is
Sol. Equation of any line perpendicular to x + y + 4 = 0 is
bx − ay + λ = 0
x −y + λ =0 ...(i)
where, λ is some constant.
which is general equation of the line.
Aliter : The given line is
Also Eq. (i) passes through (1, 2), then
ax + by + c = 0 ...(i) 1−2+ λ =0
a
Its slope = − ∴ λ =1
b Then from Eq. (i), required line is
b x −y +1=0
Slope of perpendicular line of Eq. (i) is .
a
Thus any line perpendicular to Eq. (i) is given by y Example 54. Show that the equation of the line
passing through the point (a cos 3 θ, a sin 3 θ ) and
b 
y =   x + λ1 perpendicular to the line
a 
x sec θ + y cosec θ = a is
⇒ bx − ay + aλ 1 = 0
x cosθ − y sinθ = a cos2θ
or bx − ay + λ = 0 (writing λ for aλ 1 )
Sol. The given equation x sec θ + y cosec θ = a can be written as
where, λ is some constant. x sin θ + y cos θ = a sin θ cos θ ...(i)
Corollary 1 : The equation of the line through ( x 1 , y 1 ) ∴ equation of perpendicular line of Eq. (i) is
and perpendicular to ax + by + c = 0 is x cos θ − y sin θ = λ ...(ii)
b ( x − x 1 ) − a (y − y 1 ) = 0 Also it is pass through (a cos 3 θ, a sin 3 θ )
Corollary 2 : Also equation of the line perpendicular to ∴ a cos 3 θ ⋅ cos θ − a sin 3 θ ⋅ sin θ = λ
ax + by + c = 0 is written as ⇒ λ = a (cos 4 θ − sin 4 θ )
x y = a (cos 2 θ + sin 2 θ ) (cos 2 θ − sin 2 θ )
− + k = 0, where k is some constant.
a b
= a ⋅ 1 ⋅ cos 2θ = a cos 2θ
Working Rule : From Eq. (ii), the required equation of the line is
(i) Interchange the coefficients of x and y and changing x cos θ − y sin θ = a cos 2θ
sign of one of these coefficients. Aliter : (From corollary (2) of Theorem (2)
(ii) Changing the constant term. Equation of any line perpendicular to the line
x sec θ + y cosec θ = a , is
(iii) The value of λ can be determined from an additional
x y
condition given in the problem. − =k
sec θ cosec θ
y Example 52. Find the general equation of the line or x cos θ − y sin θ = k ...(iii)
which is parallel to 3x − 4 y + 5 = 0. Also find such line Also, it pass through (a cos 3 θ, a sin 3 θ )
through the point ( −1 , 2) .
∴ a cos 3 θ ⋅ cos θ − a sin 3 θ ⋅ sin θ = k
Sol. Equation of any parallel to 3x − 4y + 5 = 0 is
or k = a (cos 4 θ − sin 4 θ )
3x − 4y + λ = 0 ...(i)
= a (cos 2 θ + sin 2 θ ) (cos 2 θ − sin 2 θ )
which is general equation of the line.
Also Eq. (i) passes through ( −1, 2), then = a ⋅ 1 ⋅ cos 2θ
3 ( − 1) − 4 ( 2) + λ = 0 = a cos 2θ
From Eq. (iii), the required equation of the line is
∴ λ = 11
x cos θ − y sin θ = a cos 2θ
Then from Eq. (i) required line is
3x − 4y + 11 = 0
102 Textbook of Coordinate Geometry

Aliter I : Let PM makes an angle θ with positive


Distance of a Point From a Line direction of X-axis.
Theorem : The length of perpendicular from a point Then, equation of PM in distance form will be
( x 1 , y 1 ) to the line ax + by + c = 0 is x − x1 y − y1
= =p (QPM = p )
| ax 1 + by 1 + c | cos θ sin θ
(a 2 + b 2 ) Therefore coordinates of M will be
( x 1 + p cos θ, y 1 + p sin θ )
Proof : Given line is ax + by + c = 0
Since, M lies on ax + by + c = 0, then
x y
⇒ + =1 a ( x 1 + p cos θ ) + b (y 1 + p sin θ ) + c = 0
 c  c
−  −  or p (a cos θ + b sin θ ) = − (ax 1 + by 1 + c ) …(iii)
 a  b
a
Y
Since, slope of AB = −
b
B(0, –c/b)
b
∴ Slope of PM =
P(x1,y1) a
b
M ∴ tan θ = (Q PM makes an angle θ with
a positive direction of X-axis)
A(–c/a,0) A
X′ X
Y′
(a2+b 2 )
b
Let the given line intersects the X-axis and Y-axis at A and
 c  θ
B respectively, then coordinates of A and B are  − , 0 
 a  B a C

 c b
and  0, −  respectively. then sin θ =
 b (a 2 + b 2 )
Draw PM perpendicular to AB . a
Now, Area of ∆ PAB and cos θ =
(a 2 + b 2 )
1  c c  c 
= x 1  0 +  −  − − y 1  + 0 (y 1 − 0 ) Now, from Eq. (iii),
2  b a  b 
1 c   a2 b2 
=  | ax 1 + by 1 + c | ...(i) p +  = − (ax 1 + by 1 + c )
2 ab  (a 2 + b 2 ) 2 
(a + b ) 
2

Let PM = p (ax 1 + by 1 + c )
Also, area of ∆ PAB or p=−
(a 2 + b 2 )
1  c
2 2
1   c 
= ⋅ AB ⋅ PM =  − − 0  +  0 +   ⋅p Since, p is positive
2 2  a   b 
| ax 1 + by 1 + c |
∴ p=
1 c 
=   (a 2 + b 2 ) ⋅ p ... (ii) (a 2 + b 2 )
2 ab
Aliter II : Let Q ( x , y ) be any point on the line
From Eqs. (i) and (ii), we have
ax + by + c = 0
1c 
 (a 2 + b 2 ) ⋅ p = 1c 
| ax + by + c |
1 1 Hence, the length of perpendicular from P on AB will be
2 ab 2 ab least value of PQ.
| ax 1 + by 1 + c |
or p=
(a 2 + b 2 )
Chap 02 The Straight Lines 103

Y Q Least value of PQ is PM
| ax 1 + by 1 + c |
B
(x,y) ∴ p = PM =
Q P(x1,y1) (a 2 + b 2 )

M Aliter III : Let M ≡ (h, k )


Since, M (h, k ) lies on AB,
X′ X ah + bk + c = 0 ..(v)
O A
Y′
Now, AB and PM are perpendicular to each other, then
Let z = ( PQ ) 2 (slope of PM ) × (slope of AB) = − 1
= ( x − x 1 ) 2 + (y − y 1 ) 2 ...(iv) y1 − k  a 
⇒ × −  = − 1
2 x1 −h  b
 c ax 
= (x − x 1 )2 +  − − − y1  x 1 − h y 1 − k a ( x 1 − h ) + b (y 1 − k )
 b b  ⇒ = =
a b a .a +b .b
Q ax + by + c = 0 
 c ax  (by law of proportion)
 ∴y = − − 
 b b  (ax 1 + by 1 + c ) − (ah + bk + c )
=
dz  c ax   a a2 + b2
∴ = 2 (x − x 1 ) + 2  − − − y1  − 
dx  b b   b
ax 1 + by 1 + c
 a2  = [from Eq. (v)]
d 2z  a  a a2 + b2
and = 2 + 2  −   −  = 2  1 + 2  = positive
dx 2  b  b  b 
∴ ( PM ) 2 = ( x − x 1 ) 2 + (y − y 1 ) 2
Q z is minimum 2
 ax + by + c 
∴ PQ is also minimum. =  1 2 12  (a 2 + b 2 )
dz  a +b 
For maximum or minimum, =0
dx ∴ Length of perpendicular
 c ax   a (ax 1 + by 1 + c )
2 (x − x 1 ) + 2  − − − y1  −  = 0 PM = ±
 b b   b (a 2 + b 2 )
 a  c ax  | ax 1 + by 1 + c )
or 2 ( x − x 1 ) + 2 (y − y 1 )  −  = 0 Qy = − −  Hence, PM = p =
 b  b b  (a 2 + b 2 )
( x − x 1 ) (y − y 1 ) a ( x − x 1 ) + b (y − y 1 )
or = = Aliter IV : Equation of AB in normal form is
a b a ⋅a + b ⋅b
a b −c
(ax + by + c ) − (ax 1 + by 1 + c ) x+ y=
= (a + b )
2 2
(a + b )
2 2
(a + b 2 )
2
(a 2 + b 2 )
c
(by law of proportion) ⇒ OL = −
0 − (ax 1 + by 1 + c ) (a + b 2 )
2
= (Qax + by + c = 0 )
(a 2 + b 2 )
Y
a (ax 1 + by 1 + c )
⇒ (x − x 1 ) = − B
(a 2 + b 2 ) P(x1,y1)
b (ax 1 + by 1 + c )
and (y − y 1 ) = − Q
(a + b )
2 2 M
L
∴ From Eq. (iv), α A
X′ X
O
(a + b )
2 2
| ax 1 + by 1 + c | Y′
PQ = (ax 1 + by 1 + c ) 2
=
(a 2 + b 2 ) 2 (a 2 + b 2 )
104 Textbook of Coordinate Geometry

Equation of line parallel to AB and passes through ( x 1 , y 1 ) Working Rule :


is (i) Put the point ( x 1 , y 1 ) for ( x , y ) on the LHS while the
a ( x − x 1 ) + b (y − y 1 ) = 0 RHS is zero.
or ax + by = ax 1 + by 1 (ii) Divide LHS after Eq. (i) by (a 2 + b 2 ) , where a and b
Normal form is are the coefficients of x and y respectively.
a b ax 1 + by 1
x+ y= y Example 55. Find the sum of the abscissas of all the
(a + b )
2 2
(a + b )
2 2
(a 2 + b 2 ) points on the line x + y = 4 that lie at a unit distance
ax 1 + by 1 from the line 4 x + 3y − 10 = 0.
⇒ OQ =
(a 2 + b 2 ) Sol. Any point on the line x + y = 4 can be taken as ( x 1, 4 − x 1 ).
As it is at a unit distance from the line 4 x + 3y − 10 = 0, we
ax 1 + by 1 + c get
∴ PM = QL = OQ − OL =
| 4 x 1 + 3 ( 4 − x 1 ) − 10 |
(a 2 + b 2 ) =1
( 4 2 + 32 )
Hence, required perpendicular distance
⇒ | x1 + 2 | = 5 ⇒ x1 + 2 = ± 5
| ax 1 + by 1 + c |
p= ⇒ x 1 = 3 or −7
(a 2 + b 2 ) ∴ Required sum = 3 − 7 = − 4.
Aliter V : The equation of line through P ( x 1 , y 1 ) and y Example 56. If p and p ′ are the length of the
perpendicular to ax + by + c = 0 is
perpendiculars from the origin to the straight lines
b ( x − x 1 ) − a (y − y 1 ) = 0 ...(vi) whose equations are x sec θ + y cos ec θ = a and
If this perpendicular meet the line ax + by + c = 0 in x cos θ − y sin θ = a cos 2θ , then find the value of
M ( x 2 , y 2 ) then ( x 2 , y 2 ) lie on both the lines 4p 2 + p ′ 2 .
ax + by + c = 0 and Eq. (vi), then |−a |
Sol. We have, p =
b ( x 2 − x 1 ) − a (y 2 − y 1 ) = 0, ax 2 + by 2 + c = 0 (sec θ + cosec 2θ )
2

ax 2 + by 2 + c = a ( x 2 − x 1 ) + b(y 2 − y 1 ) + ax 1 + by 1 + c = 0 a2 a 2 sin 2 θ cos 2 θ


∴ p2 = =
or b ( x 2 − x 1 ) − a (y 2 − y 1 ) = 0 ...(vii) sec 2 θ + cosec 2θ 1
and a ( x 2 − x 1 ) + b (y 2 − y 1 ) = − (ax 1 + by 1 + c ) …(viii) ⇒ 4 p 2 = a 2 sin 2 2θ …(i)
On squaring and adding Eqs. (vii) and (viii), we get | − a cos 2θ |
and p′ = = | − a cos 2θ |
(a 2 + b 2 ) (( x 2 − x 1 ) 2 + (y 2 − y 1 ) 2 ) = (ax 1 + by 1 + c ) 2 (cos 2 θ + sin 2 θ )
or PM = ( x 2 − x 1 ) 2 + (y 2 − y 1 ) 2 ∴ ( p ′ )2 = a 2 cos 2 2θ …(ii)
| ax 1 + by 1 + c | ∴ Adding Eqs. (i) and (ii), we get
= 4 p 2 + p ′2 = a2
(a 2 + b 2 )
Hence, length of perpendicular y Example 57. If p is the length of the perpendicular
x y
| ax 1 + by 1 + c | from the origin to the line + = 1, then prove that
PM = p = a b
(a 2 + b 2 ) 1 1 1
+ = .
Corollary 1 : The length of perpendicular from the origin a2 b2 p2
to the line ax + by + c = 0 is Sol. p = length of perpendicular from origin to
| a ⋅0 + b ⋅0 + c | | c| x y
+ =1
i.e. a b
(a + b )
2 2
(a + b 2 )
2
| 0 + 0 − 1| 1
= =
Corollary 2 : The length of perpendicular from ( x 1 , y 1 ) to  1
2
 1
2
1 1
the line x cos α + y sin α = p is   +   2 + 2
 a b  a b 
| x 1 cos α + y 1 sin α − p |
= | x 1 cos α + y 1 sin α − p | or
1
=
1
+
1
or
1
+
1
=
1
(cos 2 α + sin2 α ) p2 a2 b2 a2 b2 p2
Chap 02 The Straight Lines 105

y Example 58. Prove that no line can be drawn y Example 59. Find the distance between the lines
through the point (4, − 5) so that its distance from 5x − 12y + 2 = 0 and 5x − 12y − 3 = 0.
( −2 , 3) will be equal to 12. Sol. The distance between the lines
Sol. Suppose, if possible. 5x − 12y + 2 = 0 and 5x − 12y − 3 = 0 is
Equation of line through ( 4, − 5) with slope of m is | 2 − ( − 3) | 5
=
y + 5 = m (x − 4) (5)2 + ( −12)2 13
⇒ mx − y − 4m − 5 = 0
Aliter I : The constant term in both equations are 2 and −3
| m ( − 2) − 3 − 4m − 5 |
Then, = 12 which are of opposite sign. Hence origin lies between them.
m2 + 1 | 2| + | − 3| 5
∴ Distance between lines is =
⇒ | − 6m − 8 | = 12 (m 2 + 1) (5) + ( −12)
2 2 13
3
On squaring, (6m + 8)2 = 144 (m 2 + 1) Aliter II : Putting y =0 in 5x − 12y − 3 = 0 then x =
5
⇒ 4 (3m + 4 )2 = 144 (m 2 + 1) 3 
∴  , 0 lie on 5x − 12y − 3 = 0
⇒ (3m + 4 ) = 36 (m + 1)
2 2 5 
Hence, distance between the lines
⇒ 27m 2 − 24m + 20 = 0 ...(i)
5x − 12y + 2 = 0 and (5x − 12y − 3 = 0)
Since, the discriminant of Eq. (i) is ( −24 )2 − 4 ⋅27 ⋅ 20 = − 1584
3 
which is negative, there is no real value of m . Hence no = Distance from  , 0 to the line 5x − 12y + 2 = 0
5 
such line is possible. 3
5× −0+2
5 5
= =
Distance between Two 5 + ( −12)
2 2 13

Parallel Lines y Example 60. Find the equations of the line parallel to
5x − 12y + 26 = 0 and at a distance of 4 units from it.
Let the two parallel lines be
Sol. Equation of any line parallel to 5x − 12y + 26 = 0 is
ax + by + c = 0 and ax + by + c 1 = 0
5x − 12y + λ = 0 ...(i)
The distance between the parallel lines is the perpendicular
Since, the distance between the parallel lines is 4 units, then
distance of any point on one line from the other line.
| λ − 26 |
Let ( x 1 , y 1 ) be any point on ax + by + c = 0 =4
(5)2 + ( −12)2
∴ ax 1 + by 1 + c = 0 ...(i)
or | λ − 26 | = 52 or λ − 26 = ± 52
Now, perpendicular distance of the point ( x 1 , y 1 ) from the or λ = 26 ± 52 ∴ λ = − 26 or 78
line ax + by + c 1 = 0 is
Substituting the values of λ in Eq. (i), we get
| ax 1 + by 1 + c 1 | | c 1 − c| 5x − 12y − 26 = 0
= [from Eq. (i)]
(a + b )
2 2
(a 2 + b 2 ) and 5x − 12y + 78 = 0

This is required distance between the given parallel lines.


Aliter I : The distance between the lines is Area of Parallelogram
λ
d= Theorem : Area of parallelogram ABCD whose sides
(a + b 2 )
2
AB, BC , CD and DA are represented by a 1 x + b 1 y + c 1 = 0,
a 2 x + b 2 y + c 2 = 0, a 1 x + b 1 y + d 1 = 0
(i) λ = | c 1 − c |, if both the lines are on the same side of
the origin. and a 2 x + b 2 y + d 2 = 0 is
p1 p2 | c1 − d 1| | c2 − d 2|
(ii) λ = | c 1 | + | c |, if the lines are on the opposite side of the or
origin. sin θ a1 b1
| |
Aliter II : Find the coordinates of any point on one of a2 b2
the given lines, preferably putting x = 0 or y = 0. Then the where, p 1 and p 2 are the distances between parallel sides
perpendicular distance of this point from the other line is and θ is the angle between two adjacent sides.
the required distance between the lines.
106 Textbook of Coordinate Geometry

Proof : Since, p 1 and p 1 are the distances between the pairs Now, substitute the values of p 1 , p 2 and sin θ in Eq. (i)
of parallel sides of the parallelogram and θ is the angle
between two adjacent sides, then |c1 − d 1 | |c2 − d 2 |
a1x+b1y+d1=0 ∴ Area of parallelogram ABCD =
D C | a 1b 2 − a 2b 1 |

0
2 =0

|c − d 1 | |c2 − d 2 |

c2 =
= 1
+d

2 y+
p1 a1 b1
2y

+b
| |
+b

a2 b2

a2 x
a2 x

θ a1x+b1y+c1=0
A B
M θ Corollaries :
p2 1. If p 1 = p 2 , then ABCD becomes a rhombus
L p 12
∴ Area of rhombus ABCD =
Area of parallelogram ABCD sin θ
= 2 × Area of ∆ ABD (c 1 − d 1 ) 2
1 =
= 2 × × AB × p 1  a 2 + b 12 
2 | a 1 b 2 − a 2 b 1 |  12 
= AB × p 1  a 2 + b 22 
p  p2  2. If d 1 and d 2 are the lengths of two perpendicular
= 2 × p1 Q in ∆ ABL, sin θ = 
sin θ  AB  diagonals of a rhombus, then
p1 p2 1
= …(i) Area of rhombus = d 1 d 2
sin θ 2
Now, p 1 = Distance between parallel sides AB and DC 3. Area of the parallelogram whose sides are y = mx + a,
|a − b | |c − d |
|c1 − d 1 | y = mx + b, y = nx + c and y = nx + d is .
= |m − n|
(a 12 + b 12 )
and p 2 = Distance between parallel sides AD and BC
y Example 61. Show that the area of the parallelogram
formed by the lines x + 3y − a = 0, 3x − 2y + 3a = 0,
|c2 − d 2 | 20 2
= x + 3y + 4a = 0 and 3x − 2y + 7a = 0 is a sq units.
(a 22 + b 22 ) 11
a1  a2  Sol. Required area of the parallelogram
− − −  | −a − 4a| | 3a − 7a | 20 2
m − m 2  b1  b2  = = a sq units
Also, tan θ = 1 = 1 3 11
1 + m 1m 2  a  a  | |
1 + − 1  − 2  3 –2
 b1   b2 
y Example 62. Show that the area of the parallelogram
 a1 
 Q m 1 = slope of AB = −  formed by the lines
 b1 
x cos α + y sin α = p, x cos α + y sin α = q ,
 a2 
 and m 2 = slope of AD = −  x cos β + y sin β = r, x cos β + y sin β = s is
 b2 
|(p − q)(r − s) cosec (α − β)|.
a b − a 2 b 1
= 1 2  Sol. The equation of sides of the parallelogram are
a 1 a 2 + b 1 b 2 x cos α + y sin α − p = 0,
| a 1b 2 − a 2b 1 | x cos α + y sin α − q = 0,
∴ sin θ = x cos β + y sin β − r = 0,
(a 12 + b 12 )(a 22 + b 22 )
and x cos β + y sin β − s = 0
Chap 02 The Straight Lines 107

∴ Required area of the parallelogram |( −1 + 2)( −1 + 2)|


∴ Area of the rhombus =
| − p − ( −q )| | − r − ( −s )| | p − q | | r − s | 1 1
= =
cos α sin α | sin(β − α )|
| | |a b |
cos β sin β 1 1
= |( p − q )(r − s )cosec (α − β )| b a
a 2b 2
y Example 63. Prove that the diagonals of the = (a ≠ b )
|b − a 2 |
2
parallelogram formed by the lines
x y x y x y x y y Example 64. Show that the four lines ax ± by ± c = 0
+ = 1, + = 1, + = 2 and + = 2
a b b a a b b a 2c 2
enclose a rhombus whose area is .
are at right angles. Also find its area (a ≠ b). | ab |
Sol. The distance between the parallel sides
Sol. The given lines are
x y x y
+ = 1 and + =2 ax + by + c = 0 …(i)
a b a b
ax + by − c = 0 …(ii)
|2 − 1| 1
is = = p1 (say) ax − by + c = 0 …(iii)
1 1 1 1
ax − by − c = 0
 2 +   2 +  and …(iv)
a b2  a b2  Distance between the parallel lines Eqs. (i) and (ii) is
and the distance between the parallel sides 2c
= p 1 (say) and distance between the parallel
x y
+ = 1 and
x y
+ =2 (a + b 2 )
2

b a b a lines Eqs. (iii) and (iv) is


|2 − 1| 1 2c
is = = p2 (say) = p2 (say)
1 1 1 1 (a + b 2 )
2
 2 + 2  2 + 2
b a  a b 
Here, p 1 = p 2
Here, p1 = p 2 .
∴ it is a rhombus.
∴ Parallelogram is a rhombus.
|(c + c )(c + c )| 4c 2 2c 2
But we know that diagonals of rhombus are perpendicular ∴ Area of rhombus = = =
a b | − 2ab | | ab |
to each other. | |
a − b

Exercise for Session 2


1. The number of lines that are parallel to 2x + 6y − 7 = 0 and have an intercept 10 between the coordinate axes is
(a) 1 (b) 2 (c) 4 (d) infinitely many

2. The distance between the lines 4x + 3y = 11 and 8x + 6y = 15 is


7 7 7 9
(a) (b) (c) (d)
2 5 10 10

3. If the algebraic sum of the perpendicular distances from the points (2, 0), (0, 2) and (1, 1) to a variable straight
line is zero, then the line passes through the point
(a) (1, 1) (b) (–1, 1) (c) (–1, –1) (d) (1, –1)

4. If the quadrilateral formed by the lines ax + by + c = 0, a′ x + b ′ y + c′ = 0, ax + by + c′ = 0 and


a′ x + b ′ y + c′ = 0 have perpendicular diagonals, then
(a) b 2 + c 2 = b ′2 + c ′2 (b) c 2 + a 2 = c ′2 + a ′2 (c) a 2 + b 2 = a ′2 + b ′2 (d) None of these

5. The area of the parallelogram formed by the lines 3x − 4y + 1 = 0, 3x − 4y + 3 = 0, 4x − 3y − 1 = 0 and


4x − 3y − 2 = 0, is
1 2 3 4
(a) sq units (b) sq units (c) sq units (d) sq units
7 7 7 7
108 Textbook of Coordinate Geometry

6. Area of the parallelogram formed by the lines y = mx , y = mx + 1, y = nx and y = nx + 1equals


|m + n | 2 1 1
(a) (b) (c) (d)
(m − n )2 |m + n | |m + n | |m − n |

7. The coordinates of a point on the line y = x where perpendicular distance from the line 3x + 4y = 12 is 4 units,
are
(a)  ,
5
(b)  ,
3
(c)  − , −
8
(d)  , − 
3 3 8 32 32
  
7 7 2 2  7 7  7 7

8. A line passes through the point (2, 2) and is perpendicular to the line 3x + y = 3, then its y-intercept is
2 2 4 4
(a) − (b) (c) − (d)
3 3 3 3

9. If the points (1, 2) and (3, 4) were to be on the opposite side of the line 3x − 5y + a = 0, then
(a) 7 < a < 11 (b) a = 7 (c) a = 11 (d) a < 7 or a > 11

10. The lines y = mx , y + 2x = 0, y = 2x + k and y + mx = k form a rhombus if m equals


1
(a) –1 (b) (c) 1 (d) 2
2
x y
11. The points on the axis of x, whose perpendicular distance from the straight line + = 1is a
a b
b a
(a) (a ± (a 2 + b 2 ), 0) (b) (b ± (a 2 + b 2 ), 0)
a b
b a
(c) (a + b , 0) (d) (a ± (a 2 + b 2 ), 0)
a b

12. The three sides of a triangle are given by ( x 2 − y 2 ) (2x + 3y − 6) = 0. If the point ( −2, a ) lies inside and (b ,1) lies
outside the triangle, then
(a) a ∈  2,  ; b ∈ (−1, 1) (b) a ∈  −2,  ; b ∈  −1, 
10 10 9
 3  3  2
(c) a ∈  1,  ; b ∈ (−3, 5)
10
(d) None of these
 3

13. Are the points (3, 4) and (2, –6) on the same or opposite sides of the line 3x − 4y = 8 ?

14. If the points (4, 7) and (cos θ, sin θ), where 0 < θ < π, lie on the same side of the line x + y − 1 = 0, then prove
that θ lies in the first quadrant.

15. Find the equations of lines parallel to 3x − 4y − 5 = 0 at a unit distance from it.

16. Show that the area of the parallelogram formed by the lines 2x − 3y + a = 0, 3x − 2y − a = 0,2x − 3y + 3a = 0
2a 2
and 3x − 2y − 2a = 0 is sq units.
5

17. A line ‘L’ is drawn from P (4, 3) to meet the lines L1 : 3x + 4y + 5 = 0 and L2 : 3x + 4y + 15 = 0 at point A and B
respectively. From ‘A’ a line, perpendicular to L is drawn meeting the line L2 at A1. Similarly from point ‘B’ a line,
perpendicular to L is drawn meeting the line L1 at B1. Thus a parallelogram AABB 1 1 is formed. Find the
equation (s) of ‘L’ so that the area of the parallelogram AABB 1 1 is least.

18. The vertices of a ∆OBC are O (0, 0), B ( −3, − 1), C ( −1, − 3). Find the equation of the line parallel to BC and
1
intersecting the sides OB and OC and whose perpendicular distance from the origin is .
2
Session 3
Point of Intersection of Two Lines, Concurrent Lines
Family of Lines, How to Find Circumcentre and
Orthocentre by Slopes
Points of Intersection of a 1 b1 c 1 
a 2 b 2 c 2 = 0
Two Lines  
 a 3 b 3 c 3
Let a 1 x + b 1 y + c 1 = 0 and a 2 x + b 2 y + c 2 = 0 be two III Method : The condition for the lines P = 0, Q = 0 and
non-parallel lines. If ( x 1 , y 1 ) be the coordinates of their R = 0 to be concurrent is that three constants l, m, n (not all
point of intersection, zeros at the same time) can be obtained such that
then a 1 x 1 + b 1 y 1 + c 1 = 0 and a 2 x 1 + b 2 y 1 + c 2 = 0 lP + mQ + nR = 0
Solving these two by cross multiplication, then
Remarks
x1 y1 1
= = 1. The reader is advised to follow method I in numerical problems.
b 1c 2 − b 2c 1 c 1a 2 − c 2a 1 a 1b 2 − a 2b 1 2. For finding unknown quantity applying method II.
 b c − b 2c 1 c 1a 2 − c 2a 1  y Example 65. Show that the lines
we get (x 1 , y 1 ) ≡  1 2 , 
 a 1b 2 − a 2b 1 a 1b 2 − a 2b 1 
2 x + 3 y − 8 = 0 , x − 5 y + 9 = 0 and 3 x + 4y − 11 = 0
 b1 b2 c1 c2  are concurrent.
 
≡
c1 c2 a1 a2  Sol. I Method : Solving the first two equations, we see that
,
 a1 a2 a1 a2  their point of intersection is (1, 2) which also satisfies the
  third equation
 b1 b2 b1 b2  3 × 1 + 4 × 2 − 11 = 0
Hence the given lines are concurrent.
Remarks a1 b1 c 1 2 3 8
1. Here lines are not parallel, they have unequal slopes, then,
II Method : We have a 2 b 2 c 2 = 1 −5 9
a1b2 − a2b1 ≠ 0
2. In solving numerical questions, we should not be remember a 3 b 3 c 3 3 4 −11
the coordinates ( x1, y1 ) given above, but we solve the equations
Applying C 3 → C 3 + C 1 + 2C 2
directly.
2 3 0
= 1 −5 0 =0
Concurrent Lines 3 4 0
Hence the given lines are concurrent.
The three given lines are concurrent, if they meet in a III Method : Suppose
point. Hence to prove that three given lines are l (2x + 3y − 8) + m ( x − 5y + 9 ) +n (3x + 4y − 11) = 0
concurrent, we proceed as follows : ⇒ x (2l + m + 3n ) + y (3l − 5m + 4n ) + ( −8l + 9m − 11n ) = 0
I Method : Find the point of intersection of any two lines = 0 ⋅ x + 0 ⋅y + 0
by solving them simultaneously. If this point satisfies the On comparing,
third equation also, then the given lines are concurrent. 2l + m + 3n = 0, 3l − 5m + 4n = 0, − 8l + 9m − 11n = 0
II Method : The three lines a i x + b i y + c i = 0, i = 1 , 2 , 3 are After solving, we get l = 19, m = 1, n = − 13
concurrent if Hence, 19 (2x + 3y − 8) + ( x − 5y + 9 ) − 13 (3x + 4y − 11) = 0
Hence the given lines are concurrent.
110 Textbook of Coordinate Geometry

y Example 66. If the lines ax + y + 1 = 0 , x + by + 1 = 0  ν


⇒ (a 1 x + b 1 y + c 1 ) + λ (a 2 x + b 2 y + c 2 ) = 0  where, λ = 
and x + y + c = 0 (a, b and c being distinct and  µ
different from 1) are concurrent, then find the value of It is a first degree equation in x and y. So it represents
1 1 1
+ + . family of lines through the point of intersection of Eqs. (i)
1 − a 1 −b 1 − c and (ii).
Sol. The given lines are concurrent, then Thus, the family of straight lines through the intersection
a 1 1 a 1−a 1−a of lines
1 b 1 =0 ⇒ 1 b −1 0 =0 L1 ≡ a 1 x + b 1 y + c 1 = 0
1 1 c 1 0 c −1 and L2 ≡ a 2 x + b 2 y + c 2 = 0 is
(applying C 2 → C 2 − C 1 and C 3 → C 3 − C 1 ) (a 1 x + b 1 y + c 1 ) + λ (a 2 x + b 2 y + c 2 ) = 0
Expanding along first row i.e. L1 + λL2 = 0
⇒ a ( b − 1) ( c − 1) − ( 1 − a ) ( c − 1) − ( 1 − a ) ( b − 1) = 0
⇒ a (1 − b ) (1 − c ) + (1 − a ) (1 − c ) + (1 − a ) (1 − b ) = 0 Corollaries :
Dividing by (1 − a ) (1 − b ) (1 − c ), then 1. The equation L1 + λL2 = 0 or µL1 + νL2 = 0 represent
a 1 1 a line passing through the intersection of the lines
+ + =0
1−a 1−b 1−c L1 = 0 and L2 = 0 which is a fixed point, where λ, µ, ν
1 1 1 are constants.
⇒ −1+ + + =0
1−a 1−b 1−c 2. For finding fixed point, the number of constants in
1 1 1 family of lines are one or two. If number of constants
Hence, + + =1
1−a 1−b 1−c more than two, then convert in two or one constant
form.
y Example 67. Show that the three straight lines
2x − 3y + 5 = 0 ,3x + 4 y − 7 = 0 and 9 x − 5y + 8 = 0 y Example 68. Find the equation of the straight line
meet in a point. passing through the point (2 , 1) and through the point
of intersection of the lines x + 2y = 3 and 2x − 3y = 4.
Sol. If we multiply these three equations by 3, 1 and −1, we have
3 (2x − 3y + 5) + (3x + 4y − 7 ) − (9 x − 5y + 8) = 0 Sol. Equation of any straight line passing through the
intersection of the lines x + 2y = 3 and 2x − 3y = 4 is
which is an identity.
λ ( x + 2y − 3) + (2x − 3y − 4 ) = 0 ...(i)
Hence, three lines meet in a point.
Since, it passes through the point (2, 1)
∴ λ ( 2 + 2 − 3) + ( 4 − 3 − 4 ) = 0
Family of Lines ⇒ λ −3=0
Theorem : Any line through the point of intersection of ∴ λ =3
the lines a 1 x + b 1 y + c 1 = 0 and a 2 x + b 2 y + c 2 = 0 can be Now, substituting this value of λ in (i), we get
represented by the equation 3 ( x + 2y − 3) + (2x − 3y − 4 ) = 0
(a 1 x + b 1 y + c 1 ) + λ (a 2 x + b 2 y + c 2 ) = 0 i.e. 5x + 3y − 13 = 0
which is the equation of required line.
where λ is a parameter which depends on the other
property of line. y Example 69. The family of lines
Proof : The equations of the lines are x (a + 2b ) + y (a + 3b ) = a + b passes through the point
a 1 x + b 1y + c 1 = 0 ...(i) for all values of a and b. Find the point.
and a 2 x + b 2y + c 2 = 0 ...(ii) Sol. The given equation can be written as
a ( x + y − 1) + b (2x + 3y − 1) = 0
Multiplying µ and ν in Eqs. (i) and (ii) and adding, we get
which is equation of a line passing through the point of
µ (a 1 x + b 1 y + c 1 ) + ν (a 2 x + b 2 y + c 2 ) = 0 intersection of the lines x + y − 1 = 0 and 2x + 3y − 1 = 0.
where µ, ν are any constants not both zero. The point of intersection of these lines is (2, − 1). Hence the
given family of lines passes through the point (2, − 1) for all
Dividing both sides by µ, then values of a and b.
ν
(a 1 x + b 1 y + c 1 ) + (a 2 x + b 2 y + c 2 ) = 0
µ
Chap 02 The Straight Lines 111

y Example 70. If 3a + 2b + 6c = 0 the family of straight Sol. Any line passing through the point of intersection of the
given lines is
lines ax + by + c = 0 passes through a fixed point. Find ( x + 5y + 7 ) + λ (3x + 2y − 5) = 0
the coordinates of fixed point. ⇒ x ( 1 + 3λ ) + y ( 5 + 2λ ) + ( 7 − 5λ ) = 0 ...(i)
Sol. Given, 3a + 2b + 6c = 0 (1 + 3λ)
Its slope = −
a b ( 5 + 2λ )
or + +c =0 ...(i)
2 3 (a) Line Eq. (i) is to be parallel to 7 x + 2y − 5 = 0
and family of straight lines is ( 1 + 3λ ) 7
ax + by + c = 0 ...(ii) then − =−
( 5 + 2λ ) 2
Subtracting Eqs. (i) from (ii), then
⇒ 2 + 6λ = 35 + 14 λ
 1  1
a  x −  + b y −  = 0
 2  3 ⇒ 8λ = − 33
33
which is equation of a line passing through the point of ⇒ λ=−
intersection of the lines 8
1 1 Substituting this value of λ in Eq. (i), we get the required
x − = 0 and y − = 0 equation as 7 x + 2y − 17 = 0
2 3
 1 1 (b) Line, (i) is to be perpendicular to 7 x + 2y − 5 = 0
∴ The coordinates of fixed point are  ,  .
 2 3 ( 1 + 3λ ) 7
∴ − ×−   = −1
( 5 + 2λ )  2
y Example 71. If 4a 2 + 9b 2 − c 2 + 12ab = 0, then the or 7 + 21 λ = − 10 − 4 λ
family of straight lines ax + by + c = 0 is either 17
concurrent at ... or at .... ∴ λ=−
25
Sol. Given, 4a 2 + 9b 2 − c 2 + 12ab = 0 Substituting this value of λ is Eq. (i), we get the required
equation as
or (2a + 3b )2 − c 2 = 0
2x − 7y − 20 = 0.
or c = ± (2a + 3b ) ...(i)
Aliter :
and family of straight lines is
The point of intersection of the given lines
ax + by + c = 0 ...(ii)
x + 5y − 7 = 0 and 3x + 2y − 5 = 0 is (3, − 2).
Substituting the value of c from Eqs. (i) in (ii), then
∴ Equation of line through (3, − 2) is
ax + by ± (2a + 3b ) = 0
y + 2 = m ( x − 3) ...(ii)
⇒ a ( x ± 2) + b ( y ± 3) = 0
(a) Line (ii) is parallel to 7 x + 2y − 5 = 0
Taking ‘+’ sign : a ( x + 2) + b (y + 3) = 0
7
which is equation of a line passing through the point of ∴ m=−
intersection of the lines x + 2 = 0 and y + 3 = 0 2
∴ coordinates of fixed point are ( −2, − 3) . Hence, the equation of the required line is
Taking ‘–’ sign : a ( x − 2) + b (y − 3) = 0 7
y + 2 = − ( x − 3)
which is equation of a line passing through the point of 2
intersection of the lines or 7 x + 2y − 17 = 0
x − 2 = 0 and y − 3 = 0 (b) Line (ii) is perpendicular to 7 x + 2y − 5 = 0
∴ coordinates of fixed point are (2, 3)  7
then m ×  −  = − 1
Hence, the family of straight lines ax + by + c = 0 is either  2
concurrent at ( −2, − 3) or at (2, 3) . 2
or m=
y Example 72. Find the equation of the line passing 7
through the point of intersection of the lines Hence, the equation of the required line is
x + 5y + 7 = 0, 3x + 2y − 5 = 0 2
y + 2 = ( x − 3)
7
and (a) parallel to the line 7 x + 2 y − 5 = 0
or 2x − 7y − 20 = 0
(b) perpendicular to the line 7 x + 2 y − 5 = 0
112 Textbook of Coordinate Geometry

y Example 73. Find the equation of straight line which y Example 75. A variable straight line through the
passes through the intersection of the straight lines x y
point of intersection of the lines + = 1 and
3 x − 4y + 1 = 0 and 5 x + y − 1 = 0 a b
x y
and cuts off equal intercepts from the axes. + = 1 meets the coordinate axes in A and B. Show
Solution : Equation of any line passing through the b a
intersection of the given lines is that the locus of the mid-point of AB is the curve
( 3x − 4y + 1) + λ ( 5x + y − 1) = 0 2xy (a + b ) = ab ( x + y ).
⇒ x ( 3 + 5λ ) + y ( − 4 + λ ) + ( 1 − λ ) = 0 ...(i) Sol. Any line through the point of intersection of given lines is
x y x y  x y 
⇒ + =1  + − 1 + λ  + − 1 = 0
 λ − 1   λ − 1 a b  b a 
   
 3 + 5λ   λ − 4 
1 λ 1 λ
x  +  +y  +  = (1 + λ )
but given x-intercept = y-intercept a b  b a 
 λ − 1   λ − 1  b + aλ   a + bλ 
i.e.   =  ⇒ x  +y   = (1 + λ )
 3 + 5λ   λ − 4   ab   ab 
1 1 x y
⇒ = ⇒ + =1
3 + 5λ λ − 4  ab (1 + λ )  ab (1 + λ )
   
(λ ≠ 1Qif λ = 1 then line (i) pass through origin)  b + aλ   a + bλ 
∴ λ − 4 = 3 + 5λ
Y
or 4λ = − 7
7
∴ λ=− B
4
Substituting the value of λ in Eq. (i), we get required M(x1,y1)
equation is 23x + 23y = 11.

y Example 74. If t 1 and t 2 are roots of the equation X′ X


O A
t 2 + λt + 1 = 0, where λ is an arbitrary constant. Then Y′
prove that the line joining the points (at 12 , 2at 1 ) and This meets the X-axis at
(at 22 , 2at 2 ) always passes through a fixed point. Also  ab (1 + λ ) 
A≡ , 0
find that point.  b + aλ 
Sol. Qt 1 and t 2 are the roots of the equation t 2 + λt + 1 = 0 and meets the Y-axis at
∴ t 1 + t 2 = − λ and t 1t 2 = 1 ...(i)  ab (1 + λ )
B ≡ 0, 
Equation of the line joining the points (at 12 , 2at 1 ) and  a + bλ 
(at 22 , 2at 2 ) is Let the mid-point of AB is M ( x 1, y1 ), then
2at 2 − 2at 1 ab (1 + λ ) ab (1 + λ )
y − 2at 1 = ( x − at 12 ) x1 = and y1 =
at 22 − at 12 2 (b + aλ ) 2 (a + bλ )
1 1 2 (b + aλ ) 2 (a + bλ )
2 ∴ + = +
⇒ y − 2at 1 = ( x − at 12 ) x 1 y1 ab (1 + λ ) ab (1 + λ )
(t 2 + t 1 )
2
⇒ y (t 1 + t 2 ) − 2at 1t 2 − 2at 12 = 2x − 2at 12 = (b + aλ + a + bλ )
ab (1 + λ )
⇒ y (t 1 + t 2 ) − 2at 1t 2 = 2x
2
⇒ y ( −λ ) − 2a = 2x [from Eq. (i)] = (a + b )(1 + λ )
ab (1 + λ )
y 
or ( x + a) + λ   = 0 ( x 1 + y 1 ) 2 (a + b )
 2 ⇒ =
x 1y1 ab
which is equation of a line passing through the point of
y ⇒ 2x 1y1 (a + b ) = ab ( x 1 + y1 )
intersection of the lines x + a = 0 and = 0.
2 Hence, the locus of mid point of AB is
∴ coordinates of fixed point are ( −a , 0) . 2xy (a + b ) = ab ( x + y ) .
Chap 02 The Straight Lines 113

2β − 11
⇒ = −1
How to Find Circumcentre and 7 (2α − 5)
⇒ 2β − 11 = − 14α + 35
Orthocentre by Slopes ⇒ 14α + 2β = 46
(i) Circumcentre ∴ 7α + β = 23 …(i)
The circumcentre of a triangle is the point of intersection and O ′ E ⊥ CA
of the perpendicular bisectors of the sides of a triangle. It ∴ Slope of O ′ E × Slope of CA = − 1
is the centre of the circle which passes through the 5
β−
vertices of the triangle and so its distance from the ⇒ 2 × −1 − 6 = − 1
vertices of the triangle is the same and this distance is α−
11 5−6
known as the circumradius of the triangle. 2
2β − 5 7
A(x1,y1) ⇒ × = −1
2α − 11 1
⇒ 14 β − 35 = − 2α + 11
∴ 2α + 14 β = 46
F E ∴ α + 7β = 23 ... (ii)
O Solving Eqs. (i) and (ii), we get
23
(x2,y2)B C(x3,y3) α =β =
D 8
 23 23
Let O ( x , y ) be the circumcentre. ∴ Circumcentre =  , 
 8 8
If D , E and F are the mid points of BC , CA and AB ∴ Circumradius = O ′ A = O ′ B = O ′ C
respectively and OD ⊥ BC , OE ⊥ CA and OF ⊥ AB
= O ′ C = ( α − 6) 2 + ( β − 6) 2
∴ slope of OD × slope of BC = − 1
2 2
and slope of OE × slope of CA = − 1  23   23 
=  − 6 +  − 6
8  8 
and slope of OF × slope of AB = − 1
2 2
Solving any two, we get ( x , y ).  25  25 25 2
=   +  = units.
 8  8 8
y Example 76. Find the coordinates of the
circumcentre of the triangle whose vertices are (ii) Orthocentre
A ( 5 , − 1), B ( −1, 5) and C (6, 6 ) . Find its radius also. The orthocentre of a triangle is the point of intersection of
Sol. Let circumcentre be O ′ (α, β ) and mid points of sides altitudes.
 5 11  11 5 A
BC , CA and AB are D  ,  , E  ,  and F (2, 2)
2 2   2 2 F
E
respectively. Since O ′ D ⊥ BC .
∴ Slope of O ′ D × slope of BC = − 1 O
11
β−
⇒ 2 × 6−5 = −1
5 6+1
α−
2 B C
D
Y
Here O is the orthocentre since AD ⊥ BC , BE ⊥ CA and
C(6,6)
D CF ⊥ AB, then OA ⊥ BC, OB ⊥ CA, and OC ⊥ AB
(–1,5)B Solving any two we can get coordinates of O.
(α,β) O' E
Remarks
F 1. If any two lines out of three lines i.e. AB, BC and CAare
X′ X perpendicular, then orthocentre is the point of intersection of
O
two perpendicular lines.
A(5,–1)
2. Firstly find the slope of lines BC, CA and AB.
Y′
114 Textbook of Coordinate Geometry

y Example 77. Find the orthocentre of the triangle Y

0
6=
formed by the lines xy = 0 and x + y = 1.

2y+
Sol. Three sides of the triangle are x = 0, y = 0 and x + y =1. The

3x–
10 , 84
coordinates of the vertices are O (0, 0), A (1, 0) and B (0, 1) . (
A 23 23 )
E
The triangle OAB is a right angled triangle having right F O'

4x
angle at O. Therefore O (0, 0) is the orthocentre. Since we

+
5y
know that the point of intersection of two perpendicular

=
20
lines is the orthocentre of the triangle OAB . X′ X
O
(α,β) B
y Example 78. Find the orthocentre of the triangle ABC D
whose angular points are A (1, 2), B (2, 3) and C (4, 3)..
3−3 Y′ C
Sol. Now, Slope of BC = =0
4 −2
∴ Slope of O ′ A × Slope of BC = − 1
O' 84
−1
⇒ 23 ×m = −1
Y D 10
−1
B(2,3) 23
F C(4,3) 61
⇒ m = −1
E − 13
A(1,2) 13
X′ X ∴ m=
O 61
Y′ Let the vertex B is (α, β ) .
2−3 1 (α, β ) lies on 3x − 2y + 6 = 0
Slope of CA = =
1− 4 3 ∴ 3α − 2β + 6 = 0 …(i)
3−2 and O ′ B ⊥ AC
and Slope of AB = =1
2−1 ∴ Slope of O ′ B × slope of AC = − 1
Let orthocentre be O ′(α, β ) then β − 1  4
× −  = − 1
Slope of O ′ A × slope of BC = − 1 α − 1  5
2−β ⇒ 4β − 4 = 5α − 5
×0= −1
1−α ⇒ 5α − 4β − 1 = 0 ...(ii)
0
⇒ = −1 Solving Eqs. (i) and (ii), we get
1−α 33
α = − 13 and β = −
⇒ 1−α =0 2
∴ α =1  33 13
Since, third side passes through  − 13, −  with slope ,
and Slope of OB × slope of CA = − 1  2 61
3−β 1 therefore its equation is
⇒ × = −1
2−α 3 33 13
y+ = ( x + 13)
⇒ 3 − β = 3α − 6 2 61
⇒ 3α + β = 9 ⇒ 122y + 33 × 61 = 26x + 2 × 169
∴ β =6 (Qα = 1) ⇒ 26x − 122y − 1675 = 0
Hence, orthocentre of the given triangle is (1, 6) . Aliter : The equation of line through A. i.e. point of
intersection of AB and AC is
y Example 79. The equations of two sides of a triangle (3x − 2y + 6) + λ ( 4 x + 5y − 20) = 0 ...(i)
are 3x − 2y + 6 = 0 and 4 x + 5y = 20 and the it passes through (1, 1), then
orthocentre is (1, 1). Find the equation of the third side. (3 − 2 + 6) + λ ( 4 + 5 − 20) = 0
Sol. Let 3x − 2y + 6 = 0 and 4 x + 5y = 20 are the equations of the ⇒ 7 − 11λ = 0
sides AB and AC. The point of intersection of AB and AC is 7
∴ λ=
 10 84  11
 ,  . Let slope of BC is m. Since O ′ A ⊥ BC
 23 23 
Chap 02 The Straight Lines 115

7 Since, AD ⊥ BC
From Eq. (i), (3x − 2y + 6) + ( 4 x + 5y − 20) = 0
11 ( 2 − a )  1
∴ − × −  = − 1
⇒ 61x + 13y − 74 = 0 ( 3 − b )  2
61 ⇒ 2 − a = − 6 + 2b
∴ Slope of AD = −
13 ⇒ a + 2b = 8 ...(ii)
13 Similarly, BE ⊥ AC , we get
⇒ Slope of BC =
61 a+b=0 ...(iii)
If coordinates of B (α, β ), B lies on AB Solving Eqs. (ii) and (iii), we get
∴ 3α − 2β + 6 = 0 ...(ii) b = 8 and a = − 8
and O ′ B ⊥ CA
∴ (a, b ) is ( − 8, 8).
β − 1  4
then × −  = − 1
α − 1  5 y Example 81. If the equations of the sides of a
⇒ 5α − 4β − 1 = 0 ...(iii) triangle are ar x + b r y = 1 ; r = 1 , 2 , 3 and the orthocentre
Solving Eqs. (ii) and (iii), we get is the origin, then prove that
α = − 13 and β = −
33 a1a 2 + b1b 2 = a 2a 3 + b 2b 3 = a 3a1 + b 3b1 .
2
Sol. The equation of the line through A, i.e. the point of
∴ Equation of third side i.e. BC is intersection of AB and AC is
33 13 (a 2 x + b 2y − 1) + λ (a 3 x + b 3y − 1) = 0
y+ = ( x + 13) ...(i)
2 61 It passes through (0, 0), then
∴ 26x − 122y − 1675 = 0 A

y Example 80. If the orthocentre of the triangle


formed by the lines 2x + 3y − 1 = 0, x + 2y − 1 = 0, F E

1
3 y=

a2
ax + by − 1 = 0 is at origin, then find (a, b ).

x+
O(0,0)

b
3 x+

2 b
y=
Sol. The equation of a line through A i.e. the point of intersec- a

1
tion of AB and AC, is
(2x + 3y − 1) + λ (ax + by − 1) = 0 ...(i) B
D C
It passes through O (0, 0) , then a1x+b1y=1
−1− λ =0
−1− λ =0
∴ λ = −1
∴ λ = −1
Y
From Eq. (i), a 2 x + b 2y − 1 − a 3 x − b 3y + 1 = 0
B
∴ (a 2 − a 3 ) x + (b 2 − b 3 ) y = 0
Since, AD ⊥ BC
∴ Slope of AD × slope of BC = − 1
x+
2 y–1 (a 2 − a 3 )  a1 
=0 − × −  = − 1
(b 2 − b 3 )  b1 
X′ X
O
⇒ a1a 2 − a 3a1 = − b1b 2 + b1b 3
2x

y–1 =0
+

ax+b
3y

C ⇒ a1a 2 + b1b 2 = a 3a1 + b 3b1


–1

…(ii)
=
0

A Similarly, BE ⊥ CA , then we get


Y′ a1a 2 + b1b 2 = a 2a 3 + b 2b 3 …(iii)
From Eq. (i), From Eqs. (ii) and (iii), we get
2x + 3y − 1 − ax − by + 1 = 0 a1a 2 + b1b 2 = a 2a 3 + b 2b 3 = a 3a1 + b 3b1
⇒ (2 − a ) x + (3 − b ) y = 0
116 Textbook of Coordinate Geometry

Exercise for Session 3


1. The locus of the point of intersection of lines x cos α + y sin α = a and x sin α − y cos α = b (α is a parameter) is
(a) 2 (x 2 + y 2 ) = a 2 + b 2 (b) x 2 − y 2 = a 2 − b 2
(c) x 2 + y 2 = a 2 + b 2 (d) x 2 − y 2 = a 2 + b 2

2. If a, b , c are in AP then ax + by + c = 0 represents


(a) a straight line (b) a family of concurrent lines
(c) a family of parallel lines (d) None of these

3. If the lines x + 2ay + a = 0, x + 3by + b = 0 and x + 4cy + c = 0 are concurrent, then a, b , c are in
(a) AP (b) GP
(c) HP (d) AGP

4. The set of lines ax + by + c = 0, where 3a + 2b + 4c = 0 is concurrent at the point

(a)  ,
1
(b)  ,
3 1 3
 
4 2 2 4
(c)  − , −  (d)  − , − 
3 1 1 3
 4 2  2 4

5. If the lines ax + y + 1 = 0, x + by + 1 = 0 and x + y + c = 0, (a, b and c being distinct and different from 1) are
a b c
concurrent, then the value of + + is
a −1 b −1 c −1
(a) −2 (b) −1
(c) 1 (d) 2
a1 b1 c1
6. If u ≡ a1x + b1y + c1 = 0 and v ≡ a 2x + b 2y + c2 = 0 and = = , then u + kv = 0 represents
a 2 b 2 c2
(a) u = 0 (b) a family of concurrent lines
(c) a family of parallel lines (d) None of these

7. The straight lines x + 2y − 9 = 0, 3x + 5y − 5 = 0 and ax + by − 1 = 0 are concurrent, if the straight line


35x − 22y + 1 = 0 passes through the point
(a) (a, b ) (b) (b , a )
(c) (a, − b ) (d) (−a, b )

8. If the straight lines x + y − 2 = 0, 2x − y + 1 = 0 and ax + by − c = 0 are concurrent, then the family of lines
2ax + 3by + c = 0 (a, b , c are non-zero) is concurrent at
(b)  , 
1 1
(a) (2, 3)
 2 3

(c)  − ,−  (d)  , − 
1 5 2 7
 6 9  3 5

9. The straight line through the point of intersection of ax + by + c = 0 and a′ x + b ′ y + c′ = 0 are parallel toY-axis
has the equation
(a) x (ab ′ − a ′b ) + (cb ′ − c ′b ) = 0 (b) x (ab ′ + a ′b ) + (cb ′ + c ′b ) = 0
(c) y (ab ′ − a ′b ) + (c ′a − ca ′ ) = 0 (d) y (ab ′ + a ′b ) + (c ′a + ca ′ ) = 0

10. If the equations of three sides of a triangle are x + y = 1, 3x + 5y = 2 and x − y = 0, then the orthocentre of the
triangle lies on the line/lines
(a) 5x − 3y = 1 (b) 5y − 3x = 1
(c) 2x − 3y = 1 (d) 5x − 3y = 2
Chap 02 The Straight Lines 117

11. Find the equation of the line through the intersection of 2x − 3y + 4 = 0 and 3x + 4y − 5 = 0 and perpendicular to
6x − 7y + c = 0
(a) 199y + 120x = 125 (b) 199y − 120x = 125
(c) 119x + 102y = 125 (d) 119x − 102y = 125
x y x y 1
12. The locus of the point of intersection of the lines − = m, + = is
a b a b m
(a) a circle (b) an ellipse
(c) a hyperbola (d) a parabola

13. Find the condition on a and b, such that the portion of the line ax + by − 1 = 0, intercepted between the lines
ax + y + 1 = 0 and x + by = 0 subtends a right angled at the origin.
14. If the lines (a − b − c )x + 2ay + 2a = 0, 2bx + (b − c − a )y + 2b = 0 and (2c + 1)x + 2cy + c − a − b = 0 are
concurrent, then prove that either a + b + c = 0 or (a + b + c )2 + 2a = 0.

15. Prove that the lines ax + by + c = 0,bx + cy + a = 0 and cx + ay + b = 0 are concurrent if a + b + c = 0 or


a + bω + c ω 2 = 0 or a + bω 2 + c ω = 0, where ω is a complex cube root of unity.

16. Find the equation of the straight line which passes through the intersection of the lines x − y − 1 = 0 and
2x − 3y + 1 = 0 and is parallel to (i) X-axis (ii) Y-axis ( iii ) 3x + 4y = 14.
17. Let a, b , c be parameters. Then, the equation ax + by + c = 0 will represent a family of straight lines passing
through a fixed-point, if there exists a linear relation between a, b and c.

18. Prove that the family of lines represented by x (1 + λ ) + y (2 − λ ) + 5 = 0, λ being arbitrary, pass through a fixed
point. Also find the fixed point.
 a a
19. Prove that  −a , −  is the orthocentre of the triangle formed by the lines y = m i x + , i = 1, 2, 3; m1, m 2, m 3
 2 mi
being the roots of the equation x 3 − 3x 2 + 2 = 0.
Session 4
Equations of Straight Lines Passing Through a Given Point
and Making a Given Angle with a Given Line, A Line Equally
Inclined With Two Lines, Equation of the Bisectors, Bisector
of the Angle Containing the Origin, Equation of that Bisector
of the Angle Between Two Lines which Contains a Given Point,
How to Distinguish the Acute (Internal) and Obtuse (External)
Angle Bisectors

Equations of Straight Lines ∴ ∠ DQS = ∠ PQR + ∠ RQS = (α + θ )


∴ Equation of line CD is
Passing Through a Given Point y − y 1 = tan (θ + α ) ( x − x 1 ) ...(i)
and Making a Given Angle with and ∠ FRT = ∠ FRB + ∠ BRT
= 180 ° − α + θ
a Given Line = 180 ° + (θ − α )
Theorem : Prove that the equations of the straight lines ∴ Equation of line EF is
which pass through a given point ( x 1 , y 1 ) and make a
y − y 1 = tan (180 ° + θ − α ) ( x − x 1 )
given angle α with the given straight line y = mx + c are
or y − y 1 = tan (θ − α ) ( x − x 1 ) ...(ii)
y − y 1 = tan (θ ± α ) ( x − x 1 )
From Eqs. (i) and (ii), we get
where, m = tan θ.
y − y 1 = tan (θ ± α ) ( x − x 1 )
Proof : Let AB be the given line which makes an angle θ
with X-axis. These are the equations of the two required lines.
∴ m = tan θ y Example 82. Find the equations of the straight lines
Let CD and EF are two required lines which make angle α passing through the point (2 , 3) and inclined at π / 4
with the given line. Let these lines meet the given line AB radians to the line 2x + 3y = 5 .
at Q and R respectively Sol. Let the line 2x + 3y = 5 make an angle θ with positive
Y X -axis.
F D 2
Then tanθ = −
P(x1,y1) 3
π 2
(180°– α)
Now tan θ ⋅ tan = − × 1
B 4 3
α θ
T
α
2
θ
R = − ≠ ±1
Q
S E 3
X′ θ X
O Slopes of required lines are
A
C  π  π
tan θ +  and tan θ − 
Y′  4  4
Chap 02 The Straight Lines 119

 π  2 ⇒ (2 + 3 ) x − y = 1 + 2 3
tan θ + tan   −  + 1
 π  4  3 1 Hence, equations of other sides are
∴ tan θ +  = = =
 4  π
   2 5 (2 − 3 ) x − y = 1 − 2 3
1 − tan θ tan   1 −  −  (1)
 4  3
and (2 + 3 ) x − y = 1 + 2 3
 π
tan θ − tan  
 π  4 y Example 84. The straight lines 3x + 4 y = 5 and
and tan θ −  =
 4  π
1 + tan θ tan   4 x − 3y = 15 intersect at a point A. On these lines, the
 4
points B and C are chosen so that AB = AC . Find the
 2 possible equations of the line BC passing through the
−  −1
 3 point (1, 2) .
= = −5
 2 Sol. Clearly ∠ BAC = 90°
1 +  −  ( 1)
 3
Q AB = AC
∴ Equations of required lines are
∴ ∠ ABC = ∠ BCA = 45°
1
y − 3 = ( x − 2) and y − 3 = − 5 ( x − 2) α = 45°
5
3
i.e. x − 5y + 13 = 0 and 5x + y − 13 = 0 Q Slope of 3x + 4y = 5 is −
4
y Example 83. A vertex of an equilateral triangle is 3
Let tanθ = −
(2 , 3) and the opposite side is x + y = 2 . Find the 4
equations of the other sides. So, possible equations of BC are given by
y − 2 = tan (θ ± α ) ( x − 1)
Sol. Let A (2, 3) be one vertex and x + y = 2 be the opposite side
of an equilateral triangle. Clearly remaining two sides pass Y
3x (1,2)
through the point A (2, 3) and make an angle 60° with +
x +y =2 4y
=
5
Q Slope of x + y = 2 is −1
B
Y
Q A(2,3) X′ X
O 45°
B 60°
A(3,–1)
60°
45°
C C
x+
y=

15
2

X′ X
O =
Y′ P – 3y
4x
Let tanθ = − 1 Y′
∴ θ = 135°
∴ Equations of the other two sides are  tan θ ± tan α 
⇒ y −2=   ( x − 1)
y − 3 = tan (135° ± 60° ) ( x − 2)  1 m tan θ tan α 
i.e. sides are  
3
y − 3 = tan (195° ) ( x − 2) (taking ‘+’ sign)  − ±1 
⇒ y − 3 = tan (180° + 15° ) ( x − 2) ⇒ y −2=  4  ( x − 1)
  3 
⇒ y − 3 = tan 15° ( x − 2)  1 m  −  ( 1) 
 4 
⇒ y − 3 = ( 2 − 3 ) ( x − 2)
 −3 ± 4 
⇒ (2 − 3 ) x − y = 1 − 2 3 ⇒ y −2=   ( x − 1)
 4 m ( − 3) 
and y − 3 = tan (75° ) ( x − 2)
1
(taking ‘−’ sign) ⇒ y −2= ( x − 1) (taking upper sign)
⇒ y − 3 = cot 15° ( x − 2) 4 − ( − 3)

⇒ y − 3 = ( 2 + 3 ) ( x − 2) or x − 7y + 13 = 0
120 Textbook of Coordinate Geometry

( −3 − 4 ) y Example 85. Find the equations to the straight lines


and y −2= ( x − 1) (taking below sign)
( 4 + ( −3)) passing through the point (2 , 3) and equally inclined to
or 7x + y − 9 = 0 the lines 3x − 4 y − 7 = 0 and 12x − 5y + 6 = 0.
Hence, possible equation of the line BC are x − 7y + 13 = 0 Sol. Let m be the slope of the required line. Then its equation is
and 7 x + y − 9 = 0 y − 3 = m ( x − 2) ...(i)
It is given that line (i) is equally inclined to the lines
3x − 4y − 7 = 0 and 12x − 5y + 6 = 0 then
A Line Equally Inclined with  3   12 
 −m   −m 
Two Lines ⇒  4
1 + 3 m
 = − 5 
 1 + 12 m 
 4   5 
Theorem : If two lines with slopes m 1 and m 2 be equally
 3 
inclined to a line with slope m, then  slope of 3x − 4y − 7 = 0 is 
 4 
 m1 − m   m2 − m  12
 and slope of 12x − 5y + 6 = 0 is 
  =−   5
 1 + mm 1   1 + mm 2 
 3 − 4m   12 − 5m 
Proof : Let be two lines of slopes m 1 and m 2 intersecting ⇒   =− 
 4 + 3m   5 + 12m 
at a point P.
Let ∠ CPA = ∠ BPC = θ ⇒ (3 − 4m ) (5 + 12m ) + ( 4 + 3m ) (12 − 5m ) = 0
⇒ 63m 2 − 32m − 63 = 0
B
⇒ ( 7m − 9 ) ( 9m + 7 ) = 0
9 7
∴ m= ,−
C 7 9
m2 Putting these values of m in Eq. (i) we obtain the equations
m of required lines as 9 x − 7y + 3 = 0 and x + 9y − 41 = 0.
A
θ y Example 86. Two equal sides of an isosceles triangle
θ m1
are given by the equations 7 x − y + 3 = 0 and
P x + y − 3 = 0 and its third side passes through the
point (1, − 10) . Determine the equation of the third
 m − m1  side.
∴ tan ( ∠ CPA ) =  
 1 + mm 1  Sol. Let m be the slope of BC. Since AB = AC .
 m − m1  Therefore BC makes equal angles with AB and AC.
or tan θ =   (Qm > m 1 ) ...(i)
 1 + mm 1  Y
A
 m −m 
and tan ( ∠ BPC ) =  2 
 1 +m 2m  X′ X
O
x+

 m −m 
y–

tan θ =  2 (Qm 2 > m ) … (ii)


0

or 
3=
+3=

 1 + m 2m 
0
7x–y

θ
From Eqs. (i) and (ii), we get C

 m − m1   m2 − m   m1 − m   m2 − m  θ D(1,–10)
  =  or   =−  B
 1 + mm 1   1 + m 2m   1 + mm 1   1 + mm 2  Y′

7 −m  −1−m 
Remarks Then   =− 
1. The above equation gives two values of m which are the slopes  1 + 7m   1 + ( − 1) m 
of the lines parallel to the bisectors of the angles between the ⇒ ( 7 − m ) ( 1 − m ) − ( 1 + 7m ) ( 1 + m ) = 0
two given lines.
⇒ 6m 2 + 16m − 6 = 0
2. Sign of m in both brackets is same.
⇒ 3m 2 + 8m − 3 = 0
Chap 02 The Straight Lines 121

⇒ (3m − 1) (m + 3) = 0 y Example 87. Find the equations of the bisectors of



1
m = , −3 the angles between the straight lines 3x − 4 y + 7 = 0
3 and 12x + 5y − 2 = 0.
Equation of third side BC is y + 10 = m ( x − 1) Sol. The equations of the bisectors of the angles between
1 3x − 4y + 7 = 0 and 12x + 5y − 2 = 0 are
i.e. y + 10 = ( x − 1) and y + 10 = − 3 ( x − 1)
3 ( 3x − 4y + 7 ) (12x + 5y − 2)

or x − 3y − 31 = 0 and 3x + y + 7 = 0
( 3) + ( − 4 )
2 2
(12)2 + (5)2
( 3x − 4y + 7 ) (12x + 5y − 2)
or =±
Equation of the Bisectors or
5 13
(39 x − 52y + 91) = ± (60x + 25y − 10)
Theorem : Prove that the equation of the bisectors of the Taking the positive sign, we get
angles between the lines 21x + 77y − 101 = 0
a 1 x + b 1 y + c 1 = 0 and a 2 x + b 2 y + c 2 = 0 as one bisector.
Taking the negative sign, we get 99 x − 77y + 81 = 0
(a 1 x + b 1 y + c 1 ) (a x + b 2 y + c 2 )
are given by =± 2 as the second bisector.
(a 12 + b 12 ) (a 22 + b 22 )
Proof : Let the given lines be AA ′ and BB ′ whose
equations are
Bisector of the Angle Containing
a 1 x + b 1y + c 1 = 0 ...(i) the Origin
and a 2 x + b 2y + c 2 = 0 ...(ii)
Let equations of lines be
Since bisectors of the angles between the two lines are the
a 1 x + b 1y + c 1 = 0 ...(i)
locus of a point which moves in a plane such that whose
distance from two lines are equal. a 2 x + b 2y + c 2 = 0 ...(ii)
where c 1 and c 2 are positive.
Y
C
B A Let P ( x , y ) be taken on the bisector of the angle which
P(x,y) contains the origin.
L M Y B C A

0
a2x+b2y+c2=0 P P a1x+b1y+c1=0
a 2x

c1 =
X X
+b

D′
1 y+

D
2

X X
y+

+b

PP
c2

X′ X
a1 x
=0

O PP
D¢ X X
P(x,y) D
X X
B′ PP
A′ C′ X¢ X
Y′ O

Let CC ′ and DD ′ be the two bisectors of the angle between


the lines AA′ and BB′. Let P ( x , y ) be any point on CC ′, then B¢
Y¢ A¢ C¢'
Length of the perpendicular from P on AA′
= length of the perpendicular from P on BB ′ (i) Let P ( x , y ) lies on DD ′ ,then either O (0, 0 ) and P ( x , y )
| a 1 x + b 1y + c 1 | | a 2 x + b 2y + c 2 | will lie on the same side of the two lines Eqs. (i) and (ii),
∴ =
(a 12 + b 12 ) (a 22 + b 22 ) then
a 1 x + b 1y + c 1
(a 1 x + b 1 y + c 1 ) (a 2 x + b 2 y + c 2 ) >0
or =± 0 + 0 + c1
(a 12 + b 12 ) (a 22 + b 22 ) a 2 x + b 2y + c 2
and >0
These are the required equations of the bisectors. 0 + 0 + c2
Note or a 1 x + b 1y + c 1 > 0
The two bisectors are perpendicular to each other. and a 2 x + b 2y + c 2 > 0 (Qc 1 , c 2 > 0 )
122 Textbook of Coordinate Geometry

If the origin O (0, 0 ) and P ( x , y ) lie on the opposite side of ∴ The equation of the bisector bisecting the angle
the two lines Eqs. (i) and (ii), then containing origin is
( − 4 x − 3y + 6) (5x + 12y + 9 )
a 1 x + b 1y + c 1 a 2 x + b 2y + c 2 =
< 0 and <0 ( − 4 2 ) + ( − 3) 2 (5)2 + (12)2
0 + 0 + c1 0 + 0 + c2
a 1 x + b 1 y + c 1 < 0 and a 2 x + b 2 y + c 2 < 0  − 4 x − 3y + 6  5x + 12y + 9 
or ⇒   = 
 5   13 
(Qc 1 , c 2 > 0 )
Then equation of bisectors will be ⇒ − 52x − 39y + 78 = 25x + 60y + 45
| a 1 x + b 1y + c 1 | | a 2 x + b 2y + c 2 | ⇒ 77 x + 99y − 33 = 0 or 7 x + 9y − 3 = 0
= and the equation of the bisector bisecting the angle not
(a 12 + b 12 ) (a 22 + b 22 ) containing origin is
( − 4 x − 3y + 6) (5x + 12y + 9 )
Case I : If a 1 x + b 1 y + c 1 > 0 and a 2 x + b 2 y + c 2 > 0 =−
(a 1 x + b 1 y + c 1 ) (a 2 x + b 2 y + c 2 ) ( − 4 ) + ( − 3) )
2 2
(52 ) + (12)2
then =
 − 4 x − 3y + 6  5x + 12y + 9 
(a 12 + b 12 ) (a 22 + b 22 ) ⇒   =− 
 5   13 
Case II : If a 1 x + b 1 y + c 1 < 0 and a 2 x + b 2 y + c 2 < 0 ⇒ − 52x − 39y + 78 = − 25x − 60y − 45
(a x + b 1 y + c 1 ) (a x + b 2 y + c 2 ) ⇒ 27 x − 21y − 123 = 0 or 9 x − 7y − 41 = 0
then − 1 =− 2
(a 1 + b 1 )
2 2
(a 22 + b 22 )

i.e.
(a 1 x + b 1 y + c 1 )
=
(a 2 x + b 2 y + c 2 ) Equation of that Bisector of the
(a 12 + b 12 ) (a 22 + b 22 )
Thus is both cases equation of the bisector containing the
Angle between Two Lines which
origin, when c 1 and c 2 are positive is Contains a Given Point
(a 1 x + b 1 y + c 1 ) (a 2 x + b 2 y + c 2 )
= Let the equations of the two lines be
(a 12 + b 12 ) (a 22 + b 22 )
a 1 x + b 1y + c 1 = 0 ...(i)
and equation of the bisector of the angle between the lines and a 2 x + b 2y + c 2 = 0 ...(ii)
a 1 x + b 1 y + c 1 = 0 and a 2 x + b 2 y + c 2 = 0
The equation of the bisector of the angle between the two
which does not contain the origin when c 1 and c 2 are lines containing the points (h, k ) will be
positive is (a 1 x + b 1 y + c 1 ) (a 2 x + b 2 y + c 2 )
(a 1 x + b 1 y + c 1 ) (a 2 x + b 2 y + c 2 ) =
=− (a 12 + b 12 ) (a 22 + b 22 )
(a 12 + b 12 ) (a 22 + b 22 )
(a 1 x + b 1 y + c 1 ) (a 2 x + b 2 y + c 2 )
or =−
Working Rule :
(a 12 + b 12 ) (a 22 + b 22 )
(i) First re-write the equations of the two lines so that
their constant terms are positive. according as a 1 h + b 1 k + c 1 and a 2 h + b 2 k + c 2 are of the
same sign or opposite sign.
(ii) The bisector of the angle containing the origin and
does not containing the origin, then taking +ve y Example 89. Find the bisector of the angle between
and – ve sign in the lines 2x + y − 6 = 0 and 2x − 4 y + 7 = 0 which
(a 1 x + b 1 y + c 1 ) (a 2 x + b 2 y + c 2 ) contains the point (1, 2).
=± respectively.
(a 12 + b 12 ) (a 22 + b 22 ) Sol. Value of 2x + y − 6 at (1, 2) is – 2 (negative)
and value of 2x − 4y + 7 at (1, 2) is 1 (positive)
y Example 88. Find the equations of angular bisector i.e. opposite sign.
bisecting the angle containing the origin and not ∴ Equation of bisector containing the point (1, 2) is
containing the origin of the lines 4 x + 3y − 6 = 0 and ( 2x + y − 6) ( 2x − 4y + 7 )
=−
5x + 12y + 9 = 0 . (2 + 1 )
2 2
( 2) 2 + ( − 4 ) 2
Sol. Firstly make the constant terms (c 1, c 2 ) positive, then ⇒ 2 ( 2x + y − 6) + ( 2x − 4y + 7 ) = 0
− 4 x − 3y + 6 = 0 and 5x + 12y + 9 = 0 or 6x − 2y − 5 = 0
Chap 02 The Straight Lines 123

(a 1 x + b 1 y + c 1 ) (a 2 x `+ b 2 y + c 2 )
How to Distinguish the Acute (a 12 + b 12 )

(a 22 + b 22 )
(Internal) and Obtuse (External) Conditions Acute angle bisector Obtuse angle bisector

Angle Bisectors? a1a 2 + b1b 2 > 0


a1a 2 + b1b 2 < 0

+
+

Let the equations of the two lines be Remarks
a 1 x + b 1y + c 1 = 0 ...(i) 1. Bisectors are perpendiculars to each other.
and a 2 x + b 2y + c 2 = 0 ...(ii) 2. `+' sign gives the bisector of the angle containing origin.
3. If a1a2 + b1b2 > 0 then the origin lies in obtuse angle and if
where, c 1 > 0, c 2 > 0. a1a2 + b1b2 < 0, then the origin lies in acute angle.
Q Equations of bisectors are Explanation : Equations of given lines in normal form
(a 1 x + b 1 y + c 1 ) (a x + b 2 y + c 2 ) will be respectively
=± 2 ...(iii)
a1 x b 1y c1
(a 12 + b 12 ) (a 22 + b 22 ) − − = ( Qc 1 > 0 )
(a 1 + b 1 )
2 2
(a 1 + b 1 )
2 2 (a 2
1 + b 2
1 )
when Eq. (iii) be simplified, let the bisectors be
p 1 x + q 1 y + r1 = 0 ...(iv) a2 x b 2y c2
and − − = ( Qc 2 > 0 )
and p 2 x + q 2 y + r2 = 0 ...(v) (a 2 + b 2 )
2 2
(a 2 + b 2 )
2 2
(a 2 + b 22 )
2

Since the two bisectors are at right angles, the angle α a1 b1


If cos α = − then sin α = −
between the acute (internal ) bisector and any one of the (a 1 + b 1 )
2 2
(a 1 + b 12 )
2
given lines must lie between 0 and 45° i.e. 0 < α < 45 °.
a2 b2
∴ 0 < tan α < 1 and cos β = − then sin β = −
If m 1 and m 2 are the slopes of Eqs. (i) and (iii) respectively. (a 22 + b 22 ) (a 22 + b 22 )
a p Now, cos (α − β) = cos α cos β + sin α sin β
Then, m 1 = − 1 and m 2 = − 1 (a 1 a 2 + b 1 b 2 )
b1 q1 =
(a 12 + b 12 ) (a 22 + b 22 )
m − m2 
∴ tan α = 1 
cos (α − β) > 0 or < 0
1 + m 1 m 2
according as (α − β) is acute or obtuse.
  a1   p1   i.e. a 1 a 2 + b 1 b 2 > 0 or < 0
  − b  −  − q   a q − b p 
=
1 1 1 1 Hence, bisector of the angle between the lines will be
= 1 1

1 +  − a 1   − p 1  b 1 q 1 + a 1 p 1
the bisector of the acute or obtuse angle according as
origin lies in the acute or obtuse angle according as
  b 1   q 1  a 1 a 2 + b 1 b 2 < 0 or >0.
Hence, if 0 < tan α < 1, p 1 x + q 1 y + r1 = 0 is the acute
(internal) bisector and if tan α > 1, p 2 x + q 2 y + r2 = 0 is the y Example 90. Find the equation of the bisector of
obtuse (external ) bisector. the obtuse angle between the lines 3x − 4 y + 7 = 0 and
12x + 5y − 2 = 0.

Shortcut Method for finding Acute Sol. Firstly make the constant terms (c 1, c 2 ) positive
3x − 4y + 7 = 0 and − 12x − 5y + 2 = 0
(Internal) and Obtuse (External) Angle Q a1a 2 + b1b 2 = (3) ( − 12) + ( − 4 ) ( − 5) = − 36 + 20 = − 16
Bisectors ∴ a1a 2 + b1b 2 < 0
Let the equations of the two lines be Hence “−” sign gives the obtuse bisector.
∴ Obtuse bisector is
a 1 x + b 1y + c 1 = 0 ( 3x − 4y + 7 ) ( − 12x − 5y + 2)
a 2 x + b 2y + c 2 = 0 =−
( 3) + ( − 4 )
2 2
( − 12)2 + ( − 5)2
Taking c 1 > 0, c 2 > 0 and a 1 b 2 ≠ a 2 b 1
⇒ 13 (3x − 4y + 7 ) = − 5 ( − 12x − 5y + 2)
Then equations of the bisectors are ⇒ 21x + 77y − 101 = 0 is the obtuse angle bisector.
124 Textbook of Coordinate Geometry

y Example 91. Find the bisector of acute angle Q a1a 2 + b1b 2 = ( − 1) ( − 7 ) + ( − 1) (1) = 6 > 0
between the lines x + y − 3 = 0 and 7 x − y + 5 = 0. ∴ Acute or internal bisector is
Sol. Firstly, make the constant terms (c 1, c 2 ) positive then ( − x − y + 1) ( − 7 x + y + 15)
=−
− x − y + 3 = 0 and 7 x − y + 5 = 0 ( − 1) + ( − 1)
2 2
( − 7 ) 2 + ( 1) 2
Q a1a 2 + b1b 2 = ( − 1) (7 ) + ( − 1) ( − 1) = − 7 + 1 = − 6 ( − x − y + 1) ( − 7 x + y + 15)
i.e. a1a 2 + b1b 2 < 0 ⇒ =−
2 5 2
Hence “+” sign gives the acute bisector.
⇒ − 5x − 5y + 5 = 7 x − y − 15
− x −y +3 7x − y + 5
∴ Acute bisector is =+ or 12x + 4y − 20 = 0
( − 1) + ( − 1)
2 2
( 7 ) 2 + ( − 1) 2 or 3x + y − 5 = 0 ...(iv)
− x − y + 3 7x − y + 5 Internal bisector of BC and CA :
⇒ =
2 5 2 − 7 x + y + 15 = 0
⇒ − 5x − 5y + 15 = 7 x − y + 5 − x +y +1=0
∴ 12x + 4y − 10 = 0 or 6x + 2y − 5 = 0 Q a1a 2 + b1b 2 = ( −7 )( −1) + (1) (1) = 8 > 0
is the acute angle bisector. Q Acute or internal bisector is
( − 7 x + y + 15) ( − x + y + 1)
y Example 92. Find the coordinates of incentre of the =−
( − 7 ) + ( 1)
2 2
( − 1) 2 + ( 1) 2
triangle. The equation of whose sides are
− 7 x + y + 15 ( x − y − 1)
AB : x + y − 1 = 0,BC : 7 x − y − 15 = 0 ⇒ =
5 2 2
and CA : x − y − 1 = 0.
⇒ − 7 x + y + 15 = 5x − 5y − 5
Sol. Firstly, make the constant terms (c 1, c 2 , and c 3 ) positive
or 12x − 6y − 20 = 0
i.e. AB : − x − y + 1 = 0 ...(i) or 6x − 3y − 10 = 0 ...(v)
BC : − 7 x + y + 15 = 0 ...(ii) Finally, solve Eqs. (iv) and (v), we get
CA : − x + y + 1 = 0 ...(iii) 5
Q The incentre of triangle is the point of intersection of x = and y = 0
3
internal or acute angle bisectors.
5 
Internal bisector of AB and BC : Hence coordinates of incentre are  , 0 .
3 
− x −y +1=0
− 7 x + y + 15 = 0

Exercise for Session 4


1. The straight lines 2x + 11y − 5 = 0, 24x + 7y − 20 = 0 and 4x − 3y − 2 = 0
(a) form a triangle
(b) are only concurrent
(c) are concurrent with one line bisecting the angle between the other two
(d) None of the above

2. The line x + 3y − 2 = 0 bisects the angle between a pair of straight lines of which one has the equation
x − 7y + 5 = 0. The equation of other line is
(a) 3x + 3y − 1 = 0 (b) x − 3y + 2 = 0 (c) 5x + 5y + 3 = 0 (d) 5x + 5y − 3 = 0

3. P is a point on either of the two lines y − 3 | x | = 2 at a distance of 5 units from their point of intersection. The
coordinates of the foot of the perpendicular from P on the bisector of the angle between them are
 4 + 5 3  4 − 5 3
(a)  0,  or  0,  depending on which the point P is taken
 2   2 
 4 + 5 3  4 − 5 3  5 5 3
(b)  0,  (c)  0,  (d)  , 
 2   2  2 2 
Chap 02 The Straight Lines 125

4. In a triangle ABC, the bisectors of angles B and C lie along the lines x = y andy = 0 . If A is (1, 2), then the
equation of line BC is
(a) 2x + y = 1 (b) 3x − y = 5 (c) x − 2y = 3 (d) x + 3y = 1

5. In ∆ABC, the coordinates of the vertex A are (4, −1) and lines x − y − 1 = 0 and 2x − y = 3 are the internal
bisecters of angles B and C. Then, the radius of the incircle of triangle ABC is
5 3 6 7
(a) (b) (c) (d)
5 5 5 5

6. The equation of the straight line which bisects the intercepts made by the axes on the lines x + y = 2 and
2x + 3y = 6 is
(a) 2x = 3 (b) y = 1 (c) 2y = 3 (d) x = 1

7. The equation of the bisector of the acute angle between the lines 2x − y + 4 = 0 and x − 2y = 1 is
(a) x + y + 5 = 0 (b) x − y + 1 = 0 (c) x − y = 5 (d) x − y + 5 = 0

8. The equation of the bisector of that angle between the lines x + y = 3 and 2x − y = 2 which contains the point
(1, 1) is
(a) ( 5 − 2 2 )x + ( 5 + 2 )y = 3 5 − 2 2 (b) ( 5 + 2 2 ) x + ( 5 − 2 )y = 3 5 + 2 2
(c) 3x = 10 (d) 3x − 5y + 2 = 0

9. Find the equations of the two straight lines through (7, 9) and making an angle of 60° with the line
x − 3y − 2 3 = 0.

10. Equation of the base of an equilateral triangle is 3x + 4y = 9 and its vertex is at the point (1, 2). Find the
equations of the other sides and the length of each side of the triangle.
11. Find the coordinates of those points on the line 3x + 2y = 5 which are equidistant from the lines 4x + 3y − 7 = 0
and 2y − 5 = 0.

12. Two sides of rhombus ABCD are parallel to the lines y = x + 2 and y = 7x + 3. If the diagonal of the rhombus
intersect at the point (1, 2) and the vertex A lies on Y-axis, find the possible coordinates of A.

13. The bisector of two lines L1 and L2 are given by 3x 2 − 8xy − 3y 2 + 10x + 20y − 25 = 0. If the line L1 passes
through origin, find the equation of line L2.
14. Find the equation of the bisector of the angle between the lines x + 2y − 11 = 0 and 3x − 6y − 5 = 0 which
contains the point (1, –3).

15. Find the equation of the bisector of the angle between the lines 2x − 3y − 5 = 0 and 6x − 4y + 7 = 0 which is the
supplement of the angle containing the point (2, –1).
Session 5
The Foot of Perpendicular Drawn from the
Point (x1, y1) to the Line ax + by + c = 0, Image or
Reflection of a Point (x1, y1) about a Line Mirror,
Image or Reflection of a Point In Different Cases,
Use of Image or Reflection

The Foot of Perpendicular Aliter II : Let the coordinates of M are ( x 2 , y 2 )


PM ⊥ RS
Drawn from the Point ( x 1 , y 1 ) Q
ax + by + c = 0
to the Line ax + by + c = 0
and M lies on
i.e. ax 2 + by 2 + c = 0 ...(iii)
Let P ≡ ( x 1 , y 1 ) and let M be the foot of perpendicular  y2 − y1   a 
and   × −  = − 1 ( Q PM ⊥ RS )
drawn from P on ax + by + c = 0.  x2 − x1   b
In order to find the coordinates of M, find the equation of x2 − x1 y2 − y1
the line PM which is perpendicular to RS and passes or =
a b
through P ( x 1 , y 1 ), i.e. bx − ay = bx 1 − ay 1
x 2 − x 1 y 2 − y 1 a( x 2 − x 1 ) + b(y 2 − y 1 )
or b( x − x 1 ) − a(y − y 1 ) = 0 and solving it with or = =
a b (a 2 + b 2 )
ax + by + c = 0, then we get coordinates of M.
(By ratio proportion method)
, y 1)
P(x 1 (ax 2 + by 2 ) − (ax 1 + by 1 )
=
a2 + b2
− c − (ax 1 + by 1 )
= [from Eq. (iii)]
a2 + b2
0S
c=
by
+ x2 − x1 y2 − y1 (ax 1 + by 1 + c )
ax+ or = =−
M a b (a 2 + b 2 )

R y Example 93. Find the coordinates of the foot of the


perpendicular drawn from the point (2, 3) to the line
Aliter I : Let the coordinates of M are ( x 2 , y 2 ) then
y = 3x + 4.
M ( x 2 , y 2 ) lies on ax + by + c = 0
Sol. Given line is
⇒ ax 2 + by 2 + c = 0 ...(i)
3x − y + 4 = 0 ...(i)
and Q PM ⊥ RS Let Eq. P ≡ (2, 3).
then (Slope of PM ) ( Slope of RS ) = − 1 Let, M be the foot of perpendicular drawn from P on RS.
 y2 − y1   a  Then equation of PM passes through P (2, 3) and
⇒   × −  = − 1 perpendicular to RS is
 x2 − x1   b
x + 3y − (2 + 3 × 3) = 0
or bx 2 − ay 2 = bx 1 − ay 1 ...(ii) i.e. x + 3y − 11 = 0 ...(ii)
Solving Eqs. (i) and (ii), we get ( x 2 , y 2 ).
Chap 02 The Straight Lines 127

Y )
,y 1
S P(x 1
S

(x2,y2)M =0
+c
+ by
P(2,3) M a x
+4
y=3x

R y 2)
x 2,
Q(
X′ X
O
 x + x2 y1 + y2 
i.e. M ≡ 1 , 
R  2 2 
Y′ Aliter I : Ratio Proportion Method :
Solving Eqs. (i) and (ii), we get Q PQ ⊥ RS
1 37 ∴ ( Slope of PQ ) × ( Slope of RS) = − 1
x = − ,y =
10 10  y2 − y1   a 
 1 37 
or   × −  = − 1
∴ M ≡ − ,   x2 − x1   b
 10 10 
( x 2 − x 1 ) (y 2 − y 1 )
Aliter I : Let the coordinates of M be ( x 2 , y 2 ) then or =
M ( x 2 , y 2 ) lies on 3x − y + 4 = 0 a b
⇒ 3x 2 − y 2 + 4 = 0 …(iii) x 2 − x 1 y 2 − y 1 a( x 2 − x 1 ) + b(y 2 − y 1 )
or = =
and Q PM ⊥ RS a b a2 + b2
∴ ( Slope of PM ) × ( Slope of RS ) = − 1 a(2 x − x 1 − x 1 ) + b(2y − y 1 − y 1 )
 y 2 − 3 =
⇒   × ( 3) = − 1 a2 + b2
 x 2 − 2
Q M ( x , y ) is mid-point of P and Q 
or x 2 + 3y 2 − 11 = 0 ...(iv)  
Solving Eqs. (iii) and (iv), we get ∴ x 2 = 2 x − x 1 and y 2 = 2y − y 1 
1
x2 = − , y2 =
37 − 2ax 1 − 2by 1 + 2 (ax + by )
10 10 =
a2 + b2
 1 37 
∴ M ≡ − ,  − 2ax 1 − 2by 1 + 2( − c )
 10 10  = (Qax + by = − c )
Aliter II : By Ratio Proportion Method : a2 + b2
x 2 − 2 y 2 − 3 − (3 × 2 − 3 + 4 ) − 2(ax 1 + by 1 + c )
= = =
3 −1 32 + ( − 1) 2 (a 2 + b 2 )
x2 − 2 y2 − 3 7 x2 − x1 y2 − y1 2 (ax 1 + by 1 + c )
⇒ = =− i.e. = =−
3 −1 10 a b (a 2 + b 2 )
1 37
x2 = − and y 2 = Aliter II : By Distance form or Symmetric form or
10 10
parametric form :
 1 37 
∴ M ≡ − ,  a
 10 10  Q Slope of RS = −
b
b
∴ Slope of PQ =
Image or Reflection of a Point a

( x1 , y1 ) About a Line Mirror (a2+b 2)


b
Let Q ≡ ( x 2 , y 2 ) be the image of P ≡ ( x 1 , y 1 ) then find
coordinates of the foot of perpendicular M drawn from the θ
point P ( x 1 , y 1 ) on RS and use fact that M is the mid- point a
P and Q .
128 Textbook of Coordinate Geometry

b Aliter II :
Let tan θ =
a By distance form or Symmetric form or Parametric
b form : Let, P ≡ ( 4, − 13) and Q be the image of P with
∴ sin θ = respect to line mirror (RS) 5x + y + 6 = 0
(a + b 2 )
2
Q Slope of RS = − 5
a
and cos θ =
(a 2 + b 2 ) 26
1
Put the equation of the mirror line such that the
coefficient of y becomes negative. θ
5
Suppose if b >0
1
then ax + by + c = 0 ∴ Slope of PQ = = tanθ
5
becomes − ax − by − c = 0 1 5
∴ sinθ = and cosθ =
and p = PM = Directed distance from P ( x 1 , y 1 ) on 26 26
− ax − by − c = 0 ( i.e. p +ve or –ve) Now, put the equation of the mirror line such that the
 − ax − by − c  coefficient of y becomes negative.
= 1 1  Then, 5x + y + 6 = 0 becomes − 5x − y − 6 = 0 and
 (a + b ) 
2 2
 p = ⊥ Directed distance from P ( 4, − 13) on
( − 5x − y − 6 = 0)
 − ax − by − c  − 5 × 4 + 13 − 6
PQ = 2 PM = 2 p = 2   =r 13
∴ 1 1
= =−
 (a + b 2 ) 
2 ( − 5) + ( − 1)
2 2 26

26
⇒ Required image has the coordinates ∴ PQ = r = 2p = − = − 26
26
( x 1 + r cos θ, y 1 + r sin θ ) .
Hence, required image has the coordinates
y Example 94. Find the image of the point (4, − 13) Q ≡ ( 4 − 26 cos θ, − 13 − 26 sin θ )
with respect to the line mirror 5x + y + 6 = 0.  5 1 
i.e.  4 − 26 × , − 13 − 26 × 
Sol. Let, P ≡ ( 4, − 13) and Let, Q ≡ ( x 2 , y 2 ) be mirror image P  26 26 
with respect to line mirror 5x + y + 6 = 0. i.e. ( 4 − 5, − 13 − 1)
Let, M (α, β ) be the foot of perpendicular from P ( 4, − 13) on Hence, Q ≡ ( − 1, − 14 )
the line mirror 5x + y + 6 = 0, then
α − 4 β + 13 − (5 × 4 − 13 + 6)
= =
5 1 52 + 12 Image or Reflection of a Point
α − 4 β + 13 1
or
5
=
1
=−
2 in Different Cases
 3 27 
∴ M ≡ ,−  (i) The image or reflection of a point with
2 2
respect to X -a xis
Q M is the mid-point of P and Q, then
 3  27   Let P(α, β) be any point and Q ( x , y ) be its image about
Q ≡ ( x 2 , y 2 ) ≡ 2 × − 4, 2 ×  −  + 13
 2  2  X-axis, then ( M is the mid-point of P and Q )
i.e. Q ≡ ( − 1, − 14 ) Y
Aliter I :
By Ratio Proportion Method : Let, Q ( x 2 , y 2 ) the image P(α,β)
of P ( 4, − 13) with respect to line mirror 5x + y + 6 = 0, then
x 2 − 4 y 2 + 13 2(5 × 4 − 13 + 6) X′ X
= =− = −1 O M
5 1 52 + 12
or x 2 − 4 = − 5 and y 2 + 13 = − 1 Q(x,y)
∴ x 2 = − 1 and y 2 = − 14
Y′
Hence Q ≡ ( − 1, − 14 ) .
Chap 02 The Straight Lines 129

x = α and y = − β Y

∴ Q ≡ (α, − β) M
(α,β)P Q(x,y)
i.e. sign change of ordinate.

Remark X′ X
O
The image of the line ax + by + c = 0 about X-axis is x=a
ax − by + c = 0
Y′
(ii) The image or reflection of a point with
respect to Y-axis Let P (α, β) be any point and Q ( x , y ) be its image
about the line x = a, then y = β
Let P (α, β) be any point and Q ( x , y ) be its image about
Y-axis, then ( M is the mid-point of PQ ) ∴ Coordinates of M are (a, β)
Y Q M is the mid-point of PQ
∴ Q ≡ (2a − α, β)
(x,y)Q P(α,β)
M
Remark
The image of the line ax + by + c = 0 about the line x = λ is
X′ X a ( 2λ − x ) + by + c = 0
O
(v) The image or reflection of a point with
Y′
respect to the line y = b
x = − α and y = β Let P (α, β) be any point and Q ( x , y ) be its image about
the line y =b, then x = α
∴ Q ≡ ( − α, β )
Y Q(x,y)
i.e. sign change of abscissae.
M y=b
Remark
The image of the line ax + by + c = 0 about Y-axis is
− ax + by + c = 0
P(α,β)
X′ X
(iii) The image or reflection of a point with O
respect to origin Y′
Let P (α, β) be any point and Q ( x , y ) be its image about ∴ Co-ordinates of M are (α, b )
the origin (O is the mid point of PQ ), then
Y Q M is the mid-point of PQ ∴ Q ≡ (α, 2b − β)
P(α,β)
Remark
The image of the line ax + by + c = 0 about the line y = µ is
ax + b ( 2µ − y ) + c = 0.
X′ X
O
(vi) The image or reflection of a point with
respect to the line y = x
Q(x,y) Y′ Let P (α, β) be any point and Q ( x 1 , y 1 ) be its image about
the line y = x ( RS ), then PQ ⊥ RS
x = − α and y = − β Y
S
∴ Q ≡ ( − α, − β ) P(α,β) x
y=
i.e. sign change of abscissae and ordinate.
M
Remark
Q(x1,y1)
The image of the line ax + by + c = 0 about origin is
− ax − by + c = 0.
X′ X
O
(iv) The image or reflection of a point with respect R
to the line x = a Y′
130 Textbook of Coordinate Geometry

∴ ( Slope of PQ ) × ( Slope of RS ) = −1 x 1 = α and y1 = − β ...(i)


y1 − β Now, let P2 ( x 2 ,y 2 ) be the image of P1( x 1, y1 ) in the Y-axis.
or ×1= −1 Then
x1 −α
Y
or x1 − α = β − y1 ...(i) P(α,β)
and mid--point of PQ lie on y = x
y1 + β  x 1 + α 
i.e.   =  X' X
 2   2  O

or x1 + α = β + y1 ...(ii)
Solving Eqs. (i) and (ii), we get x 1 = β and y 1 = α P2 (x2,y2) P1(x1,y1)
Y'
∴ Q ≡ (β, α ) i.e. interchange of x and y.
x 2 = − x 1, y 2 = y1
Remark ⇒ x 2 = − α, y 2 = − β [from Eq. (i)] ...(ii)
The image of the line ax + by + c = 0 about the line y = x is further let P3 ( x 3 , y 3 ) be the image of P (α, β ) in the origin
ay + bx + c = 0. O. Then
(vii) The image or reflection of a point with x 3 = − α, y 3 = − β …(iii)
respect to the line y = x tan θ From Eqs. (ii) and (iii) ,we get
x 3 = x 2 and y 3 = y 2 .
Let P (α, β) be any point and Q ( x 1 , y 1 ) be its image about Hence the image of P2 of P after successive reflection in
the line y = x tan θ ( RS ), then PQ ⊥ RS their X-axis and Y-axis is the same as the single reflection
∴ ( Slope of PQ ) × ( Slope of RS ) = − 1 of P in the origin.
y1 − β y Example 96. Find the image of the point ( − 2 , − 7 )
or × tan θ = − 1
x1 −α
under the transformations ( x , y ) → ( x − 2y , − 3x + y ).
⇒ y 1 − β = (α − x 1 ) cot θ ...(i) Sol. Let ( x 1, y1 ) be the image of the point ( x , y ) under the given
and mid-point of PQ lie on y = x tan θ transformation. Then
y1 + β  x 1 + α  x 1 = x − 2y = ( − 2) − 2 ( − 7 ) = 12
i.e.   =  tan θ ∴ x 1 = 12 and y1 = − 3x + y = − 3 ( − 2) − 7 = − 1
 2   2 
∴ y1 = − 1
Y
Hence, the image is (12, − 1).
θ S
(α,β)P x tan y Example 97. The image of the point A(1, 2) by the line
y=
M
mirror y = x is the point B and the image of B by the
line mirror y = 0 is the point (α , β) . Find α and β.
θ Q(x1,y1) Sol. Let ( x 1, y1 ) be the image of the point (1, 2) about the line
X′ X
O
R y = x.
Y′
Then x 1 = 2, y 1 = 1 ...(i)
or y 1 + β = ( x 1 + α ) tan θ …(ii)
Y
Solving Eqs. (i) and (ii), we get
A(1,2)
x 1 = α cos 2θ + β sin 2θ
x
y=

and y 1 = α sin 2θ − β cos 2θ


∴ Q ≡ (α cos 2θ + β sin 2θ, α sin 2θ − β cos 2θ )
B(x1,y1)
y Example 95. The point P (α , β) undergoes a reflection
X′ X
O
in the X-axis followed by a reflection in the Y -axis. (α,β)
Y′
Show that their combined effect is the same as the
single reflection of P (α , β) in the origin when α , β > 0. Also given image of B( x 1, y1 ) by the line mirror y = 0 is
(α, β ). Then α = x 1 = 2
Sol. Let P1 ( x 1, y1 ) be the image of (α, β ) after reflection in the
and β = − y1 = − 1 [from Eq. (i)]
X -axis. Then
Chap 02 The Straight Lines 131

Hence, α = 2 and β = − 1
Aliter (Use of complex number) :
y Example 98. The point (4, 1) undergoes the following Let Q be the reflection of P ( 4, 1) about the line y = x , then
three transformations successively : Q ≡ (1, 4 )
(i) Reflection about the line y = x . QQ move 2 units along the +ve direction of X-axis, if new
point is ,R then R ≡ (3, 4 ) .
(ii) Translation through a distance 2 units along the
If R(3, 4 ) = R(z1 )
positive direction of X-axis.
when z1 = ( 3 + 4i )
(iii) Rotation through an angle π / 4 about the origin in
then R ′ ( x , y ) = R ′ (z 2 )
the anticlockwise direction.  π
∴ z 2 = z1e iπ / 4 Q ∠ROR ′ = 
Then, find the coordinates of the final position.  4
Sol. Let Q ( x 1, y1 ) be the reflection of P about the line y = x .  π π
= (3 + 4i )  cos + i sin 
Then  4 4
x 1 = 1
  1 i   1 7i 
y 1 = 4 = ( 3 + 4i )  +  = − + 
 2 2  2 2
∴ Coordinates of Q is (1, 4).  1 7 
Hence, new coordinates are  − , .
Given that Q move 2 units along the positive direction of  2 2
X-axis.
Y
R'
(1,4)Q
R(3,4) y=x
Use of Image or Reflection
5 To make problems simpler and easier use Image or
5
π/4 reflection.
P(4,1)
θ Types of problems : (i) If vertex of a ∆ ABC and
X′ X equations of perpendicular bisectors of AB and AC are
O
given, then B and C are the images or reflections of A
Y′
about the perpendicular bisectors of AB and AC
∴ Coordinates of R is ( x 1 + 2, y1 ) (where M and N are the mid-points of AB and AC ).
or R ( 3, 4 ) A
If OR makes an angle θ , then
4
tan θ =
3
4 3
∴ sin θ = and cos θ = M
N
5 5
π
After rotation of let new position of R is R′ and
4
B C
OR = OR ′ = 32 + 4 2 = 5
∴ OR′ makes an angle ( π / 4 + θ ) with X-axis. y Example 99. The base of a triangle passes through a
 π  π  fixed point ( f , g ) and its sides are respectively bisected
Coordinates of R ′ OR ′ cos  + θ , OR ′ sin  + θ 
 4  4  at right angles by the lines y + x = 0 and y − 9 x = 0.
  1 1  Determine the locus of its vertex.
i.e. R ′ OR ′  cos θ − sin θ ,
  2 2  Sol. Let A ≡ (α, β ) the image of A (α, β ) about y + x = 0 is B,
 1 1  then B ≡ ( − β, − α ) and if image of A (α, β ) about y − 9 x = 0
OR′ sin  cos θ + sin θ  is C ( x 2 , y 2 ), then
 2 2 
x 2 − α y 2 − β −2(β − 9α )
  3 4   3 4  = =
⇒ R′ 5  − ,5  +  −9 1 1 + 81
 5 2 5 2 5 2 5 2
9 β − 40 α 40 β + 9 α
 1 7  ∴ x2 = and y 2 =
⇒ R′  − ,  41 41
 2 2
132 Textbook of Coordinate Geometry

 9β − 40α 40β + 9α  h − 3 k + 1 − 2 (3 + 4 + 10)


∴ C ≡ ,  then = = = −2
 41 41  1 −4 12 + ( − 4 ) 2
Y i.e. h = 1, k = 7
C
∴ L ≡ (1, 7 )
A(α,β) Q F be the mid-point of AB.

0
y–9x=
Y

X′ X C L(1,7)
O
y+
x=
0 B/2 B (10,5)
(f,g)D B/2

E
F(13/2,2)
B Y′ X′ X
O
A(3,–1)
Hence, B, D , C are collinear, then
Y′
 
 −β −α 1
1 Let F ≡ ( α, β )
 f g 1 = 0
2 then B ≡ (2α − 3, 2β + 1)
9β − 40α 40β + 9α 1
 Q B lie on BE, then
 41 41  (2α − 3) − 4 (2β + 1) + 10 = 0
⇒ 4 ( α 2 + β 2 ) + ( 4 g + 5 f ) α + ( 4 f − 5g ) β = 0 i.e. 2α − 8β + 3 = 0 ...(i)
Hence, locus of vertex is and F lie on CF, then
4 ( x 2 + y 2 ) + ( 4 g + 5 f ) x + ( 4 f − 5g ) y = 0 6α + 10β − 59 = 0 ...(ii)
Solving Eqs. (i) and (ii), we get
(ii) The images or reflections of vertex A of a ∆ ABC
13
about the angular bisectors of angles B and C lie on α = ,β = 2
2
the side BC. (By congruence) A1 and A2 are the
 13 
images of A about the angle bisectors BE and CF then F ≡  , 2
2 
respectively, where M and N are the mid-points of
AA1 and AA2 . and B = (10, 5)
A Equation of AB is
2+1
y +1= ( x − 3)
13
−3
E 2
F M 6
N ⇒ y + 1 = ( x − 3)
7
B/2 C/2 or 6x − 7y − 25 = 0
B/2 C/2
B C Equation of BC is
A2 A1
7 −5
y −5= ( x − 10)
1 − 10
y Example 100. Find the equations of the sides of the
2
triangle having ( 3 , − 1) as a vertex, x − 4 y + 10 = 0 and ⇒ y −5 = − ( x − 10)
9
6 x + 10y − 59 = 0 being the equations of an angle
bisector and a median respectively drawn from or 2x + 9y − 65 = 0
different vertices. Q CA is the family of lines of CB and CF
then (2x + 9y − 65) + λ (6x + 10y − 59 ) = 0 ...(iii)
Sol. Let BE be the angle bisector and CF be the median. Given
equations of BE and CF are x − 4y + 10 = 0 and it pass through A (3, − 1)
6x + 10y − 59 = 0 respectively. then (6 − 9 − 65) + λ (18 − 10 − 59 ) = 0
Since, image of A with respect to BE lie on BC . If image of 4
∴ λ=−
A is L (h , k ). 3
Chap 02 The Straight Lines 133

From Eq. (iii), we get equation of AC is (b) Maximization : Let A and B are two given points on
18x + 13y − 41 = 0 the same side of ax + by + c = 0. Suppose we want to
determine a point P on ax + by + c = 0 such that
(iii) Optimization (Minimization or Maximization) | PA − PB | is maximum, then find the equation of line
(a) Minimization : Let A and B are two given points on AB wherever it intersects ax + by + c = 0 is the
the same side of ax + by + c = 0. Suppose we want to required point.
determine a point P on ax + by + c = 0 such that A
PA + PB is minimum. Then find the image of A or B B
about the line ax + by + c = 0 (say A′ or B ′ ) then join
P
B ′ with A or A′ with B wherever it intersects
y+c =0
ax + by + c = 0 is the required point. ax+b
A
c =0 Remark
B ax+by+
By triangle inequality
P B' Difference of two sides of a triangle < Third side
i.e. |PA − PB| = | AB| (maximum value)

y Example 102. Find a point P on the line


3x + 2y + 10 = 0 such that |PA − PB| is maximum where
A' A is (4, 2) and B is (2, 4).
∴ PA + PB = PA + PB ′ Sol. Let, L ( x , y ) = 3x + 2y + 10
or PA + PB = PA ′ + PB ∴ L ( 4, 2) = 12 + 4 + 10 = 26
and L (2, 4 ) = 6 + 8 + 10 = 24
Remark ∴ A and B lie on the same side of the line
By triangle in equality 3x + 2y + 10 = 0 ...(i)
Sum of two sides of a triangle > Third side Equation of line AB is
i.e.|PA + PB| = |PA + PB′| = | AB′| (minimum value). 4 −2
y −2= (x − 4)
y Example 101. Find a point R on the X-axis such that 2− 4
PR + RQ is the minimum, when P ≡ (1, 1) and Q ≡ ( 3, 2) . or x +y−6=0 ...(ii)
Sol. Since P and Q lie on the same side of X-axis. This line Eq. (ii) meets Eq. (i) at P ≡ ( − 22, 28) which is the
required point.
The image of Q (3, 2) about X -axis is Q ′ (3, − 2) then the
equation of line PQ ′ is Aliter
−2−1 Let P be ( x 1, y1 ) and ∠ APB = θ
y −1= ( x − 1)
3−1 ( PA )2 + ( PB )2 − ( AB )2
then cos θ =
⇒ 3x + 2y − 5 = 0 2PA ⋅ PB
Y Y B
Q (3,2)
P (1,1)

A
θ
X′ X
O R
P (x1,y1)
X′ X
O

Q' (3,–2) Y′
Y′
5 
This line meets X-axis at R  , 0 which is the required
point. 3 
134 Textbook of Coordinate Geometry

since cos θ ≤ 1 i.e. P lies on the line AB as well as on the given line.
( PA ) + ( PB ) − ( AB )2
2 2 ∴ Equation of AB is P
⇒ ≤1
2PA ⋅ PB 4 −2
y −2= (x − 4)
2− 4
⇒ ( PA − PB )2 ≤ ( AB )2 B
⇒ y −2= − x + 4
⇒ | PA − PB | ≤ | AB|
⇒ x +y =6 ...(i)
⇒ | PA − PB | ≤ 2 2
and given line
Maximum value of | PA − PB | is 2 2 3x + 2y + 10 = 0 ...(ii)
when, θ = 0. Solving Eqs. (i) and (ii), we get P ( − 22, 28). A

Exercise for Session 5


1. The coordinates of the foot of the perpendicular from (2, 3) to the line 3x + 4y − 6 = 0 are

(a)  − , −  (b)  ,− 
14 27 14 17
 25 25   25 25 

(c)  −
14 17 
(d)  , 
14 27
, 
 25 25   25 25 

2. If the foot of the perpendicular from the origin to a straight line is at the point (3, − 4). Then the equation of the
line is
(a) 3x − 4y = 25 (b) 3x − 4y + 25 = 0
(c) 4x + 3y − 25 = 0 (d) 4x − 3y + 25 = 0
a
3. The coordinates of the foot of the perpendicular from (a, 0) on the line y = mx + are
m
(a)  0,−  (b)  0,  (c)  0, −  (d)  0, 
1 a a 1
 a  m  m  a

4. If the equation of the locus of a point equidistant from the points (a1, b1) and (a 2, b 2 ) is
(a1 − a 2 ) x + (b1 − b 2 ) y + c = 0, then the value of c is
(a) a12 − a22 + b12 − b 22 (b) (a12 + b12 − a22 − b 22
1 2 1 2
(c) (a1 + a22 + b12 + b 22 ) (d) (a2 + b 22 − a12 − b12 )
2 2

5. The image of the point (3, 8) in the line x + 3y = 7 is


(a) (1, 4) (b) (4, 1) (c) (−1, − 4) (d) (−4, − 1)

6. The image of the point (4, –3) with respect to the line y = x is
(a) (−4, − 3) (b) (3, 4) (c) (−4, 3) (d) (−3, 4)

7. The coordinates of the image of the origin O with respect to the straight line x + y + 1 = 0 are

(a)  − , − 
1 1
(b) (−2, − 2)
 2 2
(c) (1, 1) (d) (−1, − 1)

8. If ( −2, 6) is the image of the point (4, 2) with respect to the line L = 0, then L ≡
(a) 6x − 4y − 7 = 0 (b) 2x − 3y − 5 = 0
(c) 3x − 2y + 5 = 0 (d) 3x − 2y + 10 = 0
Chap 02 The Straight Lines 135

9. The image of P (a, b ) on the line y = − x is Q and the image of Q on the line y = x is R. Then the mid-point of
PR is
a + b b + 2
(a) (a + b , a + b ) (b)  , 
 2 2 
(c) (a − b , b − a ) (d) (0, 0)

10. The nearest point on the line 3x − 4y = 25 from the origin is


(a) (3, 4) (b) (3, − 4) (c) (3, 5) (d) (−3, 5)

11. Consider the points A (0, 1) and B (2, 0), P be a point on the line 4x + 3y + 9 = 0. The coordinates of P such that
| PA − PB | is maximum are
(a)  − ,  (b)  − ,  (c)  − , 
12 17 84 13 6 17
(d) (0, − 3)
 5 5  5 5  5 5

12. Consider the points A (3, 4) and B (7, 13). If P is a point on the line y = x such that PA + PB is minimum, then the
coordinates of P are
(a)  ,  (b)  ,  (c)  , 
12 12 13 13 31 31
(d) (0, 0)
 7 7  7 7  7 7

13. The image of a point P(2, 3) in the line mirror y = x is the point Q and the image of Q in the line mirror y = 0 is
the point R( x , y ). Find the coordinates of R.

14. The equations of perpendicular bisector of the sides AB and AC of a ∆ ABC are x − y + 5 = 0 and x + 2y = 0
respectively. If the point A is (1, –2), find the equation of line BC.

15. In a ∆ ABC, the equation of the perpendicular bisector of AC is 3x − 2y + 8 = 0. If the coordinates of the point A
and B are (1, − 1) and (3, 1) respectively, find the equation of the line BC and the centre of the circumcircle of
∆ ABC.
16. Is there a real value of λ for which the image of the point ( λ , λ − 1) by the line mirror 3x + y = 6λ is the point
( λ2 + 1, λ ) ? If so find λ.
Session 6
Reflection of Light, Refraction of Light, Conditions of
Collinearity if Three Given Points be in Cyclic Order

Reflection of Light 3x − 2y − 5 = 0
∴ x = 1, y = − 1
When a ray of light falls on a smooth polished surface ∴ Coordinates of P are (1, − 1).
(Mirror) separating two media, a part of it is reflected back
Y
into the first medium. R
Normal A'
ence

N
n

I N R
lectio
incid

of ref

O A(3,0)
X′ X
of
Inc

ay

x
dr
Angle
Angle
ide

c te
nt

x P(1,–1)
fle
ray

i r
Re

=0
Point of incidence
0

5
=

y–
y–3
x–2

–2
I

3x
P Reflecting surface
Y′
IP is the incident ray and PR is the reflected ray. A
perpendicular drawn to the surface, at the point of Let slope of reflected ray be m.
incidence P is called the normal. Hence PN is the normal. Since, slope of line mirror is 3/2.
The angle between the incident ray and the normal ∴ Slope of PN = − 2 / 3 and
( ∠ IPN ) is called the angle of incidence which is slope of IP = 1 / 2, line PN is equally inclined to IP and PR,
represented by ∠ i then
i.e. ∠ IPN = ∠i = Angle of incidence and the angle   2   1  2 
between the reflected ray and the normal ( ∠ IPR ) is called  m − −    − −  
 3  3
  =− 2 
the angle of reflection which is represented by ∠r.   2    1  2 
 1 + m  −    1 +  −  
i.e. ∠ IPR = ∠ r = Angle of reflection.  3   2 3 
3m + 2 7
Laws of Reflection : ⇒ =−
3 − 2m 4
(i) The incident ray, normal and the reflected ray to a ⇒ 12m + 8 = − 21 + 14m
surface at the point of incidence all lie in the same ∴ 2m = 29
plane. 29
⇒ m=
(ii) The angle of incidence = angle of reflection 2
i.e. ∠i = ∠r ∴ Equation of reflected ray y + 1 =
29
( x − 1)
2
y Example 103. A ray of light is sent along the line ⇒ 2y + 2 = 29 x − 29
x − 2y − 3 = 0. Upon reaching the line 3x − 2y − 5 = 0, ⇒ 29 x − 2y − 31 = 0.
the ray is reflected from it. Find the equation of the Aliter (Image method) : Take A (3, 0) be any point on IP
line containing the reflected ray. and if A ′ (α, β ) be the image of A about the mirror line
Sol. To get coordinates of point P, we solve the given equation 3x − 2y − 5 = 0, then
of lines together as α − 3 β − 0 − 2 ( 9 − 0 − 5)
= =
x − 2y − 3 = 0 3 −2 9+4
Chap 02 The Straight Lines 137

15 16
∴ α= and β =
13 13 Refraction of Light
 15 16
∴ A′ ≡  , 
 13 13 When a ray of light falls on the boundary separating the
∴ Equation of A ′ P is the equation of the reflected ray then
two transparent media, there is a change in direction of
its equation is, ray. This phenomenon is called refraction.
 16 
 + 1

Normal
 13 

In
y +1= ( x − 1)

ci
de
 15  Angle of incidence

nt
 − 1

r
 13 

ay
i
Medium 1
29
⇒ y +1= ( x − 1) or 29 x − 2y − 31 = 0 Medium 2 Boundary
2
Deviation

Refr
y Example 104. A light beam, emanating from the point r

actio
(3, 10) reflects from the straight line 2x + y − 6 = 0 and, Angle of refraction

n ra
then passes through the point (7, 2). Find the

y
equations of the incident and reflected beams.
Sol. Let images of A and B about the line 2x + y − 6 = 0 are Laws of Refraction
A ′ (α, β ) and B′( γ , δ ) respectively.
(i) The incident ray, normal and the refracted ray to the
α − 3 β − 10 − 2 (6 + 10 − 6)
Then, = = surface separating the two transparent media all lie in
2 1 22 + 12 the same plane.
= −4 (ii) The ratio of sine of angle of incidence to the sine of
∴ α = − 5, β = 6 the angle of refraction is constant for the two given
i.e. A ′ ≡ ( − 5, 6) media. The constant is called the refractive index of
Y
medium 2 with respect to medium 1.
A(3,10)
sin i
i.e. 1 µ2 =
(–5,6)A' sin r

P y Example 105. A ray of light is sent along the line


x − 6 y = 8. After refracting across the line x + y = 1 it
B(7,2) enters the opposite side after turning by 15° away from
X′
O
X the line x + y = 1. Find the equation of the line along
which the refracted ray travels.
2x+

(–1,–2)B'
y–6

Sol. The point of intersection of x − 6y = 8 and x + y = 1 is


=

A ≡ (2, − 1). Let the required ray have the slope = m, then
0

Y′ Y
γ − 7 δ − 2 − 2 (14 + 2 − 6)
and = = =−4
2 1 22 + 12
∴ γ = − 1, δ = − 2
i.e. B′ ≡ ( − 1, − 2).
X′ X
∴ Equation of incident ray AB′ is O
15°
10 + 2 A
y +2= ( x + 1) or 3x − y + 1 = 0 (2,–1) m
3+1 6y=8
x–
and equation of reflected ray A ′ B is
x+
y=

2−6
1

y −6= ( x + 5) Y′
7 +5
m − 1
1
⇒ y − 6 = − ( x + 5) tan 15° = 6
3  m
1+
or x + 3y − 13 = 0  6
138 Textbook of Coordinate Geometry

 6m − 1 Then lf (t ) + m g(t ) + n = 0 ...(i)


⇒ 2− 3 = ±  
 m +6 where, t = a, b , c
6m − 1 i.e. a, b, c are the roots of the Eq. (i).
then, = 2 − 3 or 3 − 2
m +6
In this case Eq. (i) must be cubic in t.
70 − 37 3
⇒ m= At 3 + Bt 2 + Ct + D = 0 (say)
13
37 3 − 70 B C
or m= then a +b +c = −
, ab + bc + ca =
61 A A
Let the angle between x + y = 1 and the line through D
and abc = −
70 − 37 3 A
A(2, − 1) with the slope be α, then
13 which are the required conditions.
70 − 37 3 
 − ( − 1) y Example 106. If the points
83 − 37 3
tan α = 13  =   a 3 a 2 − 3  b 3 b 2 − 3  c 3 c 2 − 3

1 −  70 − 37 3  37 3 − 57  ,   and   are
 , , ,
  13    a − 1 a − 1  b − 1 b − 1  c − 1 c − 1 
83 − 37 3 collinear for three distinct values a, b , c and different
= from 1, then show that
37 3 − 57
abc − (bc + ca + ab ) + 3 (a + b + c ) = 0.
and if angle between x + y = 1 and the line through
37 3 − 70 Sol. Let the three given points lie on the line
A (2, − 1) with the slope be β, then
61 lx + my + n = 0, where l , m and n are constants.
37 3 − 70   t 3  t − 3
2
Then, l   +m   +n =0
 − ( − 1)  t − 1   t −1
 37 3 − 9 
tan β = 61  = 
1 −  37 3 − 70  131 − 37 3 ⇒ lt 3 + mt 2 + nt − (3m + n ) = 0

  61   for t = a, b, c
37 3 − 9 i.e. a, b, c are the roots of
=
131 − 37 3 lt 3 + mt 2 + nt − 3m − n = 0
Here tan α > tan β, ∴α > β m n
then a+b+c = − , ab + bc + ca =
70 − 37 3 l l
therefore the slope of the refracted ray =
13  3m + n 
∴ The equation of the refracted ray is and abc =  
 l 
(70 − 37 3 )
y +1= ( x − 2) Now, abc − (bc + ca + ab ) + 3 (a + b + c )
13
 3m + n  n 3m
⇒ 13y + 13 = (70 − 37 3 ) x − 140 + 74 3 =  − − =0
 l  l l
or (70 − 37 3 ) x − 13y − 153 + 74 3 = 0 Hence, abc − (bc + ca + ab ) + 3 (a + b + c ) = 0

y Example 107. If t 1 , t 2 and t 3 are distinct, the points


Conditions of Collinearity if (t 1 , 2at 1 + at 13 ), (t 2 , 2at 2 + at 23 ) and (t 3 , 2at 3 + at 33 ) are
collinear, then prove that t 1 + t 2 + t 3 = 0.
Three given points are in Sol. Let the three given points lie on the line lx + my + n = 0,
Cyclic Order where l, m and n are constants. Then,
l (t ) + m (2at + at 3 ) + n = 0
Let the three given points
⇒ (am ) t 3 + (2am + l ) t + n = 0 ...(i)
A ≡ ( f (a ), g(a )), B ≡ ( f (b ), g(b ))
for t = t 1, t 2 , t 3
and C ≡ ( f (c ), g(c )) lie on the line i .e ., t 1, t 2 , t 3 are the roots of Eq. (i), then
lx + my + n = 0, where l, m and n are constants. t1 + t 2 + t 3 = 0
Chap 02 The Straight Lines 139

Exercise for Session 6


1. A ray of light passing through the point (1, 2) is reflected on the X-axis at a point P and passes through the
point (5, 3). The abscissae of the point P is
13 13 13
(a) 3 (b) (c) (d)
3 5 4

2. The equation of the line segment AB is y = x . If A and B lie on the same side of the line mirror 2x − y = 1, then
the image of AB has the equation
(a) x + y = 2 (b) 8x + y = 9 (c) 7x − y = 6 (d) None of these

3. A ray of light travelling along the line x + y = 1is incident on the X-axis and after refraction it enters the other
side of the X-axis by turning π / 6 away from the X-axis. The equation of the line along which the refracted ray
travels is
(a) x + (2 − 3) y = 1 (b) x (2 + 3 ) + y = 2 + 3
(c) (2 − 3 )x + y = 1 (d) x + (2 + 3)y = (2 + 3 )

4. All the points lying inside the triangle formed by the points (0, 4), (2, 5) and (6, 2) satisfy
(a) 3x + 2y + 8 ≥ 0 (b) 2x + y − 10 ≥ 0
(c) 2x − 3y − 11≥ 0 (d) −2x + y − 3 ≥ 0

5. Let O be the origin and let A(1, 0), B(0, 1) be two points. If P ( x , y ) is a point such that xy > 0 and x + y < 1then
(a) P lies either inside in ∆OAB or in third quadrant (b) P cannot be inside in ∆OAB
(c) P lies inside the ∆OAB (d) None of these

6. A ray of light coming along the line 3x + 4y − 5 = 0 gets reflected from the line ax + by − 1 = 0 and goes along
the line 5x − 12y − 10 = 0 then
64 112 64 8
(a) a = ,b = (b) a = − ,b =
115 15 115 115
64 8 64 −8
(c) a = ,b = (d) a = − ,b =
115 115 115 115

7. Two sides of a triangle have the joint equation x 2 − 2xy − 3y 2 + 8y − 4 = 0. The third side, which is variable,
always passes through the point ( −5,−1). Find the range of values of the slope of the third side, so that the origin
is an interior point of the triangle.

8. Determine the range of values of θ ∈[0, 2π ] for which (cos θ, sin θ ) lies inside the triangle formed by the lines
x + y − 2 = 0, x − y − 1 = 0 and 6x + 2y − 10 = 0.
 3   3
9. Let P(sin θ, cos θ ), where 0 ≤ θ ≤ 2π be a point and let OAB be a triangle with vertices (0, 0),  , 0 and 0, .
 2   2
Find θ if P lies inside the ∆ OAB.

10. Find all values of θ for which the point (sin2 θ, sin θ ) lies inside the square formed by the line xy = 0 and
4xy − 2x − 2y + 1 = 0.
11. Determine whether the point ( −3, 2) lies inside or outside the triangle whose sides are given by the equations
x + y − 4 = 0, 3x − 7y + 8 = 0, 4x − y − 31 = 0.
12. A ray of light is sent along the line x − 2y + 5 = 0, upon reaching the line 3x − 2y + 7 = 0, the ray is reflected
from it. Find the equation of the line containing the reflected ray.
140 Textbook of Coordinate Geometry

Shortcuts and Important Results to Remember


1. Area of parallelogram formed by the lines 6. If A ≡ ( x1, y1 ), B ≡ ( x2 , y2 ) and C ≡ ( x3 , y3 ) are the vertices of
a1 x + b1 y + c1 = 0, a2 x + b2 y + d1 = 0, a1 x + b1 y + c 2 = 0 and a ∆ ABC then the equations of medians AD, BE and CF are
a2 x + b2 y + d 2 = 0 is y ( x2 + x3 − 2 x1 ) − x ( y2 + y3 − 2 y1 )
| c1 − c 2 ||d1 − d 2 | = y1 ( x2 + x3 ) − x1 ( y2 + y3 );
a1 b1
| | y ( x3 + x1 − 2 x2 ) − x ( y3 + y1 − 2 y2 )
a2 b2
= y2 ( x3 + x1 ) − x2 ( y3 + y1 )
2. Area of parallelogram formed by the lines and y ( x1 + x2 − 2 x3 ) − x ( y1 + y2 − 2 y3 ) = y3 ( x1 + x2 )
y = m1 x + c1, y = m2 x + d1, y = m1 x + c 2 and y = m2 x + d 2 − x3 ( y1 + y2 ) respectively.
is A
| c1 − c 2 ||d1 − d 2 |
| m1 − m2 |
E
3. If A ≡ ( x1, y1 ), B ≡ ( x2 , y2 ) and C ≡ ( x3 , y3 ) are the vertices of F
G
a ∆ ABC, then angle A is acute or obtuse according as
( x1 − x2 ) ( x1 − x3 ) + ( y1 − y2 ) ( y1 − y3 ) > 0 or <0
Similarly, for ∠ B
B D C
( x2 − x3 ) ( x2 − x1 ) + ( y2 − y3 ) ( y2 − y1 ) > 0 or <0
and for ∠ C Where, G is the centroid of ∆ ABC.
( x3 − x1 ) ( x3 − x2 ) + ( y3 − y1 ) ( y3 − y2 ) > 0 or <0 7. If A ≡ ( x1, y1 ), B ≡ ( x2 , y2 ) and C ≡ ( x3 , y3 ) are the vertices of
4. If the origin lies in the acute angle or obtuse angle a ∆ ABC then the equations of the altitudes AL, BM and
between the lines CN are
a1 x + b1 y + c1 = 0 y ( y2 − y3 ) + x ( x2 − x3 ) = y1 ( y2 − y3 ) + x1 ( x2 − x3 );
and a2 x + b2 y + c 2 = 0 y ( y3 − y1 ) + x ( x3 − x1 ) = y2 ( y3 − y1 ) + x2 ( x3 − x1 )
according as (a1a2 + b1b2 ) c1c 2 < 0 or > 0. A
5. If A ≡ ( x1, y1 ), B ≡ ( x2 , y2 ) and C ≡ ( x3 , y3 ) are the vertices of M
a ∆ ABC then the equations of the right bisectors N
(perpendicular bisectors) of the sides BC, CA and AB are O

 x22 − x32   y22 − y32 


y ( y2 − y3 ) + x ( x 2 − x 3 ) =   + ;
 2   2 

A
B C
L

and y ( y1 − y2 ) + x ( x1 − x2 ) = y3 ( x1 − x2 ) + x3 ( x1 − x2 )
respectively. Where O is the orthocentre of ∆ ABC.
8. If sides of a triangle ABC are represented by
O
BC : a1 x + b1 y + c1 = 0,
CA : a2 x + b2 y + c 2 = 0
B C
and AB : a3 x + b3 y + c 3 = 0
 x 2 − x12   y32 − y12  then | BC|:|CA|:| AB|
y ( y3 − y1 ) + x ( x3 − x1 ) =  3  + 
 2   2  a2 b2
= (a12 + b12 ) | |
a3 b3
 x 2 − x22   y12 − y22 
and y ( y1 − y2 ) + x ( x1 − x2 ) =  1  +  a3 b3 a1 b1
 2   2  : (a22 + b22 )| |: (a32 + b32 )| |
a1 b1 a2 b2
respectively. Where O is the circumcentre of ∆ ABC.
JEE Type Solved Examples :
Single Option Correct Type Questions
n This section contains 10 multiple choice examples. Each or ( λ2 + 2 λ + 1 ) x + λy − 2 λ2 − 2 = 0
example has four choices (a), (b), (c) and (d) out of which or ( λ2 + 1 )( x − 2 ) + λ (2 x + y ) = 0
ONLY ONE is correct.
∴ For fixed point
x − 2 = 0 and 2 x + y = 0
l Ex. 1 A rectangle ABCD has its side AB parallel to y = x
∴ Fixed point is (2, − 4 )
and vertices A, B and D lie on y =1, x = 2 and x = − 2 ∴ Equation of required line is y + 4 = 2( x − 2 )
respectively, then locus of vertex C is or y = 2x − 8
(a) x = 5 (b) x − y = 5
(c) y = 5 (d) x + y = 5 l Ex. 3 A man starts from the point P ( −3, 4 ) and reaches
Sol. (c) Since AB is parallel to y = x. point Q(0, 1) touching X-axis at R such that PR + RQ is
∴ Equation of AB is y = x + a (say) minimum, then the point R is
Q A lies on y = 1 3   3   2 
(a)  , 0 (b)  − , 0 (c)  − , 0 (d) ( −2, 0)
∴ A ≡ (1 − a, 1 ) 5   5   5 
C Sol. (b) Let R = (α , 0 )
Y For PR + PQ to be minimum it should be the path of light and
y=x thus we have
B P(–3, 4)
D

Q(0, 1)

y=1
A
q q X
X¢ X A R B
O (–3, 0) (0, 0)
(a, 0)
x=–2 x=2
∆APR ~ ∆BQR

AR PA α+3 4 3
⇒ = ⇒ = ⇒α=−
Again, B lies on x = 2 RB QB 0 −α 1 5
∴ B = (2, 2 + a )  3 
⇒ Hence, R ≡  − , 0
Equation of AD is  5 
y − 1 = − [ x − (1 − a )] or y = 2 − x − a
Q D lies on x = − 2 l Ex. 4 If the point P (a , a 2 ) lies inside the triangle formed
∴ D ≡ ( −2, 4 − a ) by the lines x = 0, y = 0 and x + y = 2, then exhaustive range
Let C ≡ (h, k ) of ‘a’ is
Q Diagonals of rectangle bisects to each other
(a) (0, 1) (b) (1, 2 )
∴ h + 1 −a =2 −2 ⇒ a =1 + h
and k + 1 =2 + a + 4 −a (c) ( 2 − 1, 1) (d) ( 2 − 1, 2)
⇒ k =5 Sol. (a) Since the point P (a, a ) lies on y = x 2
2

∴ Locus of C is y = 5 Solving, y = x2
and x + y = 2, we get
l Ex. 2 The line ( λ + 1) 2 x + λy − 2 λ 2 − 2 = 0 passes x2 + x − 2 = 0
through a point regardless of the value λ. Which of the ⇒ ( x + 2 )( x − 1 ) = 0
following is the line with slope 2 passing through the point? ⇒ x = − 2, 1
(a) y = 2x − 8 (b) y = 2x − 5 It is clear from figure,
(c) y = 2x − 4 (d) y = 2x + 8 A ≡ (1, 1 )
Sol. (a) also a > 0 for I quadrant.
Q ( λ + 1 ) 2 x + λy − 2 λ2 − 2 = 0 ∴ a ∈( 0, 1 )
142 Textbook of Coordinate Geometry

l Ex. 5 If 5a + 4b + 20c = t , then the value of t for which the l Ex. 8 Through the point P(α, β), when αβ > 0, the straight
line ax + by + c − 1 = 0 always passes through a fixed point is x y
line + =1 is drawn so as to form with
(a) 0 (b) 20 a b
(c) 30 (d) None of these coordinate axes a triangle of area ∆ . If ab > 0, then the least
Sol. (b) Equation of line
ax
+
by
+ 1 = 0 has two independent value of ∆ is
c −1 c −1 (a) αβ (b) 2αβ
parameters. It can pass through a fixed point if it contains (c) 4αβ (d) 8αβ
only one independent parameter. Now there must be one Sol. (b) Given line is
a b
relation between and independent of a, b and c so x y
c −1 c −1 + =1 …(i)
a b
a b
that can be expressed in terms of and straight line ∴ A ≡ (a, 0 ), B ≡ ( 0, b )
c −1 c −1
contains only one independent parameter. Now that given Y
5a 4b t − 20c
relation can be expressed as + = . B(0, b)
c −1 c −1 c −1
RHS is independent of c if t = 20. P(a, b)

l Ex. 6 If the straight lines, ax + amy + 1 = 0,


bx + (m + 1)by + 1 = 0 and cx + (m + 2 )cy + 1 = 0, m ≠ 0 are X¢ X
Y¢ A(a, 0)
concurrent, then a , b, c are in
(a) AP only for m = 1 (b) AP for all m Q Area of ( ∆AOB ) = ∆
(c) GP for all m (d) HP for all m 1
∴ | ab| = ∆ ⇒ ab = 2 ∆ (Qab > 0)
Sol. (d) The three lines are concurrent if 2
a am 1 Since, the line (i) passes through the point P(α , β ).
b m + 1 1 = 0 α β α aβ  2∆ 
  ∴ + =1 ⇒ + =1 Q b = 
c (m + 2 )c 1 a b a 2∆  a
Applying C 2 → C 2 − mC1, then ⇒ α 2β − 2a∆ + 2 ∆α = 0
a 0 1 Qa is real
b b 1 = 0 ∴ D≥0
 
c 2c 1 ⇒ 4 ∆2 − 4β (2 ∆α ) ≥ 0 or ∆ ≥ 2αβ
or a(b − 2c ) − 0 + 1(2bc − bc ) = 0 ∴ Least value of ∆ is 2αβ.
2ac
or b = , which is independent of m .
a+c l Ex. 9 The coordinates of the point P on the line

∴ a, b, c are in HP for all m. 2 x + 3y + 1 = 0, such that | PA − PB | is maximum,


where A is (2, 0) and B is (0, 2) is
l Ex. 7 If a ray travelling the line x =1 gets reflected the (a) (5, − 3 ) (b) (7, − 5)
line x + y =1, then the equation of the line along which the (c) ( 9, − 7 ) (d) (11, − 9)
reflected ray travels is Sol. (b) | PA − PB| ≤ | AB |
(a) y = 0 (b) x − y = 1 Maximum value of | PA − PB | is | AB |, which is possible only
(c) x = 0 (d) None of these when P , A, B are collinear
Sol. (a) Reflected ray is X-axis. if P ( x, y ), then equation AB is
x y
Y + =1
x=1 2 2
(0, 1)
⇒ x + y =2 …(i)
45° Now solving Eq. (i)
45° and 2 x + 3y + 1 = 0 …(ii)
X
O Then, we get,
(1, 0) x+y=1
x = 7, y = − 5
∴ P ≡ (7, − 5 )
∴ Equation y = 0
Chap 02 The Straight Lines 143

l Ex. 10 Equation of the straight line which belongs to the The line of this family which is farthest from ( 4, − 3 ) is the line
through (1, 1) and perpendicular to the line joining (1, 1) and
system of straight lines a ( 2 x + y − 3 ) + b (3 x + 2y − 5 ) = 0
( 4, − 3 )
and is farthest from the point (4, −3) is ∴ The required line is
(a) 4 x + 11y − 15 = 0 (b) 3 x − 4y + 1 = 0 3
(c) 7 x + y − 8 = 0 (d) None of these y − 1 = (x − 1)
4
Sol. (b) The system of straight lines or 3 x − 4y + 1 = 0
a (2 x + y − 3 ) + b (3 x + 2y − 5 ) = 0 ) passes through the point
of intersection of the lines 2 x + y − 3 = 0 and 3 x + 2y − 5 = 0
i.e. (1, 1)

JEE Type Solved Examples :


More than One Correct Option Type Questions
n This section contains 5 multiple choice examples. Each x y
l Ex. 12 Line + =1 cuts the coordinate axes at A(a , 0 )
example has four choices (a), (b), (c) and (d) out of which a b
MORE THAN ONE may be correct. x y
and B(0, b ) and the line + = −1 at A ′ ( −a ′ , 0 ) and
a′ b′
l Ex. 11 The vertices of a square inscribed in the triangle
B ′(0, − b ′ ). If the points A, B, A ′ , B ′ are concyclic, then the
with vertices A(0, 0 ), B( 2, 1) and C(3, 0 ), given that two of its orthocentre of the triangle ABA′ is
vertices are on the side AC, are
(a) (0, 0) (b) (0, b ′ )
3  3 3   9 3 9 
(a)  , 0 (b)  ,  (c)  ,  (d)  , 0  −aa ′   bb ′ 
2   2 4  4 4 4  (c) 0,  (d) 0, 
 b   a 
Sol. (a, b, c, d)
Sol. (b, c)
Let PQRS be a square inscribed in ∆ABC and
Q A, B, A′ , B′ are concyclic then,
PQ = QR = RS = SP = λ (say)
Let P ≡ (a, 0 ), Y

∴ Q ≡ (a + λ , λ ), R ≡ (a + λ , λ ) and S ≡ (a, λ )
Y B(0, b)

B(2, 1)
S R X¢ X
A¢(–a¢, 0) O A(a, 0)

X¢ X
A (0, 0) P Q C(3, 0)
B¢(0,–b¢)

Now equation of AB is Y¢
x − 2y = 0 …(i)
OA ⋅ OA′ = OB ⋅ OB′
and equation of BC is
or (a ) ⋅ ( −a′ ) = (b ) ⋅ ( −b′ )
x + y −3 = 0 …(ii)
or aa′ = bb′ …(i)
Q S lies on AB, then
a − 2λ = 0 …(iii) The equation of altitude through A′ is
and R lies on BC, then a
y − 0 = ( x + a′ )
a + λ + λ − 3 = 0 or a + 2 λ − 3 = 0 …(iv) b
3 3 It intersects the altitude
From Eqs. (iii) and (iv), we get a = , λ = aa′
2 4 x = 0 at y =
3  9  b
Hence, P ≡  , 0 , Q ≡  , 0 ,
2  4   aa′ 
∴ Orthocentre is  0,  or ( 0, b′ ) [from Eq. (i)]
 b 
 9 3  3 3
R≡ , , S = , 
 4 2  2 4
144 Textbook of Coordinate Geometry

l Ex. 13 Two straight lines u = 0 and v = 0 passes through y=Ö3 x


7 Y
the origin and angle between them is tan −1   . If the ratio
 9 B

9
of the slope of v = 0 and u = 0 is , then their equations are C
2
(a) y = 3 x and 3y = 2x
(b) 2y = 3 x and 3y = x 2 D y= x
Ö3
(c) y + 3 x = 0 and 3y + 2x = 0
(d) 2y + 3 x = 0 and 3y + x = 0 2 A
Sol. (a, b, c, d) X¢ X
9m O
Let the slope of u = 0 be m, then the slope of v = 0 is .
2 Y¢
 m − 9m 
 Coordinates of A and C are ( 3, 1 ) and (1, 3 ) in I quadrant and
Therefore, 2 = 7
 9m 9 in III quadrant are ( − 3, − 1 ) and (−1, − 3 )
1+m×
 2 Hence, coordinates of B are ( 3 + 1, 3 + 1 ) and
 −7m  7 ( − 3 − 1, − 3 − 1 )
or  =
2 + 9m  9
2

⇒ 9m 2 + 9m + 2 = 0 or 9m 2 − 9m + 2 = 0
l Ex. 15 A and B are two fixed points whose coordinates
2 1 2 1 are (3, 2) and (5, 4) respectively. The coordinates of a point P,
⇒ m = − , − or m = , if ABP is an equilateral triangle are
3 3 3 3
Therefore, the equation of lines are (a) ( 4 − 3 , 3 + 3 ) (b) ( 4 + 3 , 3 − 3 )
(i) 2 x + 3y = 0 and 3 x + y = 0 (c) (3 − 3 , 4 + 3 ) (d) (3 + 3 , 4 − 3 )
(ii) x + 3y = 0 and 3 x + 2y = 0 Sol. (a, b)
(iii) 2 x = 3y and 3x = y Q AB = AP = BP = 2 2
(iv) x = 3y and 3 x = 2y
∴ Coordinates of P are (3 + 2 2 cos105 °, 2 + 2 2 sin 105 ° )
l Ex. 14 Two sides of a rhombus OABC (lying entirely in P
the first or third quadrant) of are equal to 2 sq units are
x
y= , y = 3 x . Then the possible coordinates of B is/are
3
B(5, 4)
(O being the origin)
(a) (1 + 3 , 1 + 3 ) (b) ( −1 − 3 , − 1 − 3 )
(c) (3 + 3 , 3 + 3 ) (d) ( 3 − 1, 3 − 1) 60°
Sol. (a, b) 45°
X
A(3, 2)
Here, ∠COA = 30 °
Let OA = AB = BC = CO = x or (3 − ( 3 − 1 ), 2 + 3 + 1 )
Q Area of rhombus OABC
or (4 − 3, 3 + 3 )
1
= 2 × × x × x sin 30 ° If P below AB, then coordinates of P are
2
x 2 (3 + 2 2 cos15 °, 2 − 2 2 sin 15 °)
= =2 [given]
2 or [(3 + 3 + 1, 2 − ( 3 − 1 )]
∴ x =2 or (4 + 3, 3 − 3 )
Chap 02 The Straight Lines 145

JEE Type Solved Examples :


Paragraph Based Questions
n This section contains 2 solved paragraphs based upon 16. (d) Q M lie on y = 1
each of the paragraph 3 multiple choice questions have ∴ Length of ⊥ from M to OA is 1.
to be answered. Each of these question has four choices 17. (c) λ = Perimeter of region R
(a), (b), (c) and (d) out of which ONLY ONE is correct.
= OA + AD + DE + EO
Paragraph I =3 + 2 + 1 + 2
(Q. Nos. 16 to18) =4+2 2
Let d (P , OA ) ≤ min⋅ {d (P , AB ), d (P , BC ), d (P , OC )}, where d 18. (a) ∆ = Area of region R
denotes the distance from the point to the corresponding the 1
= (OA + ED ) × 1
line and R be the region consisting of all those points P inside 2
the rectangle OABC such that O ≡ (0, 0 ), A ≡ (3, 0 ), B ≡ (3, 2 ) 1
= (3 + 1 ) = 2
and C ≡ (0, 2 ). Let M be the peak of region R. 2
16. Length of the perpendicular from M to OA is
(a) 4 (b) 3 (c) 2 (d) 1
Paragraph II
(Q. Nos. 19 to 21)
17. If λ be the perimeter of region R, then λ is A variable straight line ‘L’ is drawn through O(0, 0 ) to meet
(a) 4 − 2 (b) 4 + 2 (c) 4 + 2 2 (d) 10 this lines L1 : y − x − 10 = 0 and L 2 : y − x − 20 = 0 at the
18. If ∆ be the area of region R, then ∆ is points A and B respectively.
(a) 2 (b) 4 (c) 6 (d) 8 2 1 1
19. A point P is taken on ‘L’ such that = + ,
Sol. Let P ≡ ( x, y ) OP OA OB
Q d ( P , OA ) ≤ min ⋅ {d ( P , AB ), d ( P , BC ), d ( P , OC )} then the locus of P is
Y (a) 3 x + 3y − 40 = 0 (b) 3 x + 3y + 40 = 0
(c) 3 x − 3y − 40 = 0 (d) 3 x − 3y + 40 = 0
(0, 2) C B(3, 2)
20. A point P is taken on ‘L’ such that (OP ) 2 = OA ⋅ OB,
P (x, y) then the locus of P is
(a) (y − x )2 = 25 (b) (y − x )2 = 50
X¢ X
O A(3, 0) (c) (y − x )2 = 100 (d) (y − x )2 = 200
Y¢ 21. A point P is taken on ‘L’ such that
⇒ | y | ≤ min {|3 − x |, |2 − y |, | x| } 1 1 1
As the rectangle OABC lies in I quadrant, 2
= 2
+ , then locus of P is
(OP ) (OA ) (OB ) 2
∴ y ≤ min ⋅ [3 − x, 2 − y . x ]
We draw the graph of (a) (y − x )2 = 32 (b) (y − x )2 = 64
y = 3 − x, y = 2 − y , y = x (c) (y − x )2 = 80 (d) (y − x )2 = 100
or x + y = 3, y = 1, y = x Sol. Let the equation of line ‘L’ through origin is
Y x−0 y −0
= =r
x
y=

cosθ sin θ
x+
y=

∴ P ≡ (r cosθ, r sin θ )
3

Let OA = r1 and OB = r2
(1, 1) E D (2, 1) y=1
∴ A ≡ (r1 cosθ, r1 sin θ )
and B ≡ (r2 cosθ, r2 sin θ )
X¢ X A lies on L1 :y − x − 10 = 0
O A
∴ r1 sin θ − r1 cosθ − 10 = 0
Y¢ 10
⇒ r1 = …(i)
∴ E ≡ (1, 1 ) and D ≡ (2, 1 ) sin θ − cosθ
146 Textbook of Coordinate Geometry

⇒ B lies on L1 : y − x − 20 = 0 r sin θ − r cosθ r sin θ − r cosθ


or 2 = +
L 10 20
y −x y −x
or 2= + [Q P ≡ (r cos, r sin θ )]
Y B 10 20
∴ Locus of P is 3 x − 3y + 40 = 0
20. (d) Q (OP ) 2 = OA ⋅ OB
⇒ r 2 = r1 ⋅ r2
0 A 10 20
=
⇒ r2 = ⋅ [from Eqs. (i) and (ii)]
20

(sin θ − cosθ ) (sin θ − cosθ )


x–

=
0 P
y–

(r sin θ − r cosθ ) 2 = 200


10

or
x–

∴ Locus of P is (y − x ) 2 = 200
y–

q
X¢ X 1 1 1
O 21. (c) Q = +
(OP ) 2 (OA ) 2 (OB ) 2

1 1 1
⇒ = +
∴ r2 sin θ − r2 cosθ − 20 = 0 r 2 r12 r22
20
⇒ r2 = …(ii) 1 (sin θ − cosθ ) 2 (sin θ − cosθ ) 2
sin θ − cosθ ⇒ = +
r2 100 400
2 1 1 2 1 1 400 = 4(r sin θ − r cosθ ) + (r sin θ − r cosθ ) 2
2
19. (d) Q = + ⇒ = + or
OP OA OB r r1 r2
∴ Locus of P is (y − x ) 2 = 80
2 sin θ − cosθ sin θ − cosθ
⇒ = + [from Eqs. (i) and (ii)]
r 10 20

JEE Type Solved Examples :


Single Integer Answer Type Questions
n
This section contains 2 examples. The answer to each example is a single digit integer, ranging from 0 to 9
(both inclusive).

l Ex. 22 P ( x , y ) is called a natural point if x , y ∈N . The l Ex. 23 The distance of the point ( x , y ) from the origin is
total number of points lying inside the quadrilateral formed defined as d = max⋅ {| x |, | y | }. Then the distance of the
by the lines 2 x + y = 2, x = 0, y = 0 and x + y = 5 is common point for the family of lines
Sol. (6) First, we construct the graph of the given quadrilateral. x (1 + λ ) + λy + 2 + λ = 0 (λ being parameter) from the
Y origin is
Sol. (2) Given family of lines is
5 x(1 + λ ) + λy + 2 + λ = 0
4 ⇒ (x + 2) + λ(x + y + 1) = 0
3
for common point or fixed point
x+

x+2=0
y=

2
5

x=0 and x+y +1 =0


1 or x = − 2, y = 1

O 1 2 3 4 5
X ∴ Common point is ( −2, 1 )
y=0 or d = max{| −2|, |1| }
Y¢ 2x+y
=2
= max{2, 1 } = 2
It is clear from the graph that there are six points lying inside
the quadrilateral.
Chap 02 The Straight Lines 147

JEE Type Solved Examples :


Matching Type Questions
n This section contains 2 examples. Examples 24 and 25 l Ex. 25 The equation of the sides of a triangle are
has four statements (A, B, C and D) given in Column I x + 2y + 1 = 0, 2 x + y + 2 = 0 and px + qy + 1 = 0 and area of
and four statements (p, q, r and s) in Column II. Any triangle is ∆ .
given statement in Column I can have correct matching
with one or more statement(s) given in Column II. Column I Column II
(A) p = 2, q = 3, then 8∆ is divisible by (p) 3
l Ex. 24 Consider the following linear equations x and y
(B) p = 3, q = 2, then 8∆ is divisible by (q) 4
ax + by + c = 0 (C) p = 3, q = 4, then 10∆ is divisible by (r) 6
bx + cy + a = 0
(D) p = 4, q = 3, then 20∆ is divisible by (s) 9
cx + ay + b = 0
Sol. (A) → ( p ); (B) → ( p, q, r ); (C) → ( p, r ); (D) → ( p, s )
Column I Column II
(A) a + b + c ≠ 0 and (p) Lines are sides of a triangle D2
Q ∆= , where
a 2 + b 2 + c 2 = ab + bc + ca 2| C1 C 2 C 3 |
(B) a + b + c = 0 and (q) Lines are different and
1 2 1
a 2 + b 2 + c 2 ≠ ab + bc + ca concurrent
D = 
2 1 2 = 3( p − 1 )
(C) a + b + c ≠ 0 and (r) Number of pair ( x , y ) satisfying
a 2 + b 2 + c 2 ≠ ab + bc + ca the equations are infinite p q 1
(D) a + b + c = 0 and (s) Lines are identical and C1, C 2, C 3 are co-factors of third column, then
a 2 + b 2 + c 2 = ab + bc + ca C1 = 2q − p, C 2 = 2 p − q, C 3 = − 3
3( p − 1 ) 2
Sol. (A) → (r , s ); (B) → (q ); (C) → ( p ); (D) → (r ) ∴ ∆=
2|2q − p||2 p − q|
(A) if a + b + c ≠ 0 and a 2 + b 2 + c 2 = ab + bc + ca
(A) for p = 2, q = 3
1
or {(a − b ) 2 + (b − c ) 2 + (c − a ) 2 } = 0 3
2 ⇒ ∆=
8
or a − b = 0, b − c = 0, c − a = 0
∴ 8∆ = 3
or a = b = c ⇒ All the lines an identical
(B) for p = 3, q = 2
and number of pair ( x, y ) are infinite.
3
(B) If a + b + c = 0 and a 2 + b 2 + c 2 ≠ ab + bc + ca ⇒ ∆=
2
⇒a + b + c = 0, but a, b, c are not simultaneously equal. ∴ 8 ∆ = 12
Hence, lines are different and concurrent. (C) for p = 3, q = 4
(C) If a + b + c ≠ 0 and a 2 + b 2 + c 2 ≠ ab + bc + ca 6
⇒ ∆=
a b c 10
⇒ ∆ =  ∴ 10 ∆ = 6
b c a ≠ 0 and a, b, c are not all simultaneously equal.
(D) for p = 4, q = 3
c a b
27
∴ Lines are sides of a triangle. ⇒ ∆=
20
(D) If a + b + c = 0 and a 2 + b 2 + c 2 = ab + bc + ca
∴ 20 ∆ = 27
⇒ ∆ = 0 and a = b = c
∴ Equations are satisfied for any ( x, y ) .
148 Textbook of Coordinate Geometry

JEE Type Solved Examples :


Statement I and II Type Questions
n Directions (Ex. Nos. 26 and 27) are Assertion-Reason Sol. (b) Let L1 ≡ 3 x + 4y = 0,
type examples. Each of these examples contains two L2 ≡ 5 x − 12y = 0 and L3 ≡ y − 15 = 0
statements. 3 + 32
Length of ⊥ from P to L1 = =7
Statement I (Assertion) and Statement II (Reason) 5
Each of these examples also has four alternative choices. |5 − 96|
Length of ⊥ from P to L2 = =7
Only one of which is the correct answer. You have to select 13
the correct choice as given below. |8 − 15|
(a) Statement I is true, statement II is true; statement II and Length of ⊥ from P to L3 = =7
1
is a correct explanation for statement I.
∴ Statement II is true
(b) Statement I is true, statement II is true; statement II
is not a correct explanation for statement I. Y
(c) Statement I is true, statement II is false.
B (–20, 15) L 3 =0 A (36, 15)
(d) Statement I is false, statement II is true.

x y x y
l Ex. 26 Consider the lines, L1 : + = 1; L 2 : + = 1; P
3 4 4 3 L1=0
L2=0
x y x y
L3 : + = 2 and L 4 : + = 2
3 4 4 3
Statement I : The quadrilateral formed by these four lines is a X¢ X
O (0, 0)
rhombus.
Statement II : If diagonals of a quadrilateral formed by any
four lines are unequal and intersect at right angle, then it is a Y¢
rhombus. 1
Sol. (c) Q L1, L3 are parallel. Also, Area of ∆OPA = × OA × 7
2
1 12 1
∴ Distance between L1 and L3 = = and = × 39 × 7 = ∆1
1 1 5 2
 + 
 9 16 1
Area of ∆OPB = × OB × 7
L2, L4 are parallel. 2
1 12 1
∴ Distance between L2 and L4 = = = × 25 × 7 = ∆ 2
 1 1 5 2
 +  1
 16 9 and Area of ∆APB = × AB × 7
2
∴ Distance between L1 and L3 = Distance between L2 and L4 .
1
∴ Quadrilateral formed by L1, L2, L3, L4 is a rhombus. = × 56 × 7 = ∆ 3
2
Hence, statement I is true and statement II is false. 7
∴ ∆1 + ∆ 2 + ∆ 3 = (39 + 25 + 56 )
2
l Ex. 27 7 × 120 1
Statement I : Incentre of the triangle formed by the lines = = × 56 × 15 = Area of ∆AOB
2 2
whose sides are 3 x + 4y = 0; 5 x − 12y = 0 and y − 15 = 0 is the
⇒ P inside the triangle.
point P whose coordinates are (1, 8).
Hence, both statements are true and statement II is not correct
Statement II : Point P equidistant from the three lines
explanation of statement I.
forming the triangle.
Chap 02 The Straight Lines 149

Subjective Type Examples


n In this section, there are 15 subjective solved examples. l Ex. 29. Let the sides of a parallelogram be

u = p , u = q , v = r and v = s where, u = lx + my + n and


l Ex. 28. If x-coordinates of two points B and C are the
v = l ′ x + m ′ y + n ′ . Show that the equation of the diagonal
roots of equation x 2 + 4 x + 3 = 0 and their y-coordinates are
through the point of intersection of u = p and v = r and u = q
the roots of equation x 2 − x − 6 = 0. If x-coordinate of B is
less than x-coordinate of C and y-coordinate of B is greater u v 1
and v = s , is given by p r 1 = 0
than the y-coordinate of C and coordinates of a third point  
A be (3, − 5 ). Find the length of the bisector of the interior q s 1
angle at A . Sol. Equation of the line through point of intersection D of lines
Sol. Q x 2 + 4x + 3 = 0 ⇒ x = − 1, − 3 u − p = 0 and v − r = 0 is
and x2 − x − 6 = 0 ⇒ x = − 2, 3 (u − p ) + λ ( ν − r ) = 0 ...(i)
it is also passing through u = q and v = s , then Eq. (i) becomes
Also given that x and y-coordinates of B are respectively less
than and greater than the corresponding coordinates of C. (q − p ) + λ (s − r ) = 0
(q − p )
∴ B ≡ ( − 3, 3 ) and C ≡ ( − 1, − 2 ) ∴ λ=− …(ii)
(s − r )
Now AB = (3 + 3 ) 2 + ( − 5 − 3 ) 2 = 10
u=p
D C
and AC = (3 + 1 ) 2 + ( − 5 + 2 ) 2 = 5
AB 2
∴ =
AC 1

v=s
v=r
Y
B(–3,3)

A u=q B

From Eqs. (i) and (ii), we get


X′
(q − p )
X (u − p ) − (v − r ) = 0
O (s − r )
– 5,– 1 D
3 3
⇒ u (s − r ) − p (s − r ) − v (q − p ) + r (q − p ) = 0
(–1,–2)C ⇒ u (r − s ) − v ( p − q ) + ps − qr = 0
x
x
u v 1
A(3,–5) ⇒ p r 1 = 0
 
Y′ q s 1

Let AD be the bisector of ∠ BAC, then


BD AB 2
l Ex. 30. The vertices B and C of a triangle ABC lie on the
= = lines 3y = 4 x and y = 0 respectively and the side BC passes
DC AC 1
Thus D divides BC internally in the ratio 2 : 1  2 2
through the point  ,  . If ABOC is a rhombus, O being
 2 ( − 1 ) + 1 ( − 3 ) 2 ( − 2 ) + 1 (3 ) 3 3
∴ D≡ , 
 2+1 2+1  the origin, find the equation of the line BC and the
 5 1 coordinates of A.
Thus, D ≡ − , − 
 3 3 Sol. Let the side of the rhombus be a
2 2 ∴ OB = BA = AC = CO = a
 5  1
Now, AD = 3 +  +  − 5 +   4x 
 3  3 Co-ordinates of A is a + x1, 1 
 3 
196 196 14 2
= + = units. ∴ (OB ) 2 = ( BA ) 2 = ( AC ) 2 = (CO ) 2 = a 2 ⇒(OB ) 2 = a 2
9 9 3
150 Textbook of Coordinate Geometry

16 x12 25 x12 5x Y
⇒ x12 + = a2 ⇒ = a2 ⇒ a = 1
9 9 3
 5 x1   8 x1 4 x1 
∴ C ≡ , 0 , A ≡  , ,
 3   3 3  (0,b)B P(a,b)
 4x 
B ≡  x1, 1  , O ≡ ( 0, 0 )
 3  Q
Y X′ X
O A(a,0)
Y′
(x1, 4x31)
B A Q PQ ⊥ AB
( 32 , 32 ) ∴ Slope of PQ × Slope of AB = − 1
4x
3 y=

 b − y1   b − 0
⇒   ×   = −1
 a − x1   0 − a
X′ X
O a C(a,0) ⇒ ax1 − by1 = a 2 − b 2 ...(ii)
Y′ x y
Q Equation of AB is + = 1
a b
∴ Equation of BC
x y
4 x1 But Q lies on AB then 1 + 1 = 1
−0 a b
 5 x1 
y −0= 3 x −  ⇒ bx1 + ay1 = ab
5 x1  3  ...(iii)
x1 −
3 From Eqs. (ii) and (iii), we get
10 x1 a3 b3
⇒ y = − 2x + ...(1) x1 = 2 , y 1 =
3 a + b2 a2 + b2
 2 2 (a 2 + b 2 )
it is passing through  ,  , then x12/3 + y12/3 =
 3 3 Now,
(a 2 + b 2 ) 2/3
2 4 10 x1
=− + = (a 2 + b 2 )1/3 = c 2/3 [from Eq. (i)]
3 3 3
10 x1 Hence, required locus is x 2/ 3
+y 2/ 3
=c 2/ 3
.
⇒ 2=
3 Aliter :
3 Since, AB = c
∴ x1 =
5 Let ∠ BAO = θ
Hence, coordinates are ∴ OA = c cosθ and OB = c sin θ
8 4 Q OB = PA = c sinθ
C ≡ (1, 0 ), A ≡  , 
5 5 Y
 3 4
B ≡  ,  , O ≡ ( 0, 0 )
 5 5
(0,b)B P(a,b)
From Eq. (i), equation of BC is
y = − 2x + 2
Q
⇒ 2x + y = 2
X′ X
O A(a,0)
l Ex. 31 The ends AB of a straight line segment of constant
Y′
length c slide upon the fixed rectangular axes OX and OY
QA QA
respectively. If the rectangle OAPB be completed, then show In ∆ PQA, sin θ = =
PA c sin θ
that the locus of the foot of perpendicular drawn from P to
AB is x 2 /3 + y 2 /3 = c 2 /3 . ∴ QA = c sin 2 θ
QM QM
Sol. Let A ≡ (a , 0 ) , B ≡ ( 0 , b ) then P ≡ (a , b ) Now, in ∆QAM, sin θ = =
QA c sin 2 θ
Since AB = c
∴ QM = c sin 3 θ
a2 + b2 = c
MA MA
or a2 + b2 = c2 ...(i) and cosθ = =
QA c sin 2 θ
and let Q ≡ ( x1, y1 )
⇒ MA = c sin 2 θ cosθ
Chap 02 The Straight Lines 151

If coordinates of Q be ( x1, y1 ) l Ex. 33 In a ∆ ABC, A ≡ (α , β), B ≡ (1 , 2 ), C ≡ ( 2 , 3 ) and


then x1 = OM = OA − MA = c cos θ − c sin 2 θ cosθ
point A lies on the line y = 2 x + 3 , where α, β ∈I . If the area
x1 = c cos3 θ ...(i) of ∆ ABC be such that [ ∆ ] = 2 , where [.] denotes the greatest
and y1 = MQ = c sin θ 3
...(ii) integer function, find all possible coordinates of A.
From Eqs. (i) and (ii), Sol. Q (α , β ) lies on y = 2 x + 3
x12/3 + y12/3 = c 2/3 cos2 θ + c 2/3 sin 2 θ = c 2/3 then β = 2α + 3
⇒ x12/3 + y12/3 =c 2/ 3 Thus ,the coordinates of A are (α , 2α + 3 )
1 α 2α + 3 +  1 2+2 3 
Hence, locus of Q is x 2/3 + y 2/3 = c 2/3 ∆ = | |
2 1 2   2 3  α 2α + 3
= | 2α − 2α − 3 + 3 − 4 + 4α + 6 − 3α |
l Ex. 32 A square lies above the X-axis and has one vertex
1
at the origin. The side passing through the origin makes an ∆ = |α + 2|
2
angle α (0 < α < π / 4 ) with the positive direction of the
1 
X-axis. Prove that the equation of its diagonals are But [ ∆ ] = |α + 2| = 2 (given)
2 
y (cos α − sin α) = x (sin α + cos α) |α + 2|
∴ 2≤ <3
and y (sin α + cos α) + x (cos α − sin α) = a 2
where, is the length of each side of the square. ⇒ 4 ≤ |α + 2 | < 6
Sol. Here, OA = AB = BC = CO = a ⇒ 4 ≤ α + 2 < 6 or − 6 < α + 2 ≤ − 4
⇒ 2 ≤ α < 4 or − 8 < α ≤ − 6
Y
B ∴ α = 2, 3, − 7, − 6
then β = 7, 9, − 11, − 9
Hence, coordinates of A are (2 , 7 ), (3 , 9 ), ( − 7 , − 11 ) and
C
( − 6, − 9 ).
A
π/4 l Ex. 34 Find the values of non-negative real numbers
X′
α
X λ 1 , λ 2 , λ 3 , µ 1 , µ 2 , µ 3 such that the algebraic sum of the
O
Y′
perpendiculars drawn from points
( λ 1 , 4 ), ( λ 2 , 5 ), ( λ 3 , − 3 ), ( 2 , µ 1 ), (3 , µ 2 ) and ( 7, µ 3 ) on a
π 
Equation of OB is y − 0 = tan  + α  ( x − 0 ) variable line passing through ( 2 ,1) is zero.
4 
Sol. Let the equation of the variable line be ax + by + c = 0. It is
π  π 
⇒ y cos  + α  = x sin  + α  given that
4  4  (aλ 1 + 4b + c ) (aλ 2 + 5b + c )
⇒ +
⇒ y (cos α − sin α ) = x (cos α + sin α ) (a 2 + b 2 ) (a 2 + b 2 )
Hence, equation of diagonal OB is
(aλ 3 − 3b + c ) (2a + bµ1 + c )
x (cos α + sin α ) + y (sin α − cos α ) = 0 + +
(a + b )
2 2
(a 2 + b 2 )
Coordinates of A are (a cos α , a sin α )
(3a + bµ 2 + c ) (7a + bµ 3 + c )
Diagonal AC is perpendicular to the diagonal OB, + + =0
1 (a 2 + b 2 ) (a 2 + b 2 )
Slope of AC = −
slope of OB ⇒ a ( λ 1 + λ 2 + λ 3 + 12 ) + b (µ1 + µ 2 + µ 3 + 6 ) + 6c = 0
1 π   λ + λ 2 + λ 3 + 12  µ1 + µ 2 + µ 3 + 6 
=− = − cot  + α  ⇒ a 1  +b   + c = 0 …(i)
 π   4   6   6 
tan  + α 
4  But the line passes through (2,1), therefore
Hence, equation of diagonal AC is 2a + b + c = 0 ...(ii)
π  From Eqs. (i) and (ii), we get
y − a sin α = − cot  + α  ( x − a cos α ) λ 1 + λ 2 + λ 3 + 12 µ + µ2 + µ3 + 6
4  = 2 and 1 =1
 cos α − sin α  6 6
⇒ y − a sin α = −   ( x − a cos α ) ⇒ λ 1 + λ 2 + λ 3 = 0 and µ1 + µ 2 + µ 3 = 0
 cos α + sin α 
⇒ λ 1 = λ 2 = λ 3 and µ1 = µ 2 = µ 3 [Q λ i , µi ≥ 0 for all i ]
⇒ y (cos α + sin α ) − a sin α cos α − a sin 2 α λ1 = λ 2 = λ 3 = α (say)
= − x (cos α − sin α ) + a cos2 α − a sin α cos α and µ1 = µ 2 = µ 3 = β (say)
⇒ x (cos α − sin α ) + y (cos α + sin α ) = a where α ≥ 0 and β ≥ 0.
152 Textbook of Coordinate Geometry

Ex. 35 The three sides of a triangle are y k y k


l
⇒ − = x − h or x − = h −
Lr ≡ x cos θ r + y sin θ r − pr = 0, where r= 1,2,3. Show that m m m m
x y
the orthocentre is given by ⇒ − =1
 k  k
L1 cos (θ 2 − θ 3 ) = L 2 cos (θ 3 − θ 1 ) = L3 cos (θ 1 − θ 2 ) . h −  m h − 
 m  m
Sol. The given lines are
x y
L1 ≡ x cosθ1 + y sin θ1 − p1 = 0 ⇒ + =1
 k  (k − mh )
L2 ≡ x cosθ 2 + y sin θ 2 − p 2 = 0 h − 
 m
L3 ≡ x cosθ 3 + y sin θ 3 − p 3 = 0
Now, equation of AD is L2 + λL3 = 0 …(i) Y

⇒ ( x cosθ 2 + y sin θ 2 − p 2 ) + λ ( x cosθ 3 + y sin θ 3 − p 3 ) = 0


⇒ x (cosθ 2 + λ cosθ 3 ) + y (sin θ 2 + λ sin θ 3 ) − ( p 2 + λp 3 ) = 0 P
(cosθ 2 + λ cosθ 3 )
∴ Slope of AD = − (h,k)
(sin θ 2 + λ sin θ 3 )
cosθ1
and Slope of BC = −
sin θ1 X′ X
O Q
A Y′

k
∴ OQ = h −
and OP = k − mh
L3 =0 F m
E
L2 =0 1 1  k
O Area of triangle OPQ = . OQ. OP = . h −  (k − mh )
2 2  m
B 1 k2
⇒ = 2hk − mh 2 − 
D C 2 m
L1 =0
mh 2 k 2
Since, AD ⊥ BC ⇒ A (m ) = hk − − …(ii)
2 2m
∴ Slope of BC × slope of AD = − 1 dA
To minimise, =0
cosθ1  (cosθ 2 + λ cosθ 3 )  dm
⇒ − × − = −1
sin θ1  (sin θ 2 + λ sin θ 3 )  h2 k2 k
⇒ 0− + =0 ⇒m=±
⇒ cosθ1 cosθ 2 + λ cosθ 3 cosθ1 2 2m 2 h
= − sin θ1 sin θ 2 − λ sin θ 3 sin θ1 d 2A 2k 2
∴ = −
⇒ cos (θ1 − θ 2 ) + λ cos (θ 3 − θ1 ) = 0 dm 2 2m 3
cos (θ1 − θ 2 )
∴ λ=− d 2A h3
cos (θ 3 − θ1 ) = >0 (Qh > 0, k > 0)
dm 2 m = − k /h
k
Now from Eq. (i),
cos (θ1 − θ 2 ) k k
L2 − L3 = 0 Hence for m = − , A (m ) is minimum . Put m = − in Eq. (ii),
cos (θ 3 − θ1 ) h h
we get minimum area.
∴ L2 cos (θ 3 − θ1 ) = L3 cos (θ1 − θ 2 ) ...(ii)
hk hk
Similarly, we can obtain equation of altitude BE as ⇒ Minimum area of ∆OPQ = hk + + = 2hk
2 2
L3 cos (θ1 − θ 2 ) = L1 cos (θ 2 − θ 3 ) ...(iii)
From Eqs. (ii) and (iii), we get l Ex. 37 The distance between two parallel lines is unity. A
L1 cos (θ 2 − θ 3 ) = L2 cos (θ 3 − θ1 ) = L3 cos(θ1 − θ 2 )
point P lies between the lines at a distance a from one of
l Ex. 36 Let (h, k) be a fixed point, where h > 0, k > 0. them. Find the length of a side of an eqilateral triangle PQR,
vertex Q of which lies on one of the parallel lines and vertex
A straight line passing through this point cuts the positive
R lies on the other line.
direction of the co-ordinate axes at the points P and Q. Find
Sol. Let PQ = QR = RP = r
the minimum area of the triangle OPQ ,O being the origin.
and ∠ PQL = θ
Sol. Equation of any line passing through the fixed point (h, k )
then ∠ XQR = θ + 60 °
and having slope m can be taken as
Given PL = a and RN = 1 unit
y − k = m (x − h ) ...(i)
Chap 02 The Straight Lines 153

Now, let slope of AB is m


Y
C

R(r,θ+60°) Y

0
α

1=
y+
α

A(1,2)
x–
B
α
60°

2
α

y=
θ P(r,θ)

x–
X′ N X
(0,0) Q L X′ X
O
Y′ A'(x1,y1)

In ∆PQL Y′
PL a
sin θ = =
QP r Q OA and AB are equally inclined to normal of x − y + 1 = 0
then,
∴ a = r sinθ ...(i)
and in ∆QRN,  m1 − m2   m − m2 
  =− 
RN 1  1 + m1m2   1 + mm2 
r sin (θ + 60 ° ) = =
QR r  2 − (− 1)   m − (− 1) 
⇒   =− 
sin (θ + 60 ° ) = 1  1 + 2( − 1 )  1 + m (− 1)
(sin θ cos 60 ° + cosθ sin 60 ° ) = 1 ⇒ 3 (1 − m ) = m + 1
1 3  1
⇒ r  sin θ + cosθ = 1 or m=
2 2  2
1 a ∴ Equation of AB is
3 a2 
⇒ r  × + × 1 − 2  = 1 [from Eq. (i)] 1
2 r 2 r  y − 2 = (x − 1)
2

a
+
3
(r 2 − a 2 ) = 1 or x − 2y + 3 = 0
2 2 Now, solving x − 2y + 3 = 0
3 a and x −y =2
⇒ (r 2 − a 2 ) = 1 −
2 2 then, we get x = 7, y = 5
3 2 a2 i.e. B ≡ ( 7, 5 )
or (r − a ) = 1 +
2
−a
4 4 Q BC is parallel to OA
⇒ 3r 2 − 3a 2 = 4 + a 2 − 4a ∴ Equation of BC is
⇒ 3r 2 = 4 (a 2 − a + 1 ) y − 5 = m1 ( x − 7 )
⇒ y − 5 = 2 (x − 7)
2
∴ r= (a 2 − a + 1 ) i.e. 2x − y − 9 = 0
3
Aliter :
l Ex. 38 A ray of light travelling along the line OA
If image of A (1, 2 ) with respect to line mirror x − y = 2 be
A′ ( x1, y1 ), then
(O being origin) is reflected by the line mirror x − y + 1 = 0, x1 − 1 y1 − 2 − 2 (1 − 2 − 2 )
is the point of incidence being A (1, 2 ) the reflected ray, = =
1 −1 1+1
travelling along AB is again reflected by the line mirror
or x1 = 4, y1 = − 1
x − y = 2 , the point of incidence being B. If this reflected ray
i.e. A′ ≡ ( 4, − 1 )
moves along BC, find the equation of the line BC. Q BC is parallel to OA
2−0
Sol. Since, slope of OA = = 2 = m1 (say) then equation of BC = equation of A′ C is
1−0
y − y1 = 2 ( x − x1 )
and slope of normal to x − y + 1 = 0 is ⇒ y + 1 = 2 (x − 4)
− 1 = m2 (say) or 2x − y − 9 = 0
154 Textbook of Coordinate Geometry

l Ex. 39 Consider two lines L1 ≡ x − y = 0 and L 2 ≡ x + y = 0 l Ex. 40 A rectangle PQRS has its side PQ parallel to the
and a moving point P ( x , y ). Let d (P , Li ), i =1, 2 represents the line y = mx and vertices P , Q and S on the lines y = a , x = b
distance of the point ‘P’ from Li . If point ‘P’ moves in certain and x = − b respectively. Find the locus of the vertex R.
2 Sol. Q PQ is parallel to y = mx
region ‘R’ in such a way Σ d (P , Li ) ∈[ 2 , 4 ].
i =1 ∴ Equation of PQ is y = mx + λ ...(i)
Find the area of region ‘R’. Q Diagonals bisect to each other
2
Sol. Q Given Σ d (P , Li ) ∈ [2, 4] Y
i =1 R(h,k)
2
⇒ 2≤ Σ d (P , Li ) ≤ 4
i =1

⇒ 2 ≤ d ( P , L1 ) + d ( P , L2 ) ≤ 4
(–b,λ2)S Q(b,λ1)
| x − y| | x + y|
⇒ 2≤ + ≤4
2 2
⇒ 2 2 ≤ | x − y| + | x + y| ≤ 4 2 mx
y=
Case I : If x − y > 0, x + y > 0 P(–h,a)
X′ X
then 2 2 ≤ (x − y ) + (x + y ) ≤ 4 2 O
⇒ 2 2 ≤ 2 x ≤ 4 2 or 2 ≤ x ≤ 2 2
Y′
Case II : If x − y > 0, x + y < 0
then 2 2 ≤ (x − y ) − (x + y ) ≤ 4 2 ∴ x-coordinate of P is −h.
⇒ 2 2 ≤ − 2y ≤ 4 2 Suppose y-coordinate of Q and S are λ 1 and λ 2 respectively.
Q Eq. (i) pass through P ( − h, a )
or − 2 ≥ y ≥ − 2 2 or − 2 2 ≤ y ≤ − 2
then a = − mh + λ
Case III : If x − y < 0, x + y > 0 ∴ λ = a + mh
then 2 2 ≤ − (x − y ) + (x + y ) ≤ 4 2 ⇒ y = mx + a + mh
⇒ 2 2 ≤ 2y ≤ 4 2 or 2 ≤ y ≤ 2 2 Q also lie on it, then
Case IV : If x − y < 0, x + y < 0 λ 1 = mb + a + mh
then 2 2 ≤ − (x − y ) − (x + y ) ≤ 4 2 ∴ Q ≡ (b, mb + a + mh )
⇒ 2 2 ≤ − 2x ≤ 4 2 Also, slope of PQ × slope of QR = − 1
(k − (mb + a + mh ))
Y m× = −1
(h − b )
0
y=

⇒ mk − m 2b − am − m 2h = − h + b
x–

B 2√2 A
⇒ (m 2 − 1 ) h − mk + b (m 2 + 1 ) + am = 0
B' √2 A' ∴ Locus of R is
(m 2 − 1 ) x − my + b (m 2 + 1 ) + am = 0
X′ X
–2√2 –√2 √2 2√2

C'
–√2 D'
l Ex. 41. For points P ≡ ( x 1 , y 1 ) and Q ≡ ( x 2 , y 2 ) of the
coordinate plane, a new distance d (P , Q ) is defined by
d (P , Q ) = | x 1 − x 2 | + | y 1 − y 2 |. Let O ≡ (0, 0 ) and A ≡ (3 , 2 ).
x+

C –2√2 D
y=

Prove that the set of the points in the first quadrant which
0

Y′ are equidistant (with respect to the new distance) from O and


or − 2 ≥ x ≥ −2 2 A consists of the union of a line segment of finite length and
or −2 2 ≤ x ≤ − 2
an infinite ray. Sketch this set in a labelled diagram.
Sol. Let P ( x, y ) be any point in the first quadrant, we have
Combining all cases, we get
x > 0, y > 0
x ∈ [ − 2 2, − 2 ] ∪ [ 2, 2 2 ]
d (P , Q ) = | x − 0| + | y − 0| = | x | + | y | = x + y
and y ∈ [ − 2 2, − 2 ] ∪ [ 2, 2 2 ] and d (P , A ) = | x − 3| + | y − 2|
Hence, area of the required region Given, d (P , Q ) = d (P , A )
= ( 4 2 ) 2 − (2 2 ) 2 x + y = | x − 3| + |y − 2| ...(i)
= 32 − 8 = 24 sq units.
Chap 02 The Straight Lines 155

Case I : 0 ≤ x < 3, 0 ≤ y < 2 l Ex. 42. A line through the variable point A (k +1, 2k )
then Eq. (i) becomes
meets the lines 7 x + y − 16 = 0, 5 x − y − 8 = 0, x − 5y + 8 = 0
x + y =3 − x + 2 −y
at B, C , D respectively, prove that AC , AB, AD are in HP.
or x + y =5/2
Sol. Given lines are
Case II : 0 ≤ x < 3, y ≥ 2
7 x + y − 16 = 0 . ..(i)
then Eq. (i) becomes
5x − y − 8 = 0 ...(ii)
x + y =3 − x + y −2
x − 5y + 8 = 0 ...(iii)
1
or x= Let the equation of line passing through A (k + 1, 2k ) making
2
an angle θ with the + ve direction of X -axis, be
Case III : x ≥ 3, 0 ≤ y < 2 x − (k + 1 ) y − 2k
then Eq. (i) becomes = = r1, r2, r3 (if AB = r1, AC = r2, AD = r3 )
cos θ sin θ
x + y = x −3 + 2 −y
∴ B ≡ [(k + 1 ) + r1 cos θ, 2k + r1 sin θ ]
1
⇒ y =− (Impossible) C ≡ [(k + 1 ) + r2 cos θ, 2k + r2 sin θ ]
2
D ≡ [(k + 1 ) + r3 cos θ, 2k + r3 sin θ ]
Case IV : x ≥ 3, y ≥ 2
Points B, C , D satisfying Eqs. (i), (ii) and (iii) respectively
x + y = x −3 + y −2
9 (1 − k )
⇒ 0 = −5 (Impossible) then r1 =
7 cos θ + sin θ
Combining all cases, then
3 (1 − k )
x + y = 5 / 2; 0 ≤ x < 3, 0 ≤ y < 2 r2 =
1 5 cos θ − sin θ
and x = ; 0 ≤ x < 3, y ≥ 2 9 (1 − k )
2 and r3 =
Y 5 sin θ − cos θ
1 1 (5 cos θ − sin θ ) (5 sin θ − cos θ )
x=1 ∴ + = +
3 2 r2 r3 3 (1 − k ) 9 (1 − k )
15 cos θ − 3 sin θ + 5 sin θ − cos θ
2 =
9 (1 − k )
x+y =5
1 2 14 cos θ + 2 sin θ 2
= =
X′ X 9 (1 − k ) r1
O 1 2 3
Hence r2, r1, r3 are in HP
Y′ i.e. AC , AB, AD are in HP.
The labelled diagram is given in adjoining figure.
#L The Straight Lines Exercise 1 :
Single Option Correct Type Questions
n This section contains 30 multiple choice questions. 8. If the distance of any point ( x , y ) from the origin is
Each question has four choices (a), (b), (c), (d) out of which defined as d ( x , y ) = max {| x |, | y | }, d ( x , y ) = a non-zero
ONLY ONE is correct. constant, then the locus is
1. The straight line y = x − 2 rotates about a point where it (a) a circle (b) a straight line
cuts X-axis and becomes perpendicular on the straight (c) a square (d) a triangle
line ax + by + c = 0, then its equation is 9. If p 1 , p 2 , p 3 be the perpendiculars from the points
(a) ax + by + 2a = 0 (b) ay − bx + 2b = 0
(c) ax + by + 2b = 0 (d) None of these
(m 2 , 2m ), (mm ′ , m + m ′ ) and (m ′ 2 , 2m ′ ) respectively on
sin 2 α
2 2 1 2m the line x cos α + y sin α + = 0, then p 1 , p 2 , p 3
2. If + + = , then orthocentre of the cos α
1! 9 ! 3! 7 ! 5! 5! n !
are in
triangle having sides x − y + 1 = 0, x + y + 3 = 0 and
(a) AP (b) GP
2x + 5y − 2 = 0 is (c) HP (d) None of these
(a) (2m − 2n, m − n ) (b) (2m − 2n, n − m )
(c) (2m − n, m + n ) (d) (2m − n, m − n ) 10. ABCD is a square whose vertices A, B, C and D are (0, 0),
(2, 0), (2, 2) and (0, 2) respectively. This square is rotated
3. If f ( x + y ) = f ( x ) f (y ) ∀ x , y ∈ R and f (1) = 2 , then area in the xy plane with an angle of 30° in anti-clockwise
enclosed by 3| x | + 2| y | ≤ 8 is direction about an axis passing through the vertex A the
1 equation of the diagonal BD of this rotated square is ……
(a) f ( 4 ) sq units (b) f (6 ) sq units
2 . If E is the centre of the square, the equation of the
1 1
(c) f (6 ) sq units (d) f (5 ) sq units circumcircle of the triangle ABE is
3 3
(a) 3 x + (1 − 3 )y = 3 , x 2 + y 2 = 4
4. The graph of the function (b) (1 + 3 ) x − (1 − 2 )y = 2, x 2 + y 2 = 9
y = cos x cos( x + 2) − cos 2 ( x + 1) is
(c) (2 − 3 ) x + y = 2( 3 − 1 ), x 2 + y 2 − x 3 − y = 0
(a) a straight line passing through ( 0, − sin 2 1 ) with slope 2
(d) None of the above
(b) a straight line passing through (0, 0)
(c) a parabola with vertex (1, − sin 2 1 ) 11. The point (4, 1) undergoes the following three successive
π 
(d) a straight line passing through the point  , − sin 2 1 are transformations
2 
parallel to the X-axis. (i) reflection about the line y = x − 1.
5. A line passing through the point (2, 2) and the axes (ii) translation through a distance 1 unit along the positive
direction of X-axis.
enclose an area λ. The intercepts on the axes made by π
the line are given by the two roots of (iii) rotation through an angle about the origin in the
anti-clockwise direction 4
(a) x 2 − 2| λ | x + | λ | = 0 (b) x 2 + | λ | x + 2| λ | = 0
Then, the coordinates of the final point are
(c) x 2 − | λ | x + 2| λ | = 0 (d) None of these  7 7
(a) ( 4, 3 ) (b)  , 
6. The set of value of ‘b’ for which the origin and the point  2 2
(1, 1) lie on the same side of the straight line (c) ( 0, 3 2 ) (d) (3, 4)
a 2 x + aby + 1 = 0 ∀a ∈ R, b > 0 are 12. If the square ABCD, where A(0, 0), B(2, 0), C(2, 2) and
(a) b ∈(2, 4 ) (b) b ∈( 0, 2 )
(c) b ∈[ 0, 2 ] (d) None of these D(0, 2) undergoes the following three transformations
successively
7. Line L has intercepts a and b on the co-ordinates axes, (i) f1( x, y ) → (y , x )
when the axes are rotated through a given angle; (ii) f 2( x, y ) → ( x + 3y , y )
keeping the origin fixed, the same line has intercepts p x −y x + y
and q, then (iii) f 3( x, y ) →  , 
 2 2 
1 1 1 1
(a) a 2 + b 2 = p 2 + q 2 (b) + = + then the final figure is a
a2 b2 p2 q2
(a) square (b) parallelogram
1 1 1 1
(c) a 2 + p 2 = b 2 + q 2 (d) 2 + 2 = 2 + 2 (c) rhombus (d) None of these
a p b q
Chap 02 The Straight Lines 157

13. The line x + y = a meets the axes of x and y at A and B 20. Suppose that a ray of light leaves the point (3, 4), reflects
respectively. A triangle AMN is inscribed in the triangle off the Y-axis towards the X-axis, reflects off the X-axis,
OAB, O being the origin, with right angle at N, M and N and finally arrives at the point (8, 2). The value of x is
lie respectively on OB and AB. If the area of the triangle Y
3 AN
AMN is of the area of the triangle OAB, then is (3, 4)
8 BN
equal to
(a) 1 (b) 2 (c) 3 (d) 4 (0, y) (8, 2)
14. If P (1, 0), Q ( −1, 0) and R (2, 0) are three given points, then
the locus of point S satisfying the relation
(SQ ) 2 + (SR ) 2 = 2(SP ) 2 is O (x, 0)
X

(a) a straight line parallel to X -axis


(b) a circle through the origin 1 1 2 1
(a) 4 (b) 4 (c) 4 (d) 5
(c) a circle with centre at the origin 2 3 3 3
(d) a straight line parallel to Y -axis 21. m, n are two integers with 0 < n < m. A is the point (m, n )
sin α cos α 
15. If A  − 1, − 1 and B(1, 1), α ∈ [− π, π ] are two on the cartessian plane. B is the reflection of A in the
 3 2  line y = x . C is the reflection of B in the Y -axis, D is the
points on the same side of the line 3x − 2y + 1 = 0, then α reflection of C in the X -axis and E is the reflection of D
belongs to the interval in the Y -axis. The area of the pentagon ABCDE is
 3π   π  (a) 2m(m + n ) (b) m(m + 3n )
(a)  − π , − ∪  , π (b) [ −π , π ]
 4   4  (c) m(2m + 3n ) (d) 2m(m + 3n )
(c) φ (d) None of these 22. A straight line L with negative slope passes through the
16. The line x + y = 1 meets X-axis at A and Y-axis at B, P is point (8, 2) and cuts the positive coordinates axes at
the mid-point of AB. P1 is the foot of the perpendicular points P and Q. As L varies, the absolute minimum value
from P to OA; M 1 is that of P1 from OP ; P2 is that of M 1 of OP + OQ is (O is origin)
from OA ; M 2 is that of P2 from OP ; P3 is that of M 2 (a) 10 (b) 18 (c) 16 (d) 12
from OA and so on. If Pn denotes the nth foot of the 23. Drawn from origin are two mutually perpendicular lines
perpendicular on OA form M n − 1 , then OPn is equal to forming an isosceles triangle together with the straight
1 1 line 2x + y = a, then the area of this triangle is
(a) (b)
2n 2n a2 a2
(c) 2n − 1 (d) 2n + 3 (a) sq units (b) sq units
2 3
17. The line x = c cuts the triangle with corners (0, 0); (1, 1) (c)
a2
sq units (d) None of these
and (9, 1) into two regions. For the area of the two 5
regions to be the same, then c must be equal to 24. The number of integral values of m for which the
5
(a) (b) 3 x-coordinate of the point of intersection of the lines
2
3x + 4y = 9 and y = mx + 1 is also an integer is
7 (a) 2 (b) 0 (c) 4 (d) 1
(c) (d) 3 or 15
2
25. A ray of light coming from the point (1, 2) is reflected at
18. If the straight lines x + 2y = 9, 3x − 5y = 5 and ax + by = 1 a point A on the X -axis and then passes through the
are concurrent, then the straight line 5x + 2y = 1, passes point (5, 3). The coordinates of the point A are
through the point  13  5 
(a) (a, − b ) (b) ( −a, b ) (a)  , 0 (b)  , 0
5   13 
(c) (a, b ) (d) ( −a, − b )
(c) ( −7, 0 ) (d) None of these
19. The ends of the base of the isosceles triangle are at (2, 0)
26. Consider the family of lines
and (0, 1) and the equation of one side is x = 2, then the
5x + 3y − 2 + λ(3x − y − 4 ) = 0 and
orthocentre of the triangle is
x − y + 1 + µ(2x − y − 2) = 0. Equation of straight line
 3 3 5 
(a)  , 
 4 2
(b)  , 1
4  that belong to both families is ax + by − 7 = 0, then
a + b is
3  4 7 
(c)  , 1 (d)  ,  (a) 1 (b) 3 (c) 5 (d) 7
4   3 12
158 Textbook of Coordinate Geometry

27. In ∆ABC equation of the right bisectors of the sides AB 30. In the adjacent figure combined equation of the incident
and AC are x + y = 0 and x − y = 0 respectively. If and refracted ray is
A ≡ (5, 7 ), then equation of side BC is Y
(a) 7y = 5 x (b) 5x = y
(c) 5y = 7 x (d) 5y = x
28. Two particles start from the point (2, −1), one moving 2
units along the line x + y = 1 and the other 5 units along
the line x − 2y = 4. If the particles move towards
increasing y, then their new positions are 60°
(2, 0)
(a) (2 − 2, 2 − 1 ); (2 2 + 2, 5 − 1 ) X¢
O P
X

(b) (2 2 + 2, 5 − 1 ); (2 2, 2 + 1 ) 30°
(c) (2 + 2, 2 + 1 ); (2 2 + 2, 5 + 1 ) Y¢

(d) (2 − 2, 5 − 1 ); ( 2 − 1, 2 2 + 2 )
4
(a) ( x − 2 ) 2 + y 2 + (x − 2 )y = 0
29. Let P be (5, 3) and a point R on y = x and Q on the X -axis 3
be such that PQ + QR + RP is minimum, then the (b) ( x − 2 ) 2 + y 2 −
4
(x − 2 )y = 0
coordinates of Q are 3
 17   17  y
(a)  , 0 (b)  , 0 (c) ( x − 2 ) 2 + y 2 + (x − 2) = 0
8  4  3
 17  y
(c)  , 0 (d) (17, 0 ) (d) ( x − 2 ) 2 + y 2 − (x − 2) = 0
2  3

#L The Straight Lines Exercise 2 :


More than One Correct Option Type Questions
n
The section contains 15 multiple choice questions. 34. If the lines x − 2y − 6 = 0, 3x + y − 4 = 0 and
Each question has four choices (a), (b), (c), and (d) out of λx + 4y + λ 2 = 0 are concurrent, then
which MORE THAN ONE may be correct.
(a) λ = 2 (b) λ = − 3 (c) λ = 4 (d) λ = − 4
x y
31. The point of intersection of the lines + = 1 and 35. Equation of a straight line passing through the point of
a b
intersection of x − y + 1 = 0 and 3x + y − 5 = 0 are
x y
+ = 1 lies on perpendicular to one of them is
b a (a) x + y + 3 = 0 (b) x + y − 3 = 0
(a) x − y = 0 (c) x − 3y − 5 = 0 (d) x − 3y + 5 = 0
(b) ( x + y )(a + b ) = 2ab
(c) (lx + my )(a + b ) = (l + m )ab 36. If one vertex of an equilateral triangle of side a lies at the
(d) (lx − my )(a + b ) = (l − m )ab origin and the other lies on the line x − 3y = 0, the
32. The equations (b − c )x + (c − a )y + a − b = 0 and coordinates of the third vertex are
 3a −a   − 3a a 
(b 3 − c 3 )x + (c 3 − a 3 )y + a 3 − b 3 = 0 will represent the (a) ( 0, a ) (b)  ,  (c) ( 0, − a ) (d)  , 
 2 2  2 2
same line, if
(a) b = c (b) c = a 37. If the line ax + by + c = 0, bx + cy + a = 0 and
(c) a = b (d) a + b + c = 0 cx + ay + b = 0 are concurrent (a + b + c ≠ 0) then
33. The area of a triangle is 5. Two of its vertices are (2, 1) (a) a 3 + b 3 + c 3 − 3abc = 0 (b) a = b
and (3, −2). The third vertex lies on y = x + 3. The (c) a = b = c (d) a 2 + b 2 + c 2 − bc − ca − ab = 0
coordinates of the third vertex cannot be
38. A(1, 3) and C(7, 5) are two opposite vertices of a square.
 −3 3   3 −3 
(a)  ,  (b)  ,  The equation of a side through A is
 2 2 4 2 
(a) x + 2y − 7 = 0 (b) x − 2y + 5 = 0
 7 13  −1 11
(c)  ,  (d)  ,  (c) 2 x + y − 5 = 0 (d) 2 x − y + 1 = 0
2 2   4 4
Chap 02 The Straight Lines 159

39. If 6a 2 − 3b 2 − c 2 + 7ab − ac + 4bc = 0, then the family of 43. Consider the equation y − y 1 = m( x − x 1 ). If m and x 1
lines ax + by + c = 0 is concurrent at are fixed and different lines are drawn for different
(a) ( −2, − 3 ) (b) (3, − 1 ) values of y 1 , then
(c) (2, 3 ) (d) ( −3, 1 ) (a) the lines will pass through a fixed point
(b) there will be a set of parallel lines
40. Consider the straight lines x + 2y + 4 = 0 and
(c) all the lines intersect the line x = x1
4 x + 2y − 1 = 0. The line 6x + 6y + 7 = 0 is (d) all the lines will be parallel to the line y = x1
(a) bisector of the angle including origin
(b) bisector of acute angle 44. Let L 1 ≡ ax + by + a 3 b = 0 and L 2 ≡ bx − ay + b 3 a = 0
(c) bisector of obtuse angle be two straight lines. The equations of the bisectors of
(d) None of the above the angle formed by the foci whose equations are
41. Two roads are represented by the equations y − x = 6 λ 1 L 1 − λ 2 L 2 = 0 and λ 1 L 1 + λ 2 L 2 = 0, λ 1 and λ 2 being
non-zero real numbers, are given by
and x + y = 8. An inspection bungalow has to be so
(a) L1 = 0 (b) L2 = 0
constructed that it is at a distance of 100 from each of (c) λ 1L1 + λ 2L2 = 0 (d) λ 2L1 − λ 1L2 = 0
the roads. Possible location of the bungalow is given by
(a) (100 2 + 1, 7 ) (b) (1 − 100 2, 7 ) 45. The equation of the bisectors of the angles between the
x −3 y +5 x −3 y +5
(c) (1, 7 + 100 2 ) (d) (1, 7 − 100 2 ) two intersecting lines = and = are
cos θ sin θ cos φ sin φ
42. If (a, b ) be an end of a diagonal of a square and the other x −3 y +5 x −3 y +5
diagonal has the equation x − y = a, then another vertex = and = , then
cos α sin α β γ
of the square can be θ+φ
(a) (a − b, a ) (b) (a, 0 ) (a) α = (b) β = − sin α
2
(c) ( 0, − a ) (d) (a + b, b )
(c) γ = cosα (d) β = sin α

#L The Straight Lines Exercise 3 :


Paragraph Based Questions
n
The section contains 5 Paragraphs based upon each of (a) a line segment of finite length
the paragraphs 3 multiple choice questions have to be (b) a line of infinite length
answered. Each of these questions has four choices (a), (b), (c) a ray of finite length
(c), and (d) out of which ONLY ONE is correct. (d) a ray of infinite length

Paragraph I 48. Let T ( x , y ), such that T is equidistant from point O and C


(Q. Nos. 46 to 48) with respect to new distance and if T lie in first quadrant,
then T consists of the union of a line segment of finite
For points P ≡ ( x1 , y1 ) and Q = ( x 2 , y 2 ) of the coordinate length and an infinite ray whose labelled diagram is
plane, a new distance d ( P , Q ) is defined by
Y Y
d ( P , Q ) = | x1 − x 2 | + | y1 − y 2 |
Let O ≡ ( 0, 0), A ≡ (1, 2), B ≡ ( 2, 3) and C ≡ ( 4, 3) are four 3.5
3
fixed points on x- y plane.
(a) (b)
46. Let R( x , y ), such that R is equidistant from the point O 1/2
and A with respect to new distance and if 0 ≤ x < 1 and X X
O 1/2 3.5 O 3
0 ≤ y < 2, then R lie on a line segment whose equation is
(a) x + y = 3 (b) x + 2y = 3 Y Y
(c) 2 x + y = 3 (d) 2 x + 2y = 3
3.5
47. Let S( x , y ), such that S is equidistant from points O and B 2.5
(c) (d)
with respect to new distance and if x ≥ 2 and 0 ≤ y < 3,
1/2
then locus of S is X
O 1/2 3.5 X
O 2.5
160 Textbook of Coordinate Geometry

Paragraph II How to find the image or reflection of a curve?


(Q. Nos. 49 to 51) P

In a triangle ABC, if the equation of sides AB, BC and CA are S=0


2x − y + 4 = 0, x − 2 y − 1 = 0 and x + 3 y − 3 = 0 respectively.
49. Tangent of internal angle A is equal to L=0
(a) −7 (b) −3
1 M
(c) (d) 7
2
S′=0
50. The equation of external bisector of angle B is
(a) x − y − 1 = 0 (b) x − y + 1 = 0
(c) x + y − 5 = 0 (d) x + y + 5 = 0 Q

51. The image of point B w.r.t the side CA is Let the given curve be S : f ( x, y ) = 0 and line mirror
 3 26  3 26 L : ax + by + c = 0. We take a point P on the given curve in
(a)  − ,  (b)  − , − 
 5 5  5 5 parametric form. Suppose Q be the image or reflection of point
P about line mirror L = 0, which again contains the same
 3 26  3 26
(c)  , −  (d)  ,  parameter. Let Q ≡ ( φ( t ), ψ ( t )) , where t is parameter. Now let
5 5 5 5 
x = φ( t ) and y = ψ( t )
Paragraph III Eliminating t, we get the equation of the reflected curve S ′.
(Q. Nos. 52 to 54) 55. The image of the line 3x − y = 2 in the line y = x − 1 is
 2 −2 (a) x + 3y = 2 (b) 3 x + y = 2
A (1, 3) and C  − ,  are the vertices of a triangle ABC and
 5 5 (c) x − 3y = 2 (d) x + y = 2
the equation of the angle bisector of ∠ABC is x + y = 2. 56. The image of the circle x 2 + y 2 = 4 in the line x + y = 2
52. Equation of BC is is
(a) 7 x + 3y − 4 = 0 (b) 7 x + 3y + 4 = 0 (a) x 2 + y 2 − 2 x − 2y = 0 (b) x 2 + y 2 − 4 x − 4y + 6 = 0
(c) 7 x − 3y + 4 = 0 (d) 7 x − 3y − 4 = 0 (c) x 2 + y 2 − 2 x − 2y + 2 = 0 (d) x 2 + y 2 − 4 x − 4y + 4 = 0
53. Coordinates of vertex B are 57. The image of the parabola x 2 = 4y in the line x + y = a is
 3 17  17 3 
(a)  ,  (b)  ,  (a) ( x − a ) 2 = 4(a − y ) (b) (y − a ) 2 = 4 (a − x )
 10 10  10 10
 5 9  9 5 (c) ( x − a ) 2 = 4(a + y ) (d) (y − a ) 2 = 4(a + x )
(c)  − ,  (d)  , − 
 2 2  2 2
Paragraph V
54. Equation of AB is (Q. Nos. 58 to 60)
(a) 3 x + 7y = 24 In a ∆ABC, the equation of the side BC is 2 x − y = 3 and its
(b) 3 x + 7y + 24 = 0 circumcentre and orhtocentre are (2, 4 ) and (1, 2 ) respectively.
(c) 13 x + 7y + 8 = 0
(d) 13 x − 7y + 8 = 0 58. Circumradius of ∆ABC is
61 51 41 43
Paragraph IV (a) (b) (c) (d)
5 5 5 5
(Q. Nos. 55 to 57)
59. sin B ⋅ sin C =
Let S ′ = 0 be the image or reflection of the curve S = 0 about 9 9 9 9
(a) (b) (c) (d)
line mirror L = 0. Suppose P be any point on the curve S = 0 2 61 4 61 61 5 61
and Q be the image or reflection about the line mirror L = 0,
then Q will lie on S ′ = 0.
60. The distance of orthocentre from vertex A is
1 6 3 2
(a) (b) (c) (d)
5 5 5 5
Chap 02 The Straight Lines 161

#L The Straight Lines Exercise 4 :


Single Integer Answer Type Questions
n The section contains 10 questions. The answer to eaeh 66. In a plane there are two families of lines : y = x + n,
question is a single digit integer, ranging from 0 to 9 y = − x + n, where n ∈ {0, 1, 2, 3, 4 }. The number of squares
(both inclusive). of the diagonal of length 2 formed by these lines is
61. The number of possible straight lines passing through 67. Given A(0, 0) and B ( x , y ) with x ∈(0, 1) and y > 0. Let the
(2, 3) and forming a triangle with the coordinate axes, slope of line AB be m1 . Point C lies on line x = 1 such
whose area is 12 sq units, is that the slope of BC is equal to m 2 , where 0 < m 2 < m1 .
62. The portion of the line ax + 3y − 1 = 0, intercepted If the area of triangle ABC can be expressed as
(m1 − m 2 ) f ( x ) and the largest possible value of f ( x ) is
between the lines ax + y + 1 = 0 and x + 3y = 0 subtend a
1
right angle at origin, then the value of | a | is λ, then the value of is
λ
63. Let ABC be a triangle and A ≡ (1, 2), y = x be the
68. If ( λ , λ + 1) is an interior point of ∆ABC, where A ≡ (0, 3),
perpendicular bisector of AB and x − 2y + 1 = 0 be the B ≡ ( −2, 0) and C ≡ (6, 1), then the number of integral
angle bisector of ∠C. If the equation of BC is given by values of λ is
ax + by − 5 = 0, then the value of a − 2b is
69. For all real values of a and b, lines
64. A lattice point in a plane is a point for which both (2a + b )x + (a + 3b )y + (b − 3a ) = 0 and λx + 2y + 6 = 0 and
coordinates are integers. If n be the number of lattice λx + 2y + 6 = 0 are concurrent, then the value of | λ | is
points inside the triangle whose sides are x = 0, y = 0 and
9 x + 223y = 2007, then tens place digit in n is 70. If from point (4, 4) perpendiculars to the straight lines
3x + 4y + 5 = 0 and y = mx + 7 meet at Q and R and area
65. The number of triangles that the four lines y = x + 3, of triangle PQR is maximum, then the value of 3m is
y = 2x + 3, y = 3x + 2 and y + x = 3 form is

#L The Straight Lines Exercise 5 :


Matching Type Questions
n
The section contains 5 questions. Questions 1, 2 and 3 72. Match the Columns
have four statement (A, B, C and D) given in Column I
Column I Column II
and four statements (p, q, r and s) in Column II and
questions 74 and 75 have three statements (A, B and C) (A) Lines x − 2y − 6 = 0, 3 x + y − 4 = 0 and (p) 2
given in Column I and five statements (p, q, r, s and t) in λx + 4y + λ2 = 0 are concurrent, then the
Column II. Any given statement in Column I can have value of | λ| is
correct matching with one or more statement (s) given in (B) The variable straight lines (q) 3
Column II. 3 x (a + 1) + 4y (a − 1) − 3 (a − 1) = 0 for
different value of ‘a’ passes through a fixed
71. Let L 1 , L 2 , L 3 be three straight lines a plane and n be the point ( p , q ) if λ = p − q, then the value of 4 | λ |
number of circles touching all the lines. (C) λ (r) 4
If the line x + y − 1 − = 0 passing through
2
Column I Column II
the intersection of x − y + 1 = 0 and
(A) The lines are concurrent, then n + 1 is a (p) natural number 3 x + y − 5 = 0, is perpendicular to one of
them, then the value of | λ + 1| is
(B) The lines are parallel, then 2n + 3 is a (q) prime number
(D) If the line y − x − 1 + λ = 0 is equidistant from (s) 5
(C) Two lines are parallel, then n + 2 is a (r) composite number the points (1, − 2 ) and ( 3 , 4 ), then the value of
(D) The lines are neither concurrent nor (s) perfect number | λ| is
parallel, then n + 2 is a
162 Textbook of Coordinate Geometry

73. Consider the triangle formed by the lines y + 3x + 2 = 0, (s) 3


3y − 2x − 5 = 0 and 4y + x − 14 = 0 (t) 4

Column I Column II
75. Match the following
(A) If ( 0 , λ ) lies inside the triangle, then (p) 4
integral values are less than | 3λ| Column I Column II
(B) If (1, λ ) lies inside the triangle, then (q) 5 (A) If the distance of any point ( x , y ) (p) ( λ , µ ) lies on x = 3y
integral values are less than | 3λ| from origin is defined as
d ( x , y ) = 2 | x | + 3| y |. If perimeter
(C) If ( λ , 2 ) lies inside the triangle, then (r) 6 and area of figure bounded by
integral values of | 6λ| are d ( x , y ) = 6 are λ unit and µ sq units
respectively, then
(D) If ( λ , 7 / 2 ) lies inside the triangle, then (s) 7
integral value of | 6λ| are (B) If the vertices of a triangle are (6, 0), (q) ( λ , µ ) lies on
(0, 6) and (6, 6). If distance between x 2 − y 2 = 64
circumcentre and orthocentre and
74. Match the following distance between circumcentre and
Column I Column II centroid are λ unit and µ unit
respectively, then
(A) The area bounded by the curve (p) 0
max. {| x | , | y | } = 1 is (C) The ends of the hypotenuse of a (r) ( λ , µ ) lies on
right angled triangle are (6, 0) and x 2 + y 2 − 6 x − 6y = 0
(B) If the point (a , a ) lies between the lines (q) 1 (0, 6). If the third vertex is ( λ , µ ),
| x + y | = 6, then [| a | ] is (where [. ] then
denotes the greatest integer function)
(s) ( λ , µ ) lies on
(C) Number of integral values of b for which (r) 2 x 2 − 16y = 16
the origin and the point (1, 1) lie on the
same side of the st. line a 2 x + aby + 1 = 0 (t) ( λ , µ ) lies on
for all a ∈ R ~ { 0 } is x 2 − y 2 = 16

#L The Straight Lines Exercise 6 :


Statement I and II Type Questions
n
Directions (Q. Nos 76 to 83) are Assertion-Reason type 78. Statement I If sum of algebraic distances from points
questions. Each of these question contains two statements. A(1, 2), B (2, 3), C(6, 1) is zero on the line ax + by + c = 0,
Statement I (Assertion) and then 2a + 3b + c = 0
Statement II (Reason) Statement II The centroid of the triangle is (3, 2)
Each of these questions has four alternative choices, only
79. Statement I Let A ≡ (0, 1) and B ≡ (2 , 0) and P be a point
one of which is the correct answer.
You have to select the correct choice. on the line 4 x + 3y + 9 = 0, then the co-ordinates of P
 12 17 
(a) Statement I is true, statement II is true; statement II such that | PA − PB | is maximum is  − ,  .
is a correct explanation for statement I  5 5
(b) Statement I is true, statement II is true; statement II Statement II | PA − PB | ≤ | AB |
is not a correct explanation for statement I
(c) Statement I is true, statement II is false
80. Statement I The incentre of a triangle formed by the
 π  π
(d) Statement I is false, statement II is true line x cos   + y sin   = π,
9 9
76. Statement I The lines x (a + 2b ) + y(a + 3b ) = a + b are
concurrent at the point (2, − 1)  8π   8π 
x cos   + y sin  
9 9
Statement II The lines x + y − 1 = 0 and 2x + 3y − 1 = 0
intersect at the point (2, − 1)  13π   13π 
= π and x cos   + y sin   = π is (0, 0).
 9   9 
77. Statement I The points (3, 2) and (1, 4) lie on opposite
side of the line 3x − 2y − 1 = 0 Statement II Any point equidistant from the given
Statement II The algebraic perpendicular distance three non-concurrent straight lines in the plane is the
from the given point to the line have opposite sign. incentre of the triangle.
Chap 02 The Straight Lines 163

81. Statement I Reflection of the point (5, 1) in the line Statement II The image of point A with respect to
x + y = 0 is ( −1 , − 5). 5x + 5y + 6 = 0 lies on the side BC of the triangle.
Statement II Reflection of a point P(α, β ) in the line 83. Statement I If the point (2a − 5, a 2 ) is on the same side of
 α + α′ β + β′ the line x + y − 3 = 0 as that of the origin, then a ∈(2, 4 ).
ax + by + c = 0 is Q (α ′ , β ′ ), if  ,  lies on
 2 2  Statement II The point ( x 1 , y 1 ) and ( x 2 , y 2 ) lie on the
the line.
same or opposite sides of the line ax + by + c = 0, as
82. Statement I The internal angle bisector of angle C of a ax 1 + by 1 + c and ax 2 + by 2 + c have the same or
triangle ABC with sides AB, AC and BC as y = 0, opposite signs.
3x + 2y = 0, and 2x + 3y + 6 = 0, respectively, is
5x + 5y + 6 = 0.

The Straight Lines Exercise 7 :


Subjective Type Questions
n In this section, there are 15 subjective questions. pass through a fixed point for all θ. What are the
84. If A ( x 1 , y 1 ) , B ( x 2 , y 2 ) and C ( x 3 , y 3 ) are the vertices coordinates of this fixed point and its reflection in the
of a triangle, then show that the equation of the line line x + y = 2 ? Prove that all lines through reflection
joining A and the circumcentre is given by point can be represented by equation

 x y 1  x y 1 (2 cos θ + 3 sin θ) x + (3 cos θ − 5 sin θ) y


= ( 2 − 1) (5 cos θ − 2 sin θ)
(sin 2 B )x 1 y 1 1 + (sin 2 C )x 1 y 1 1 = 0
   
90. P is any point on the line x − a = 0. If A is the point (a , 0)
x 2 y 2 1 x 3 y 3 1
and PQ , the bisector of the angle OPA, meets the X-axis
85. Find the coordinates of the point at unit distance from in Q. Prove that the locus of the foot of the
the lines perpendicular from Q on OP is
3x − 4y + 1 = 0, 8x + 6y + 1 = 0 . ( x − a) 2 ( x 2 + y 2 ) = a 2y 2 .
86. A variable line makes intercepts on the coordinate axes, 91. Having given the bases and the sum of the areas of a
the sum of whose squares is constant and equal to k 2 . number of triangles is constant, which have a common
Show that the locus of the foot of the perpendicular vertex. Show that the locus of this vertex is a straight
from the origin to this line is line.
( x 2 + y 2 ) 2 ( x −2 + y −2 ) = k 2 .
92. A (3, 0) and B (6, 0) are two fixed points and U (α, β ) is a
87. A variable line intersects n lines variable point on the plane. AU and BU meet the y-axis
y = mx , (m = 1 , 2 , 3 , ..., n ) in the points at C and D respectively and AD meets OU at V. Prove
A 1 , A 2 , A 3 , ...., A n respectively. that CV passes through (2, 0) for any position of U in the
n 1
If Σ = c (constant). Show that line passes through plane.
p = 1 OA p
93. A variable line is drawn through O to cut two fixed
a fixed point. Find the coordinates of this fixed point straight lines L 1 and L 2 in R and S. A point P is chosen
(O being origin). m +n m n
on the variable line such that = + . Show
88. Given n straight lines and a fixed point O. A straight line OP OR OS
is drawn through O meeting these lines in the points that the locus of P is a straight line passing through the
R1 , R 2 , R 3 , ...., Rn and a point R is taken on it such that point of intersection of L 1 and L 2 .
n n 1 94. A line through A ( − 5, − 4 ) meets the lines
= Σ
OR r = 1 ORr x + 3y + 2 = 0, 2x + y + 4 = 0 and x − y − 5 = 0 at the
points B, C and D respectively, if
Prove that the locus of R is a straight line. 2 2 2
 15   10   6 
89. Prove that all lines represented by the equation   +  = 
 AB   AC   AD 
(2 cos θ + 3 sin θ) x + (3 cos θ − 5 sin θ) y
find the equation of the line.
= 5 cos θ − 2 sin θ
164 Textbook of Coordinate Geometry

95. Two fixed straight lines X -axis and y = mx are cut by a  1


97. Let O (0, 0), A (2 , 0) and B 1,
 be the vertices of a
variable line in the points A (a , 0) and B (b, mb )  3
triangle. Let R be the region consisting of all those
respectively. P and Q are the feet of the perpendiculars
drawn from A and B upon the lines y = mx and X -axis. points P inside ∆OAB which satisfy
Show that, if AB passes through a fixed point (h, k ), then d ( P, OA ) ≤ min {d ( P, OB ), d ( P, AB )}
PQ will also pass through a fixed point. Find the fixed
where d denotes the distance from the point to the
point.
corresponding line. Sketch the region R and find its area.
98. Two triangles ABC and PQR are such that the
96. Find the equation of straight lines passing through point perpendiculars from A to QR, B to RP and C to PQ are
(2 , 3) and having an intercept of length 2 units between
concurrent. Show that the perpendicular from P to BC , Q
the straight lines 2x + y = 3 , 2x + y = 5 .
to CA and R to AB are also concurrent.

#L The Straight Lines Exercise 8 :


Questions Asked in Previous 13 Year’s Exams
n This section contains questions asked in IIT-JEE, AIEEE, (a) Statement I is true, statement II is true; statement II is not
JEE Main & JEE Advanced from year 2005 to 2017. a correct explanation for statement I
(b) Statement I is true, statement II is true; statement II is not
99. The line parallel to the X-axis and passing through the a correct explanation for statement I
intersection of the lines ax + 2by + 3b = 0 and (c) Statement I is true, statement II is false
bx − 2ay − 3a = 0, where (a, b ) ≠ (0, 0) is [AIEEE 2005, 3M] (d) Statement I is false, statement II is true
3
(a) below the X -axis at a distance of
2
from it 103. Let P = ( −1, 0), Q = (0, 0) and R = (3, 3 3 ) be three point.
2 The equation of the bisector of the angle PQR is
(b) below the X -axis at a distance of from it [AIEEE 2007, 3M]
3
3 3
(c) above the X -axis at a distance of from it (a) x+y =0 (b) x + 3 y = 0
2 2
2 3
(d) above the X -axis at a distance of from it (c) 3 x + y = 0 (d) x + y =0
3 2
100. A straight line through the point A(3, 4 ) is such that its 104. Consider the lines given by
intercept between the axes is bisected at A. Its equation L 1 : x + 3y − 5 = 0
is [AIEEE 2006, 4.5M] L 2 : 3x − ky − 1 = 0
(a) x + y = 7 (b) 3 x − 4y + 7 = 0 L 3 : 5x + 2y − 12 = 0
(c) 4 x + 3y = 24 (d) 3 x + 4y = 25
Match the statements/Expressions in Column I with the
x
101. If (a, a 2 ) falls inside the angle made by the lines y = , statements/Expressions in Column II
2
x > 0 and y = 3x , x > 0, then a belong to [AIEEE 2006, 6M] Column I Column II
 1 k = −9
(a)  0,  (b) (3, ∞ ) (A) L1 , L 2 , L 3 are concurrent, if (p)
 2
(B) one of L1 , L 2 , L 3 is parallel to at (q) 6
1   1 k=−
(c)  , 3 (d)  −3, −  least one of the other two, if 5
2   2
(C) L1 , L 2 , L 3 form a triangle, if (r) 5
k=
102. Lines L 1 : y − x = 0 and L 2 : 2x + y = 0 intersect the line 6
L 3 : y + 2 = 0 at P and Q respectively. The bisector of the
(D) L1 , L 2 , L 3 do not form a triangle, if (s) k =5
acute angle between L 1 and L 2 intersects L 3 at R.
[IIT-JEE 2007, 3M] [IIT-JEE 2008, 6M]

Statement I The ratio PR : RQ equals 2 2 : 5 because 105. The perpendicular bisector of the line segment joining
P (1, 4) and Q (k , 3) has y-intercept −4. Then a possible
Statement II In any triangle, bisector of an angle
value of k is [AIEEE 2008, 3M]
divides the triangle into two similar triangles. (a) 1 (b) 2 (c) −2 (d) −4
Chap 02 The Straight Lines 165

106. The lines p ( p 2 + 1)x − y + q = 0 and 111. A ray of light along x + 3y = 3 gets reflected upon
( p + 1) x + ( p + 1)y + 2q = 0 are perpendicular to a
2 2 2
reaching X -axis, the equation of the reflected ray is
common line for [AIEEE 2009, 4M] [JEE Main 2013, 4M]
(a) exactly one values of p (b) exactly two values of p (a) y = x + 3 (b) 3y = x − 3
(c) more than two values of p (d) no value of p (c) y = 3 x − 3 (d) 3y = x − 1
x y
107. The line L given by + = 1 passes through the point 112. For a > b > c > 0, the distance between (1, 1) and the
5 b
point of intersection of the lines ax + by + c = 0 and
(13, 32). The line K is parallel to L and has the equation bx + ay + c = 0 is less than 2 2. Then
x y
+ = 1. Then the distance between L and K is [JEE Advanced 2013, 3M]
c 3 [AIEEE 2010, 4M]
(a) a + b − c > 0 (b) a − b + c < 0
17 (c) a − b + c > 0 (d) a + b − c < 0
(a) 17 (b)
15 113. Let PS be the median of the triangle with vertices P(2, 2),
23 23
(c) (d) Q(6, − 1) and R(7, 3). The equation of the line passing
17 15
through (1, − 1) and parallel to PS is [JEE Main 2014, 4M]
108. A straight line L through the point (3, − 2) is inclined at (a) 4 x + 7y + 3 = 0 (b) 2 x − 9y − 11 = 0
(c) 4 x − 7y − 11 = 0 (d) 2 x + 9y + 7 = 0
an angle 60° to the line 3x + y = 1. If L also intersects
the X -axis, then the equation of L is [IIT-JEE 2011, 3M] 114. Let a, b, c and d be non-zero numbers. If the point of
(a) y + 3 x + 2 − 3 3 = 0 (b) y − 3 x + 2 + 3 3 = 0 intersection of the lines 4ax + 2ay + c = 0 and
(c) 3y − x + 3 + 2 3 = 0 (d) 3y + x − 3 + 2 3 = 0 5bx + 2by + d = 0 lies in the fourth quadrant and is
equidistant from the two axes, then [JEE Main 2014, 4M]
109. The lines L 1 : y − x = 0 and L 2 : 2x + y = 0 intersect the (a) 3bc − 2ad = 0 (b) 3bc + 2ad = 0
line L 3 : y + 2 = 0 at P and Q respectively. The bisector of (c) 2bc − 3ad = 0 (d) 2bc + 3ad = 0
the acute angle between L 1 and L 2 intersects L 3 at R . 115. For a point P in the plane, let d 1 ( P ) and d 2 ( P ) be the
[AIEEE 2011, 4M] distance of the point P from the lines x − y = 0 and
Statement I : The ratio PR : RQ equals 2 2 : 5 x + y = 0 respectively. The area of the region R
consisting of all points P lying in the first quadrant of
Statement II : In any triangle, bisector of an angle
the plane and satisfying 2 ≤ d 1 ( P ) + d 2 ( P ) ≤ 4, is
divides the triangle into two similar triangles.
[JEE Advanced 2014, 3M]
(a) Statement I is true, statement II is true; statement II is not
a correct explanation for statement I. 116. The number of points, having both co-ordinates as
integers, that lie in the interior of the triangle with
(b) Statement I is true, statement II is false.
vertices (0, 0), (0, 41) and (41, 0) is
(c) Statement I is false, statement II is true. [JEE Advanced 2015, 4M]
(d) Statement I is true, statement II is true; statement II is a (a) 820 (b) 780
correct explanation for statement I (c) 901 (d) 861
117. Two sides of a rhombus are along the lines, x − y + 1 = 0
110. If the line 2x + y = k passes through the point which
and 7 x − y − 5 = 0. If its diagonals intersect at ( −1, − 2),
divides the line segment joining the points (1, 1) and then which one of the following is a vertex of this
(2, 4) in the ratio 3 : 2, then k equals [AIEEE 2012, 4M] rhombus? [JEE Main 2016, 4M]
29  1 8  10 7
(a) (b) 5 (a)  , −  (b)  − , − 
5  3 3  3 3
11
(c) 6 (d) (c) ( −3, − 9 ) (d) ( −3, − 8 )
5
166 Textbook of Coordinate Geometry

Answers
Exercise for Session 1 Exercise for Session 6
1. (c) 2. (b) 3. (b) 4. (d) 5. (b) 1. (c) 2. (c) 3. (a,b) 4. (a) 5. (a)
7. m ∈  −1, 
6. (d) 7. (c) 8. (d) 9. (a,d) 10. (c) 1
6. (c)
11. (d) 12. y = 9 13. (2 + 2 3 , 4) and (2 − 2 3 , 0)  5
 5π −1  π 5π π
9. θ ∈  0,  ∪ 
132
14. PQ = 15. 4 2 units 8. θ ∈  0, − tan 3 , 
12 3 + 5  6   12   12 2 
π 5π
16. 83x − 35 y + 92 = 0 17. x + y − 11 = 0 10. θ ∈  ∪  2nπ, 2nπ +   ∪  ∪  2mπ + , 2mπ  
n = z  6  m = z  6 
Exercise for Session 2 11. Outside 12. 29x − 2 y + 33 = 0
1. (b) 2. (c) 3. (a) 4. (c) 5. (b) 6. (d)
7. (c,d) 8. (d) 9. (d) 10. (d) 11. (b) 12. (d) Chapter Exercises
13. The two points are on the opposite side of the given line. 1. (b) 2. (a) 3. (c) 4. (d) 5. (c) 6. (b)
15. 3x − 4 y = 0 and 3x − 4 y − 10 = 0 7. (b) 8. (b) 9. (b) 10. (c) 11. (c) 12. (b)
17. 7x + y − 31 = 0 18. 2x + 2 y + 2=0 13. (c) 14. (d) 15. (a) 16. (b) 17. (b) 18. (c)
19. (b) 20. (b) 21. (b) 22. (b) 23. (c) 24. (a)
Exercise for Session 3 25. (a) 26. (b) 27. (a) 28. (a) 29. (b) 30. (a)
31. (a,b,c,d) 32. (a,b,c,d) 33. (a,c) 34. (a,d)
1. (c) 2. (b) 3. (c) 4. (a) 5. (d) 6. (a)
35. (b,d) 36. (a,b,c,d) 37. (a,c,d) 38. (a,d) 39. (a,b) 40. (a,b)
7. (a) 8. (c) 9. (a) 10. (a,b) 11. (c) 12. (c) 41. (a,b,c,d) 42. (b,d) 43. (a,b,c) 44. (a,b) 45. (a,b,c)
13. 2a + b2 + b = 0 16. (i) y = 3 (ii) x = 4, (iii) 3x + 4 y = 24 46. (d) 47. (d) 48. (a) 49. (a) 50. (d) 51. (a)
−5 −5 
18.  , 
52. (b) 53. (c) 54. (a) 55. (c) 56. (d) 57. (b)
 3 3 58. (a) 59. (a) 60. (b) 61. (3) 62. (6) 63. (5)
64. (8) 65. (3) 66. (9) 67. (8) 68. (2) 69. (2)
Exercise for Session 4 70. (4) 71. (A) → (p); (B) → (p, q); (C) → (p, r) (D) → (p, r, s)
1. (c) 2. (d) 3. (b) 4. (b) 5. (c) 6. (b) 72. (A) → (p, r); (B) → (q); (C) → (q, s) (D) → (p)
7. (c) 8. (a) 9. x = 7 and x + 3 y = 7 + 9 3 73. (A) → (p, q); (B) → (p, q, r, s); (C) → (p, q, r, s); (D) → (p, q, r, s)
10. x (4 3 + 3) + y(4 − 3 3 ) = 11 − 2 3 and 74. (A) → (t); (B) → (p, q, r); (C) → (s)
4 3 75. (A) → (q, s); (B) → (p, t); (C) → (r) 76. (a) 77. (a)
y (4 + 3 3 ) − x (4 3 − 3) = 11 + 2 3,
15 78. (d) 79. (d) 80. (c) 81. (b) 82. (b) 83. (d)
 6 −1  2 −13   3   −8 3 
11.  − ,  and  ,  85.  ,  ,  − ,
1 73 1 77  ,  0,  ,  , 
 14 28   16 32   5 10   5 10   2   5 10 
 n 1 n 
12.  0,  and (0, 0) 13. x + 2 y − 6 = 0
p
5 ±∑ ±∑ 
 2  p = 1 (1 + p )
2
p =1 (1 + p 2
) 
87.  ,  94. 2x + 3 y + 22 = 0
14. 3x = 19 15. 10x − 10 y − 3 = 0  c c 
 
 
Exercise for Session 5  h + mk mh − k 
95.  ,  96. 3x + 4 y − 18 = 0 and x − 2 = 0
1. (d) 2. (a) 3. (b) 4. (d) 5. (c)  1 + m2 1 + m2 
6. (d) 7. (d) 8. (c) 9. (d) 10. (b) 97. (2 − 3 ) sq units. 99. (a) 100. (c) 101. (c) 102. (c) 103. (c)
11. (b) 12. (c) 13. (3, − 2) 14. 14x + 23 y − 40 = 0
103. (c) 104. (A) → (s); (B) → (p, q); (C) → (r); (D) → (p, q, s)

15. 4x − y + 6 = 0,  − , 
4 14 105.(a) 106. (a) 107. (c) 108. (b) 109. (b) 110. (c) 111. (b)
16. (2)
 5 5 112.(a) 113. (d) 114. (a) 115. (6) 116. (b) 117. (a)
Solutions
1. Equation of line passing through (2, 0) and perpendicular to 4. We have, y = cos x cos( x + 2 ) − cos2( x + 1 )
ax + by + c = 0 1
Then, required equation is y = {2 cos x cos( x + 2 ) − 2 cos2( x + 1 )}
2
b 1
y − 0 = (x − 2) = {cos(2 x + 2 ) + cos2 − 1 − cos(2 x + 2 )}
a 2
ay = bx − 2b 1
= (cos2 − 1 )
⇒ ay − bx + 2b = 0 2
2 2 1 2m 1
2. Q + + = = (1 − 2 sin 2 1 − 1 )
1 !9 ! 3 !7 ! 5 !5 ! n ! 2
1 2 × 10 ! 2 × 10 ! 10 !  2m = − sin 2 1
 + + =
10 !  1 !9 ! 3 !7 ! 5 !5 !  n ! which is a straight line passing through ( λ , − sin 2 1 ); ∀ λ ∈ R
1 10 2m and parallel to the X -axis.
⇒ {2 C1 + 210C 3 + 10C 5 } =
10 ! n! x y
5. Let line + =1 …(i)
1 10 2m a b
{ C1 + 10C 3 + 10C 5 + 10C 7 + 10C 9 } =
10 ! n! Its passes through (2, 2), then
1 10 − 1 2 m 2 2
⇒ (2 ) = + =1
10 ! n! a b
∴ m = 9 and n = 10 ⇒ 2(a + b ) = ab …(ii)
Hence, x − y + 1 = 0 and x + y + 3 = 0 are perpendicular to Y
each other, then orthocentre is the point of intersection which
is ( −2, − 1 ) (0, b) B
∴ −2 = 2m − 2n and −1 = m − n
∴ Point is (2m − 2n, m − n ).
3. ∴ Required area
6
1 8  64 2
= 4 ×  × 4 = = …(i)
2 3  3 3 X¢
O (a, 0) A
X

Q f ( x + y ) = f ( x ) f (y ) Y¢
Y 1
Q Area of ∆AOB = ab = | λ | (given)
2
(0, 4) ∴ ab = 2| λ |
from Eq. (ii), a + b = | λ |
Hence, required equation is
(–8/3, 0) (8/3, 0)

x 2 − (a + b ) x + ab = 0
X
O
or x 2 − | λ | x + 2| λ | = 0
Value of (a 2x + aby + 1 ) at (1, 1 )
6. >0
(0, –4) Value of (a 2x + aby + 1 ) at ( 0, 0 )
a 2 + ab + 1
or > 0; ∀ a ∈ R
Y¢ 1
or a 2 + ab + 1 > 0; ∀ a ∈ R
∴ f (2 ) = f (1 ) f (1 ) = 2 2
∴ D<0
f (3 ) = f (1 + 2 ) = f (1 ) f (2 ) = 2 3
………………………………………… ⇒ b2 − 4 < 0
………………………………………… ⇒ −2 < b < 2 but b > 0
∴ f (n ) = 2n ∴ 0 <b <2
2 6 f (6 ) i.e. b ∈( 0, 2 )
∴ Area = = sq units
3 3
168 Textbook of Coordinate Geometry

7. Equation of L is a +
x y
= 1 and let the axis be rotated through 3 − 1 ( 3 − 1 )( 3 − 1 )
b Slope of BD = = = 3 −2
−1 − 3 −2
an angle θ and let ( X , Y ) be the new coordinates of any point
P ( x, y ) in the plane, then Y
C
x = X cosθ − Y sin θ, y = X sin θ + Y cosθ, the equation of the
line with reference to original coordinates is
2
x y
+ =1 D
a b E
X cosθ − Y sin θ X sin θ + Y cosθ 90° B
i.e. + =1 …(i) 2
a b 2
75°
and with reference to new coordinates is 30°
X
X Y (0, 0) A
+ =1 …(ii)
p q
Comparing Eqs. (i) and (ii), we get ∴ Equation of BD is
cosθ sin θ 1 y − 1 = ( 3 − 2 )( x − 3 )
+ = …(iii)
a b p ⇒ (2 − 3 ) x + y = 2( 3 − 1 )
sin θ cosθ 1
and − + = …(iv) and equation of the circumcircle of the triangle ABE
a b q (Apply diametric form as AB is diameter)
Squaring and adding Eqs. (iii) and (iv), we get ( x − 0 )( x − 3 ) + (y − 0 )(y − 1 ) = 0
1 1 1 1 ⇒ x2 + y 2 − x 3 − y = 0
+ = +
a2 b2 p2 q2
11. If (α, β) be the image of (4, 1) w.r.t y = x − 1, then (α, β) = (2, 3),
8. d (x, y ) = max{| x |, | y | } …(i) say point Q
but d ( x, y ) = a …(ii) Y
From Eqs. (i) and (ii), we get R
a = max{| x |, | y | } R¢ Q (3, 3)
(2, 3)
if | x | > | y |, then a = | x |
∴ x = ±a P
45°
and if | y | > | x |, then a = | y | (4, 1)
45°
∴ y =±a O (1, 0)
X
Therefore locus represents a straight line.
sin 2 α After translation through a distance 1 unit along the positive
9. P1 = | m 2 cosα + 2m sin α + | direction of X -axis at the point whose coordinate are R ≡ (3, 3 ).
cosα π
(m cosα + sin α ) 2 After rotation through are angle about the origin in the
= 4
| cosα | anticlockwise direction, then R goes toR ′ such that
 sin 2 α OR = OR′ = 3 2
p 2 = mm′ cos α + (m + m′ )sin α + 
 cosα  ∴The coordinates of the final point are ( 0, 3 2 ).
|(m cosα + sin α )| | m′ cosα + sin α | 12. Q A ≡ ( 0, 0 ); B ≡ (2, 0 ); C ≡ (2, 2 ); D ≡ ( 0, 2 )
=
| cosα | (i) f1( x, y ) → (y , x ), then
sin 2 α A ≡ ( 0, 0 ); B ≡ ( 0, 2 ); C ≡ (2, 2 ), D ≡ (2, 0 )
and p 3 = | m′ 2 cosα + 2m′ sin α + |
cosα (ii) f 2( x, y ) → ( x + 3y , y ), then
(m′ cosα + sin α ) 2 A ≡ ( 0, 0 ); B ≡ (6, 2 ); C ≡ (8, 2 ), D ≡ (2, 0 )
= x −y x + y
| cosα | (iii) f 3( x, y ) →  ,  , then
 2 2 
Q p 22 = p1 p 3
A ≡ ( 0, 0 ); B ≡ (2, 4 ); C ≡ (3, 5 ), D ≡ (1, 1 )
Hence, p1, p 2, p 3 are in GP.
Now, AB = DC = 2 5, AD = BC = 2
10. Side of the square = 2 unit
and AC = 34, BD = 10
Coordinates of B, C and D are ( 3, 1 ), ( 3 − 1, 3 + 1 ) and
i.e. AC ≠ BD
( −1, 3 ) respectively.
∴ Final figure is a parallelogram.
Chap 02 The Straight Lines 169

AN It is clear from the figure


13. Let =λ
BN  −3 π  π 
α ∈  −π,  ∪  , π .
 a aλ   4  4 
Then, coordinate of N are  , 
1 + λ 1 + λ 16. Q Equation of AB is x + y = 1, then coordinates of A and B are
Q Slope of AB = − 1 (1, 0) and (0, 1) respectively.
Y  1 1
∴ Coordinates of P are  , 
 2 2
B(0, a)
Q PP1 is perpendicular to OA
N Y
x+y= a
M B(0, 1)
X
O A(a, 0)

∴ Slope of MN = 1 P
∴ Equation on MN is

x+
M1

y=
M2
aλ a 1−λ
y − =x− ⇒ x − y = a

1
 A(1, 0)
1+λ 1+λ  λ + 1 X
O P3 P2 P1
  λ − 1 
So, the coordinates of M are  0,a  
  λ + 1 
Equation of OP is y = x
3
Therefore, area of ∆AMN = area of ∆OAB Then, OP1 = PP1 =
1
8 2
1 3 1
⇒ ⋅ AN ⋅ MN = ⋅ a ⋅ a We have, (OMn − 1 ) 2 = (OPn ) 2 + ( Pn Mn − 1 ) 2
2 8 2
1 aλ 2 a 2  = 2(OPn ) 2 {Qy = x}
⇒ ⋅ ⋅ = 3 ⋅ 1 a ⋅ a
2 1 + λ 1 + λ 8 2 = 2α n2 (say)

a 2λ 3 1 Also, (OPn − 1 ) = (OMn − 1 ) 2 + ( Pn − 1Mn − 1 ) 2


2
⇒ = ⋅ a2
(1 + λ ) 2 8 2 1
α n2 − 1 = 2α n2 + α n2 − 1
1 2
∴ λ = 3 or λ = 1 2
3 ⇒ α n − 1 = 2α n2
1 2
For λ = , then M lies outside the segment OB and hence the
3 1
⇒ αn = αn − 1
required value of λ = 3. 2
14. Let S = (x, y ), given (SQ )2 + (SR )2 = 2(SP )2 1
∴ OPn = α n = α n − 1
2
⇒ ( x + 1 ) 2 + y 2 + ( x − 2 ) 2 + y 2 = 2[( x − 1 2 ) + y 2 ]
1 1
⇒ 2 x 2 + 2y 2 − 2 x + 5 = 2 ( x 2 + y 2 − 2 x + 1 ) = 2 αn − 2 = 3 αn − 3
2 2
3 ……………………………………………
⇒ 2x + 3 = 0 ⇒ x = −
2 ……………………………………………
A straight line parallel to Y -axis. ……………………………………………
Value of (3 x − 2y + 1 ) at A 1
15. >0 = n − 1 α1
Value of (3 x − 2y + 1 ) at B 2
(sin α − 3 ) − (cosα − 2 ) + 1 1  1 1
⇒ >0 = n −1   = n.
(3 − 2 + 1 ) 2  2 2
⇒ sin α − cosα > 0 ⇒ sin α > cosα 17. Let O ≡ (0, 0), A ≡ (1, 1) and B ≡ (9, 1)
Y 1
Area of ∆OAB = ×8 ×1 = 4
2
It is clear that 1 < c < 9
–p –3p/4
X¢ X  c
–p O p p p and M ≡ (c, 1 ) and N ≡ c, 
2 4 2  9


170 Textbook of Coordinate Geometry

Y 20. Let A ≡ (3, 4), B ≡ (0, y ), C ≡ (x, 0), D ≡ (8, 2)


∴ Slope of AB = − Slope of BC
x=c
y −4 0 −y
A(1, 1) (a, 1) ⇒ =− 
M B 0 −3  x − 0
x
N y= 9 or 4 x − xy = 3y …(i)
X
O and slope of BC = − slope of CD
0 −y  2 − 0
⇒   = − 
 x − 0 8 − x
∴ Area of ∆BMN = 2 (given) or 2 x + xy = 8y …(ii)
1  c adding Eqs. (i) and (ii), we get
⇒ × (9 − c ) × 1 −  = 2
2  9 6 x = 11y …(iii)
or (9 − c ) = 36
2 from Eqs. (ii) and (iii), we get
13 1
or 9 − c = ± 6 ⇒ c = 3 or 15 x= =4
3 3
but 1 <c <9
∴ c =3 21. Y y=x

18. The three lines are concurrent if (–n, m) C B (n, m)


1 2 −9
3 −5 −5 = 0 A (m, n)
a b −1
X¢ X
or 5a + 2b = 1 O
which is three of the line 5 x + 2y = 1 passes through (a, b ).
19. Q BC = AC
⇒ 2 2 + ( λ − 1 ) 2 = λ2
D (–n, –m) E (n,–m)
⇒ 4 = λ2 − ( λ − 1 ) 2 Y¢
= (2 λ − 1 )(1 )
Area of rectangle BCDE = (2n )(2m )
5
∴ λ= = 4mn
2
1
and area of ∆ABE = × 2m × (m − n )
C 2
(2, l)
= m(m − n )
∴ Area of pentagon = 4mn + m(m − n )
B
(0, 1) = m (m + 3n )
22. The equation of the line L, be y − 2 = m(x − 8), m < 0
D
 2 
coordinates of P and Q are P 8 − , 0 and Q( 0, 2 − 8m ).
A (2, 0)
 m 
O
x=2 2
So, OP + OQ = 8 − + 2 − 8m
m
x y  1
Q Equation. of AB is + = 1, D ≡ 1,  (mid-point of AB) 2
2 1  2 = 10 + + 8( −m ) ≥
( −m )
∴ Equation of CD is 2x − y = µ
2
QCD pass through D, thus 10 + 2 × 8( −m ) ≥ 18
( −m )
1 3
2 − = µ or µ = So, absolute minimum value of OP + OQ = 18
2 2
3 23. Let the two perpendiculars through the origin intersect
∴ Equation of CD is 2x − y = …(i) 2x + y = a at A and B so that the triangle OAB is isosceles.
2
and Eq. (i) of line ⊥ to AC and pass through B is y = 1 …(ii) OM = length of perpendicular from O to
a
from Eqs. (i) and (ii), we get AB, OM = .
5
5 
Orthocentre ≡  , 1
4 
Chap 02 The Straight Lines 171

Y 27. Q B is the reflection of A(5, 7) w.r.t the line x + y = 0


B ∴ B ≡ ( −7, − 5 )
and C is the reflection of A(5, 7 ) w.r.t the line x − y = 0
A (5, 7)
M

= 0
2x+y=a
x+ x–y
O A X M y =0 N

Also, AM = MB = OM (0, 0)
2a
⇒ AB = B C
5
∴ C ≡ (7, 5 )
1
Area of ∆OAB = ⋅ AB ⋅ OM 5+5
2 ∴ Equation of BC is y + 5 = ( x + 7 ) or 7y = 5 x
7+7
1 2a a a2
= ⋅ ⋅ = sq units 28. Let P ≡ (2, − 1)
2 5 5 5
24. Solving given equations, we get P(2, − 1 ) goes 2 units along x + y = 1 upto A and 5 units along
5 x − 2y = 4 upto B.
x= Now, slope of x + y = − 1 is −1 = tanθ (say)
3 + 4m
∴ θ = 135 °
x is an integer, if 3 + 4m = 1, − 1, 5, −5 1
−2 −4 2 −8 and slope x − 2y = 4 is = tan φ (say)
or m= , , , 2
4 4 4 4 1 2
1 1 ∴ sin φ = , cos φ =
or m = − , − 1, , − 2 5 5
2 2
The coordinates of A
Hence, m has two integral values.
i.e. (2 + 2 cos135 °, − 1 + 2 sin 135 ° )
25. Let the coordinates of A be (a, 0). Then the slope of the or (2 − 2, 2 − 1 )
reflected ray is
3−0 The coordinates of B
= tanθ (say) …(i) i.e. (2 + 5 cos φ, − 1 + 5 sin φ ) or (2 + 2 5, 5 − 1 )
5 −a
Then the slope of the incident ray 29. Q P ≡ (5, 3)
2−0 Let P′ and P′′ be the images of P w.r.t y = x and y = 0 (X -axis)
= = tan( π − θ )
1 −a respectively, then P′ ≡ (3, 5 ) and P′′ ≡ (5, − 3 )
From Eqs. (i) and (ii), we get Q PQ + QR + RP is minimum
tan θ + tan( π − θ ) = 0 ∴ P ′ , R, Q, P ′′ are collinear.
3 2 P¢(3, 5)
⇒ + =0 Y x
5 −a 1 −a y=
⇒ 3 − 3a + 10 − 2a = 0 R P(5, 3)
13
a=
5
X¢ X
 13  Q
Thus, the coordinate of A is  , 0 O
5 
26. Lines 5x + 3y − 2 + λ(3x − y − 4) = 0 are concurrent at (1, −1) P¢¢(5, –3)
and lines

x − y + 1 + µ(2 x − y − 2 ) = 0 are concurrent at (3, 4).
∴ Equation of P ′ P ′′ is
Thus equation of line common to both family is
4+1  5 + 3
y +1= (x − 1) y +3=  (x − 5)
3 −1  3 − 5
or 5 x − 2y − 7 = 0 or 4 x + y = 17
∴ a = 5, b = − 2 ⇒a + b = 3  17 
∴ Q ≡  , 0 (QQ on Y -axis)
4 
172 Textbook of Coordinate Geometry

30. Equation of incident ray is 35. Equation of any line through the point of intersection of the
y − 0 = tan(90 ° + 60 ° )( x − 2 ) given lines is (3 x + y − 5 ) + λ ( x − y + 1 ) = 0.
1 Since this line is perpendicular to one of the given lines
or y =− (x − 2) 3+λ 1
3 = − 1 or
or (x − 2) + y 3 = 0 λ −1 3

and equation of refracted ray is ⇒ λ = − 1 or −5, therefore the required straight line is
y − 0 = − tan 60 °( x − 2 ) x + y −3 = 0
or y = − 3(x − 2) or x − 3y + 5 = 0
y 36. If B lies on Y -axis, then coordinates of B are (0, a ) or (0, − a )
or (x − 2) + =0
3 Y
∴ Combined equation is B
 y  60°
[( x − 2 ) + y 3 ] ( x − 2 ) +  =0
 3
a A
4 A 60° x– 3y=0
i.e. (x − 2)2 + y 2 + ( x − 2 )y = 0 a
3 60°
O
x y x y 30° 30°
31. Point of intersection of + = 1 and + = 1 is X¢
30° 30°
X
a b b a
 ab 60°
ab  A¢¢
P ,  , this point P satisfies alternates (a), (b), (c) A¢
a + b a + b

and (d).
32. The two lines will be identical if their exists some real number B¢
k such that

b 3 − c 3 = k(b − c ), c 3 − a 3 = k(c − a ) and a 3 − b 3 = k(a − b )
⇒ b − c = 0 or b 2 + c 2 + bc = k If third vertex in IV quadrant or in II quadrant, then its
coordinates are (a cos30 °, − a sin 30 ° ) and ( −a cos30 °, a sin 30 ° )
c − a = 0 or c 2 + a 2 + ca = k a 3 a  a 3 a
i.e.  , −  and  − , ,
and a − b = 0 or a 2 + b 2 + ab = k  2 2  2 2
⇒ a = b or b = c or c = a
37. Since, ax + by + c = 0, bx + cy + a = 0 and cx + ay + b = 0 are
or b 2 + c 2 + bc = c 2 + a 2 + ca
concurrent
⇒ b = c or c = a
a b c 
or a = b or a + b + c = 0 b c a = 0
∴  
33. As the third vertex lies on the line y = x + 3, its coordinates c a b
are of the form ( x, x + 3 ). The area of the triangle with vertices
(2, 1), (3, −2) and ( x, x + 3 ) is given by ⇒ 3abc − a 3 − b 3 − c 3 = 0
x x + 3 1 ⇒ − (a + b + c )(a 2 + b 2 + c 2 − ab − bc − ca) = 0
1 
|2 1 1| = |2 x − 2| = 5 (given) a+b+c≠0
2
3 −2 1 ∴ a + b + c − ab − bc − ca = 0
2 2 2

−3 7 1
∴ 2x − 2 = ± 5 ⇒ x = , {(a − b ) 2 + (b − c ) 2 + (c − a ) 2 } = 0
2 2 2
 7 13 As a, b, c are real numbers
Thus, the coordinates of the third vertex are  ,  or
2 2  ∴ b − c = 0, c − a = 0, a − b = 0
 −3 3  ⇒ a =b =c
 , .
 2 2
38. Q E ≡ (4, 4)
1 −2 −6
∴ zC = 7 + 5i , z E = 4 + 4i
34. 3 1 −4= 0 Now, (in ∆BEC)
λ 4 λ2  π
zB − zE i
⇒ λ + 2λ − 8 = 0
2 =e 2 =i
zC − z E
∴ ( λ + 4 )( λ − 2 ) = 0 ⇒ z B − 4 − 4i = i (7 + 5i − 4 − 4i )
⇒ λ = − 4, 2 or z B = 3 + 7i
Chap 02 The Straight Lines 173

∴ B ≡ (3, 7 ), then D ≡ (5, 1 ) After solving, we get


B C (7, 5) x1 = 1 ± 100 2, 1
and y1 = 7, 7 ± 100 2
Hence, (1 + 100 2, 7 ), (1 − 100 2, 7 ),

E
(1, 7 + 100 2 ), (1, 7 − 100 2 )
42. Equation of the other diagonal is x + y = λ which pass
through (a, b ), then
A(1, 3) D a+b=λ
∴ Equation of other diagonal is
Equation of AB is
7 −3 x + y =a + b
y −3 = ( x − 1 ) or 2 x − y + 1 = 0 i.e. then centre of the square is the point of intersection of
3 −1
 b b
and equation of AD is x − y = a and x + y = a + b is a + ,  , then vertex
 2 2
1 −3
y −3 = ( x − 1 ) or x + 2y − 7 = 0
5 −1 b b
a+ ,
B 2 2
39. Given,
6a 2 − 3b 2 − c 2 + 7ab − ac + 4bc = 0 A(a, b)
⇒ 6a + (7b − c )a − (3b − 4bc + c ) = 0
2 2 2

− (7b − c ) ± (7b − c ) 2 + 24 (3b 2 − 4bc + c 2 ) O


⇒ a=
12
⇒ 12a + 7b − c = ± (11b − 5c ) C
⇒ 12a − 4b + 4c = 0
D
or 12a + 18b − 6c = 0
⇒ 3a − b + c = 0 C ≡ (2a + b − a, b − b )
or −2a − 3b + c = 0 ∴ C ≡ (a + b, 0 )
Hence (3, − 1 ) or ( −2, − 3 ) lies on the line ax + by + c = 0, If B ≡z
40. x + 2y + 4 = 0 and 4x + 2y − 1 = 0  b ib 
z − a + +  π
⇒ x + 2y + 4 = 0  2 2 BO i 2
Then, = e = i (Q BO = AO)
and −4 x − 2y + 1 = 0  b ib  AO
(a + ib ) − a + + 
Here, (1 )( −4 ) + (2 )( −2 ) = − 8 < 0  2 2
∴Bisector of the angle including the acute angle bisectors and  b ib   b ib  ib b
⇒ z − a + +  = i  − +  = − −
origin is  2 2  2 2 2 2
x + 2y + 4 ( −4 x − 2y + 1 ) ∴ z =a
=
5 2 5 ∴ B ≡ (a, 0 )
⇒ 6 x + 6y + 7 = 0 then, D ≡ (a + b, b )
41. Let position of bunglow is P (x1, y1 ), then PM = 100 and Hence, other vertices are (a + b, 0 ), (a, 0 ) and (a + b, b ).
PN = 100 43. (y − y1 ) − m(x − x1 ) = 0 is family of lines
x1 + y1 − 8
∴ = ± 100 ∴ y − y1 = 0, x − x1 = 0
2
Then, y = y1 and x = x1
x1 − y1 + 6
and = ± 100 44. Given lines L1 = 0 and L2 = 0 are perpendicular and given
2
Y bisectors are λ 1L1 − λ 2L2 = 0 and λ 1L1 + λ 2L2 = 0
∴bisectors are perpendicular to each other.
(0, 8) (0, 6) Hence, bisectors of λ 1L1 − λ 2L2 = 0 and λ 1L1 + λ 2L2 = 0 are
M L1 = 0 and L2 = 0.
 θ + φ
y–x=6 P(x, y) x+y=8 45. ∴ One bisector makes an angle   with X -axis, then
 2 
X′ X  θ + φ
(–6, 0) O (8, 0) other bisector makes an angle 90 ° +   with X -axis.
 2 
Y′
174 Textbook of Coordinate Geometry

f–q
48. QOT = CT
2 ⇒ | x − 0| + | y − 0| = | x − 4| + | y − 3|
Q x ≥ 0, y ≥ 0
⇒ x + y = | x − 4| + | y − 3|
f–q f+q Case I : If 0 ≤ x ≤ 4 and 0 ≤ y ≤ 3
q+ =
q f 2 2 x + y = 4 − x + 3 −y
(3, –5) 7
⇒ x+y =
2
Case II : If 0 ≤ x ≤ 4 and y ≥ 3
∴ Equations of bisectors are x + y = 4− x + y − 3
x −3 y +5 1
= …(i) ⇒ x=
 θ + φ   θ + φ 2
cos  sin  
 2   2  Case III : If x ≥ 4 and 0 ≤ y ≤ 3
x −3 y +5 x + y = x − 4 + 3 −y
and =
 π θ + φ  π θ + φ y = −1 / 2 (impossible)
cos +  sin  + 
2 2  2 2  Case IV : If x ≥ 4 and y ≥ 3
x −3 y +5 x + y = x − 4 + y −3
⇒ = …(ii)
 θ + φ  θ + φ ⇒ 0 = −7 (impossible)
− sin   cos 
 2   2  Combining all cases, we get
x −3 y + 5 7
But given bisector are = x + y = , ∀ 0 ≤ x ≤ 4 and 0 ≤ y ≤ 3
cosα sin α 2
θ+φ x −3 y + 5 1
∴ α= and = [from Eq. (i)]…(iii) and x = , ∀ 0 ≤ x ≤ 4 and y ≥ 3
2 β γ 2
 θ + φ Y
∴ β = − sin   = − sin α [from Eq. (ii)]
 2 
 θ + φ 3
and γ = cos   = cosα
 2 
46. QOR = AR
⇒ | x − 0| + | y − 0| = | x −1| + | y − 2| X
O 1/2 3.5
⇒ | x| + | y | = | x − 1| + | y − 2|
Q 0 ≤ x < 1 and 0 ≤ y < 2 Sol. (Q. Nos. 49 to 51)
∴ x + y = − ( x − 1 ) − (y − 2 ) AB : 2 x − y + 4 = 0,
⇒ 2 x + 2y = 3 BC : x − 2y − 1 = 0
47. OS = BS and CA : x + 3y − 3 = 0
⇒ | x − 0| + | y − 0| = | x − 2| + | y − 3| Y
Y

C
X¢ O
infinte ray X

X
O 1 2 B

∴ mAB = m1 = 2
⇒ | x| + | y | = | x − 2| + | y − 3| 1
Q x ≥ 2 and 0 ≤ y < 3 mBC = m2 =
2
∴ x + y = x −2 + 3 −y 1
⇒ 2y = 1 and mCA = m3 = −
3
1
∴ y = Q m1 > m2 > m3
2
Chap 02 The Straight Lines 175

49. Q ∠A is obtuse 2 9
− −
m − m1 5  5
∴ tan A = 3 or y − 2 − = 5 2 x + 
2 − + 2 5  2
1 + m3 m1
5 2
1
− −2 or 7 x + 3y + 4 = 0
= 3 = −7
2 53. Coordinates of vertex B are (λ, 2 − λ )
1−
3  5 9
i.e. − ,  [from Eq. (i)]
50. For external bisector of B  2 2
AB : 2 x − y + 4 = 0
54. A ≡ (1, 3) and B ≡  − , 
5 9
BC : − x + 2y + 1 = 0  2 2
Q (2 )( −1 ) + ( −1 )(2 ) = − 4 < 0 ∴ Equation of AB is
∴ External bisector of B is 9
−3
 2x − y + 4 ( − x + 2y + 1 )
  =− y −3 = 2 (x − 1)
 5  5 5
− −1
or x+y +5=0 2
or 3 x + 7y = 24
51. Let (α, β) be the image of B(−3, − 2) w.r.t. the line
x + 3y − 3 = 0, then 55. Any point on the line 3x − y = 2 is (t, 3t − 2), t being parameter.
α + 3 β + 2 −2( −3 − 6 − 3 ) If ( x, y ) be image of the point (t, 3t − 2 ) in the line y = x − 1 or
= = x − y − 1 = 0, then
1 3 1+9
x − t y − (3t − 2 )
α + 3 β + 2 12 =
or = = 1 −1
1 3 5 2(t − 3t + 2 − 1 )
3 26 =−
or α = − and β = 1+1
5 5
x − t y − 3t + 2
 3 26 ⇒ = = 2t − 1
∴ Required image is  − ,  , 1 −1
 5 5
or x − t = 2t − 1
Sol. (Q. Nos. 52 to 54)
⇒ x + 1 = 3t …(i)
Let B ≡ (λ, 2 − λ ) (Q B lies on x + y = 2)
and y − 3t + 2 = − 2t + 1
1+λ
Slope of line AB = m1 = ⇒ y + 1 =t …(ii)
1−λ
From Eqs. (i) and (ii), we get
5 λ − 12
and Slope of line BC = m2 = x + 1 = 3 (y + 1 )
−5 λ − 2
⇒ x − 3y = 2
12 − 5 λ
= 56. Any point on the circle x 2 + y 2 = 4 is (2 cosθ, 2sin θ ),θ being
2 + 5λ
parameter.
Let slope of bisector ( x + y = 2 ) = m3 = − 1 If ( x, y ) be image of the point (2 cosθ, 2 sin θ ), in the line
m3 − m1 m − m3 x + y = 2, then
Now, = 2
1 + m3m1 1 + m2m3 x − 2 cosθ y − 2 sin θ
=
1 + λ 12 − 5 λ 1 1
−1 − +1
1−λ 2 + 5λ −2(2 cosθ + 2 sin θ − 2 )
⇒ = =
1+λ 12 − 5 λ 1+1
1− 1−
1−λ 2 + 5λ or x − 2 cosθ = y − 2 sin θ
−2 14 = − 2 cosθ − 2 sin θ + 2 …(i)
or =
−2 λ −10 + 10 λ or x − 2 cosθ = − 2 cosθ − 2 sin θ + 2
or 14 λ = − 10 + 10 λ ⇒ x − 2 = − 2sinθ
−5 and y − 2 sin θ = − 2 cosθ − 2 sin θ + 2
∴ λ= …(i)
2 ⇒ y − 2 = − 2 cosθ …(ii)
52. Equation of BC is From Eqs. (i) and (ii),
2 ( x − 2 ) 2 + (y − 2 ) 2 = 4
− − (2 − λ )
y − (2 − λ ) = 5 (x − λ ) ⇒ x 2 + y 2 − 4 x − 4y + 4 = 0
2
− −λ
5
176 Textbook of Coordinate Geometry

57. Any point on the parabola x 2 = 4y is (2t, t 2 ), t being parameter. ⇒ cos A = 2 cos B cosC
If ( x, y ) be image of the point (2t, t ) in the x + y = a, then
2 ⇒ − cos( B + C ) = 2 cos B cosC (QA + B + C = π )
⇒ − (cos B cosC − sin B sin C ) = 2 cos B cosC
x − 2t y − t 2
+ or sin B sin C = 3 cos B cosC
1 1 3
−2 (2t + t 2 − a ) =3 ×
= 2R 5
1+1 
9 61 
= Q R = 
= − 2t − t 2 + a 2 61  5
or x − 2t = − 2t − t 2 + a 60. Q AO = 2R cos A
⇒ x − a = −t2 …(i) 3
=2 × [from Eq. (i)]
and y − t 2 = − 2t − t 2 + a 5
⇒ y − a = −2t …(ii) 6
=
From Eqs. (i) and (ii) we get 5
(y − a ) 2 = 4t 2 = − 4( x − a ) 61. The equation of straight line through (2, 3) with slope m is
or (y − a ) = 4(a − x )
2 y − 3 = m( x − 2 )
or mx − y = 2m − 3
Sol. (Q. Nos. 58 to 60)
x y
Given orthocentre O ≡ (1, 2 ) or + =1
 2m − 3  (3 − 2m )
and circumcentre  
 m 
O′ = (2, 4 )
2m − 3
A Here, OA = or OB = 3 − 2m
m
E Q The area of ∆OAB = 12
⇒ 1 × OA × OB = 12
F 2 

O 1  2m − 3
A A or   (3 − 2m ) = ± 12
2 m 
or (2m − 3 ) 2 = ± 24m
B D M 2x–y=3 C Taking positive sign, we get 4m 2 − 36m + 9 = 0
Here D > 0, This is a quadratic in m which given two value of
Q Slope of OO′ = Slope of (2 x − y = 3 )
m, and taking negative sign, we get (2m + 3 ) 2 = 0.
3
and OD = O ′ M = −3
5 This gives one line of m as .
2
Let R be the circumradius
Hence, three straight lines are possible.
∴ O ′ M = R cos A
3 62. Q Point of intersection of ax + 3y − 1 = 0 and ax + y + 1 = 0 is
⇒ R cos A = …(i)  2 
5 A  − , 1 and point of intersection of ax + 3y − 1 = 0 and
 a 
58. R = AO ′ = (AO )2 + (OO ′ )2  1 1 
x + 3y = 0 is B  ,− 
= (2 R cos A ) 2 + 5  a − 1 3(a − 1 )

 6 
2 a
=   +5 [from Eq. (i)] ⇒ Slope of OA is mOA = −
 5 2
1
61 and Slope of OB is mOB =−
= 3
5
Q mOA × mOB = −1
59. QOD = 2R cos B cosC a 1
∴ − × − = −1
3 2 3
∴ 2 R cos B cosC =
5 or a = −6
= R cos A [from Eq. (i)] …(ii) ∴ |a | = 6
Chap 02 The Straight Lines 177

63. Here, B is the image of A w.r.t line y = x 66. Let a be the length of side of square
∴ B ≡ (2, 1 ) and C is the image of A w.r.t line x − 2y + 1 = 0 if ∴ a 2 + a 2 = 22 ⇒ a = 2
C ≡ (α , β ), then i.e. distance between parallel lines is 2
α − 1 β − 2 −2(1 − 4 + 1 ) Now, let two lines of family y = x + n are y = x + n, and
= =
1 −2 1+4 y = x + n2, where
9 2 n1, n2 ∈ { 0, 1, 2, 3, 4 }
or α = and β =
5 5 | n1 − n2|
∴ = 2
 9 2 2
∴ C ≡ , 
 5 5 or | n1 − n2| = 2
⇒ Equation of BC is ⇒ {n1, n2 } are { 0, 2 }, {1, 3 } and {2, 4 }
2  Hence, both the family have three such pairs. So, the number
 − 1 of squares possible is 3 × 3 = 9.
5 
y −1 = (x − 2)
9  67. Let the coordinate of C be (1, c ), then
 − 2
5  c −y
m2 =
or 3x − y − 5 = 0 (Q Eq. of BC is ax + by − 5 = 0) 1−x
Here, a = 3, b = − 1 c − m1x
or m2 = (Q slope of AB = m1)
∴ a − 2b = 5 1−x
64. On the line y = 1, the number of lattice points is ⇒ m2 (1 − x ) = c − m1x
2007 − 223  or c = (m1 − m2 ) x + m2
= 198
 9  1
Now, the area of ∆ABC is | cx − y |
Y 2
1
= ((m1 − m2 ) x + m2 ) x − m1x| (Qy = m1x)
(0, 9) B
2
1
= (m1 − m2 )( x − x 2 ) [Qm1 > m2 and x ∈( 0, 1 )]
2
1
2 Hence, f (x ) = (x − x 2 )
1 2
X
O A df ( x ) 1
(223, 0) ∴ = (1 − 2 x )
dx 2
2
d f (x )
Hence, the total number of points and = −1 < 0
dx 2
2007 − 223y 
8
= ∑  9 
For maximum of
df ( x ) 1
y =1
f ( x ), =0 ⇒ x=
= 198 + 173 + 148 + 123 + 99 + 74 + 49 + 24 = 888 dx 2
Hence, tens place digit is 8. 1  1 1
∴ f ( x ) max =  − 
2  2 4
65. A rough sketch of the lines is given.
1
There are three triangle namely ABC, BCD and ABD = =λ (given)
8
A
1
⇒ =8
λ
68. Equation of AB is 3x − 2y + 6 = 0
Y
Y
A (0, 3)
C
B D
P(l, l+1)
2
y= +3
3

3x+

C (6, 1)
x+
x+

2x

y=
y=

y=

X¢ X
3

X¢ X B O
O (–2, 0)


178 Textbook of Coordinate Geometry

Equation of BC is x − 8y + 2 = 0, 71. (A)


Equation of CA is x + 3y − 9 = 0 L1
Let P ≡ (λ, λ + 1)
QB and P lie on one side of AC, then L2
λ + 3( λ + 1 ) − 9
>0
−2 + 0 − 9
or 4λ − 6 < 0 L3
3
or λ< (i)
2 In this case no circle
and C and P lie on one side of AB, then ∴ n = 0 ⇒ n + 1 =1
3 λ − 2( λ + 1 ) + 6 (B)
>0
18 − 2 + 6
or λ+4>0
or λ > −4 …(ii) In this case no circle
Finally, A and P lie on one side of BC, then ∴ n = 0 ⇒ 2n + 3 = 3
λ − 8( λ + 1 ) + 2
>0 (C)
0 − 24 + 2
or −7 λ − 6 < 0
6
or λ>− …(iii)
7
From Eqs. (i), (ii) and (iii), we get
6 3 In this case two circle which are touching all three lines
− <λ< ∴ n =2 ⇒ n + 2 = 4
7 2
Integral values of λ are 0 and 1. (D)
Hence, number of integral values of λ is 2.
69. Lines
(2a + b ) x + (a + 3b )y + b − 3a = 0
or a (2 x + y − 3 ) + b ( x + 3y + 1 ) = 0
are concurrent at the point of intersection of lines
2 x + y − 3 = 0 and x + 3y + 1 = 0 which is (2, − 1 ).
Now, line λx + 2y + 6 = 0 must pass through (2, − 1 ), therefore,
2 λ − 2 + 6 or λ = − 2
In this case four circle which are touching all three lines
∴ | λ| = 2
∴ n = 4⇒ n + 2 =6
70. Since, PQ is of fixed length.
72. (A) The given lines an concurrent. So,
1
Area of ∆PQR = | PQ | | RP |sinθ 1 −2 −6
2
3 1 −4 = 0
This will be maximum, if sinθ = 1 and RP is maximum.
λ 4 λ2
R or λ2 + 2 λ − 8 = 0
or λ = 2, − 4
P (4, 4)
∴ | λ | = 2, 4
q
(B) Given family is
(0, 7)R¢
3 x(a + 1 ) + 4y (a − 1 ) − 3(a − 1 ) = 0
Q 3x+4y+5=0 or a(3 x + 4y − 3 ) + (3 x − 4y + 3 ) = 0
for fixed point=
3 x + 4y − 3 = 0
Since, line y = mx + 7 rotates about ( 0, 7 ), if PR′ is
and 3 x − 4y + 3 = 0
perpendicular to the line than PR′ is maximum value of PR.
3
 4 − 0 4 ∴ x = 0, y =
∴ m=−  = 4
 4 − 7 3
 3
Fixed point is  0,  ,
Hence, 3m = 4  4
Chap 02 The Straight Lines 179

3 −4 1
Here p = 0, q = ∴ < λ < or −8 < 6 λ < 3
4 3 2
∴ 4| λ | = 4| p − q | = 3 Integral values of 6λ are
(C) The point of intersection of x − y + 1 = 0 and −7, −6, −5, −4, −3, −2, −1, 0, 1, 2
3 x + y − 5 = 0 is (1, 2). It lies on the line ∴ |6λ | = 7, 6, 5, 4, 3, 2, 1, 0
λ (D)Q A ≡ ( −2, 4 )
x + y − 1 −  = 0
2 7
The points on the line y = , whose x-coordinates lies between
λ 2
⇒ 1 + 2 − 1 −  = 0 7
2 0 (put y = in 4y + x − 14 = 0)
2
or | λ | = 4 or λ = − 4, 4
−11 7
∴ λ + 1 = − 3, 5 or | λ + 1| = 3, 5 and (put y = in y + 3 x + 2 = 0)
6 2
(D) The mid-point of (1, − 2 ) and (3, 4) will satisfy
−11
y − x −1 + λ = 0 ∴ <λ<0
6
or 1 −2 −1 + λ = 0
or −11 < 6 λ < 0
∴ λ = 2 or | λ | = 2
Integral value of 6λ are
73. −10, − 9, − 8, − 7, −6, −3, − 2, − 1
y+

Y
3x+

∴ |6 λ | = 10, 9, 8, 7, 6, 5, 4, 3, 2, 1
x=1
2=

74. (A)Q max. {| x|,| y | } = 1


0

A (0, 7/2)
E If | x| = 1 and if | y | = 1
C 4 y +x then x = ± 1 and y = ± 1
–1 4 =
0
D y=2 Y
(0,5/3) y=1
B
X¢ X
O
0
5=
2x–
3 y–
X¢ X
Y¢ x=–1 O x=1
(A) The points on the line x = 0, whose y-coordinate lies
5 7
between and inside the triangle ABC.
3 2 y=–1
5 7 Y¢
∴ <λ< or 5 < 3 λ < 10.5
3 2
∴ Required area = 2 × 2 = 4 sq units
∴ |3 λ | = 6, 7, 8, 9, 10
(B) The line y = x cuts the lines | x + y | = 6
(B)QC ≡ (2, 3 )
i.e. x + y = ±6
The points on the line x = 1, whose y-coordinate lies between
at x = ± 3, y = ± 3
8
( put x = 1 in 3y − 2 x − 5 = 0) or ( −3, − 3 ) and (3, 3)
3
then −3 < a < 3
13
and (put x = 1 in 4y + x − 14 = 0) ∴ 0 ≤ | a| < 3
4
8 13 ∴ [| a| ] = 0, 1, 2
∴ <λ< or 8 < 3 λ < 9.75 (C) Since (0, 0) and (1, 1) lie on the same side.
3 4
∴ |3 λ | = 9 So, a 2 + ab + 1 > 0
(C)Q B ≡ ( −1, 1 ) Q Coefficient of a 2 is > 0
The point on the line y = 2, whose x-coordinate lies between ∴ D<0
−4 b 2 − 4 < 0 or −2 < b < 2
(put y = 2 in y + 3 x + 2 = 0)
3
⇒ b = − 1, 0, 1
1
and (put y = 2 in 3y − 2 x − 5 = 0) ∴ Number of values of b is 3.
2
180 Textbook of Coordinate Geometry

75. (A) Q d (x, y ) = 2| x| + 3| y | = 6 (given) (C)∴ Slope of AC × slope of BC = − 1


| x| | y | Y
∴ + =1
3 2 C(l, m)
Y

(0, 6) B
B (0, 2)
13 13
C A
X′ Q X
(–3, 0) (3, 0) X
O A(6, 0)
13 13
D (0, –2)  µ − 0  µ − 6 
⇒   ×  = −1
 λ − 6  λ − 0
Y′ ⇒ µ 2 − 6µ = − λ2 + 6 λ
∴Perimeter, λ = 4 13 or λ2 + µ 2 − 6 λ − 6µ = 0

and area,
1
µ = 4 × × 3 × 2 = 12 Hence, locus of ( λ , µ ) is
2 x 2 + y 2 − 6 x − 6y = 0
λ2
then −µ =1 76. Q x(a + 2b ) + y (a + 3b ) = a + b
16
⇒ a ( x + y − 1 ) + b (2 x + 3y − 1 ) = 0
and λ2 − µ 2 = 64
then x + y − 1 = 0 and 2 x + 3y − 1 = 0
Hence, locus of ( λ , µ ) are ∴ point of intersection is (2, − 1 )
x 2 − 16y = 16 Hence, both statement are true and statement II is correct
and x 2 − y 2 = 64 explanation for statement I.
(B) It is clear that orthocentre is (6, 6) 77. Q Algebraic perpendicular from (3, 2) to the line
O′ ≡ (6, 6 ), 9−4+1 6
3 x − 2y + 1 = 0 is i.e. = p1 (say)
Circumcentre is C′ ≡ (3, 3 ) and centroid is G′ ≡ ( 4, 4 ) (9 + 4 ) 13
Y and algebraic perpendicular distance from (1, 4) to the line
3 x − 2y + 1 = 0 is
B (6, 6) 3 −8 + 1 −4
(0, 6) C i.e. = p2 (say)
9+4 13
6 −4 −24
∴ p1p 2 = × = <0
13 13 13
Hence, both statements are true and statement II is a correct
explanation for statement I.
78. Sum of algebraic distances from points A(1, 2), B(2, 3), C(6, 1) to
X
O A (6, 0) the line ax + by + c = 0 is zero (given), then
a + 2b + c (2a + 3b + c ) (6a + b + c )
+ + =0
∴ λ = O′ C ′ = ( 0 − 3 ) 2 + (6 − 3 ) 2 (a 2 + b 2 ) (a 2 + b 2 ) (a 2 + b 2 )
= 9 + 9 =3 2 ⇒ 9a + 6b + 3c = 0
and µ = C ′ G′ = ( 4 − 3 ) + ( 4 − 3 )
2 2 or 3a + 2b + c = 0
∴ Statement I is false.
= 1+1= 2
 1 + 2 + 6 2 + 3 + 1
Also, centroid of ∆ABC is  , 
∴ λ − µ = 16 and λ = 3µ
2 2
 3 3 
Hence, locus of ( λ , µ ) are i.e. (3, 2)
x 2 − y 2 = 16 and x = 3y ∴ Statement II is true.
Chap 02 The Straight Lines 181

79. Equation of AB is Also, (2a − 5, a 2 ) and ( 0, 0 ) on the same side of x + y − 3 = 0,


0 −1 then
y −1 = ( x − 0 ) ⇒ x + 2y − 2 = 0
2−0 2a − 5 + a 2 − 3
>0
Q | PA − PB| ≤ | AB| 0 + 0 −3
⇒| PA − PB| to be maximum, then A, B and P must be ⇒ a 2 + 2a − 8 < 0
collinear.
or (a + 4 )(a − 2 ) < 0
Solving x + 2y − 2 = 0
∴ a ∈ ( −4, 2 )
and 4 x + 3y + 9 = 0,
⇒ Statement I is false
 24 17
we get, P ≡ ,  Hence, statement I is false and statement II is true.
5 5
BD R
Hence, Statement I is false and Statement II is obviously true. 84. In ∆OBD, = ...(i)
sin ( π − 2 C ) sin θ
80. Statement II is false as the point satisfying such a property can DC R
be the excentre of the triangle. In ∆ODC, = ...(ii)
sin ( π − 2 B ) sin ( π − θ )
 π  π
Let L1 ≡ x cos  + y sin   − π = 0, BD sin 2 C
 9  9 From Eqs. (i) and (ii), =
DC sin 2 B
 8π   8π 
L2 ≡ x cos  + y sin   − π = 0 and
 9  9 (x1,y1)A

 13 π   8π 
L3 ≡ x cos  + y sin   − π = 0
 9   9
h R
and P ≡ ( 0, 0 ) 2C 2B
Length ⊥ from P to L1 = Length of ⊥ from P to L2 = Length of ⊥ R O R
from P to L3 = π and P lies inside the triangle. π–2C π–2B
θ π– θ
∴P( 0, 0 ) is incentre of triangle. (x2,y2)B C(x3,y3)
D
Hence, statement I is true and statement II is false.
81. Q Mid-point of (5, 1) and (−1, − 5) i.e. (2, − 2) lies on x + y = 0
and (slope of x + y = 0) × (slope of line joining (5, 1) ∴ Coordinates of D are
−6  x 2 sin 2 B + x 3 sin 2 C y 2 sin 2 B + y 3 sin 2 C 
and (−1, − 5)) = ( −1 ) ×  , 
−6  sin 2 B + sin 2 C sin 2 B + sin 2C 
∴ Statement I is true.
Let ( x, y ) be any point on AD, then equation of AD is
Statement II is also true.
 
Hence, both statements are true but statement II is not correct  x y 1
explanation of statement I.  x1 y1 1= 0
82. Equation of AC and BC are 3x + 2y = 0 and 2x + 3y + 6 = 0  
Q(3 )(2 ) + (2 )(3 ) = 12 > 0  x 2 sin 2 B + x 3 sin 2 C y 2 sin 2 B + y 3 sin 2 C 1
 sin 2 B + sin 2 C sin 2 B + sin 2 C 
∴Internal angle bisector of C is
 3 x + 2y   2 x + 3y + 6  x y
  = −  or 
 13   13   x 1 y 1

or 5 x + 5y + 6 = 0 x 2 sin 2 B + x 3 sin 2 C y 2 sin 2 B + y 3 sin 2 C


⇒ Statement I is true. 1 
Also, the image of A about the angle bisectors of angle B and C 1 = 0

lie on the side BC. (by congruence). sin 2 B + sin 2 C 
∴ Statement II is true.  x y 1 
Both statements are true and statement II is not correct or 
 x1 y 1 1  
explanation of statement I.
x 2 sin 2 B y 2 sin 2 B sin 2 B
83. Q Points (x1, y1 ) and (x 2, y 2 ) lie on the same or opposite sides of
 x y 1 
the line

+  x1 1 
ax + by + c = 0, as
y1 = 0
ax1 + by1 + c x 3 sin 2 C y 3 sin 2 C sin 2 C
> 0 or < 0
ax 2 + by 2 + c x y 1  x y 1
∴ Statement II is true. or (sin 2 B ) 
x1 y1 1
 + (sin 2 C ) x1 y1 1 = 0
 
x 2 y 2 1 x 3 y 3 1
182 Textbook of Coordinate Geometry

85. Let (x1, y1 ) be the coordinates of a point at unit distance from ∴ Equation of OP is y = x tan (90 ° − α )
each of the given lines. y
or cot α =
|3 x1 − 4y1 + 1| |8 x1 + 6y1 + 1| x
⇒ = 1 and =1
3 +4
2 2
82 + 62 x
∴ sin α =
⇒ 3 x1 − 4y1 + 1 = ± 5 and 8 x1 + 6y1 + 1 = ± 10 (x + y 2 )
2

⇒ 3 x1 − 4y1 − 4 = 0 ...(i) and cos α =


y
...(ii)
or 3 x1 − 4y1 + 6 = 0 ...(ii) (x + y 2 )
2

8 x1 + 6y1 − 9 = 0 ...(iii) Substituting the values of sin α and cos α from Eq. (i) in (i)
or 8 x1 + 6y1 + 11 = 0 ...(iv) then we get the required locus of P
(1) ∩ (3) x y
∴ + =k
⇒ x1 / 60 = y1 / − 5 = 1 / 50, y / (x + y ) x / (x 2 + y 2 )
2 2

6 1
∴ ( x1, y1 ) =  , −  ⇒ ( x 2 + y 2 ) ( x 2 + y 2 ) = kxy
 5 10
(1) ∩ (4) Squaring both sides, we get
⇒ x1 / − 20 = y1 / − 65 = 1 / 50, ( x 2 + y 2 ) 2 ( x 2 + y 2 ) = k 2x 2y 2
 2 13  x2 y2 
∴ ( x1, y1 ) =  − , −  or (x 2 + y 2 )2  2 2 + 2 2  = k 2
 5 10 x y xy 
(2) ∩ (3) or ( x 2 + y 2 ) 2 ( x −2 + y −2 ) = k 2
⇒ x1 / 0 = y1 / 75 = 1 / 50, ∴ ( x1, y1 ) = ( 0, 3 / 2 )
(2) ∩ (4)
87. Let the equation of variable line be
ax + by = 1. Then the coordinates of Ap will be
 8 3
⇒ x1 / − 80 = y1 / 15 = 1 / 50, ∴ ( x1, y1 ) =  − ,   1 p 
 5 10 Ap ≡  , 
 a + bp a + bp 
Hence, the required four points have the coordinates
6 1   2 13  3  8 3 

nx
 , −  ,  − , −  ,  0,  ,  − ,  .

y=
5 10  5 10  2  5 10

3x
Y
2x

y=
y=
86. Let ∠OAB = α y=
x
An
∴ OA = AB cos α and OB = AB sin α
A3
Q (OA ) 2 + (OB ) 2 = k 2 A2
A1
Y ax
+
by
=
X′ X
1
B O

P
Y′
90°– α α
X′ X (1 + p 2 )
O A ∴ OAp = ...(i)
Y′ | a + bp |
n
1
i.e. ( AB ) 2 (cos2 α + sin 2 α ) = k 2 Given, Σ =c
p =1 OAp
or AB = k
OA = k cos α and OB = k sin α
n
| a + bp |
then ⇒ Σ =c [from Eq. (1)]
x y p =1 (1 + p 2 )
∴ Equation of AB is + =1
k cos α k sin α  n   n 
1 p
x y ⇒ a ±

Σ  +b ± Σ  =c
or +
cos α sin α
=k ...(i)  p =1 (1 + p 2 )  
 p =1 (1 + p 2 ) 
 n
1   n
p 
Let P be the foot of perpendicular from O on AB.  Σ   Σ 
 p =1 (1 + p ) 2
 p =1 (1 + p 2 ) 
or a  ±  +b ±  =1
(x 2+y 2) c c
x    
   
   
α
y
Chap 02 The Straight Lines 183

So, line always passes through a fixed point whose coordinates y1


a+b
are n x1
then = [from Eq. (iv)]
  ( x12 + y12 ) y 
2
 n 1 n p  1 +  1
± Σ
 p =1 ± Σ
2   x1 
 (1 + p )
2 p = 1 (1 + p ) 
, ⇒ n = ay1 + bx1
 c c 
  Hence, locus of point R is bx + ay = n.
88. Let the equation of given n lines be 89. First equation can be expressed as
y = m r x + cr , (2 x + 3y − 5 ) cos θ + (3 x − 5y + 2 ) sin θ = 0
where r = 1, 2, 3, .... ,n ...(i) ⇒ (2 x + 3y − 5 ) + (3 x − 5y + 2 ) tan θ = 0
Let equation of line through origin O is It is clear that these lines will pass through the point of
y = mx ...(ii) intersection of the lines
2 x + 3y − 5 = 0 
Y′  ...(i)
3 x − 5y + 2 = 0 
y = mx
Rn for all values of θ.
y = mnx+cn Solving the system of Eq. (i), we get (1, 1).
Hence, the fixed point is P (1, 1 ). Let Q (α , β ) be the reflection of
R3 P (1, 1 ) in the line x + y = 2.
α − 1 β − 1 − 2 (1 + 1 − 2 )
R2 Then = = = 2 −2
y = m3x+c3 1 1 12 + 12
R
R1 y = m2x+c2 ∴ α = 2 − 1, β = 2 − 1
i.e. Q ≡ ( 2 − 1, 2 − 1 )
y = m1x+c1 If the required family of lines is
X′ X (2 cos θ + 3 sin θ ) x + (3 cos θ − 5 sin θ ) y = λ
O
in order that each member of the family pass through Q, we
Y′ have
λ = ( 2 − 1 ) (2 cos θ + 3 sin θ + 3 cos θ − 5 sin θ )
Solving Eqs. (i) and (ii) , we get
 cr mcr  λ = ( 2 − 1 ) (5 cos θ − 2 sin θ )
Rr ≡  , 
 m − mr m − mr  Hence, equation of required family is
2 2 (2 cos θ + 3 sin θ ) x + (3 cos θ − 5 sin θ ) y
 cr   m cr  = ( 2 − 1 ) (5 cos θ − 2 sin θ ).
∴ OR r =   + 
 m − mr   m − mr 
90. Let R (h, k ) be the foot of perpendicular from Q on OP.
 c 
=  r  (1 + m 2 ) ...(iii) Let equation of OP be
m − mr
Y
Let R ≡ ( x1, y1 )
y1
∴ y1 = mx1 ⇒ m = ...(iv) P(a,ma)
x1
mx x x
n n
1 y=
Given, = Σ R
OR r = 1 OR r x–a=0

m − mr
Σ  
n
n 1
⇒ = [from Eq . (iii)] θ
( x12 + y12 ) r = 1 cr  (1 + m 2 ) X′ X
O Q A(a,0)
  n  1 
1 n
 mr   Y′
= m  Σ  ±   + Σ  m  
(1 + m )  r = 1  cr   r = 1  cr  
2
y = mx
1 then k = mh
= (ma + b ) k
(1 + m 2 ) or m= ...(i)
h
 n
 1 n
 mr 
where a = Σ  ±  and b = Σ  m  and coordinates of P ≡ (a , ma )
r = 1  cr  r =1  cr 

184 Textbook of Coordinate Geometry

Q PQ is the bisector of OPA  3β 


Similarly, the coordinates of C are  0, 
∴ ∠APQ = ∠RPQ  3 − α
and ∠PAQ = ∠QRP = 90 ° Now, the equation of AD is
∴ PA = PR x (6 − α )
+ y =1 ...(i)
then | ma | = (h − a ) 2 + (k − ma ) 2 3 6β
2 and the equation of OU is
From Eq. (i), ak = (h − a ) 2 + k − ak 
h   h βx = αy ...(ii)
Solving Eqs. (i) and (ii), we get
⇒ a | k | = |(h − a )| (h 2 + k 2 ) 6α 6β
x= ,y =
Hence, required locus is 6+α 6+α
( x − a ) 2 ( x 2 + y 2 ) = a 2y 2  6α 6β 
Hence, coordinates of V are  , 
91. Let the coordinates of the vertex be (h, k ) and equations of the 6 + α 6 + α
bases be Then, the equation of CV is
x cos α r + y sin α r − pr = 0 where r = 1 , 2 , 3 , ...., n 6β 3β

and their lengths be respectively l1, l 2, l 3, ...., ln . 3β 6 + α 3 −α
y − = (x − 0)
Q Length of perpendicular from (h, k ) on 3 −α 6α
−0
x cos α r + y sin α r − pr = 0 is 6+α
| h cos α r + k sin α r − pr | 3β − 9α β
, ⇒ y − = x
(cos2 α + sin 2 α ) 3 − α 6 α (3 − α )
i.e. | h cos α r + k sin α r − pr | 3β  x
⇒ y = 1 − 
Given, sum of areas of all triangles = constant (3 − α )  2
then which pass through the point (2, 0) for all values of (α , β ).
n
1
Σ lr ⋅ | h cos αr + k sin αr − pr | = C ′ 93. Let the equation of the variable line through `O' be
r =1 2
x y
n
1 =
⇒ Σ . lr . (± (h cos αr + k sin αr − pr )) = C ′ cos θ sin θ
r =1 2
and let OR = r1, OS = r2 and OP = r3
 n 1   n 1 
⇒ h  Σ ± lr cos α r  + k  Σ ± lr . sin α r  L2
 r = 1 2   r = 1 2  Y
n
1
= Σ ± lr . pr + C ′
r =1 2 S
⇒ Ah + Bk = − C R y=c
L1
∴ Required locus is
P(h,k)
Ax + By + C = 0
X′ X
where A, B, C are constants. O
ax
+

92. The equation of BU is


by

Y′
=

0 −β
1

y −β = (x − α )
6 −α Then coordinates of R , S and P are :
 6β  R (r1 cos θ, r1 sin θ ), S (r2 cos θ, r2 sin θ ), P (r3 cos θ, r3 sin θ )
So that the coordinates of D are  0,  R lies on L1 and S lies on L2.
 6 − α
Let L1 ≡ y − c = 0
Y
and L2 ≡ ax + by − 1 = 0
C ∴ r1 sin θ = c and ar2 cos θ + br2 sin θ = 1
c 1
D ∴ r1 = and r2 =
U(α,β) sin θ a cos θ + b sin θ
V From the given condition
m+n m n
= +
X′ X r3 r1 r2
O (2,0) A(3,0) B(6,0)
m + n m sin θ
Y′
⇒ = + n (a cos θ + b sin θ ) ...(i)
r3 c
Chap 02 The Straight Lines 185

Let the coordinates of P be (h, k ), then Let P ≡ (α , β )


h = r3 cos θ, k = r3 sin θ
Y
mr sin θ mx
From Eq. (i), m + n = 3 + n (ar3 cos θ + br3 sin θ ) y=
c
mk P B(b,mb)
⇒ m+n= + n (ah + bk )
c
my R(h,k)
Locus of P is n (ax + by ) + = (m + n )
c
m X′ X
⇒ n (ax + by − 1 ) + (y − c ) = 0 O (b,0)Q A(a,0)
c
m
⇒ (ax + by − 1 ) + (y − c ) = 0 Y′
nc
Q P be the foot of perpendicular from A on y = mx, then
 m
⇒ L2 + λL1 = 0  where, λ =  α − a β − 0 − ( 0 − ma )
 nc  = =
−m 1 (1 + m 2 )
Hence, locus of P is a point of intersection of L1 and L2.
a am
94. The given lines are ∴ α= , β=
1 + m2 1 + m2
x + 3y + 2 = 0 ...(i)
 a am 
2x + y + 4 = 0 ...(ii) i.e. P ≡ , 
x −y −5 = 0 ...(iii)  1 + m2 1 + m2 
Equation of the line passing through A ( − 5 , − 4 ) and making ∴ Equation of PQ is
an angle θ with the positive direction of X-axis is am
−0
x+5 y +4 1 + m2
= = r ( AB, AC , AD ) ...(iv) y −0= (x − b )
cos θ sin θ a
− b
∴ Points ( − 5 + AB cos θ, − 4 + AB sin θ ), 1 + m2
( − 5 + AC cos θ, − 4 + AC sin θ ) and a
( − 5 + AD cos θ, − 4 + AD sin θ ) lie on Eqs. (i), (ii) and (iii) ⇒ (y − mx ) + am − (1 + m 2 ) y = 0 ...(ii)
b
respectively.
Adding Eqs. (i) and (ii), then
( − 5 + AB cos θ ) + 3 ( − 4 + AB sin θ ) + 2 = 0
a
⇒ AB (cos θ + 3 sin θ ) = 15 (y − mx + k ) + (mh − k − (1 + m 2 ) y ) = 0
b
15
⇒ = cos θ + 3 sin θ ⇒ (mh − k − (1 + m 2 ) y ) + λ (y − mx + k ) = 0
AB
10  a
Similarly, = 2 cos θ + sin θ  where, λ = 
AC  b
6 Hence PQ pass through a fixed point.
and = cos θ − sin θ
AD For fixed point
From given condition mh − k − (1 + m 2 ) y = 0, y − mx + k = 0
2 2 2
 15   10   6  mh − k h + mk
  +  =  y = ,x=
 AB   AC   AD  (1 + m 2 ) (1 + m 2 )
we get (cos θ + 3 sin θ ) 2 + (2 cos θ + sin θ ) 2 = (cos θ − sin θ ) 2  h + mk mh − k 
Hence, fixed point is  , .
⇒ 4 cos2 θ + 9 sin 2 θ + 12 sin θ cos θ = 0  1 + m2 1 + m2 
⇒ (2 cos θ + 3 sin θ ) 2 = 0 96. Given lines are parallel and distance between them < 2
2 Given lines are
∴ tan θ = −
3 2x + y = 3 ...(i)
Hence the equation of the line from Eq. (iv) is and 2x + y = 5 ...(ii)
2 Equation of any line through Eqs. (ii) and (iii) is
y + 4 = − ( x + 5 ) ⇒ 2 x + 3y + 22 = 0
3 y − 3 = m (x − 2)
95. Q A, R and B are collinear or y = mx − 2m + 3 ...(iii)
k − 0 mb − 0 Let line (iii) cut lines (i) and (ii) at A and B respectively.
then, =
h −a b −a Solving Eqs. (i) and (iii), we get
a  2m 6 − m 
∴ k − am + mh − k = 0 ...(i) A≡ , 
b  m + 2 m + 2
186 Textbook of Coordinate Geometry

and solving Eqs. (ii) and (iii), we get Also tanθ = − 2 (slope of 2 x + y = 5)
 2m + 2 m + 6 Now, equation of required lines are
B≡ , 
 m + 2 m + 2 y − 3 = tan (θ ± α ) ( x − 2 )
According to question AB = 2  tan θ ± tan α 
⇒ y −3 =   (x − 2)
⇒ ( AB ) 2 = 4  1 m tan θ tan α 
2 2 1
 2   2m  ( −2 ) ±
⇒   +  =4 2 (x −2)
 m + 2  m + 2 ⇒ y −3 =
 1
1 m ( −2 )  
⇒ 1 + m 2 = m 2 + 4m + 4 ...(iv)  2
Case I : When m is finite (line is not perpendicular to X-axis)  1
 −2 ±

then from Eq. (iv).  2
⇒ y −3 = (x − 2)
1 = 4m + 4 1 m ( −1 )
3
∴ m=−  1
4 ⇒ (1 m ( −1 )) (y − 3 ) =  −2 ±  (x − 2)
 2
Case II : When m is infinite (line is perpendicular to X-axis)
3
then from Eq. (iv), ⇒ x − 2 = 0 and 2y − 6 = − (x − 2)
1 4 4 2
+ 1 =1 + + i.e. x − 2 = 0 and 3 x + 4y − 18 = 0
m2 m m2
0 + 1 =1 + 0 + 0 Aliter II : Any line through (2 , 3 ) is
1 = 1 which is true x −2 y −3
= =r
Hence m → ∞ acceptable. cosθ sin θ
Hence, equation of the required lines are Suppose this line cuts 2 x + y = 5 and 2 x + y = 3 at D and C
3 respectively but given DC = 2
y − 3 = − (x − 2)
4 then D ≡ (2 + r cosθ, 3 + r sin θ )
y −3 and C ≡ (2 + (r + 2 ) cosθ, 3 + (r + 2 ) sin θ )
and = x −2 ⇒x −2 = 0
∞ Q D and C lies on
i.e. 3 x + 4y = 18 and x − 2 = 0 2 x + y = 5 and 2 x + y = 3
Aliter I : then 2 (2 + r cosθ ) + (3 + r sin θ ) = 5 ... (v)
Q 2 x + y = 3 cuts Y-axis at ( 0, 3 ) and line 2 x + y = 5 cuts and 2 (2 + (r + 2 ) cosθ ) + (3 + (r + 2 ) sin θ ) = 3 ... (vi)
Y-axis at ( 0, 5 ) Subtracting Eq. (v) from Eq. (vi), then
Y 4 cosθ + 2 sin θ = − 2
or 2 cosθ + sin θ = − 1
 2  θ    θ 
2  1 − tan     2 tan   
 2   2
2
C α  +  = −1

 2  θ   2  θ 
+     +  
  2  
1 tan 1 tan
D P(2,3)   2  
 θ  θ  θ
A ⇒ 2 − 2 tan 2   + 2 tan   = − 1 − tan 2  
 2  2  2
M
X′ X  θ  θ
O α 2 ⇒ tan 2   − 2 tan   − 3 = 0
2x+y=5  2  2
Y′ B  θ
2x+y=3 ∴ tan   = − 1 or 3
 2
Therefore intercept on Y-axis is 2. 3
∴ tanθ = ∞ or −
Also, AM = distance between parallel lines 4
| −5 + 3| 2 ∴ Required lines are
= =
2 +1
2 2 5 y − 3 = ∞ (x − 2)
3
∴ MB = ( AB ) 2 − ( AM ) 2 = 4 −
4
=
4 and y − 3 = − (x − 2)
5 5 4
AM 1 i.e. x −2 = 0
then tanα = = and 3 x + 4y − 18 = 0
MB 2
Chap 02 The Straight Lines 187

97. If I be the incentre of ∆OAB. 99. The line passing through the intersection of lines
If inradius =r ax + 2by + 3b = 0 and bx − 2ay − 3a = 0 is
then ID = IE = IF = r ax + 2by + 3b + λ(bx − 2ay − 3a ) = 0
Y
⇒(a + bλ ) x + (2b − 2aλ )y + 3b − 3 λa = 0
As this line is parallel to X-axis.
a
B 1,
1 ∴ a + bλ = 0 ⇒ λ = −
√3 b
F r r E a
⇒ ax + 2by + 3a − (bx − 2ay − 3a ) = 0
I b
15°
15°
r 2a 2 3a 2
X′ 15° 15°
X ⇒ ax + 2by + 3b − ax + y + =0
O D A(2,0) b b
 2a 2  3a 2
Y′ y 2b +  + 3b + =0
 b  b
If P at I,then
d ( P , OA ) = d ( P , OB ) = d ( P , AB ) = r  2b 2 + 2a 2   3b 2 + 3a 2 
y  = − 
But d ( P , OA ) ≤ min{d ( P , OB ), d ( P , AB )}  b   b 
which is possible only when P lies in the ∆OIA. −3(a 2 + b 2 ) −3
ID r y = =
∴ tan15 ° = = 2(b 2 + a 2 ) 2
OD 1 3
⇒ r = (2 − 3 ) So, it is units below X-axis.
2
1
∴ Required area = ⋅ 2 ⋅ r = r = (2 − 3 ) sq units. 100. Y
2
98. Let A ≡ (x1, y1 ), B ≡ (x 2, y 2 ) and C ≡ (x 3, y 3 ) are the vertices of P(0, b)
a triangle ABC and P ≡ (a1, b1 ), Q ≡ (a 2, b2 ) and R ≡ (a 3, b3 ) are
the vertices of triangle PQR.
Equation of perpendicular from A to QR is A(3, 4)
(a − a 3 )
y − y1 = − 2 ( x − x1 )
(b2 − b3 )
Q(a, 0)
or (a 2 − a 3 ) x + (b2 − b3 ) y − x1 (a 2 − a 3 ) − y1(b2 − b3 ) = 0 …(i) X
O
Similarly, equations of perpendiculars from B to RP and C to
PQ are respectively, Q A is the mid-point of PQ, therefore
(a 3 − a1 ) x + (b3 − b1 ) y − x 2 (a 3 − a1 ) − y 2 (b3 − b1 ) = 0 ...(ii) a+0 0+b
= 3, =4
and (a1 − a 2 ) x + (b1 − b2 ) y − x 3 (a1 − a 2 ) − y 3 (b1 − b2 ) = 0 ...(iii) 2 2
Given that lines (i), (ii) and (iii) are concurrent, then adding, ⇒ a = 6, b = 8
we get x y
( x 2 − x 3 ) a1 + ( x 3 − x1 ) a 2 + ( x1 − x 2 ) a 3 + (y 2 − y 3 ) b1 + ∴Equation of line is + = 1
6 8
(y 3 − y1 )b2 + (y1 − y 2 ) b3 = 0 ...(iv) or 4 x + 3y = 24
Now, equation of perpendicular from P to BC is
101. Clearly for point P,
(x − x 3 )
y − b1 = − 2 ( x − a1 ) y
(y 2 − y 3 ) y=3x
or ( x 2 − x 3 ) x + (y 2 − y 3 ) y − a1
( x 2 − x 3 ) − b1(y 2 − y 3 ) = 0 ...(v)
P(a, a2)
Similarly, equations of perpendiculars from Q to CA and R to
AB are respectively,
( x 3 − x1 ) x + (y 3 − y1 ) y − a 2 y= x
2
( x 3 − x1 ) − b2 (y 3 − y1 ) = 0 ...(vi)
O
and ( x1 − x 2 ) x + (y1 − y 2 ) y − a 3 x
( x1 − x 2 ) − b3 (y1 − y 2 ) = 0 ...(vii)
Adding Eqs. (v), (vi) and (vii), we get a
a 2 − 3a < 0 and a 2 − >0
LHS = 0 (identically) [ from Eq. (iv)] 2
Hence perpendiculars from P to BC, Q to CA and R to AB are 1
concurrent. ⇒ <a <3
2
188 Textbook of Coordinate Geometry

102. Point of intersection of L1 and L2 is A(0, 0). (C) Lines are not concurrent or not parallel, then
6
Also P( −2, − 2 ), Q(1, − 2 ) k ≠ 5, k ≠ − 9, k ≠ −
5
A 5
(0, 0) ∴ k=
6
(D) The given lines do not form a triangle if they are

L2
=0

:2
concurrent or any two of them are parallel.
–x

x–
:y

y=
∴ k = 5, k = − 9, k = −
1
L

0
5
R 3−4 −1
(– 2, –2) P L3 : y+2=0 Q (1, –2) 105. Slope of PQ = =
k −1 k −1
∴ Slope of perpendicular bisector of PQ = (k − 1 )
Q AR is the bisector of ∠PAQ, therefore R divides PQ in the
 k + 1 7
same ratio as AP : AQ. Also mid-point of PQ  , 
 2 2
Thus PR : RQ = AP : AQ = 2 2 : 5
∴ Statement I is true. ∴ Equation of perpendicular bisector is
7  k + 1
Statement II is clearly false. y − = (k − 1 )  x − 
2  2 
103. Given : The coordinates of points P , Q, R are (−1, 0), (0, 0),
(3, 3 3 ) respectively. ⇒ 2y − 7 = 2(k − 1 ) x − (k 2 − 1 )
Y ⇒ 2 (k − 1 ) x − 2y + (8 − k 2 ) = 0
R (3,3Ö3)
8 −k2
M ∴ Y -intercept = − = −4
−2
⇒ 8 − k 2 = −8 or k 2 = 16 ⇒ k = ± 4
106. If the line p(p 2 + 1)x − y + q = 0
2p/3 p/3
X¢ X and ( p 2 + 1 ) 2 x + ( p 2 + 1 )y + 2q = 0
P (–1, 0) Q (0, 0)
are perpendicular to a common line, then these lines must be
Y¢ parallel to each other,
y 2 − y1 3 3 p( p 2 + 1 ) (p 2 + 1)2
Slope of equation QR = = ∴ m1 = m2 ⇒ − =− 2
x 2 − x1 3 −1 p +1
π ⇒ ( p 2 + 1 )( p + 1 ) = 0
⇒ tanθ = 3 ⇒ θ =
3 ⇒ p = −1
π ∴p can have exactly one value.
⇒ ∠RQX =
3 b
π 2π
107. Slope of line L = −
∴ ∠RQP = π − = 5
3 3 3
Slope of line K = −
Let QM bisects the ∠PQR, c
2π Line L is parallel to line K .
∴ Slope of the line QM = tan =− 3
3 b 3
⇒ = ⇒ bc = 15
∴ Equation of line QM is (y − 0 ) = − 3( x − 0 ) 5 c
⇒ y = − 3x ⇒ 3x + y = 0 (13, 32) is a point on L.
13 32 32 8
104. (A)Q L1, L2, L3 are concurrent, then ∴ + =1 ⇒ =−
5 b b 5
1 3 −5 3
⇒ b = − 20 ⇒ c = −
3 −k −1 = 0 ⇒k = 5 4
5 2 −12 Equation of K : y − 4 x = 3
(B) slope of ( L1 ) = slope of ( L2 ) ⇒ 4x − y + 3 = 0
1 3 |52 − 32 + 3|
⇒ − = ∴ k = −9 Distance between L and K =
3 k 17
and slope of ( L3 ) = slope of ( L2 ) =
23
5 3 6 17
⇒ − = ∴ k=−
2 k 5
Chap 02 The Straight Lines 189

108. Let the slope of line L be m. 111. Suppose B(0, 1) be any point on given line and coordinate of A
m + 3 
 is ( 3, 0 ). So, equation of
Then = 3
1 − 3m
Y (0, 1)
B

Ö3x+y=1
L
(0, 1)
X¢ X A (3, 0)
O 60°
(3, –2)
Y¢ B¢ (0, –1)
⇒ m + 3 = ± ( 3 − 3m ) −1 − 0 y −0
Reflected ray is =
⇒ 4m = 0 or 2m = 2 3 0− 3 x− 3
⇒ m = 0 or m = 3 ⇒ 3y = x − 3
Q L intersects X-axis,
 −c −c 
∴ m= 3 112. The intersection point of two lines is  , 
a + b a + b
∴Equation of L is y + 2 = 3( x − 3 )
 −c −c 
or 3 x − y − (2 + 3 3 ) = 0 Distance between (1, 1) and  ,  <2 2
a + b a + b
109. L3 2
L1  c 
⇒ 21 +  <8
 a + b
=0 P (–2, –2) c
y– x ⇒ 1+ <2
R (–1, –2) a+b
O ⇒ a + b −c > 0
(0, 0) 2x+
y= 113. Let P , Q, R, be the vertices of ∆PQR
0
P (2, 2)
L2

L1 : y − x = 0, L2 : 2 x + y = 0, L3 : y + 2 = 0
On solving the equation of lines L1 and L2, we get their point of
intersection (0, 0) i.e. origin O.
On solving the equation of lines L1 and L3,
we get P = ( −2, − 2 )
Similarly, we get Q = ( −1, − 2 )
R (7, 3)
We know that bisector of an angle of a triangle, divide the Q (6, –1) S
opposite side the triangle in the ratio of the sides including the
angle [Angle Bisector Theorem of a Triangle] Since, PS is the median, S is mid-point of QR
PR OP ( −2 ) + ( −2 ) 2
2 2
2
 7 + 6 3 − 1  13 
∴ = = = So, S = ,  =  , 1
RQ OQ ( −1 ) + ( −2 )
2 2 5  2 2  2 
2 −1 2
110. Let the joining points be A(1, 1) and B(2, 4). Now, slope of PS = =−
13 9
Let point C divides line AB in the ratio 3 : 2. So, by section 2−
2
formula we have
Since, required line is parallel to PS therefore slope of required
 3 × 2 + 2 × 1 3 × 4 + 2 × 1  8 14 line = slope of PS Now, eqn of line passing through (1, − 1 ) and
C = , = , 
 3+2 3+2  5 5  2
having slope − is
 8 14 9
Since Line 2x + y = k passes through C  ,  2
y − ( −1 ) = − ( x − 1 )
5 5 
9
∴C satisfies the equation 2x + y = k. 9y + 9 = − 2 x + 2
2 + 8 14
⇒ + = k ⇒k = 6 ⇒ 2 x + 9y + 7 = 0
5 5
190 Textbook of Coordinate Geometry

114. Given lines are 116. Total number of integral points inside the square OABC
4ax + 2ay + c = 0 = 40 × 40 = 1600
5bx + 2by + d = 0 Number of integral points on AC
The point of intersection will be = Number of integral points on OB
x −y 1 = 40 [namely (1, 1), (2, 2) … (40, 40)]
= =
2ad − 2bc 4ad − 5bc 8ab − 10ab
2(ad − bc ) bc − ad (0, 41) C
(41, 41)
⇒ x= = B
−2ab ab
5bc − 4ad 4ad − 5bc
⇒ y = =
−2ab 2ab
Q Point of intersection is in fourth quadrant so x is positive
and y is negative.
Also distance from axes is same
O A
So x = − y (Q distance from X-axis is −y as y is (0, 0) (41, 0)
negative)
bc − ad 5bc − 4ad
= ⇒ 3bc − 2ad = 0 ∴ Number of integral points inside the ∆OAC
ab 2ab
115. Let the point P be (x, y ) 1600 − 40
= = 780
x − y x + y 2
Then d 1( P ) =  and d 2( P ) = 
 2   2  117. x–y+1=0
D C
For P lying in first quadrant x > 0, y > 0.
Also 2 ≤ d 1( P ) + d 2( P ) ≤ 4
x −y x+y

m=0

=0
⇒ 2≤ + ≤4

7x–y–5
2 2 O (–1, –2)
7x–y+
If x > y , then
x −y + x + y
2≤ ≤ 4 or 2 ≤ x ≤ 2 2
2
A x–y+l=0 B
If x < y , then
y −x+x+y Let other two sides of rhombus are
2≤ ≤ 4 or 2 ≤ y ≤ 2 2
2 x −y + λ = 0
The required region is the shaded region in the figure given and 7x − y + µ = 0
below. then O is equidistant from AB and DC and from AD and BC
Y
∴ | −1 + 2 + 1 | = | −1 + 2 + λ | ⇒ λ = −3
y=x and | −7 + 2 − 5 | = | −7 + 2 + µ| ⇒ µ = 15
y=2 2 ∴Other two sides are
x −y −3 = 0
y= 2 and 7 x − y + 15 = 0
On solving the equation of sides pairwise, we get the vertices
 1 −8   −7 −4 
X as  ,  , (1, 2),  ,  , ( −3, − 6 )
O x= 2 x=2 2 3 3   3 3

∴ Required area = (2 2 ) 2 − ( 2 ) 2 = 8 − 2 = 6 sq units


CHAPTER

03
Pair of
Straight Lines
Learning Part
Session 1
● Introduction ● Homogeneous Equation in Two Variables
Session 2
2
● Angle between the Pair of Lines ax + 2hxy + by 2
Session 3
● Bisectors of the Angle between the Lines Given by a Homogeneous Equation

Session 4
● General Equation of Second Degree ● Important Theorems

Session 5
● To Find the Point of Intersection of Lines Represented by

ax 2 + 2hxy + by 2 + 2 gx + 2 fy + c = 0 with the Help of Partial Differentiation


● Removal of First Degree Term
● Equation of the Lines Joining the Origin to the Points of Intersection of a
Given Line and a Given Curve

Practice Part
● JEE Type Examples
● Chapter Exercises

Arihant on Your Mobile !


Exercises with the #L
symbol can be practised on your mobile. See inside cover page to activate for free.
Session 1
Introduction, Homogeneous Equation in
Two Variables
Introduction i.e. ( x − 4y ) ( x − 2y ) = 0
⇒ x − 4y = 0 and x − 2y = 0
Let the equation of two lines be Aliter :
ax + by + c = 0 ...(i) We have, x 2 − 6xy + 8y 2 = 0
and a 1 x + b 1y + c 1 = 0 ...(ii) 6y ± ( − 6y )2 − 4 ⋅ 8y 2
By Shri Dharacharya method, x =
Hence, (ax + by + c ) (a 1 x + b 1 y + c 1 ) = 0 is called the joint 2
equation of lines Eqs. (i) and (ii) and conversely, if joint ⇒ x = 3y ± y (9 − 8)
equation of two lines be
x = 3y ± y
(ax + by + c ) (a 1 x + b 1 y + c 1 ) = 0, ∴ x = 4y and x = 2y
then their separate equations will be Hence, the lines are
ax + by + c = 0 and a 1 x + b 1 y + c 1 = 0 x − 4y = 0 and x − 2y = 0.

Remark
In order to find the joint equation of two lines, make RHS of two
lines equal to zero and then multiply the two equations.
Homogeneous Equation in
y Example 1 Find the joint equation of lines y = x and
Two Variables
y = − x. An equation of the form
Sol. The given lines can be rewritten as a 0 y n + a 1 y n − 1 x + a 2 y n − 2 x 2 + .... + a n x n = 0 …(i)
x − y = 0 and x + y = 0
∴ Joint equation of lines is ( x − y ) ( x + y ) = 0 in which the sum of the powers of x and y in every term
is the same (here n), is called a homogeneous equation
or x2 − y2 = 0
(of degree n).
Wrong process : Since, the lines are
We will prove that Eq. (i) represents n straight lines
y = x and y = − x
passing through the origin.
Then joint equation is y2 = − x2
a 0 y n + a 1 y n − 1 x + a 2 y n − 2 x 2 + .... + a n x n = 0
⇒ x2 + y2 = 0
This process is wrong, since RHS of two equations are not Dividing each term by x n , we get
equal to zero. n n −1 n −2
y  y  y 
a 0   + a1   + a2   + ... + a n = 0
Remark x x x
In order to find the separate equations of two lines when their joint y
equation is given, first of all make RHS equal to zero and then Above is an equation of nth degree in . Let the roots of
resolve LHS into two linear factors or use Shri Dharacharya method. x
The two factors equated to zero will give the separate equations of this equation be m 1 , m 2 , m 3 , ..., m n .
lines. Then, the above equation will be identical with
y Example 2 Find the separate equation of lines y  y  y  y 
a 0  − m1   − m2   − m 3  ...  − m n  = 0
represented by the equation x 2 − 6 xy + 8 y 2 = 0. x  x  x  x 
Sol. Separate equation of lines represented by the equation ⇒ a 0 (y − m 1 x ) (y − m 2 x ) (y − m 3 x ) .... (y − m n x ) = 0
x 2 − 6xy + 8y 2 = 0
Chap 03 Pair of Straight Lines 193

Hence, Eq. (i) represents n straight lines pass through the origin, let their equations be
y = m1x and y = m2x
y − m 1 x = 0, y − m 2 x = 0,
then, ( y − m1x ) and ( y − m2x )
y − m 3 x = 0,..., y − m n x = 0 must be factors of ax 2 + 2hxy + by 2 = 0
all of which clearly pass through the origin. then ax 2 + 2hxy + by 2 = b( y − m1x ) ( y − m2x )
Corollary : Since, ax 2 + 2hxy + by 2 = 0 is a homogeneous [Making coefficient of y 2 equal on both sides]
equation of second degree, it represents two straight lines Now, comparing both sides, we get
through origin. The given equation is 2h = − b ( m1 + m2 ) and a = bm1m2
2h a
ax 2 + 2hxy + by 2 = 0 ...(i) ∴ m1 + m2 = − and m1m2 =
b b
2
Dividing by x , we get
2
y Example 3 Find the condition that the slope of one
y  y  of the lines represented by ax 2 + 2hxy + by 2 = 0 should
a + 2h   + b   = 0
x x be n times the slope of the other.
2
y  y  Sol. Let the lines represented by
⇒ b   + 2h   + a = 0 ...(ii) ax 2 + 2hxy + by 2 = 0 are y = m1x and y = m 2 x .
x x
2h
y Therefore, m1 + m 2 =− ...(i)
Putting =m b
x a
and m1m 2 = ...(ii)
then, bm 2 + 2hm + a = 0 ...(iii) b
If m 1 and m 2 be two roots, then Given, m2 = nm1
2h
2h coefficient of xy From Eq. (i), m1 + nm1 =−
m1 + m2 = − =− b
b coefficient of y 2 2h
∴ m1 = − …(iii)
a coefficient of x 2 b (1 + n )
and m 1m 2 = =
b coefficient of y 2 a
and from Eq. (ii), m1(nm1 ) =
b
∴ |m 1 − m 2 | = {(m 1 + m 2 ) 2 − 4m 1m 2 } a
∴ nm12 =
2 b
= (h 2 − ab ) 2
| b|  − 2h  a
⇒ n  = [from Eq. (iii)]
Thus, y = m 1 x and y = m 2 x are two straight lines which  b(1 + n ) b
are given by Eq. (i). Also, from Eq. (iii), 4nh 2 a
⇒ =
b (1 + n )
2 2
b
− 2h ± 2 (h 2 − ab )
m= ⇒ 4nh 2 = ab(1 + n )2 ...(iv)
2b
This is the required condition.
− h ± (h 2 − ab ) y  y Corollary : If slope of one line is double of the other, then
= = Q m = x 
b x   put n = 2 in Eq. (iv), we have
8h 2 = 9ab.
∴ by = {− h + (h 2 − ab ) }x

and by = {− h − (h 2 − ab ) } x y Example 4 If the slope of one of the lines


represented by ax 2 + 2hxy + by 2 = 0 be the nth power
are two lines represented by Eq. (i). 1 1
(i) The lines are real and distinct, if h 2 − ab > 0. of the other, prove that, (ab ) n n +1
+ (a b )
n n +1
+ 2h = 0.
(ii) The lines are coincident, if h 2 − ab = 0.
(iii) The lines are imaginary, if h 2 − ab < 0. Sol. Let m and mn be the slopes of the lines represented by
ax 2 + 2hxy + by 2 = 0
Remarks
2h
1. In further discussions, we will consider only real cases. then m + mn = − ...(i)
b
2. Two very useful identities When lines represented by
ax 2 + 2hxy + by 2 = 0
194 Textbook of Coordinate Geometry

a a y Example 6 Find the condition that one of the lines


and m ⋅ mn = ⇒ mn + 1 =
b b given by ax 2 + 2hxy + by 2 = 0
1
 a  (n + 1) may be perpendicular to one of the lines given by
⇒ m=  ...(ii)
b a ′ x 2 + 2h ′ xy + b ′ y 2 = 0.
Substituting the value of m from Eq. (ii) in Eq. (i), then Sol. Since, both pair are passing through origin, let y = mx be
1  n  one of the lines represented by
 
 a (n + 1)  a   n + 1 2h
  +  =− ax 2 + 2hxy + by 2 = 0
b b b
1 n n 1 then, ax 2 + 2hx (mx ) + b (mx )2 = 0
n +1 n +1 n +1 n +1 ⇒ bm 2 + 2hm + a = 0
⇒ a ⋅b +a ⋅b + 2h = 0 ...(i)
1
Corollary : If slope of one line is square of the other, then then, y = − x be one of the line represented by
put n = 2, then m
1 1 a ′ x 2 + 2h ′ xy + b ′ y 2 = 0
(ab 2 ) 3 + (a 2b ) 3 = − 2h  1 
On cubing both sides, we get Qy = − m x is perpendicular to 
1 1  1 1  
y = mx and passing through origin
ab 2 + a 2b + 3 (ab 2 ) 3 (a 2b ) 3 (ab 2 ) 3 + (a 2b ) 3  = − 8h 3
  2
   1   1 
then a ′ x 2 + 2h ′ x  − x + b′ − x = 0
⇒ ab (a + b ) + 3ab ( − 2h ) = − 8h 3  m   m 
(a + b ) 8h 2 ⇒ a ′ m 2 − 2h ′ m + b ′ = 0 ...(ii)
∴ + = 6.
h ab On solving Eqs. (i) and (ii), by cross-multiplication rule i.e.
m2 m 1 m2 m
y Example 5 Find the product of the perpendiculars
b 2h a b 2h
drawn from the point ( x 1 , y 1 ) on the lines
ax 2 + 2hxy + by 2 = 0. a′ − 2h ′ b ′ a′ − 2h ′
m2 m 1
Sol. Let the lines represented by ax 2 + 2hxy + by 2 = 0 be ⇒ = =
2hb ′ + 2h ′ a aa ′ − bb ′ − 2h ′ b − 2a ′ h
y = m1x and y = m 2 x .
(hb ′ + h ′ a ) (bb ′ − aa ′ )
Therefore, m1 + m 2 = −
2h
and m1m 2 =
a ∴ m2 = − ,m =
b b (h ′ b + a ′ h ) 2 (a ′ h + h ′ b )
The lengths of the perpendiculars from ( x 1, y1 ) on these On eliminating m, we obtain
lines are 4 (ha ′ + h ′ b ) (h ′ a + hb ′ ) + (bb ′ − aa ′ )2 = 0.
| y1 − m1x 1| | y1 − m 2 x 1|
and
1 + m1
2
1 + m 22 y Example 7 Show that the centroid ( x ′ , y ′ ) of the
| y1 − m1x 1| | y1 − m 2 x 1| triangle with sides ax 2 + 2hxy + by 2 = 0 and
Their product = ×
1+ m12 1 + m 22 lx + my = 1, is given by
x′ y′ 2
=
|(y1 − m1x 1 ) (y1 − m 2 x 1 )| = = .
bl − hm am − hl 3 (am − 2hlm + bl 2 )
2
(1 + m12 ) (1 + m 22 )
Sol. Let the lines represented by
| y12 − (m1 + m 2 ) x 1y1 + m1m 2 x 12 |
= ax 2 + 2hxy + by 2 = 0
1 + m12 + m 22 + m12 m 22
be y = m1x
| y12 − (m1 + m 2 ) x 1y1 + m1m 2 x 12 | and y = m2x .
=
1 + (m1 + m 2 )2 − 2m1m 2 + (m1m 2 )2 Y
B
2h a
| y12+ x 1y1 + x 12 |
2x

lx+

= b b
y=m

 4h 2 2a a 2 
y=

1 + 2 − +  G(x′,y′ )
1

 b b b2 
x A
y=m 1
| ax 12 + 2hx 1y1 + by12 | X
= O
{(a − b ) + 4h }
2 2
Chap 03 Pair of Straight Lines 195

2h Y
Therefore, m1 + m 2 = − B
b
a

2x
m1m 2 =

lx+
and

y=m
b

m
y=
1
Coordinates of A and B are
 1 m1   1 m2  x A
 ,  and  ,  y=m 1
 l + mm1 l + mm1   l + mm 2 l + mm 2  O
X

Since, centroid = ( x ′ , y ′ ),
 1 1  Coordinates of A and B are
 + + 0
l + mm l + mm  −n − nm1   −n − nm 2 
then, x ′ =  1 2   ,  and  , ,
 3   l + mm1 l + mm1   l + mm 2 l + mm 2 
 
  respectively.
 2l + m(m1 + m 2 )  Then, required area
= 2 
 3 {l + ml (m1 + m 2 ) + m m1 m 2 ) }
2 1 − n   − nm 2   − n   − nm1 
=    −   
  2 l + mm1   l + mm 2   l + mm 2   l + mm1 
 2l −
2hm 
[Q if coordinates are (0, 0), ( x 1, y1 )and ,
 b 
=  1
 2hml m a 2 ( x 2 , y 2 ) , then area = | x 1y 2 − x 2y1 |]
 3 l 2 − +   2
  b 
 b
1 n 2 (m 2 − m1 ) 
2 (bl − hm ) = 2 
= ⋅ ...(i) 2l + lm (m1 + m 2 ) + m 2m1m 2
3 (am − 2hlm + bl 2 )
2

 m1  1 n 2 (m1 + m 2 )2 − 4m1m 2 
+
m2
+ 0 = 2 
 2l + lm (m1 + m 2 ) + m 2m1m 2
l + mm l + mm
and y ′ =  1 2 
 3 
   4h 2 4a 
  n2  2 − 
1  b b
 l (m1 + m 2 ) + 2mm1m 2  =
= 2  2 2hlm m 2a
 3 {l + lm (m1 + m 2 ) + m m1m 2 ) }
2
l2 − +
b b
 
 −
2hl 2ma
+ 
 b b  n 2 (h 2 − ab )
= =
 2 2hlm m 2 a   |(am 2 − 2hlm + bl 2 )|
 3 l − + 
  b  
 b
y Example 9 Show that the two straight lines
2 (am − hl )
= ⋅ ...(ii) x 2 (tan 2 θ + cos 2 θ ) − 2xy tan θ + y 2 sin 2 θ = 0
3 (am − 2hlm + bl 2 )
2
move with the axis of x angles such that the difference
From Eqs. (i) and (ii), we get of their tangents is 2.
x′ y′ 2
= = Sol. Given equation is
bl − hm am − hl 3(am 2 − 2hlm + bl 2 )
x 2 (tan 2 θ + cos 2 θ ) − 2xy tan θ + y 2 sin 2 θ = 0 ...(i)
y Example 8 Show that the area of the triangle formed and homogeneous equation of second degree
by the lines ax 2 + 2hxy + by 2 = 0 and lx + my + n = 0 ax 2 + 2hxy + by 2 = 0 ...(ii)
On comparing Eqs. (i) and (ii), we get
n 2 (h 2 − ab )
is . a = tan 2 θ + cos 2 θ
|(am 2 − 2hlm + bl 2 )|
h = − tan θ
Sol. Let equation of lines represented by and b = sin 2 θ
ax 2 + 2hxy + by 2 = 0 be y = m1x and y = m 2 x Let separate lines of Eq. (ii) are
therefore, m1 + m 2 = −
2h a
and m1m 2 = . y = m1x
b b and y = m2x
196 Textbook of Coordinate Geometry

where, m1 = tan θ1 and m 2 = tan θ 2 2


= tan 2 θ − sin 2 θ (tan 2 θ + cos 2 θ )
2h 2 tan θ sin θ
2
therefore, m1 + m 2 = − =
b sin 2 θ 2 sin θ
= (sec 2 θ − tan 2 θ − cos 2 θ )
a tan θ + cos θ 2 2
sin 2 θ
and m1 ⋅ m 2 = =
b sin 2 θ 2 sin θ
= (1 − cos 2 θ )
∴ m1 ~ m 2 = (m1 + m 2 ) − 4m1m 2 2 sin 2 θ
2
4 tan 2 θ 4 (tan 2 θ + cos 2 θ ) = sin θ = 2
⇒ tan θ1 ~ tan θ 2 = − sin θ
sin 4 θ sin 2 θ

Exercise for Session 1


1. The lines given by the equation (2y 2 + 3xy − 2x 2 ) ( x + y − 1) = 0 form a triangle which is
(a) equilateral (b) isosceles
(c) right angled (d) obtuse angled

2. Area of the triangle formed by the lines y 2 − 9xy + 18x 2 = 0 and y = 9 is


(a) 27/4 (b) 0
(c) 9 / 4 (d) 27

3. The equation 3x 2 + 2hxy + 3y 2 = 0 represents a pair of straight lines passing through the origin. The two lines
are
(a) real and distinct, if h 2 > 3 (b) real and distinct, if h 2 > 9
(c) real and coincident, if h 2 = 3 (d) real and coincident, if h 2 > 3

4. If one of the lines of the pair ax 2 + 2hxy + by 2 = 0 bisects the angle between positive directions of the axes,
then a, b , h satisfy the relation
(a) a + b = 2 |h | (b) a + b = − 2h
(c) a − b = 2 |h | (d) (a − b )2 = 4h 2

5. If the slope of one of the lines given by a 2 x 2 + 2hxy + b 2 y 2 = 0 be three times of the other, then h is equal to
(a) 2 3ab (b) − 2 3ab
2 2
(c) ab (d) − ab
3 3

6. Find the separate equations of two straight lines whose joint equation isab ( x 2 − y 2 ) + (a 2 − b 2 ) xy = 0.

7. Find the coordinates of the centroid of the triangle whose sides are 12x 2 − 20xy + 7y 2 = 0 and 2x − 3y + 4 = 0.

8. If the lines ax 2 + 2hxy + by 2 = 0 be two sides of a parallelogram and the line lx + my = 1be one of its diagonal,
show that the equation of the other diagonal is y (bl − hm ) = x (am − hl ).

9. Find the condition that one of the lines given by ax 2 + 2hxy + by 2 = 0 may coincide with one of the lines given
by a′ x 2 + 2h′ xy + b ′ y 2 = 0.
Session 2
Angle between the Pair of Lines ax2 + 2hxy + by2

Angle between the Pair of Corollary 1 Condition for the lines to be perpendicular.

Lines ax 2 + 2 hxy + by 2 The lines are perpendicular if the angle between them is
π
.
Theorem The angle θ between the pair of lines 2
represented by ax 2 + 2hxy + by 2 = 0 π
i.e. θ=
2
2 (h 2 − ab ) 
is given by θ = tan − 1  . π
⇒ cot θ = cot
 | a + b |  2
Proof Let y = m 1 x and y = m 2 x be the lines represented by ⇒ cot θ = 0
ax 2 + 2hxy + by 2 = 0. | a + b|
⇒ =0 ⇒ a +b =0
2h a 2 (h 2 − ab )
Then, m 1 + m 2 = − , m 1m 2 =
b b i.e. Coefficient of x 2 + Coefficient of y 2 = 0
Since, θ be the angle between the lines Hence, the lines represented by ax 2 + 2hxy + by 2 = 0 are
y = m 1 x and y = m 2 x . perpendicular, iff a + b = 0 i.e. coefficient of x 2 +
Y
coefficient of y 2 = 0.
2x

Remark
m
y=

Pair of any two perpendicular lines through the origin.


x Q Lines represented by ax 2 + 2hxy + by 2 = 0
θ y=
m 1
be perpendicular, then a + b = 0 or b = − a
X′ X Hence, the equation becomes ax 2 + 2hxy − ay 2 = 0

x 2 +   xy − y 2 = 0
Y′ 2h

 a
m − m 2  (m 1 + m 2 ) 2 − 4m 1m 2 ⇒ x 2 + pxy − y 2 = 0 ,
Then, tan θ = 1 = (Remember)
1 + m 1m 2 |1 + m 1m 2 | where, p is any constant.
2
 −2h  a  Corollary 2 Pair of lines perpendicular to the lines
  −4 
 b  b  2 (h − ab )
2
represented by
= =
1 + a 
  |a + b | ax 2 + 2hxy + by 2 = 0
 b and through origin.
2 (h − ab )  2 Let lines represented by
∴ θ = tan − 1  
 | a + b|  ax 2 + 2hxy + by 2 = 0 be y = m 1 x and y = m 2 x
2h
then m1 + m2 = −
Remark b
 2 h2 − ab  a
θ = sin−1   and m 1m 2 =
 ( a − b) 2 + 4 h2  b
198 Textbook of Coordinate Geometry

lines perpendiculars to y = m 1 x On comparing Eqs. (i) and (ii), we get


and y = m2 x a = sin 2 α − cos 2 β, h = sin β cos β,
and passing through origin are b = sin 2 α − sin 2 β
1 Let the angle between the lines representing by Eq. (i) is θ.
y =− x h 2 − ab
m1 ∴ tan θ = 2
|a + b |
1
and y =− x
m2 sin 2 β cos 2 β − (sin 2 α − cos 2 β ) (sin 2 α − sin 2 β )
=2
|sin 2 α − cos 2 β + sin 2 α − sin 2 β |
 x  x 
then, pair is y +  y +  =0 {sin 2 β cos 2 β − sin 4 α + sin 2 α sin 2 β
 m1   m2 
+ sin 2 α cos 2 β − sin 2 β cos 2 β}
⇒ x 2 + xy (m 1 + m 2 ) + m 1m 2 y 2 = 0 =2
|(2 sin 2 α − 1)|
2hxy a 2
⇒ x −
2
+ y =0 sin 2 α (1 − sin 2 α )
b b =2
| − cos 2α|
∴ bx 2 − 2hxy + ay 2 = 0
2 sin α cos α
= = tan 2α
Aid to memory For perpendicular pairs interchange the | − cos 2α|
coefficients of x 2 and y 2 and change the sign of xy. ∴ θ = 2α
Corollary 3 Condition for the lines to be coincident.
y Example 11 Show that the angle between the lines
The lines are coincident, if the angle between them is given by (a + 2hm + bm 2 ) x 2 + 2 {(b − a ) m − (m 2 − 1) h }
0° (or π )
xy + (am 2 − 2hm + b )y 2 = 0 is the same whatever be
i.e. θ = 0 (or π )
the value of m.
∴ tan θ = 0 °
Sol. Given equation is
2 (h 2 − ab ) (a + 2hm + bm 2 )x 2 + 2 { (b − a )m − (m 2 − 1)h }xy +
⇒ =0
| a + b| (am 2 − 2hm + b )y 2 = 0 ...(i)
⇒ h − ab = 0
2
The homogeneous equation of second degree
Ax 2 + 2Hxy + By 2 = 0 ...(ii)
⇒ h 2 = ab
On comparing Eqs. (i) and (ii), we get
Hence, the lines represented by ax 2 + 2hxy + by 2 = 0
A = a + 2hm + bm 2 , H = (b − a )m − (m 2 − 1)h ,
are coincident, iff h 2 = ab, then ax 2 + 2hxy + by 2
B = am 2 − 2hm + b
is a perfect square.
Let the angle between the lines representing by Eq. (i) is θ.
Remark 2 ( H 2 − AB )
∴ tan θ =
The parallel lines will be coincident only as both pass through a | A + B|
point.
{(b − a ) m − (m 2 − 1)h }2 − (a + 2hm + bm 2 )
y Example 10 Find the angle between the lines 2
(am 2 − 2hm + b )
( x 2 + y 2 ) sin 2 α = ( x cos β − y sin β) 2 . =
| a + 2hm + bm 2 + am 2 − 2hm + b |
Sol. Given equation is
2 (m 2 + 1)2 (h 2 − ab ) 2 (h 2 − ab )
( x 2 + y 2 ) sin 2 α = ( x cos β − y sin β )2 = =
| a + b | ( m 2 + 1) |a + b |
⇒ x 2 (sin 2 α − cos 2 β ) + 2xy sin β cos β
 2 (h 2 − ab ) 
+y 2 (sin 2 α − sin 2 β ) = 0 …(i) ⇒ θ = tan − 1  
|a + b | 
The homogeneous equation of second degree is  
ax 2 + 2hxy + by 2 = 0 …(ii) which is independent of m. Hence, the angle between the
lines representing by Eq. (i) is same for all values of m.
Chap 03 Pair of Straight Lines 199

y Example 12 Show that the straight lines y Example 13 Show that the condition that two of the
x 2 + 4 xy + y 2 = 0 and the line x − y = 4 form an three lines represented by ax 3 + bx 2 y + cxy 2 + dy 3 = 0
equilateral triangle. may be at right angles is a 2 + ac + bd + d 2 = 0.
Sol. Equation x 2 + 4 xy + y 2 = 0 ...(i) Sol. The given equation being homogeneous of third degree
is a homogeneous equation of second degree in x and y. represents three straight lines through the origin. Since,
Therefore, it represents two lines OP and OQ through the two of these lines are to be at right angles.
origin. Let pair of these lines be ( x 2 + pxy − y 2 ), p is constant and
Equation, x −y = 4 ...(ii) the other factor is (ax − dy ).
represent the line PQ . Hence, ax 3 + bx 2y + cxy 2 + dy 3 = ( x 2 + pxy − y 2 ) (ax − dy )
Y Comparing the coefficients of similar terms, we get
b = ap − d ...(i)
X′ X
O 15° y+(2–√3)x=0 c = − pd − a ...(ii)
60°
Multiplying Eq. (i) by d and Eq. (ii) by a and adding, we get
bd + ac = − d 2 − a 2
15 °

P
⇒ a 2 + ac + bd + d 2 = 0
y+(2

Aliter :
y=
+√3)

x–

Let y = m1x , y = m 2 x and y = m 3 x be the lines represented


x=0

by the equation
Q ax 3 + bx 2y + cxy 2 + dy 3 = 0.
Y′
Then
Let ∠POQ = θ. ax 3 + bx 2y + cxy 2 + dy 3 = d (y − m1x )
2 [(2)2 − 1 ⋅ 1] (y − m 2 x ) (y − m 3 x )
then, tan θ = = 3 ...(i)
| 1 + 1| 3 2
On equating the coefficients of x , x y and xy on both2

∴ θ = 60° sides, we get


From Eq. (i), x 2 + 4 xy + y 2 = 0 c
m1 + m 2 + m 3 = −
2 d
y  y 
⇒   + 4   +1=0 b
x x m1m 2 + m 2m 3 + m 3m1 =
d
y − 4 ± (42 − 4) a
⇒ = = −2± 3 and m1m 2m 3 = − ...(ii)
x 2 d
⇒ y = (− 2 ± 3) x Let the perpendicular lines be y = m1x and y = m 2 x , then
i.e. OP : y + (2 − 3 ) x = 0 m1m 2 = − 1
OQ : y + (2 + 3 ) x = 0 a
and From Eq. (ii), m3 = ...(iii)
d
Q Slope of PQ = 1 and Slope of OP = − (2 − 3 ).
On putting y = m 3 x in Eq. (i), we get
If ∠OPQ = α
ax 3 + bm 3 x 3 + cm 32 x 3 + dm 33 x 3 = 0
1 − ( − 2 + 3 ) 3− 3
 =
then tan α = = 3 ⇒ dm 33 + cm 32 + bm 3 + a = 0
 1 − 2 + 3  − 1 + 3 3 2
a a a
∴ α = 60° ⇒ d   +c   +b  +a=0 [from Eq. (iii)]
d  d  d 
Hence, ∠OQP = 180° − (60° + 60°) = 60°
Hence, ∆OPQ is an equilateral triangle. Hence, a 2 + ac + bd + d 2 = 0
200 Textbook of Coordinate Geometry

Exercise for Session 2


1. The angle between the pair of straight lines y 2 sin2 θ − xy sin2 θ + x 2(cos 2 θ − 1) = 0 is
π π π 2π
(a) (b) (c) (d)
4 2 3 3

2. The angle between the lines given by the equation ay 2 − (1 + λ 2 ) xy − ax 2 = 0 is same as the angle between
the lines
(a) 5x 2 + 2xy − 3y 2 = 0 (b) x 2 − 2xy − 3y 2 = 0
(c) x 2 − y 2 = 100 (d) xy = 0

3. Which of the following pair of straight lines intersect at right angles ?


(a) 2x 2 = y (x + 2y ) (b) (x + y )2 = x (y + 3x )
(c) 2y (x + y ) = xy (d) y = m 2x

4. If h 2 = ab , then the lines represented by ax 2 + 2hxy + by 2 = 0 are


(a) parallel (b) perpendicular
(c) coincident (d) None of these

5. Equation ax − 9x y − xy + 4y = 0 represents three straight lines. If the two of the lines are perpendicular,
3 2 2 3

then a is equal to
(a) −5 (b) 5
(c) −4 (d) 4

6. Find the angle between the lines whose joint equation is2x 2 − 3xy + y 2 = 0.

7. Show that the lines (1 − cos θ tan α ) y 2 − (2 cos θ + sin2 θ tan α ) xy + cos θ (cos θ + tan α ) x 2 = 0
include an angle α between them.

8. Find the angle between the lines represented by the equation x 2 − 2pxy + y 2 = 0.

9. Show that the lines x 2 − 4xy + y 2 = 0 and x + y = 1form an equilateral triangle and find its area.

10. Prove that the triangle formed by the lines ax 2 + 2hxy + by 2 = 0 and lx + my = 1is isosceles,
if h (l 2 − m 2 ) = (a − b ) m .
Session 3
Bisectors of the Angle between the Lines
Given by a Homogeneous Equation

Bisectors of the Angle ∴ The pair of bisectors is


 (y − m x ) (y − m x )   (y − m x ) (y − m x ) 
between the Lines Given by a  1
 (1 + m 2 )
+ 2 
2  
1
− 2
2 
 =0
 +   + 2
+ 2 
Homogeneous Equation 1 (1 m 2 ) (1 m 1 ) (1 m )
(y − m 1 x ) 2 (y − m 2 x ) 2
Theorem The joint equation of the bisectors of the angles ⇒ − =0
(1 + m 12 ) (1 + m 22 )
between the lines represented by the equation
x 2 − y 2 xy ⇒ (1 + m 22 ) (y 2 + m 12 x 2 − 2m 1 xy )
ax + 2hxy + by = 0 is
2 2
= .
a −b h − (1 + m 12 ) (y 2 + m 22 x 2 − 2m 2 xy ) = 0
Proof Let the lines represented by ax 2 + 2hxy + bySession-
2
=0 ⇒ (m 22 − m 12 )y 2 − (m 22 − m 12 ) x 2
be y − m 1 x = 0 and y − m 2 x = 0, then +2 xy (m 2 − m 1 ) − 2m 1m 2 (m 2 − m 1 ) xy = 0
2h a
m1 + m2 = − and m 1m 2 = ⇒ (m 2 + m 1 ) (y − x 2 ) + 2 xy − 2m 1m 2 xy = 0
2
b b
[Qm 1 − m 2 ≠ 0 ]
Since, the bisectors of the angles between the lines are the
locus of a point which is equidistant from the two given  2h 
 Qm 1 + m 2 = − 
lines.  2 h   a  b
⇒ ( x 2 − y 2 )  −  = 2 xy  1 −   
Y
 b   b   mm = a 
 1 2
b 
2x
m
y=

x 2 − y 2 xy
∴ = [b ≠ 0 ]
N ,k) x
P(h m 1
a −b h
y=
M Aliter :
X¢ X Let the equation ax 2 + 2hxy + by 2 = 0 represent two lines
O
L1 OM 1 and L2 OM 2 making angles θ 1 and θ 2 with the

positive direction of X-axis.
Let P (h, k ) be a point on a bisector of the angle between If slopes of L1 OM 1 and L2 OM 2 are m 1 and m 2 , then
the given lines. Then, PM = PN
m 1 = tan θ 1 and m 2 = tan θ 2
| k − m 1 h| | k − m 2 h|
⇒ = and
2h
m 1 + m 2 = − , m 1m 2 =
a
...(i)
(1 + m 12 ) (1 + m 22 ) b b
(k − m 1 h ) (k − m 2 h ) Let NON 1 and KOK 1 are the required bisectors,
⇒ =± θ − θ1
(1 + m 12 ) (1 + m 22 ) Since ∠ NOL1 = ∠ NOL2 = 2
2
Hence, the locus of a P (h, k ) is
θ − θ1 θ1 + θ2
(y − m 1 x ) (y − m 2 x ) ∠ NOX = θ 1 + 2 =
=± 2 2
(1 + m 12 ) (1 + m 22 ) π
Since, ∠ NOK =
2
202 Textbook of Coordinate Geometry

Y Remark
L2
x 2 − y 2 xy
N The joint equation of the bisectors is =
a− b h
L1
K or hx 2 − ( a − b) xy − hy 2 = 0
i.e. coefficient of x + coefficient of y 2 = 0.
2

θ1 θ2
Hence, the bisectors of the angle between the lines are always
X′ X perpendicular to each other.
O
M1
N1 K1 Corollaries
M2 Y′

π π  θ + θ2  1. If a = b, the bisectors are x 2 − y 2 = 0


∴ ∠ KOX = + ∠ NOX = +  1 
2 2  2  i.e. x − y = 0, x + y = 0
2. If h = 0, the bisectors are xy = 0
 θ + θ2 
Equation of bisectors arey = x tan  1  i.e. x = 0, y = 0
 2 
 θ + θ2  y Example 14 Find the equation of the bisectors of the
⇒ y − x tan  1  =0 …(ii) angle between the lines represented by
 2 
 π θ + θ2  3 x 2 − 5 xy + 4y 2 = 0.
and y = x tan  + 1 
2 2  Sol. Given equation is
 θ + θ2  3x 2 − 5xy + 4y 2 = 0 ...(i)
⇒ y = − x cot  1 
 2  Comparing it with the equation
ax 2 + 2hxy + by 2 = 0 ...(ii)
 θ + θ2 
⇒ y + x cot  1  =0 ...(iii) 5
 2  then a = 3, h = − , b = 4
2
∴ Pair of bisectors Hence, the equation of bisectors of the angle between the
pair of the lines (i) is
  θ1 + θ2     θ1 + θ2  
y − x tan  2   y + x cot  2   = 0 x2 − y2
=
xy
    3− 4 −5 / 2
  θ + θ2   θ1 + θ2   x 2 − y 2 2xy
⇒ y 2 − x 2 + xy  cot  1  − tan    =0 ⇒ =
  2   2  −1 −5
∴ 5x 2 − 2xy − 5y 2 = 0
 2  θ1 + θ2  
 1 − tan  
  2  y Example 15 Show that the line y = mx bisects the
⇒ x − y = xy
2 2
  θ1 + θ2   angle between the lines
 tan   
 2   ax 2 − 2hxy + by 2 = 0,
 2  if h (1 − m 2 ) + m (a − b ) = 0.
⇒ x 2 − y 2 = xy  
 tan (θ 1 + θ 2 )  Sol. Equation of pair of bisectors of angles between lines
ax 2 − 2hxy + by 2 = 0 is
 1 − tan θ 1 tan θ 2 
⇒ x 2 − y 2 = 2 xy   x 2 − y 2 xy
 tan θ 1 + tan θ 2  =
a−b −h
 1 − m 1m 2  ⇒ − h ( x 2 − y 2 ) = (a − b ) xy ...(i)
⇒ x 2 − y 2 = 2 xy  
 m1 + m2  But y = mx is one of these lines, then it will satisfy
it. Substituting y = mx in Eq. (i),
1 −a / b 
⇒ x 2 − y 2 = 2 xy   − h ( x 2 − m 2 x 2 ) = (a − b )x ⋅ mx
 − 2h / b 
Dividing by x 2 , h (1 − m 2 ) + m (a − b ) = 0
( x − y ) xy
2 2
∴ =
(a − b ) h
Chap 03 Pair of Straight Lines 203

y Example 16 If pairs of straight lines Y S


Q
x 2 − 2pxy − y 2 = 0 and x 2 − 2qxy − y 2 = 0 be such
A
that each pair bisects the angle between the other
pair, then prove that pq = − 1. P
B
Sol. According to the question, the equation of the bisectors of
the angle between the lines x R
x 2 − 2pxy − y 2 = 0 ...(i) x
is x 2 − 2qxy − y 2 = 0 …(ii) X′
O
X
R′
∴ The equation of bisectors of the angle between the lines P′
A′
(i) is S′ B'
x 2 − y 2 xy Q′ Y′
=
1 − ( − 1) − p which are also the bisectors of the second pair.
Let P ′ OP , Q ′ OQ , R ′ OR, S ′ OS be the lines of the Eqs. (i) and
⇒ − px − 2xy + py = 0
2 2
…(iii)
(ii) pairs respectively and A ′ OA and B ′ OB be their
Since, Eqs. (ii) and (iii) are identical, comparing Eqs. (ii) and bisectors. We have,
(iii), we get ∠ ROA = ∠ SOA
1 −2q −1 ∠ POA = ∠QOA
= = ⇒ pq = − 1 and
−p −2 p ⇒ ∠ ROA − ∠ POA = ∠SOA − ∠QOA
⇒ ∠ ROP = ∠SOQ
Remark Hence the result.
By taking the bisectors of the angles between the pair of lines (ii),
we will get the same result. y Example 19 If the lines represented by
y Example 17 Prove that the angle between one of the x 2 − 2pxy − y 2 = 0 are rotated about the origin
lines given by ax 2 + 2hxy + by 2 = 0 and one of the through an angle θ , one in clockwise direction and the
lines ax 2 + 2hxy + by 2 + λ ( x 2 + y 2 ) = 0 is equal to other in anti-clockwise direction, then find the
equation of the bisectors of the angle between the
angle between other two lines of the system. lines in the new position.
Sol. The equation of the bisectors of the angle between the lines
Sol. Since, lines represented by x 2 − 2pxy − y 2 = 0
ax 2 + 2hxy + by 2 = 0 ...(i)
are perpendicular to each other. The bisectors of the angles
x 2 − y 2 xy between the lines in new position are same as the bisectors
is =
a−b h of the angles between their old positions. i.e.
OC , OD ; OA , OB be the old and new pairs respectively and
and the equation of bisectors of the angle between the lines
OE and OF be their bisectors, we have
ax 2 + 2hxy + by 2 + λ ( x 2 + y 2 ) = 0 …(ii) ∠COE = ∠ DOE
⇒ (a + λ) x 2 + 2hxy + (b + λ )y 2 = 0 and ∠COA = ∠ DOB = θ [given]
x2 − y2 xy x 2 − y 2 xy D Y B
is = ⇒ = E
(a + λ ) − (b + λ ) h a−b h
A
∴ Bisectors of angles between lines given by Eqs. (i) and
(ii) are the same. Hence the result. F
θ
y Example 18 Show that the pair of lines given by θ C
a 2 x 2 + 2h(a + b )xy + b 2 y 2 = 0 is equally inclined to the X′ X
O
pair given by ax 2 + 2hxy + by 2 = 0.
Y′
Sol. Given pair of lines are
⇒ ∠COE − ∠COA = ∠DOE − ∠ DOB
a 2 x 2 + 2h (a + b )xy + b 2y 2 = 0 ...(i)
⇒ ∠ AOE = ∠ BOE
and ax 2 + 2hxy + by 2 = 0 ...(ii)
x 2 − y 2 xy
Equation of bisectors of first pair is Therefore, the required equation is =
1 − ( − 1) − p
x2 − y2 xy x 2 − y 2 xy
= ⇒ = i.e. px 2 + 2xy − py 2 = 0.
a 2 − b 2 h (a + b ) a−b h
204 Textbook of Coordinate Geometry

Exercise for Session 3


1. If coordinate axes are the angle bisectors of the pair of lines ax 2 + 2hxy + by 2 = 0, then
(a) a = b (b) h = 0
(c) a + b = 0
2
(d) a + b 2 = 0

2. If the line y = mx is one of the bisector of the lines x 2 + 4xy − y 2 = 0, then the value of m is
5−1 5+ 1
(a) (b)
2 2
 5 + 1  5 − 1
(c) −   (d) −  
 2   2 

3. If one of the lines of my 2 + (1 − m 2 ) xy − mx 2 = 0 is a bisector of the angle between the lines xy = 0, then
cos −1(m ) is
(a) 0 (b) π/2
(c) π (d) 3 π / 2

4. The bisectors of the angles between the lines (ax + by )2 = c (bx − ay )2, c > 0 are respectively parallel and
perpendicular to the line
(a) bx − ay + µ = 0 (b) ax + by + λ = 0
(c) ax − by + ν = 0 (d) bx + ay + τ = 0

5. If the pairs of straight lines ax + 2hxy − ay = 0 and bx 2 + 2gxy − by 2 = 0 be such that each bisects the angles
2 2

between the other, then prove that hg + ab = 0.

6. Prove that the lines 2x 2 + 6xy + y 2 = 0 are equally inclined to the lines 4x 2 + 18xy + y 2 = 0.

7. Show that the lines bisecting the angle between the bisectors of the angles between the lines
ax 2 + 2hxy + by 2 = 0 are given by (a − b ) ( x 2 − y 2 ) + 4hxy = 0.

8. Prove that the bisectors of the angle between the lines ax 2 + acxy + cy 2 = 0 and
 1 2  1 2
3 +  x + xy + 3 +  y = 0 are always the same.
 c   a

9. The lines represented by x 2 + 2λxy + 2y 2 = 0 and the lines represented by (1 + λ )x 2 − 8xy + y 2 = 0 are equally
inclined, find the values of λ.
Session 4
General Equation of Second Degree,
Important Theorems

General Equation of Second Now, ax 12 + 2hx 1 y 1 + by 12 + 2 gx 1 + 2 fy 1 + c = 0


⇒ x 1 (ax 1 + hy 1 + g ) + y 1 (hx 1 + by 1 + f )
Degree + ( gx 1 + fy 1 + c ) = 0
The equation ax 2 + 2hxy + by 2 + 2 gx + 2 fy + c = 0 ⇒ x 1 ⋅ 0 + y 1 ⋅ 0 + gx 1 + fy 1 + c = 0
[from Eqs. (iii) and (iv)]
is the general equation of second degree and represents a
conics (pair of straight lines, circle, parabola, ellipse, ⇒ gx 1 + fy 1 + c = 0 ...(vi)
hyperbola). It contains six constants a, b, c , f , g, h. On eliminating x 1 , y 1 from Eqs. (iii), (iv) and (vi), we get
the determinant
i.e. a = coefficient of x 2 , b = coefficient of y 2 , a h g
c = constant term, g = half the coefficient of x, h b f =0
f = half the coefficient of y, g f c
h = half the coefficient of xy. ∴ abc + 2 fgh − af − bg 2 − ch 2 = 0,
2

Theorem The necessary and sufficient condition for as the required condition.
ax 2 + 2hxy + by 2 + 2 gx + 2 fy + c = 0 Remarks
to represent a pair of straight lines is that 1. Without using determinant On solving Eqs. (iii) and (iv),
we get
a h g  hf − bg gh − af 
( x1, y1 ) =  , 
abc + 2 fgh − af − bg − ch = 0 or h
2 2 2
b f =0.  ab − h ab − h 
2 2

g f c and then substituting the values of x1 and y1 in Eqs. (vi), we


obtain
Proof Necessary condition : Let the equation be
 hf − bg   gh − af 
g   + f   + c =0
ax 2 + 2hxy + by 2 + 2 gx + 2 fy + c = 0 …(i) 2
 ab − h 
2
 ab − h 
represent a pair of lines. Assuming that these lines are not ⇒ abc + 2fgh − af 2 − bg 2 − ch2 = 0
parallel, we suppose further that their point of intersection 2. By making ax 2 + 2hxy + by 2 + 2gx + 2fy + c = 0
is ( x 1 , y 1 ). Shifting the origin at ( x 1 , y 1 ) without rotating
homogeneous with the help of a new variable z, i.e.
the coordinate axes, we have the Eq. (i) transforms to
ax 2 + 2hxy + by 2 + 2gxz + 2fyz + cz 2 = 0
a( X + x 1 ) 2 + 2h( X + x 1 )(Y + y 1 ) + b(Y + y 1 ) 2 Let f ( x, y, z ) ≡ ax 2 + 2hxy + by 2 + 2gxz + 2fyz + cz 2 = 0
+ 2 g( X + x 1 ) + 2 f (Y + y 1 ) + c = 0 ...(ii) ∴
∂f
= 2ax + 2hy + 2gz = 0
∂x
Now this Eq. (ii) represents a pair of lines through the new
∂f
origin and consequently, it is homogeneous in X and Y. = 2hx + 2by + 2fz = 0
∂y
Hence, the coefficients of X and Y and the constant term ∂f
= 2gx + 2fy + 2cz = 0
in Eq. (ii) must vanish separately. ∂z
i.e. coefficient of X = coefficient of Y = constant term = 0 and finally putting z = 1, we obtain equations
ax + hy + g = 0, hx + by + f = 0, gx + fy + c = 0
⇒ ax 1 + hy 1 + g = 0 ...(iii)
which are same as Eqs. (iii), (iv) and (vi), respectively.
hx 1 + by 1 + f = 0 ...(iv) 3. If ab − h2 = 0, the lines given by Eq. (i) are parallel. In this
and ax 12 + 2hx 1 y 1 + by 12 + 2 gx 1 + 2 fy 1 + c = 0 ...(v) case, the method followed in the above proof fails and we
follow the following method.
206 Textbook of Coordinate Geometry

Aliter I : (Proof) Case I If a ≠ 0, then writing Eq. (i) as a quadratic


Let the lines represented by equation in x, we get
ax 2 + 2hxy + by 2 + 2 gx + 2 fy + c = 0 ...(i) ax 2 + 2 x (hy + g ) + by 2 + 2 fy + c = 0
be lx + my + n = 0 and l ′ x + m ′ y + n ′ = 0 Solving, we have
then ax 2 + 2hxy + by 2 + 2 gx + 2 fy + c −2(hy + g ) ± 4(hy + g ) 2 − 4a(by 2 + 2 fy + c )
x=
≡ (lx + my + n ) (l ′ x + m ′ y + n ′ ) ...(ii) 2a
Comparing the coefficients of similar terms in both sides − (hy + g ) ± {(h − ab )y 2 + 2( gh − af )y + ( g 2 − ac ) }
2
of Eq. (ii), we get ∴x =
a
ll ′ = a, mm ′ = b, nn ′ = c 
  Eq. (i), will represent two straight lines, if LHS of Eq. (i),
lm ′ + l ′ m = 2h, ln ′ + l ′ n = 2 g , …(iii) can be resolved into two linear factors, therefore the
mn ′ + m ′ n = 2 f 
  expression under the square root should be a perfect
We now eliminate l, m, n, l ′ , m ′ and n′ from these equations, square.
l l′ 0 l′ l 0 [QAx 2 + Bx + C = 0 is a perfect
we have m m′ 0 × m′ m 0 =0 square ⇔ B 2 − 4 AC = 0 ]
n n′ 0 n′ n 0 Hence, 4( gh − af ) 2 − 4(h 2 − ab )( g 2 − ac ) = 0
[Qeach determinant = 0] or abc + 2 fgh − af 2 − bg 2 − ch 2 = 0 ...(ii)
2ll ′ lm ′ + l ′ m ln ′ + l ′ n
Ths is called discriminant of the Eq. (i).
⇒ ml ′ + m ′ l 2mm ′ mn ′ + m ′ n = 0
Case II If a = 0, b ≠ 0, then writing Eq. (i) as a quadratic
nl ′ + n ′ l nm ′ + n ′ m 2nn′ equation in y
2a 2h 2g i.e. by 2 + 2y (hx + f ) + 2 gx + c = 0
⇒ 2h 2b 2f =0 and proceeding above we get the condition
2g 2 f 2c 2 fgh − bg 2 − ch 2 = 0
a h g which is condition obtained by putting a = 0 in Eq. (ii).
⇒ ∆= h b f =0 Case III If a = 0, b = 0 but h ≠0, then Eq. (i) becomes
g f c 2hxy + 2 gx + 2 fy + c = 0
∴ ∆ = abc + 2 fgh − af 2 − bg 2 − ch 2 = 0, Multiplying by
h
[Qh ≠ 0 ]
which is the required necessary condition. 2
ch
⇒ h 2 xy + hgx + hfy + = 0
Remark 2
Without using determinant ch
Now, ( lm′ + l ′ m) (ln′ + l ′ n) ( mn′ + m′ n) = 2h ⋅ 2g ⋅ 2f ⇒ (hx + f ) (hy + g ) = fg −
2
⇒ 2ll ′ mm′ nn′ + ll ′ ( m2n′2 + m′2 n2 )
+ mm′( n2l ′2 + n′2 l 2 ) + nn′ ( l 2m′2 + l ′2 m2 ) = 8fgh
Above equation represents two straight lines, if
⇒ 2ll ′ mm′ nn′ + ll ′{( mn′ + m′ n) 2 − 2mm′ nn′}
ch
fg − = 0 ⇒ 2 fgh − ch 2 = 0
+ mm′{( nl ′ + n′ l ) 2 − 2nn′ ll ′} + nn′ {( lm′ + l ′ m) 2 − 2ll ′ mm′} 2
= 8fgh [from Eq. (iii)] which is condition obtained by putting a = 0, b = 0 in
⇒ 2abc + a( 4 f − 4 bc ) + b( 4 g − 4 ca) + c( 4 h − 4 ab) = 8 fgh
2 2 2 Eq. (ii).
Hence in each case, the condition that
∴ abc + 2fgh − af 2 − bg 2 − ch2 = 0,
which is the required necessary condition.
ax 2 + 2hxy + by 2 + 2 gx + 2 fy + c = 0
Aliter II : (Proof) represents two straight lines is
Given equation is abc + 2 fgh − af 2 − bg 2 − ch 2 = 0
ax 2 + 2hxy + by 2 + 2 gx + 2 fy + c = 0 ...(i) which is the required necessary condition.
Chap 03 Pair of Straight Lines 207

Sufficient condition (Conversely)


Here, we have to show that the equation Important Theorems
ax 2 + 2hxy + by 2 + 2 gx + 2 fy + c = 0 represents a pair of Theorem 1 The angle between the lines represented
straight lines. by
a h g ax 2 + 2hxy + by 2 + 2 gx + 2 fy + c = 0
h b f =0 is given by
g f c  (h 2 − ab ) 
−1  2
i.e. lines ax + hy + g = 0, hx + by + f = 0, gx + fy + c = 0 θ = tan  .
 |a +b | 
are concurrent. 
Let the point of concurrency be ( x 1 , y 1 ). Proof Let y = m 1 x + c 1
Then, ax 1 + hy 1 + g = 0 ...(iii) and y = m2 x + c 2
hx 1 + by 1 + f = 0 ...(iv) be the lines represented by
and gx 1 + fy 1 + c = 0 ...(v) ax 2 + 2hxy + by 2 + 2 gx + 2 fy + c = 0
Y
Now, shifting the origin at ( x 1 , y 1 ) without rotating the
coordinate axes the equation P
ax 2 + 2hxy + by 2 + 2 gx + 2 fy + c = 0 reduces to

1
1 x+c
2
c
θ

+
2x
a( X + x 1 ) 2 + 2h( X + x 1 ) (Y + y 1 ) + b

y=m
y=
(Y + y 1 ) 2 + 2 g( X + x 1 ) + 2 f (Y + y 1 ) + c = 0 β α
X′ X
O R Q
⇒ aX + 2hXY + bY + 2 X (ax 1 + hy 1 + g )
2 2

+ 2Y (hx 1 + by 1 + f ) + x 1 (ax 1 + hy 1 + g ) Y′

+ y 1 (hx 1 + by 1 + f ) + ( gx 1 + fy 1 + c ) = 0 where, m 1 = tan α, m 2 = tan β


Then, ax 2 + 2hxy + by 2 + 2 gx + 2 fy + c
⇒ aX 2 + 2hXY + bY 2 + 0 + 0 + 0 + 0 + 0 = 0
[from Eqs. (iii), (iv) and (v)] ≡ (y − m 1 x − c 1 ) (y − m 2 x − c 2 )
i.e. aX + 2hXY + bY = 0
2 2 Comparing coefficients of like powers, we obtain
2h a
It is homogeneous equation of second degree. So, it m 1 + m 2 = − , m 1m 2 =
represents a pair of straight lines through the new origin. b b
Now, if θ be the acute angle between the lines
Hence, the equation ax 2 + 2hxy + by 2 + 2 gx + 2 fy + c = 0
y = m 1 x + c 1 and y = m 2 x + c 2 , then
represents a pair of straight lines, if
m − m 2  (m 1 + m 2 ) 2 − 4m 1m 2
abc + 2 fgh − af 2 − bg 2 − ch 2 = 0. tan θ = 1 =
1 + m 1m 2 | 1 + m 1m 2 |
Some useful identities If y = m 1 x + c 1 , y = m 2 x + c 2 be
2
lines represented by Eq. (i). Then,  2h  a 
−  − 4  
ax 2 + 2hxy + by 2 + 2 gx + 2 fy + c  b b  2 (h 2 − ab )
= =
= b (y − m 1 x − c 1 ) (y − m 2 x − c 2 )  a
 |a +b |
1+ 
= b (y 2 − (m 1 + m 2 ) xy + m 1m 2 x 2  b
+ (m 1 c 2 + m 2 c 1 ) x − (c 1 + c 2 )y + c 1 c 2 )  (h 2 − ab ) 
−1  2
On equating coefficients, we get ∴ θ = tan  
 |a +b | 
2h a 2g 
m 1 + m 2 = − ,m 1m 2 = ,m 1 c 2 + m 2 c 1 = ,
b b b Corollary 1. The angle between the lines represented by
2f c ax 2 + 2hxy + by 2 + 2 gx + 2 fy + c = 0
c1 + c2 = − , c 1c 1 =
b b is the same as the angle between the lines represented by
These five relations are very useful to solve many problems.
ax 2 + 2hxy + by 2 = 0
208 Textbook of Coordinate Geometry

Corollary 2. The lines represented by then ax 2 + 2hxy + by 2 + 2 gx + 2 fy + c


ax 2 + 2hxy + by 2 + 2 gx + 2 fy + c = 0 ≡ (lx + my + n ) (l ′ x + m ′ y + n ′ )
are perpendicular iff a + b = 0 and parallel iff h = ab. 2 Comparing the coefficients of similar terms in both sides
then ll ′ = a, mm ′ = b, nn ′ = c
Theorem 2 The lines represented by
lm ′ + l ′ m = 2h, ln′ + l ′ n = 2 g, mn ′ + m ′ n = 2 f
ax 2 + 2hxy + by 2 + 2 gx + 2 fy + c = 0
⇒ (lm ′ − l ′ m ) = (lm ′ + l ′ m ) 2 − 4ll ′ mm ′ = 2 (h 2 − ab )
will be coincident, if h − ab = 0, g − ac = 0
2 2
⇒ (nl ′ − n ′ l ) = (ln ′ + l ′ n ) 2 − 4ll ′ nn ′ = 2 ( g 2 − ac )
and f − bc = 0.
2

and (mn ′ − m ′ n ) = (mn ′ + m ′ n ) 2 − 4 mm ′ nn ′


Proof Let the lines represented by
ax 2 + 2hxy + by 2 + 2 gx + 2 fy + c = 0 = 2 ( f 2 − bc )
be lx + my + n = 0 and l ′ x + m ′ y + n ′ = 0 Now, solving lx + my + n = 0 and l ′ x + m ′ y + n ′ = 0
x y 1
then ax 2 + 2hxy + by 2 + 2 gx + 2 fy + c then = =
(mn ′ − m ′ n ) (nl ′ − n ′ l ) (lm ′ − l ′ m )
≡ (lx + my + n ) (l ′ x + m ′ y + n ′ )
x y 1
Comparing the coefficients of similar terms in both sides, ⇒ = =
we get 2 ( f − bc )
2
2 ( g − ac )
2
2 (h − ab )
2

ll ′ = a, mm ′ = b, nn ′ = c    f 2 − bc   g 2 − ac  
 ∴ (x , y ) =   2 ,  
lm ′ + l ′ m = 2h, ln′ + l ′ n = 2 g    h − ab   h 2 − ab  
 
mn ′ + m ′ n = 2 f 

Q Lines lx + my + n = 0 and l ′ x + m ′ y + n ′ =0 are  f 2 − bc  ( f 2 − bc ) (h 2 − ab )
Also, x =  2  =
coincident, then =
l m n
= .  h − ab  (h 2 − ab )
l ′ m ′ n′
f 2 h 2 − abf 2 − bch 2 + b (abc )
Taking the ratios in pairs, then =
lm ′ − l ′ m = 0,mn ′ − m ′ n = 0, ln′ − l ′ n = 0 (h 2 − ab )

⇒ (lm ′ + l ′ m ) 2 − 4ll ′ mm ′ = 0, f 2 h 2 − abf 2 − bch 2 + b (af 2 + bg 2 + ch 2 − 2 fgh )


=
(h 2 − ab )
(mn ′ + m ′ n ) 2 − 4 mm ′ nn ′ = 0
[Qabc + 2 fgh − af 2 − bg 2 − ch 2 = 0 ]
and (ln ′ + l ′ n ) 2 − 4ll ′ nn ′ = 0
( f 2 h 2 + b 2 g 2 − 2bfgh ) (bg − hf ) 2
i.e. ( 4h − 4ab ) = 0, ( 4 f − 4bc ) = 0
2 2 = =
(h 2 − ab ) (h 2 − ab )
and ( 4 g − 4ac ) = 0
2
 bg − hf 
i.e. h − ab = 0, f − bc = 0, g − ac = 0
2 2 2 = 2 
 h − ab 
Theorem 3 The point of intersection of the lines  af − gh 
represented by Similarly, y = 2 
 h − ab 
ax 2 + 2hxy + by 2 + 2 gx + 2 fy + c = 0 is
 bg − hf af − gh 
  f 2 − bc   g 2 − ca    bg − hf af − gh  Hence, (x , y ) =  2 , .
  ,    or  2 , .  h − ab h 2 − ab 
  h 2 − ab   h 2 − ab    h − ab h 2 − ab 
 
Remembering Method (For second point)
Proof Let the lines represented by a h g
ax 2 + 2hxy + by 2 + 2 gx + 2 fy + c = 0 Since, ∆= h b f
be lx + my + n = 0 and l ′ x + m ′ y + n ′ = 0 g f c
Chap 03 Pair of Straight Lines 209

taking first two rows (repeat first column) Theorem 4 The pair of bisectors of the lines
a h g a represented by
h b f h ax 2 + 2hxy + by 2 + 2 gx + 2 fy + c = 0,
⇒ ab − h 2 , hf − bg, gh − af ( x − α ) 2 − ( x − β) 2 ( x − α ) (y − β)
is =
⇒ h 2 − ab, bg − hf , af − gh (a − b ) h
bg − hf af − gh where (α, β) be the point of intersection of the pair of
⇒ 1, , straight lines represented by Eq. (i).
h 2 − ab h 2 − ab
Proof Since (α, β) be the point of intersection of the lines
 bg − hf af − gh  represented by
Hence, point of intersection is  2 , .
 h − ab h 2 − ab  ax 2 + 2hxy + by 2 + 2 gx + 2 fy + c = 0 …(i)
OR Shifting the origin at (α, β) without rotating the coordinate
Cofactors of third column are C 13 , C 23 , C 33 axes, the Eq. (i) reduces to
h b a( X + α ) 2 + 2h( X + α ) (Y + β)
Q C 13 = = hf − bg
g f + b(Y + β) 2 + 2 g( X + α ) + 2 f (Y + β) + c = 0

C 23 = −
a h
= hg − af [Qx = X + α and y =Y + β]
g f ⇒ (aX + 2hXY + bY ) + 2 X (aα + hβ + g )
2 2

and C 33 =
a h
= ab − h 2 + 2Y (hα + bβ + f )
h b + aα + 2hαβ + bβ + 2 gα + 2 f β + c = 0…(ii)
2 2

C C  This equation represents a pair of straight lines passing


Point of intersection is  13 , 23  i.e.
 C 33 C 33  through the new origin. So, it must be homogeneous
equation of second degree in X and Y.
 hf − bg hg − af   bg − hf af − hg 
 ,  or  2 , . ∴ aα + hβ + g = 0 ...(iii)
 ab − h 2 ab − h 2   h − ab h 2 − ab 
hα + bβ + f = 0 ....(iv)
Remembering Mehod (For first point) and aα 2 + 2hαβ + bβ2 + 2 gα + 2 f β + c = 0 …(v)
Cofactors of leading diagonal are
Now, from Eq. (ii), aX + 2hXY + bY = 0
2 2
...(vi)
C 11 , C 22 , C 33
b f The equation of the bisectors of the angles between the
Q C 11 = = bc − f 2 , lines given by Eq. (vi) is
f c
X 2 − Y 2 XY
a g = ...(vii)
C 22 = = ac − g 2 a −b h
g c [with reference to new origin]
a h Replacing X by x − α and Y by y − β in Eq. (vii), then
and C 33 = = ab − h 2
h b ( x − α ) 2 − (y − β) 2 ( x − α ) (y − β)
=
 C 11 C 22  (a − b ) h
∴ Point of intersection are  , 
 C 33 C 33  [with reference to old origin]
which is the required equation of the bisectors of the
  bc − f 2   ac − g 2  
i.e.   ,   angles between the lines given by Eq. (i).
  ab − h 2   ab − h 2  
 
Remark
  f 2 − bc   g 2 − ac   If ax 2 + 2hxy + by 2 + 2gx + 2fy + c = 0
or   ,  
  h 2 − ab   h 2 − ab   represents two straight lines, then the equation of lines through
  the origin and parallel to them is ax 2 + 2hxy + by 2 = 0.
210 Textbook of Coordinate Geometry

y Example 20 For what value of λ does the To find the angle between the lines
equation 12x 2 − 10xy + 2y 2 + 11x − 5y + λ = 0 If θ be the angle between the lines, then
represent a pair of straight lines? Find their equations 2 h 2 − ab
tan θ =
and the angle between them. |a + b |
Sol. Comparing the given equation with the equation 2 25 − 24 1
= =
ax 2 + 2hxy + by 2 + 2gx + 2 fy + c = 0, | 12 + 2| 7
11 5
we get a = 12, h = − 5, b = 2, g = , f = − and c = λ − 1  1
∴ θ = tan  .
2 2 7
If the given equation represents a pair of straight lines, then
abc + 2 fgh − af 2 − bg 2 − ch 2 = 0 y Example 21 Prove that the equation
 5 11 25 8 x 2 + 8 xy + 2y 2 + 26 x + 13y + 15 = 0 represents a pair
⇒ 12 × 2 × λ + 2 ×  −  × × ( − 5) − 12 ×
 2 2 4 of parallel straight lines. Also, find the
121 perpendicular distance between them.
−2× − λ × 25 = 0
4 Sol. Given equation is
∴ λ = 2, also h 2 − ab = 25 − 24 = 1 > 0 8x 2 + 8xy + 2y 2 + 26x + 13y + 15 = 0 …(i)
∴ The given equation will represent a pair of straight lines, Writing Eq. (i) as quadratic equation in x, we get
if λ = 2. 8x 2 + 2x ( 4y + 13) + 2y 2 + 13y + 15 = 0
To find the two lines
− 2 ( 4y + 13) ± 4 ( 4y + 13)2 − 32 ( 2y 2 + 13y + 15)
First method ∴ x=
16
Substituting λ = 2 in the given equation, we get
− ( 4y + 13) ± ( 4y + 13)2 − 8 (2y 2 + 13y + 15)
12x 2 − 10xy + 2y 2 + 11x − 5y + 2 = 0 …(i) ⇒ x=
8
Since, 12x 2 − 10xy + 2y 2 = 2 (3x − y ) (2x − y ) − ( 4y + 13) ± 7
⇒ x=
factors of Eq. (i) can be taken as 8
2 ( 3x − y + l ) ( 2x − y + m ) ⇒ 8x = − 4y − 13 + 7, i.e. 4 x + 2y + 3 = 0
= 12x 2 − 10xy + 2y 2 + 2 (2l + 3m )x + 2 ( −l − m )y + 2lm and 8x = − 4y − 13 − 7, i.e. 2x + y + 5 = 0
11 5 i.e. the given Eq. (i) represents two straight lines
On comparing, 2l + 3m = , l + m = , lm = 1
2 2 2x + y + 5 = 0
1 and 4 x + 2y + 3 = 0
Solving, we get l = 2, m = .
2 3
i.e. 2x + y + = 0
Thus, the factors of Eq. (i) are 2
 1 both lines are parallel.
2(3x − y + 2) 2x − y +  = 0 3
 2 5−
2 7
or (3x − y + 2) ( 4 x − 2y + 1) = 0. ∴ Distance between them = =
∴ The two straight lines represented by the given equation 22 + 12 2 5
are 13
Aliter : Here, ∆ = 8 × 2 × 15 + 2 × × 13 × 4
3x − y + 2 = 0 and 4 x − 2y + 1 = 0. 2
Second Method 2
 13
− 8 ×   − 2 × (13)2 − 15 × ( 4 )2 = 0
Writing Eq. (i) as quadratic equation in x, we get  2
12x 2 + ( − 10y + 11)x + 2y 2 − 5y + 2 = 0 and h 2 = ( 4 )2 = 16 = 8 × 2 = ab
∴ ∴ Given equation
− ( − 10y + 11 ) ± ( −10y + 11)2 − 48 (2y 2 − 5y + 2) 8x 2 + 8xy + 2y 2 + 26x + 13y + 15 = 0
x= …(i)
24
represents two parallel straight lines.
i.e. 24 x = (10y − 11) ± ( 4y 2 + 20y + 25) Since, 8x 2 + 8xy + 2y 2 = 2(2x + y )2
= (10y − 11) ± (2y + 5) factors of Eq. (i) can be taken as
∴ 24 x = 12y − 6, i.e. 4 x − 2y + 1 = 0 2(2x + y + l ) (2x + y + m )
and 24 x = 8y − 16, i.e. 3x − y + 2 = 0 = 8x 2 + 8xy + 2y 2 + 2(2m + 2l )x + 2 (m + l ) y + 2lm
are the required lines.
Chap 03 Pair of Straight Lines 211

On comparing, we get l + m =
13
and lm =
15 Sol. Given ax 2 + 2hxy + by 2 + 2gx + 2 fy + c = 0 …(i)
2 2 The point of intersection of the lines given by Eq. (i) are
∴ Distance between them
 f 2 − bc   g 2 − ca 
l −m (l + m )2 − 4lm α=  2  ,β =  2 
= =  h − ab   h − ab 
( 22 + 12 ) 5
Y
169 60 A(a,b)

= 4 2 = 7
5 2 5

Bis
to
ec

ec
Bis

to
Remark

r
For comparing coefficients write equation in form
X¢ X
2x + y + l O B(x1,0) M C(x2,0)

+ y + Y'
2x m
coefficient of x is 2m + 2l , cofficient of y is l + m and coefficients Hence, equation of the bisectors of the lines given by Eq. (i)
of constant term is lm. is
13 15
i.e. l + m = , lm = ( x − α )2 − (y − β) 2 ( x − α) (y − β)
2 2 =
a −b h
y Example 22 Find the combined equation of the For X-axis, y = 0.
straight lines passing through the point (1, 1) and ( x − α) 2 − β 2 − β( x − α )
∴ =
parallel to the lines represented by the equation a−b h
x 2 − 5xy + 4 y 2 + x + 2y − 2 = 0. or h ( x − α) 2 + β ( x − α) (a − b ) − h β 2 = 0 …(ii)
Sol. Given equation of lines is Eq. (ii) is a quadratic in ( x − α ) and let two values of x be x 1
and x 2 , so that its roots are
x 2 − 5xy + 4y 2 + x + 2y − 2 = 0. …(i)
x 1 − α and x 2 − α
Since, x − 5xy + 4y = ( x − 4y )( x − y )
2 2
− β( a − b )
∴ ( x 1 − α) + ( x 2 − α) = Sum of roots =
Factors of Eq. (i) taken as ( x − 4y + l ) ( x − y + m ) . h
Now, equation of line through (1, 1) and parallel to ( x 1 − α) ( x 2 − α) = Products of roots = − β 2
x − 4y + l = 0 is x − 4y + λ = 0
∴ x 2 − x 1 = |( x 2 − α) − ( x 1 − α) |
i.e. 1− 4 + λ =0
∴ λ =3 = [( x 2 − α) + ( x 1 − α) ]2 − 4 ( x 2 − α) ( x 1 − α)
then line is x − 4y + 3 = 0 …(ii)  β 2 (a − b )2 
and equation of line through (1, 1) and parallel to ∴ | x 2 − x 1| =  2
+ 4 β2
x − y + m = 0 is x − y + µ = 0  h 
i.e. 1−1+µ =0 β
= ( a − b ) 2 + 4h 2
∴ µ =0 h
then line is x −y =0 …(iii) 1
∴ Area of ∆ABC = BC AM
Hence, equation of lines Eqs. (ii) and (iii) is 2
( x − 4y + 3) ( x − y ) = 0 1
= x 2 − x1 β
i.e. x − 5xy + 4y 2 + 3x − 3y = 0
2
2
1 β
= ( a − b ) 2 + 4h 2 × | β |
y Example 23 If ax 2 + 2hxy + by 2 + 2gx + 2 fy + c = 0 2 h

represents a pair of lines, prove that the area of ( a − b ) 2 + 4h 2 2


= .β
the triangle formed by their bisectors and axis of x 2h

(a − b ) 2 + 4h 2 ca − g 2 (a − b )2 + 4h 2 ca − g 2
is . . = .
2h ab − h 2 2h  ab − h 2
212 Textbook of Coordinate Geometry

Exercise for Session 4


1. If λx 2 + 10xy + 3y 2 − 15x − 21y + 18 = 0 represents a pair of straight lines. Then, the value of λ is
(a) – 3 (b) 3 (c) 4 (d) – 4

2. The point of intersection of the straight lines given by the equation 3y − 8xy − 3x − 29x + 3y − 18 = 0 is
2 2

(a)  1,  (b)  1, −  (c)  − ,


5
(d)  − , − 
1 1 3 3 5

 2  2  2 2  2 2

3. If the equation 12x 2 + 7xy − py 2 − 18x + qy + 6 = 0 represents two perpendicular lines, then the value of p and
q are
23 23
(a) 12, 1 (b) 12, – 1 (c) 12, (d) 12, −
2 2

4. If the angle between the two lines represented by 2x 2 + 5xy + 3y 2 + 7y + 4 = 0 is tan−1(m), then m is equal to
1 1 3 3
(a) − (b) (c) − (d)
5 5 5 5

5. The equation of second degree x 2 + 2 2xy + 2y 2 + 4x + 4 2 y + 1 = 0 represents a pair of straight lines, the
distance between them is
(a) 2 (b) 2 3 (c) 4 (d) 4 3

6. Find the area of the parallelogram formed by the lines


2x 2 + 5xy + 3y 2 = 0 and 2x 2 + 5xy + 3y 2 + 3x + 4y + 1 = 0.

7. Find the locus of the incentre of the triangle formed by


xy − 4x − 4y + 16 = 0 and x + y = a (a > 4, a ≠ 4 2 and a is the parameter).

8. If the equation 2hxy + 2gx + 2fy + c = 0 represents two straight lines, then show that they form a rectangle of
fg
area with the coordinate axes.
h2

9. Find the area of the triangle formed by the lines represented by ax 2 + 2hxy + by 2 + 2gx + 2fy + c = 0 and axis
of x.

10. Find the equations of the straight lines passing through the point (1, 1) and parallel to the lines represented by
the equation x 2 − 5xy + 4y 2 + x + 2y − 2 = 0.
Session 5
To Find the Point of Intersection of Lines
Represented by ax2 + 2hxy + by2 + 2gx+ 2fy + c = 0
with the Help of Partial Differentiation, Removal
of First Degree Terms, Equation of the Lines
Joining the Origin to the Points of Intersection
of a Given Line and a Given Curve
To Find the Point of Intersection from first two rows
of Lines Represented by and
a h g ⇒ ax + hy + g = 0
h b f ⇒ hx + by + f = 0 and then solve.
ax 2 + 2 hxy + by 2 + 2 gx y Example 24 Find the point of intersection of lines
+ 2 fy + c = 0 represented by 2x 2 − 7 xy − 4 y 2 − x + 22y − 10 = 0.
Let φ ( x , y ) ≡ ax 2 + 2hxy + by 2 + 2 gx + 2 fy + c = 0 Sol. Let φ ≡ 2x 2 − 7 xy − 4y 2 − x + 22y − 10 = 0
∂φ
∂φ ∴ ≡ 4 x − 7y − 1 = 0
∴ = 2ax + 2hy + 2 g [treating y as constant] ∂x
∂x ∂φ
∂φ and ≡ − 7 x − 8y + 22 = 0
and = 2hx + 2by + 2 f [treating x as constant] ∂y
∂y then, the point of intersection is ( x , y ) = (2, 1).
∂φ ∂φ
For point of intersection = 0 and = 0,
∂x ∂y
we obtain ax + hy + g = 0 and hx + by + f = 0
Removal of First Degree Terms
x y 1 Let point of intersection of lines represented by
Solving them = =
f h − bg gh − af ab − h 2 ax 2 + 2hxy + by 2 + 2gx + 2 fy + c = 0 …(i)
is (α, β).
 bg − fh af − gh 
(x , y ) =  2 ,   bg − fh af − gh 
 h − ab h 2 − ab  Here, (α, β) =  2 , 
 h − ab h 2 − ab 
Working rule In practice, therefore, the general equation of For removal of first degree terms, shift the origin to
second degree φ = 0, represents a pair of straight lines, we (α, β).
∂φ ∂φ i.e. Replacing x by (X + α) and y be (Y + β) in Eq. (i).
solve its partial derivatives = 0, = 0 for their intersecting
∂x ∂y Aliter : Direct equation after removal of first degree
point ( x , y ) = (α, β). terms is
aX 2 + 2hXY + bY 2 + ( gα + f β + c ) = 0
Remembering Method (without use of partial derivatives)
bg − fh
a h g where, α=
h 2 − ab
Since, ∆= h b f
af − gh
g f c and β= .
h 2 − ab
214 Textbook of Coordinate Geometry

y Example 25 Find the new equation of curve π  3 π


Taking θ=  we can also take θ =
12x 2 + 7 xy − 12y 2 − 17 x − 31y − 7 = 0 after removing 4  4 
1 1
the first degree terms. ⇒ cos θ = , sin θ =
2 2
Sol. Let φ ≡ 12x 2 + 7 xy − 12y 2 − 17 x − 31y − 7 = 0 …(i)
Now, if ( X ′ , Y ′ ) be the coordinates of the point when the
∂φ π
∴ = 24 x + 7y − 17 = 0 axes are rotated through θ = , we have
∂x 4
∂φ X ′ −Y ′ X ′ +Y ′
and ≡ 7 x − 24y − 31 = 0 X = ,Y =
∂y 2 2
Their point of intersection is ( x , y ) ≡ (1, − 1) then Eq. (i) becomes
Here, α = 1, β = − 1 Y ′2 X ′2 Y ′2 X ′2
− =1 ⇒ − 2 =1
Shift the origin to (1, − 1) then replacing x = X + 1 and 6 2 b2 a
y = Y − 1 in Eq. (i) the required equation is where, a 2 = 2 and b 2 = 6.
12( X + 1)2 + 7( X + 1) (Y − 1) − 12(Y − 1)2 − 17( X + 1)
− 31(Y − 1) − 7 = 0
i.e. 12X + 7 XY − 12Y = 0
2 2
Equation of the Lines Joining
Aliter :
Here, α = 1 and β = − 1
the Origin to the Points of
and
17 31
g = − , f = − ,c = − 7
2 2
Intersection of a Given Line
∴ gα + f β + c = −
17
2
×1−
31
2
× −1−7 =0 and a Given Curve
∴ Removed equation is Theorem The combined equation of the straight lines
aX 2 + 2hXY + by 2 + ( g α + f β + c ) = 0 joining the origin to the points of intersection of a second
degree curve
12X 2 + 7 XY − 12Y 2 + 0 = 0
ax 2 + 2hxy + by 2 + 2 gx + 2 fy + c = 0
12X 2 + 7 XY − 12Y 2 = 0.
and a straight line lx + my + n = 0 is :
y Example 26 Transform the equation x + 4 xy + y 2 2
 lx + my   lx + my 
ax 2 + 2hxy + by 2 + 2 gx   + 2 fy  
Y ′2 X ′2  −n   −n 
− 2x + 2y + 4 = 0 into the form 2 − 2 = 1.
b a 2
 lx + my 
+c   =0
Sol. To remove the first degree terms, we shift the origin to the  −n 
point (α, β).
bg − fh 1 × ( − 1) − 1 × 2 Proof The equation of the curve ( PAQ ) is
Then, α= 2 = = −1
h − ab 4 −1×1 ax 2 + 2hxy + by 2 + 2 gx + 2 fy + c = 0 …(i)
af − gh 1 × 1 − ( − 1) × (2)
and β= 2 = =1 and the equation of the line PQ be
h − ab 4 −1×1
lx + my + n = 0 …(ii)
then, the transformed equation is Y
X 2 + 4 XY + Y 2 + ( g α + f β + c ) = 0 P
⇒ X + 4 XY + Y + ( − 1 × ( − 1) + 1 × 1 + 4 ) = 0
2 2

∴ X 2 + 4 XY + Y 2 + 6 = 0 …(i)
A
Now, to remove the XY term from Eq. (i), we rotate the axes Q
through an angle θ given by
a−b X′
cot2θ = =0 [Q here a = b] O
X
2h Y′
⇒ cot2θ = 0
π 3π From the equation of the line Eq. (ii), find the value of ‘1’
⇒ 2θ = or in terms of x and y,
2 2
π 3π lx + my
⇒ θ= or i.e. =1 …(iii)
4 4 −n
Chap 03 Pair of Straight Lines 215

Now, the Eq. (i) can be written as If θ is the acute angle between pair of lines of Eq. (iii), then
or ax + 2hxy + by + (2 gx + 2 fy )(1) + c (1) = 0
2 2 2
2 h 2 − ab 2 (1 + 7 ) 2 8 4 2 2 2
tan θ = = = = =
 lx + my  a + b 7 − 1 6 6 3
or ax 2 + 2hxy + by 2 + (2 gx + 2 fy )  
 −n  2 2
∴ θ = tan −1  
 lx + my 
2  3 
+c   = 0 …(iv)
 −n 
y Example 28 Find the condition that the pair of
lx + my straight lines joining the origin to the intersections of
[replacing 1 by from Eq. (iii)]
−n the line y = mx + c and the circle x 2 + y 2 = a 2 may be
Hence, the Eq. (iv) is homogeneous equation of second at right angles.
degree. Above Eq. (iv) on simplification will be of the form Sol. The equations of the line and the circle are
Ax 2 + 2 Hxy + By 2 = 0 and will represent the required y = mx + c …(i)
straight lines. If θ be the angle between them, then and x 2 + y 2 = a2 …(ii)
 2 ( H 2 − AB )  The pair of straight lines joining the origin to the
θ = tan −1   intersections of Eqs. (i) and (ii), is obtained by making
  A + B 
  homogeneous Eq. (ii) with the help of Eq. (i).
y − mx
Hence, the equation of pairs of straight lines passing Q y = mx + c ⇒ =1
c
through the origin and the points of intersection of a 2
 y − mx 
curve and a line is obtained by making the curve ∴ x 2 + y 2 = a 2 ( 1) 2 ⇒ x 2 + y 2 = a 2  
 c 
homogeneous with the help of the line.
⇒ x 2 (c 2 − a 2m 2 ) + 2ma 2 xy + y 2 (c 2 − a 2 ) = 0 …(iii)
y Example 27 Prove that the angle between the lines The lines given by Eq. (iii), are at right angles, then
joining the origin to the points of intersection of the coefficient of x 2 + coefficient of y 2 = 0
straight line y = 3x + 2 with the curve
⇒ c 2 − a 2m 2 + c 2 − a 2 = 0
2 2
x 2 + 2xy + 3y 2 + 4 x + 8 y − 11 = 0 is tan −1  . ∴ 2c 2 = a 2 (1 + m 2 )
 3 
which is the required condition.
Sol. Equation of curve is x 2 + 2xy + 3y 2 + 4 x + 8y − 11 = 0 …(i)
y − 3x y Example 29 Prove that the pair of lines joining the
and line y = 3x + 2 ⇒ =1 …(ii) 2
2 x2 y
origin to the intersection of the curve 2 + 2 = 1 by
Making Eq. (i) homogeneous with the help of Eq. (ii), then a b
 y − 3x   y − 3x  the line lx + my + n = 0 are coincident, if a
x 2 + 2xy + 3y 2 + 4 x   + 8y  
 2   2  a 2l 2 + b 2m 2 = n 2 .
2
 y − 3x  x2 y2
−11   =0 =1 +
 2  Sol. The given curve is 2
…(i)
a b2
⇒ x 2 + 2xy + 3y 2 + 2xy − 6x 2 + 4y 2 − 12xy and line lx + my + n = 0
11 ⇒ lx + my = − n
− ( y − 3x ) 2 = 0
4 lx + my
⇒ =1 …(ii)
⇒ − 5x 2 − 8xy + 7y 2 −
11 2
(y − 6xy + 9 x 2 ) = 0 −n
4 Making Eq. (i) homogeneous with the help of Eq. (ii), then
⇒ − 20x 2 − 32xy + 28y 2 − 11y 2 + 66xy − 99 x 2 = 0 x2 y2  lx + my 
2
+ = 
⇒ 119 x − 34 xy − 17y = 0
2 2
a2 b 2  −n 
2 2
⇒ 7 x − 2xy − y = 0
2 2
…(iii) n x n 2y 2
⇒ + = l 2 x 2 + m 2y 2 + 2lmxy
This is the equation of lines joining the origin to the points a2 b2
of intersection of Eqs. (i) and (ii). n2 2 2 n2 2 2
⇒  2 − l  x − 2lmxy +  2 − m  y = 0 …(iii)
Comparing Eq. (iii) with ax 2 + 2hxy + by 2 = 0 a  b 
a = 7, h = − 1, b = − 1
216 Textbook of Coordinate Geometry

This is of the form Ax 2 + 2Hxy + By 2 = 0, y Example 30 Show that the straight lines joining the
n 2 origin to the points of intersection of curves
then A= − l 2 , H = − lm
a2 ax 2 + 2hxy + by 2 + 2gx = 0
n2 and a ′ x 2 + 2h ′ xy + b ′ y 2 + 2g ′ x = 0
and B= − m2
b2 are at right angles, if g ′ (a + b ) = g (a ′ + b ′ ).
The lines given by Eq. (iii) will be coincident, if Sol. The two curves meet in two points and the required lines
H 2 − AB = 0 ⇒ H 2 = AB joining the origin to these points will be obtained by making
one equation homogeneous with the help of the other.
n2  n2 
⇒ l 2m 2 =  2 − l 2   2 − m 2  ax 2 + 2hxy + by 2 + 2gx = 0 …(i)
a  b 
a ′ x 2 + 2h ′ xy + b ′ y 2 + 2g ′ x = 0 …(ii)
4 2 2 2 2
⇒ l 2m 2 =
n

n m

l n
+ l 2m 2 Multiplying Eq. (i) by g ′ and Eq. (ii) by g and subtracting,
a 2b 2 a2 b2 we get (ag ′ − a ′ g )x 2 + (2hg ′ − 2h ′ g )xy + (bg ′ − b ′ g )y 2 = 0
n4 n 2m 2 n 2l 2 If the lines are at right angles, then coefficient of
⇒ = +
2 2
ab a b2 2 x 2 + coefficient of y 2 = 0
⇒ n 2 = b 2m 2 + a 2l 2 ⇒ ag ′ − a ′ g + bg ′ − b ′ g = 0
∴ (a + b )g ′ = (a ′ + b ′ )g .
⇒ a l +b m =n
2 2 2 2 2

Exercise for Session 5


1. If the straight lines joining origin to the points of intersection of the line x + y = 1with the curve
x 2 + y 2 + x − 2y − m = 0 are perpendicular to each other, then the value of m should be
1 1
(a) − (b) 0 (c) (d) 1
2 2
2. The pair of straight lines joining the origin to the common points of x 2 + y 2 = 4 and y = 3x + c are
perpendicular, if c 2 is equal to
(a) –1 (b) 6 (c) 13 (d) 20

3. Mixed term xy is to be removed from the general equation of second degree


ax 2 + 2hxy + by 2 + 2gx + 2fy + c = 0, one should rotate the axes through an angle θ, then tan 2θ is equal to
(a − b ) 2h (a + b ) 2h
(a) (b) (c) (d)
2h (a + b ) 2h (a − b )

4. The lines joining the origin to the points of intersection of 2x 2 + 3xy − 4x + 1 = 0 and 3x + y = 1 are given by
(a) x 2 − y 2 − 5xy = 0 (b) x 2 − y 2 + 5xy = 0 (c) x 2 + y 2 − 5xy = 0 (d) x 2 + y 2 + 5xy = 0

5. The equation of the line joining the origin to the point of intersection of the lines 2x 2 + xy − y 2 + 5x − y + 2 = 0 is
(a) x + y = 0 (b) x − y = 0 (c) x − 2y = 0 (d) 2x + y = 0
6. Find the equation of the lines joining the origin to the points of intersection of 3x − 2y = 1 with
3x 2 + 5xy − 3y 2 + 2x + 3y = 0 and show that they are at right angles.

7. If the straight line joining the origin and the points of intersection of y = mx + 1and x 2 + y 2 = 1 be perpendicular
to each other, then find the value of m.
8. Prove that the straight lines joining the origin to the points of intersection of the straight line kx + hy = 2hk with
the curve ( x − h )2 + ( y − k )2 = c 2 are at right angles, if h 2 + k 2 = c 2.
9. Show that for all values of λ, the lines joining the origin to the points common to x 2 + 2hxy − y 2 + gx + fy = 0
and fx − gy = λ are at right angles.
10. Find the equations of the straight lines joining the origin to the points of intersection of x 2 + y 2 − 4x − 2y = 0
and x 2 + y 2 − 2x − 4y = 4.
Shortcuts and Important Results to Remember
1 If slope of one of the lines represented by 5 If the equation ax 2 + 2 hxy + by 2 + 2 gx + 2 fy + c = 0
ax 2 + 2 hxy + by 2 = 0 should be n times the slope of the represents a pair of parallel lines, then
other, then 4nh 2 = ab(1 + n )2 . (i) h 2 = ab, bg 2 = af 2 .
2 If the slope of one of the lines represented by (g 2 − ac )
ax 2 + 2 hxy + by 2 = 0 be the nth power of the other, then (ii) the distance between them = 2 .
a(a + b)
(abn )1/ n + 1 + (an b)1/ n + 1 + 2 h = 0
6 If ax 2 + 2 hxy + by 2 + 2 gx + 2 fy + c = 0 and
3 If two of the three lines represented by
ax 2 + 2 hxy + by 2 − 2 gx − 2 fy + c = 0 each represent a pair
ax 3 + bx 2 y + cxy 2 + dy 3 = 0 may be at right angles, then
of lines, then the area of the parallelogram enclosed by
a2 + ac + bd + d 2 = 0
2|c|
4 If pairs of straight lines x 2 + 2 m1 xy − y 2 = 0 and them is .
(h 2 − ab )
x 2 + 2 m2 xy − y 2 = 0 be such that each pair bisects the
angle between the other pair, then m1m2 = − 1
JEE Type Solved Examples :
Single Option Correct Type Questions
n This section contains 6 multiple choice examples. Each l Ex. 3 The pair of lines 3 x 2 - 4 xy + 3y 2 = 0 are
example has four choices (a), (b), (c) and (d) out of which p
ONLY ONE is correct. rotated about the origin by in the anti-clockwise sense.
6
The equation of the pair in the new position is
l Ex. 1 If the pairs of lines x 2 + 2 xy + ly 2 = 0 and
(a) x 2 - 3 xy = 0 (b) y 2 - 3 xy = 0
lx 2 + 2 xy + y 2 = 0 have exactly one line in common, then
the joint equation of the other two lines is given by (c) 3 x 2 - xy = 0 (d) 3y 2 - xy = 0
(a) 3 x 2 + 8 xy - 3y 2 = 0 (b) 3 x 2 + 10xy + 3y 2 = 0 Sol. (c) The given equation of pair of straight lines can be written
as
(c) x 2 + 2xy - 3y 2 = 0 (d) 3 x 2 + 2xy - y 2 = 0
( 3x - y ) ( x - 3y ) = 0
Sol. (b) Let y = mx, be a line common to the given pairs of lines,
1
then Þ y = 3x and y = x
3
lm 2 + 2m + 1 = 0 and m 2 + 2m + l = 0
Þ y = x tan 60° and y = x tan 30°
m2 m 1 After rotation, the separate equations are
Þ = =
2( l - 1) (1 - l2 ) 2( l - 1)
y = x tan 90° and y = x tan 60°
1
Þ m 2 = 1 and m = - ( l + 1) Þ x = 0 and y = x 3
2
2 Hence, the combined equation in the new position is
Þ ( l + 1) = 4 Þ l + 1 = 2 , - 2
x ( 3x - y ) = 0
\ l = 1 or - 3
But for l = 1, the two pairs have both the lines common. or 3x 2 - xy = 0.
So, l = - 3 and the slope m of the line common to both the
pairs is 1. Now, l Ex. 4 If the pair of lines ax 2 - 2 xy + by 2 = 0 and
x 2 + 2xy + ly 2 = x 2 + 2xy - 3y 2 = ( x - y ) ( x + 3y ) bx 2 - 2 xy + ay 2 = 0 be such that each pair bisects the angle
and lx 2 + 2xy + y 2 = - 3x 2 + 2xy + y 2 = - ( x - y ) (3x + y ) between the other pair, then | a - b | equals to
(a) 1 (b) 2
Hence, the joint equation of other two lines is
( x + 3y ) (3x + y ) = 0 or 3x 2 + 10xy + 3y 2 = 0. (c) 3 (d) 4
Sol. (b) According to the example, the equation of the bisectors of
l Ex. 2 The combined equation of the lines l1 and l 2 is the angle between the lines

2 x 2 + 6 xy + y 2 = 0 and that of the lines m1 and m 2 is ax 2 - 2xy + by 2 = 0 ...(i)


2 2
4 x 2 + 18 xy + y 2 = 0. If the angle between l1 and m 2 is a , is bx - 2xy + ay = 0 ...(ii)
then the angle between l 2 and m1 will be \The equation of bisectors of the angle between the lines
p p Eq. (i) is
(a) - a (b) + a (c) a (d) 2a
2 4 x 2 - y 2 xy
=
Sol. (c) The combined equation of the bisectors of the angles a-b -1
between the lines of the first pair is
Þ x 2 + (a - b )xy - y 2 = 0 ...(iii)
x 2 - y 2 xy 1
= Þ x 2 - y 2 = xy Since, Eqs. (ii) and (iii) are identical, comparing Eqs. (ii) and
2-1 3 3
(iii), we get
and the combined equation of the bisectors of the angles b -2 a
between the lines of the second pair is = =
1 a-b -1
x 2 - y 2 xy 1
= Þ x 2 - y 2 = xy Þ (a - b )b = - 2
4 -1 9 3
and (a - b )a = 2
It is clear that the two pairs are equally inclined to each
other. \ (a - b )2 = 4
Hence, the angle between l 2 and m1 is a. or a-b =2
Chap 03 Pair of Straight Lines 219

l Ex. 5 The equation of line which is parallel to the line (a) g 2 + f 2


=c (b) g 2 - f 2
=c
common to the pair of lines given by 3 x 2 + xy - 4y 2 = 0 and 2
(c) g - f 2
= 2c 2
(d) g + f 2
= c2
6 x 2 + 11xy + 4y 2 = 0 and at a distance of 2 units from it is
Sol. (c) Given,
(a) 3 x - 4y = - 10 (b) x - y = 2
x 2 + y 2 + 2gx + c = 0 ...(i)
(c) 3 x + 4y = 10 (d) 2x + y = - 2
2 2
Sol. (c) We have, 3 x 2 + xy - 4y 2 = 0 and x + y + 2 fy - c = 0 ...(ii)
or ( x - y ) ( 3x + 4y ) = 0 ...(i) On subtracting Eq. (ii) from Eq. (i), we get
and 6x 2 + 11xy + 4y 2 = 0 2gx - 2 fy + 2c = 0
or ( 2x + y ) ( 3x + 4y ) = 0 ...(ii) fy - gx
or =1 ...(iii)
Equation of line common to Eqs. (i) and (ii) is c
3x + 4y = 0 ...(iii) On adding Eqs. (i) and (ii), we get
Equation of any line parallel to Eq. (iii) is 2( x 2 + y 2 + gx + fy ) = 0
3x + 4y = l or x 2 + y 2 + gx + fy = 0 ...(iv)
Since, its distance from Eq. (iii) is 2, we have
Homogenising Eq. (iv) with the help of Eq. (iii), then
l -0
= 2 or l = ± 10 æ fy - gx ö
( 32 + 4 2 ) x 2 + y 2 + ( gx + fy ) ç ÷ =0
è c ø
Hence, required lines are 3x + 4y = ± 10. \The lines will be at right angles, when
æ g2 ö æ f 2ö
l Ex. 6 The lines joining the origin to the points of ç1 - ÷ + ç1 + ÷ =0
è c ø è c ø
intersection of x 2 + y 2 + 2 gx + c = 0 and
x 2 + y 2 + 2 fy - c = 0 are at right angles, if Þ g2 - f 2
= 2c

JEE Type Solved Examples :


More than One Correct Option Type Questions
n
This section contains 3 multiple choice examples. Each l Ex. 8 The lines (lx + my ) 2 - 3(mx - ly ) 2 = 0 and
example has four choices (a), (b), (c) and (d) out of which lx + my + n = 0 form
MORE THAN ONE may be correct.
(a) an isosceles triangle (b) a right angled triangle
(c) an equilateral triangle (d) None of these
l Ex. 7 The lines joining the origin to the point of
Sol. (a,c) (lx + my ) 2 - 3(mx - ly ) 2 = 0
intersection of 3 x 2 + mxy - 4 x + 1 = 0 and 2 x + y - 1 = 0 are
Þ (l 2 - 3m 2 )x 2 + 8mlxy + (m 2 - 3l 2 )y 2 = 0
at right angles. Which of the following is/are possible value
of m? Þ {(l + m 3 )x + (m - l 3 )y } {(l - 3m )x + (m + 3l )y } = 0
(a) - 4 (b) 3 (c) 4 (d) 7 Let slope of (l + m 3 )x + (m - l 3 )y = 0 be m1 and slope of
Sol. (a,b,c,d) Given line is 2 x + y = 1 ...(i) (l - m 3 )x + (m + l 3 )y = 0 be m 2 and slope of
and curve is 3x 2 + mxy - 4 x + 1 = 0 ...(ii) lx + my + n = 0 is m 3 .
Homogenising Eq. (ii) with the help of Eq. (i), then l l +m 3
2 2
+-
3x + mxy - 4 x (2x + y ) + (2x + y ) = 0 m 3 - m1 m m -l 3
Now, tan q 1 = =
2 2 1 + m 3m1 l æl + m 3ö
or - x + mxy + y = 0 1+ ×ç ÷
m èm - l 3ø
the lines are at right angles as a + b = 0, when h 2 > ab
3( l 2 + m 2 )
m2 = = 3
i.e. +1>0 (l 2 + m 2 )
4
which is true for all m Î R. Þ q 1 = 60°
220 Textbook of Coordinate Geometry

m 2 - m1 Sol. (a,c) Q m and m 2 are the roots of the equation


Similarly, tan q 2 = = 3 2
1 + m 2m1 æy ö æy ö
ç ÷ -6ç ÷ +a=0
Þ q 2 = 60° èx ø èx ø
Hence, an equilateral triangle is formed which is also an \ m + m 2 = 6 and m × m 2 = a ...(i)
isosceles one. Now, 2 3
( m + m ) = ( 6) 3

l Ex. 9 If the equation ax 2 - 6 xy + y 2 + bx + cy + d = 0 Þ m + m + 3m × m (m + m 2 ) = 216


3 6 2

represents a pair of lines whose slopes are m and m 2 , then Þ a + a 2 + 3a (6) = 216 [from Eq. (i)]
2
the value(s) of a is/are Þ a + 19a - 216 = 0
(a) - 27 (b) - 8 Þ (a + 27 ) (a - 8) = 0
(c) 8 (d) 27 \ a = - 27, 8

JEE Type Solved Examples :


Paragraph Based Questions
n
This section contains one solved paragraph based upon 10. (d) For D = 0,
each paragraph 2 multiple choice questions have to be
9 l l2
answered. Each of these questions has four choices (a), (b), 0+2´ ´ - 4 ´ - 0 - 0 + 12 ´ =0
2 2 4
(c) and (d) out of which ONLY ONE is correct.
Þ 3l2 - 18l = 0
Paragraph \ l = 0, 6
(Q. Nos. 10 to11) Hence, l =6 [Q for l = 0, it will give an
Consider the equation of a pair of straight lines as equation of first degree]
lxy - 8x + 9 y - 12 = 0 11. (b) Let f ( x, y ) = 6 xy - 8 x + 9y - 12 [Q l = 6]
10. The value of l is ¶f ¶f
\ = 6y - 8 and = 6x + 9
(a) 0 (b) 2 ¶x ¶y
(c) 4 (d) 6 ¶f ¶f
For point of intersection = 0 and = 0, we get
11. The point of intersection of lines is (a, b), then the ¶x ¶y
equation whose roots are a, b, is 3 4
x = - ,y =
(a) 4 x 2 + x - 8 = 0 (b) 6 x 2 + x - 12 = 0 2 3
3 4
(c) 4 x 2 - x - 8 = 0 (d) 6 x 2 - x - 12 = 0 \ a = - and b =
2 3
Sol. Given equation is, Hence, required equation is
lxy - 8x + 9y - 12 = 0 æ 3 4ö
x 2 - ç- + ÷x - 2 = 0
Here, a = 0, b = 0, c = - 12, è 2 3ø
9 l
f = , g = - 4 and h = Þ 6x 2 + x - 12 = 0
2 2
Chap 03 Pair of Straight Lines 221

JEE Type Solved Examples :


Single Integer Answer Type Questions
n This section contains 2 examples. The answer to each l Ex. 13 If one of the lines given by 6 x 2 - xy + 4cy 2 = 0 is
example is a single digit integer, ranging from 0 to 9 3 x + 4y = 0, then the value of | c | is
(both inclusive). Sol. (3) Q 3 x + 4y = 0 is one of the lines given by
6 x 2 - xy + 4cy 2 = 0, then
l Ex. 12 If the sum of the slopes of the lines given by
2
x 2 - 2cxy - 7y 2 = 0 is four times their product, then the æ 3x ö æ 3x ö é 3x ù
6x 2 - x ç - ÷ + 4c ç - ÷ =0 êëQy = - 4 úû
value of c is è 4 ø è 4 ø
Sol. (2) Given, m1 + m2 = 4m1m2 3 9c
2h a Þ 6+ + =0 ; \ c = -3
Þ - = 4 × Þ h = - 2a 4 4
b b Hence, |c| = 3
\ -c = -2´1 Þ c =2

JEE Type Solved Examples :


Matching Type Questions
n
This section contains one solved example. This example 2h
m + m2 = - ...(i)
has four statements (A, B, C and D) is Column I and four b
statements (p, q, r and s) in Column II. Any given a
and m × m2 = ...(ii)
statements in Column I can have correct matching with b
one or more statement(s) given in Column II. 3
æ 2h ö
From Eq. (i), (m + m 2 )3 = ç - ÷
è bø
l Ex. 14 Match the following
8h 3
Column I Column II Þ m 3 + m 6 + 3m × m 2 (m + m 2 ) = -
b3
(A) If the slope of one of the lines represented by (p) Odd prime a a 2 3a æ 2h ö 8h 3
ax 2 + 2hxy + by 2 = 0 is the square of the number Þ + 2 + ç - ÷ = - 3 [from Eqs. (i) and (ii)]
b b b è bø b
a + b 8h 2
other, then + is a 8h 3
h ab Þ ab + a 2 - 6ah = -
b
(B) The product of perpendiculars drawn from (q) Composite
the point (1, 2 ) to the pair of lines number a + b 8h 2
\ + =6
x 2 + 4 xy + y 2 = 0 is l units, then [ l ] is a h ab
(where [×] denotes the greatest integer (B) Let y = m1x and y = m 2 x lines represented by
function). x 2 + 4 xy + y 2 = 0, then m1 + m 2 = - 4, m1m 2 = 1. Then,
(C) Distance between two lines represented by (r) Even prime product of perpendiculars drawn from the point (1, 2)
the line pairx 2 - 4 xy + 4y 2 + x - 2y - 6 = 0 number
(2 - m1 ) (2 - m 2 )
is l unit, then [ l ] is a (where [×] denotes the =
greatest integer function). (1 + m12 ) (1 + m 22 )
(D) If the pairs a1x 2 + 2h1xy + b1y 2 = 0 and (s) Perfect 4 - 2 (m1 + m 2 ) + m1m 2
number =
a 2x 2 + 2h2xy + b2y 2 = 0 have one line 1 + (m1 + m 2 )2 - 2m1m 2 + (m1m 2 )2
(a1b2 - a 2b1 ) 2
common, then is a 4 +8+1 13
(b1h2 - b2h1 ) (a 2h1 - a1h2 ) = = units
(1 + 16 - 2 + 1) 4
Sol. (A) ® (q, s); (B) ® (p); (C) ® (r); (D) ® (q) 13
2 Q l=
(A) Let m and m be the slopes of the lines represented by 4
ax 2 + 2hxy + by 2 = 0, then \ [l] = 3
222 Textbook of Coordinate Geometry

(C) Given, line pair is x 2 - 4 xy + 4y 2 + x - 2y - 6 = 0 (D) Let y = mx be the common line, then
b1m 2 + 2h1m + a1 = 0 and b 2m 2 + 2h 2m + a 2 = 0.
Þ ( x - 2y )2 + ( x - 2y ) - 6 = 0
2
Þ ( x - 2y + 3) ( x - 2y - 2) = 0 b1 2h1 2h1 a1 a1 b1
Þ ´ =
\ Lines are x - 2y + 3 = 0 and x - 2y - 2 = 0 b 2 2h 2 2h 2 a 2 a2 b2
3 - ( - 2) Þ 4(b1h 2 - b 2h1 ) (a 2h1 - a1h 2 ) = (a1b 2 - a 2b1 )2
Hence, distance between lines = = 5 unit
1+ 4 (a1b 2 - a 2b1 )2
\ =4
\ l= 5 (b1h 2 - b 2h1 ) (a 2h1 - a1h 2 )
Hence, [ l] = [ 5 ] = 2

JEE Type Solved Examples :


Statement I and II Type Questions
n Directions (Ex. Nos. 15-16) Each of these examples contains Sol. (a) The pair of bisectors of 3 x 2 + 6 xy + 2y 2 = 0 is
two statements. x 2 - y 2 xy
= , i.e. 3 x 2 - xy - 3y 2 = 0 and pair of bisectors of
Statement I (Assertion) and 3 -2 3
Statement II (Reason) x 2 - y 2 xy
5 x 2 + 18 xy + 2y 2 = 0 is =
Each of these examples also has four alternative choices, 5 -2 9
only one of which is the correct answer. You have to select
i.e. 3 x 2 - xy - 3y 2 = 0 are coincides.
the correct choice, as given below :
(a) Statement I is true, Statement II is true and Statement II is a \ Angle between l1, m2 is same as angle between l 2, m1.
correct explanation for Statement I \ Both statements are true and Statement II is a correct
(b) Statement I is true, Statement II is true and Statement II is explanation for Statement I.
not a correct explanation for Statement I Ex. 16 Statement I The equation
(c) Statement I is true, Statement II is false 2 x 2 - 3 xy - 2y 2 + 5 x - 5y + 3 = 0 represents a pair of
(d) Statement I is false, Statement II is true
perpendicular straight lines.
Ex. 15 Statement I The combined equation of l1 , l 2 is Statement II A pair of lines given by
3 x 2 + 6 xy + 2y 2 = 0 and that of m1 , m 2 is ax 2 + 2hxy + by 2 + 2 gx + 2 fy + c = 0 are perpendicular if
5 x 2 +18 xy + 2y 2 = 0. If angle between l1 , m 2 is q, then angle a + b = 0.
between l 2 , m1 is q. Sol. (d) Here, D ¹ 0, h 2 > ab and a + b = 0
Statement II If the pairs of lines l1l 2 = 0, m1m 2 = 0 are \ 2x 2 - 3xy - 2y 2 + 5x - 5y + 3 = 0
equally inclinded that angle between l1 and m 2 = angle represents a rectangular hyperbola.
between l 2 and m1 . \ Statement I is false and Statement II is true.

Subjective Type Examples


n
In this section, there are 10 subjective solved examples. and the diagonal 5x + 2y = 1 does not pass through origin,
hence it is AC.
l Ex. 17 If the lines represented by 2 x 2 - 5 xy + 2y 2 = 0 be On solving, x - 2y = 0 and 5x + 2y = 1, we get
the two sides of a parallelogram and the line 5 x + 2y = 1 be æ1 1 ö
one of its diagonal. Find the equation of the other diagonal, A=ç , ÷
è 6 12 ø
and area of the parallelogram.
and solving 2x - y = 0 and 5x + 2y = 1 , we get
Sol. Given pair of lines, 2 x 2 - 5 xy + 2y 2 = 0
æ1 2ö
i.e. ( x - 2y ) (2x - y ) = 0 C º ç , ÷.
è9 9 ø
\ x - 2y = 0 and 2x - y = 0
Chap 03 Pair of Straight Lines 223

Y B Þ {(1 + m1 )y - (m1 - 1)x } {(1 + m 2 )y - (m 2 - 1)x } = 0


C Þ (1 + m1 ) (1 + m 2 )y 2 - xy {(1 + m1 )(m 2 - 1)
+ (m1 - 1) (1 + m 2 )} + (m1 - 1) (m 2 - 1)x 2 = 0

5x+2
0
y=
2x– H
Þ (1 + m1 + m 2 + m1m 2 )y 2 - 2xy (m1m 2 - 1)
y =1 A
y=0 + {m1m 2 - (m1 + m 2 ) + 1} x 2 = 0
X′ x–2
X
O æ 2h a ö 2 æa ö æ a 2h ö
Y′ Þ ç1 - + ÷y - 2xy ç - 1÷ + ç + + 1÷ x 2 = 0
è b b ø èb ø èb b ø
Since, diagonals of parallelogram bisect each other, if bisect
at H . \ (a + 2h + b )x 2 - 2(a - b )xy + (a - 2h + b )y 2 = 0
ì 1 æ 1 1 ö 1 æ 1 2 öü
Then, H º í ç + ÷ , ç + ÷ý l Ex. 19 If u º ax 2 + 2hxy + by 2 + 2 gx + 2 fy + c = 0
î 2 è 6 9 ø 2 è 12 9 øþ
represents a pair of straight lines, prove that the equation of
æ 5 11 ö the third pair of straight lines passing through the points
i.e. H =ç , ÷
è 36 72 ø where these meet the axes is cu + 4( fg - ch ) xy = 0.
Hence, the equation of other diagonal which passes through Sol. u º ax 2 + 2hxy + by 2 + 2 gx + 2 fy + c = 0
O and H is
Q u represents a pair of straight line, then D = 0
11
-0 \ abc + 2 fgh - af 2
- bg 2 - ch 2 = 0 …(i)
y - 0 = 72 ( x - 0)
5 Combined equation of axes is xy = 0
-0
36 Now, the curve through the intersection of
11 u = 0 and xy = 0 is
Þ y= x
10 u + lxy = 0 …(ii)
Þ 11x - 10y = 0 Y

Now, area of parallelogram = 2 ´ Area of D OAC


11 2 1 1 1
=2´ ´ - ´ = sq units Third pair
2 6 9 9 12 36

l Ex. 18 Prove that the equation


(a + 2h + b ) x 2 - 2(a - b ) xy + (a - 2h + b ) y 2 = 0 represents X'
O
X

a pair of lines each inclined at an angle of 45° to one or


Y'
other of the lines given by, ax 2 + 2hxy + by 2 = 0.
i.e. ax 2 + 2hxy + by 2 + 2gx + 2 fy + c + lxy = 0
Sol. Given equation is
Þ ax 2 + xy (2h + l ) + by 2 + 2gx + 2 fy + c = 0
ax 2 + 2hxy + by 2 = 0 …(i)
If it represents a pair of lines, then
Let the lines represented by Eq. (i) are
2
y - m1x = 0 …(ii) æ lö 2 æ lö
abc + 2 fg çh + ÷ - af - bg 2 - c çh + ÷ = 0
and y - m2 x = 0 …(iii) è 2ø è 2ø
2h a 2 cl2
therefore, m1 + m 2 = - , m1m 2 = Þ (abc + 2 fgh - af - bg 2 - ch 2 ) + l( fg - ch ) - =0
b b 4
If m1 = tanq 1 and m 2 = tanq 2 , the equation of lines through cl2
the origin making 45° with the lines Eqs. (ii) and (iii) will be Þ 0 + l ( fg - ch ) - =0 [from Eq. (i)]
4
y - x tan(q 1 - 45° ) = 0 4( fg - ch )
\ l= …(iii)
and y - x tan(q 2 - 45° ) = 0 c
Their combined equation is Hence, the equation of third pair from Eqs. (ii) and (iii) is
{y - x tan(q 1 - 45° )} {y - x tan(q 2 - 45° ) } = 0 4( fg - ch )
u+ xy = 0
æ m -1 ö æ m -1 ö c
Þ çy - 1 x ÷ çy - 2 x÷ = 0
è 1 + m1 ø è 1 + m2 ø \ cu + 4( fg - ch ) xy = 0
224 Textbook of Coordinate Geometry

l Ex. 20 If the equation ax 2 + 2hxy + by 2 + 2 gx we show that by an example that these conditions are not
sufficient because
+ 2 fy + c = 0 represents a pair of parallel lines, prove that h2 = ab Þ h= ± ab
(i) h = ab and g b = f a or (h = - ab and 2 2
and bg = af Þ g b=±f a
g b = - f a ).
Þ h= ab, g b = f a or ( h = - ab, g b = - f a)
æ g 2 - ac ö
(ii) the distance between them is 2 ç ÷. Consider, for example, 4 x + 4 xy + y 2 + 4 x - 2 y + 5 = 0
2

è a (a + b ) ÷ø Here, a = 4, h = 2, b = 1 , g = 2, f = - 1, c = 5 and
Sol. Given equation is, h= ab, g b = - f a
ax 2 + 2hxy + by 2 + 2gx + 2 fy + c = 0 …(i) But abc + 2fgh - af 2 - bg 2 - ch2
Let the equation of the parallel lines represented = 4 ´ 1 ´ 5 + 2 ´ ( -1) ´ 2 ´ 2 - 4 ´ ( -1) 2 - 1 ´ ( 2) 2 - 5 ´ ( 2) 2
by Eq. (i) be = 20 - 8 - 4 - 4 - 20 = - 16 ¹ 0
lx + my + n = 0 and lx + my + n1 = 0. Hence, the Eq. (i) does not represent a pair of lines at all.
Then, (lx + my + n ) (lx + my + n1 )
º ax 2 + 2hxy + by 2 + 2gx + 2 fy + c l Ex. 21 A parallelogram is formed by the lines
Equating the coefficients, we get ax 2 + 2hxy + by 2 = 0 and the lines through ( p, q ) parallel to
l2 = a …(ii) them. Show that the equation of the diagonal of the
parallelogram which does not pass through origin is
m2 = b …(iii)
( 2 x - p )(ap + hq ) + ( 2y - q ) (hp + bq ) = 0.
nn1 = c …(iv) Show also that the area of the parallelogram is
2lm = 2h …(v) |(ap 2 + 2hpq + bq 2 )| / 2 (h 2 - ab ) .
(n + n1 )l = 2g …(vi) Sol. The combined equation of AB and AD is
m( n + n1 ) = 2 f …(vii) S1 º ax 2 + 2hxy + by 2 = 0
Eq. (i) From Eq. (v)
Now, equation of lines through ( p , q ) and parallel to S1 º 0
Þ h = lm = ± ab is S 2 º a( x - p )2 + 2h ( x - p )(y - q ) + b(y - q )2 = 0
Now, h = ab or h = - ab C(p,q)
2 2 2
Q abc + 2 fgh - af - bg - ch = 0 D
2
Þ abc + 2 fg ab - af - bg 2 - c × ab = 0
[substituting the value of h = ab ]
Þ - ( f a - g b )2 = 0 Þ f a=g b B

Thus, the given equation represents a pair of lines. A(0,0)


Also, if h = - ab , then g b = - f a
Hence, the equation of diagonal BD is S1 - S 2 = 0
(ii) The distance between parallel lines
a( -2xp + p 2 ) + 2h ( - py - qx + pq ) + b ( -2qy + q 2 ) = 0
| n - n1| (n + n1 )2 - 4nn1
= =
-ap (2x - p ) - hq (2x - p ) - hp (2y - q ) - bq (2y - q ) = 0
(l 2 + m 2 ) l 2 + m2
[Q2hpq is written as hpq + hpq]
æ 4g 2 ö
ç 2 - 4c ÷ Hence, diagonal of BD is
è l ø (2x - p ) (ap + hq ) + (2y - q ) (bq + hp ) = 0
= [from Eqs. (vi), (ii) and (iii)]
(a + b ) pp
Area of parallelogram = 1 2
æ g 2 - ac ö sinq
=2 ç ÷ [Ql 2 = a ] Þ p 1p 2 = product of perpendiculars from ( p , q) on AB and
è a(a + b ) ø
AD whose combined equation is
ax 2 + 2hxy + by 2 = 0
Remark
| ap 2 + 2hpq + bq 2 |
In some books, the conditions for parallel lines are stated as \ p 1p 2 =
h2 = ab and bg 2 = af 2 {(a - b )2 + 4h 2 }
Chap 03 Pair of Straight Lines 225

2 (h 2 - ab ) On squaring both sides, we get


\ tan q =
|(a + b )| ( x 12 + y12 ) (h 2 - ab ) = k 2 {(a - b )2 + 4h 2 }

2 (h 2 - ab ) Hence, locus of P ( x 1, y1 ) is
\ sin q = ( x 2 + y 2 ) (h 2 - ab ) = k 2 {(a - b )2 + 4h 2 }.
( a - b ) 2 + 4h 2
| ap 2 + 2hpq + bq 2 | l Ex. 23 Show that if two of the lines ax 3 + bx 2 y + cxy 2
\ Required area =
2
2 (h - ab ) + dy 3 = 0 (a ¹ 0 ) make complementary angles with X-axis in
anti-clockwise sense, then a (a - c ) + d (b - d ) = 0.
l Ex. 22 A point moves so that the distance between the Sol. The given equation can be written as
feet of perpendiculars drawn from it to the lines ax 2 + 2hxy y
dm 3 + cm 2 + bm + a = 0 , where m =
+ by 2 = 0 is a constant 2k. Show that the equation of its x
locus is ( x 2 + y 2 ) (h 2 - ab ) = k 2 {(a - b ) 2 + 4h 2 }. Let its roots be m1, m 2 , m 3 .
p a
Sol. Q ÐONP = Ð OMP = \ m1 m 2 m 3 = - …(i)
2 d
\ O, M, P, N are concyclic with diameter OP. If m1 = tana, then m 2 = tan(90° - a )
Y N [Q two lines makes complementary
angles with X-axis]
k P(x1,y1) \ m 2 = cota then m1m 2 = 1
r L a
θ From Eq. (i), m3 = -
k
O' θ d
r Since, m 3 is root of the above cubic, we have
θ r M
æ a3 ö æ a2 ö æ aö
X′ X d ç- 3 ÷ + c ç 2 ÷ + bç- ÷ + a = 0
O è dø
è d ø èd ø
Y′
æ a 3 ö æ ca 2 ö æ ab ö
Let P º ( x 1, y1 ) Þ ç- 2 ÷ + ç 2 ÷ - ç ÷ + a = 0
æx y ö è d ø èd ø èd ø
\ Coordinates of centre are O ¢ = ç 1 , 1 ÷
è2 2ø Þ - a 3 + ca 2 - abd + ad 2 = 0
\ Radius r = O ¢ N = OM = OO ¢ On dividing each by a, we get
2 2
æx ö æy ö ( x 12 + y12 ) -a 2 + ca - bd + d 2 = 0
= ç 1÷ + ç 1÷ =
è2ø è2ø 2 \ a (a - c ) + d (b - d ) = 0.
Let Ð MON = q
\ Ð MO ¢ N = 2q Þ Ð NO ¢ L = Ð MO ¢ L = q
l Ex. 24 Show that the equation
and given MN = 2k , then NL = LM = k a ( x 4 + y 4 ) - 4bxy ( x 2 - y 2 ) + 6cx 2 y 2 = 0 represents two
Now, in DNO ¢ L, pairs of lines at right angles and that if 2b 2 = a 2 + 3ac , the
NL k k 2k two pairs will coincide.
sin q = = = = …(i)
O¢N r 2 2
( x 1 + y1 ) ( x 1 + y12 )
2 Sol. Given equation is
2 a( x 4 + y 4 ) - 4bxy ( x 2 - y 2 ) + 6c x 2y 2 = 0 …(i)
But angle between the lines represented by Eq. (i) is a homogeneous equation of fourth degree and
2
2 (h - ab ) since it represents two pairs at right angles. i.e. sum of the
ax 2 + 2hxy + by 2 = 0 is tan q = . coefficients of x 2 and y 2 should be zero.
|a + b |
Let a( x 4 + y 4 ) - 4bxy ( x 2 - y 2 ) + 6cx 2y 2
2 (h 2 - ab )
\ sin q = …(ii)
( a - b ) 2 + 4h 2 = (ax 2 + pxy - ay 2 )( x 2 + qxy - y 2 ),
where, p and q are constants.
From Eqs. (i) and (ii), we get
On comparing similar powers, we get
2 (h 2 - ab ) 2k
= p + aq = - 4b …(ii)
2 2
( a - b ) + 4h ( x 12 + y12 ) -2a + pq = 6c …(iii)
226 Textbook of Coordinate Geometry

Again, given two pairs coincide, then = k 3 (a 2 + b 2 - 2ac + c 2 - 2bd + d 2 )


p
=q = k 3 {(a - c )2 + (b - d )2 }
a
or p = aq …(iv) Hence, locus of P is
From Eqs. (ii) and (iv), (ay 3 + by 2 x + cyx 2 + dx 3 ) = k 3 {(a - c )2 + (b - d )2 }
2b
q=-
a l Ex. 26 If one of the lines given by the equation
and p = - 2b ax 2 + 2hxy + by 2 = 0 coincides with one of the lines given
Substituting the values of p and q in Eq. (iii), we get by a ¢ x 2 + 2h ¢ xy + b ¢ y 2 = 0 and the other lines represented
2
4b by them be perpendicular, then .
-2a + = 6c
a ha ¢ b ¢ h ¢ ab 1
= = ( -aa ¢ bb ¢ )
Þ -a 2 + 2b 2 = 3ac b¢ - a¢ b - a 2
\ 2b 2 = a 2 + 3ac Sol. Let the two lines represented by
ax 2 + 2hxy + by 2 = 0 …(i)
l Ex. 25 Show that the locus of a point such that the be y = m1x
product of the perpendiculars let fall from it on three lines and y = m2x
represented by ay 3 + by 2 x + cyx 2 + dx 3 = 0 is 2h
\ m1 + m 2 = - …(ii)
constant = k 3 , is b
a
ay 3 + by 2 x + cyx 2 + dx 3 = k 3 (a - c ) 2 + (b - d ) 2 . and m1m 2 = …(iii)
b
Sol. Let the three represented lines are and the lines represented by
y = m1x , y = m 2 x and y = m 3 x a ¢ x 2 + 2h ¢ xy + b ¢ y 2 = 0 …(iv)
ay + by 2 x + cyx 2 + dx 3 = a(y - m1x ) (y - m 2 x ) (y - m 3 x )
3
1
be y=- x and y = m 2 x
On comparing similar powers on both sides m1
b 1 2h ¢
Þ m1 + m 2 + m 3 = - …(i) \ - + m2 = - …(v)
a m1 b¢
c æ 1 ö a¢
m1m 2 + m 2m 3 + m 3m1 = …(ii) and - ç ÷m 2 = …(vi)
a è m1 ø b¢
d
m1m 2m 3 = - …(iii) From Eqs. (iii) and (vi), we get
a
aa ¢ ( -aba ¢ b ¢ )
Let the point P ( x 1, y1 ), from given condition - m 22 = Þ m2 =
bb ¢ bb ¢
(y1 - m1x 1 ) (y1 - m 2 x 1 ) (y1 - m 3 x 1 )
. . = k3 ( -aba ¢ b ¢ )
(1 + m12 ) (1 + m 22 ) (1 + m 32 ) From Eqs. (iii), we get m1 = -
a¢b
1
Þ (ay13 + by12 x 1 + cy1x 12 + dx 13 ) Substituting these values of m1 and m 2 in Eqs. (ii) and (v),
a we get
= k 3 (1 + m12 ) (1 + m 22 ) (1 + m 32 ) -aba ¢ b ¢ -aba ¢ b ¢ 2h
- + =-
3 2 2
[Qay + by x + cyx + dx 3 a¢b bb ¢ b
ha ¢ b ¢ 1
= a (y - m1x ) (y - m 2 x )(y - m 3 x )] Þ = -aa ¢ bb ¢ …(vii)
b¢ - a¢ 2
Þ (ay13 + by12 x 1 + cy1x 12 + dx 13 )
a¢b -aba ¢ b ¢ 2h ¢
= ak 3 (1 + Sm12 + Sm12m 22 + m12m 22m 32 ) and + =-
-aba ¢ b ¢ bb ¢ b¢
{1 + ( Sm1 )2 - 2 Sm1m 2 + ( Sm1m 2 )2 h ¢ ab 1
= - aba ¢ b ¢ …(viii)
= ak 3 b-a 2
- 2m1m 2m 3 Sm1 + (m1m 2m 3 )2 }
2
From Eqs. (vii) and (viii), we get
3 b2
2c c 2 2d æ b ö æ d ö ha ¢ b ¢ h ¢ ab 1
= ak 1+ 2 - + + .ç- ÷ + ç- ÷ = = - aba ¢ b ¢
a a a2 a è aø è aø b¢ - a¢ b - a 2
#L Pair of Straight Lines Exercise 1 :
Single Option Correct Type Questions
n This section contains 12 multiple choice questions. (a) 5 (b) 2 5
Each question has four choices (a), (b), (c), (d) out of which (c) 3 5 (d) 4 5
ONLY ONE is correct.
8. The four straight lines given by the equations
1. If the sum of the slopes of the lines given by 12x 2 + 7 xy − 12y 2 = 0 and
4 x + 2λxy − 7y = 0 is equal to the product of the
2 2
12x 2 + 7 xy − 12y 2 − x + 7y − 1 = 0 lie along the sides of a
slopes, then λ is equal to (a) square (b) rhombus
(a) − 4 (b) − 2 (c) 2 (d) 4 (c) rectangle (d) parallelogram
2. The equation 3ax 2 + 9 xy + (a 2 − 2)y 2 = 0 represents two 9. Distance between the parallel lines
perpendicular straight lines for 4 x 2 + 20xy + 25y 2 + 2x + 5y − 12 = 0 is
(a) only one value of a (b) for all values of a 3 5
(c) for only two values of a (d) for no value of a (a) (b)
29 29
3. The image of the pair of lines represented by 7 9
(c) (d)
ax 2 + 2hxy + by 2 = 0 by the line mirror y = 0 is 29 29
(a) ax 2 − 2hxy − by 2 = 0 (b) bx 2 − 2hxy + ay 2 = 0 10. The point of intersection of the two lines given by
(c) bx 2 + 2hxy + ay 2 = 0 (d) ax 2 − 2hxy + by 2 = 0 2x 2 − 5xy + 2y 2 + 3x + 3y + 1 = 0 is
(a) ( − 2, 2 ) (b) ( − 3, 3 )
4. Number of points lying on the line 7 x + 4y + 2 = 0 which
(c) (3, 3 ) (d) (2, 2 )
is equidistant from the lines 15x 2 + 56xy + 48y 2 = 0 is
11. If α, β > 0 and α < β and
(a) 0 (b) 1
(c) 2 (d) 4 αx 2 + 4 γxy + βy 2 + 4 p ( x + y + 1) = 0 represents a pair
5. Orthocentre of the triangle formed by the lines of straight lines, then
xy − 3x − 5y + 15 = 0 and 3x + 5y = 15 is (a) α ≤ p ≤ β (b) p ≤ α
(a) ( − 5, − 3 ) (b) (5, 3 ) (c) p ≥ α (d) p ≤ α or p ≥ β
(c) ( − 3 , − 5 ) (d) (3, 5 ) 12. If the equation of the pair of straight lines passing
6. Two of the straight lines given by through the point (1, 1), one making an angle θ with the
3x + 3x y − 3xy + dy = 0 are at right angles, if d
3 2 2 3 positive direction of the X -axis and the other making the
same angle with the positive direction of the Y -axis, is
equal to
x 2 − (a + 2)xy + y 2 + a ( x + y − 1) = 0, a ≠ − 2 , then the
(a) − 4 (b) − 3
(c) − 2 (d) − 1 value of sin 2θ is
(a) a − 2 (b) a + 2
7. Two lines are given by ( x − 2y ) 2 + λ( x − 2y ) = 0. The 2 2
(c) (d)
value of | λ | so that the distance between them is 3, is (a + 2 ) a

#L Pair of Straight Lines Exercise 2 :


More than One Option Correct Type Questions
n
This section contains 6 multiple choice questions. Each 14. The equation ax 2 + by 2 + cx + cy = 0 represent a pair of
questions has four choices (a), (b), (c), (d) out of which straight lines, if
MORE THAN ONE may be correct. (a) a + b = 0
13. The equation of image of pair of lines y = | x − 1 | in (b) c = 0
(c) a + c = 0
Y -axis is (d) c (a + b ) = 0
(a) y = | x + 1 | (b) y = | x − 1 | + 3
(c) x 2 − y 2 + 2 x + 1 = 0 (d) x 2 − y 2 + 2 x − 1 = 0
228 Textbook of Coordinate Geometry

15. If x 2 + αy 2 + 2βy = a 2 represents a pair of 17. Two pair of straight lines have the equations
perpendicular straight lines, then y 2 + xy − 12x 2 = 0 and ax 2 + 2hxy + by 2 = 0.
(a) α = 1, β = a One line will be common among them, if
(b) α = 1, β = − a (a) a = − 3(2h + 3b ) (b) a = 8(h − 2b )
(c) α = − 1, β = − a (c) a = 2(b + h ) (d) a = − 3(b + h )
(d) α = − 1, β = a
18. The combined equation of three sides of a triangle is
16. If the pair of lines ax 2 + 2hxy + by 2 + 2gx + 2 fy + c = 0 ( x 2 − y 2 ) (2x + 3y − 6) = 0. If ( − 2 , a ) is an interior and
intersect on Y -axis, then
(b, 1) is an exterior point of the triangle, then
(a) f 2 = bc 10 10
(b) abc = 2 fgh (a) 2 < a < (b) − 2 < a <
3 3
(c) bg 2 ≠ ch 2 9
(c) − 1 < b < (d) − 1 < b < 1
(d) 2 fgh = bg 2 + ch 2 2

#L Pair of Straight Lines Exercise 3 :


Paragraph Based Questions
n This section contains 3 paragraphs based upon each of (c) ax 2 + 2hxy + by 2 = 0
the paragraph, 3 multiple choice questions have to be (d) None of the above
answered. Each of these question has four choices (a), (b),
23. If f i + 1 ( x , y ) = 0 represents the equation of a pair of
(c) and (d) out of which ONLY ONE is correct.
perpendicular lines, thenf 3 ( x , y ) = 0 is same as
Paragraph I (a) f1( x, y ) = 0
(Q. Nos. 19 to 21) (b) f 2( x, y ) = 0
(c) hx 2 − (a − b ) xy − hy 2 = 0
Consider the equation of a pair of straight lines as
(d) None of the above
x 2 − 3xy + λy 2 + 3x − 5 y + 2 = 0
5 f n + 2 ( x, y )
19. The value of λ is 24. The value of ∑ f n ( x, y )
is
(a) 1 (b) 2 n=2
(c) 3 (d) 4 (a) 14 (b) 4
(c) 54 (d) 6
20. The point of intersection of lines is (α, β), then the value
of α 2 + β 2 is Paragraph III
(a) 2 (b) 5 (Q. Nos. 25 to 27)
(c) 10 (d) 17 Consider a pair of perpendicular straight lines
21. The angle between the lines is θ, then the value of cos 2θ 2x 2 + 3xy + by 2 − 11x + 13 y + c = 0
is
1 2 25. The value of c is
(a) (b) (a) − 2 (b) 2
3 3
3 4 (c) − 3 (d) 3
(c) (d)
5 5 26. The value of | b + 2c | is
Paragraph II (a) 4 (b) 6
(c) 8 (d) 10
(Q. Nos. 22 to 24)
27. If point of intersection of lines is C and points of
Let f1 ( x, y ) ≡ ax 2 + 2hxy + by 2 = 0 and let f i + 1 ( x, y ) = 0 intersection of the lines with the X -axis are A and B, if
denote the equation of the bisectors of f i ( x, y ) = 0 for all distance between the orthocentre and the circumcentre
i = 1, 2, 3, ... of ∆ABC is λ, then [λ] is (where [. ] denotes the greatest
22. f 3 ( x , y ) = 0 is integer function
(a) 2 (b) 3
(a) hx 2 − (a − b ) xy − hy 2 = 0 (c) 4 (d) 5
(b) (a − b ) x 2 + 4hxy − (a − b )y 2 = 0
Chap 03 Pair of Straight Lines 229

#L Pair of Straight Lines Exercise 4 :


Single Integer Answer Type Questions
n This section contains 5 questions. The answer to each 30. The lines represented by x 2 + 2λxy + 2y 2 = 0 and ( λ + 1)
question is a single digit integer, ranging from 0 to 9 x 2 − 8xy + y 2 = 0 are equally inclined, then the value of
(both inclusive).
| λ | is
28. Equation λx 3 − 10x 2 y − xy 2 + 4y 3 = 0 represented
31. If the lines joining the origin to the intersection of the
three straight lines, out of these three, two lines makes line y = nx + 2 and the curve x 2 + y 2 = 1 are at right
equal angle with y = x and λ > 0, then the value of λ is
angles, then the value of n 2 is
29. Area enclosed by curves y 2 − 5xy + 6x 2 + 3x − y = 0 and
32. If area of the triangle formed by the line x + y = 3 and
y 2 − 5xy + 6x 2 + 2x − y = 0 is λ sq units, then the value
the angle bisectors of the pair of straight lines
of λ is
x 2 − y 2 + 2y = 1 is λ sq units, then the value of λ is

#L Pair of Straight Lines Exercise 5 :


Matching Type Questions
n
This section contains one question. This question has four statements (A,B,C and D) given in Column I and four
statements (p, q, r and s) in Column II. Any given statements in Column I can have correct matching with one or more
statement(s) given in Column II.
33. Match the following
Column I Column II
(A) The pair of lines joining the origin to the (p) 2
points of intersection of the curve
9x 2 + 16 y2 = 144 by the line 2x + 2 y + λ = 0 are coincident, then | λ | is divisible by

(B) If the straight lines joining the origin to the points of intersection of the straight line 4 x + 3 y = 24 (q) 3
and the curve (x − 3)2 + ( y − 4 )2 = λ2, are at right angles, then | λ | is divisible by

(C) The two line pairs y2 − 4 y + 3 = 0 and x 2 + 4 xy + 4 y2 − 5x − 10 y + 4 = 0 enclose a 4 sided (r) 5


convex polygon, if area of polygon is λ sq units, then λ is divisible by
(D) If the pairs of lines 3x 2 − 2 pxy − 3 y2 = 0 and 5x 2 − 2qxy − 5 y2 = 0 are such that each pair bisects (s) 6
the angle between the other pair. If λ = | pq | , then λ is divisible by
230 Textbook of Coordinate Geometry

#L Pair of Straight Lines Exercise 6 :


Statement I and II Types Questions
n Directions (Q. Nos. 34-37) are Assertion-Reason type 35. Statement I Two of the straight lines represented by
questions. Each of these questions contains two dx 3 + cx 2 y + bxy 2 + ay 3 = 0 will be at right angles if
statements:
d 2 + bd + bc + a 2 = 0.
Statement I (Assertion) and Statement II (Reason)
Each of these questions also has four alternative choices, Statement II Product of the slopes of two perpendicular
lines is − 1.
only one of which is the correct answer. You have to select
the correct choice as given below : 36. Statement I If αβ = − 1, then the pair of straight lines
(a) Statement I is true, Statement II is true; Statement II is a x 2 − 2αxy − y 2 = 0 and y 2 + 2βxy − x 2 = 0 are the angle
correct explanation for Statement I bisector of each other.
(b) Statement I is true, Statement II is true; Statement II is not a Statement II Pair of angle bisector lines of the pair of
correct explanation for Statement I lines ax 2 + 2hxy + by 2 = 0 is h( x 2 − y 2 ) = (a − b )xy.
(c) Statement I is true, Statement II is false
(d) Statement I is false, Statement II is true 37. Statement I If a + b = − 2h, then one line of the pair of
lines ax 2 + 2hxy + by 2 = 0 bisects the angle between
34. Statement I The four straight lines given by coordinate axes in positive quadrant.
6x 2 + 5xy − 6y 2 = 0 and 6x 2 + 5xy − 6y 2 − x + 5y − 1 = 0 Statement II If ax + y(2h + a ) = 0 is a factor of
are the sides of a square. ax 2 + 2hxy + by 2 = 0, then b + 2h + a = 0.
Statement II The lines represented by general equation
of second degree ax 2 + 2hxy + by 2 + 2gx + 2 fy + c = 0
are perpendicular if a + b = 0.

#L Pair of Straight Lines Exercise 7 :


Subjective Type Questions
n
In this section, there are 7 subjective questions. 42. If ax 2 + 2hxy + by 2 + 2gx + 2 fy + c = 0 and
38. Prove that the straight lines represented by ax 2 + 2hxy + by 2 − 2gx − 2 fy + c = 0 each represents a
(y − mx ) 2 = a 2 (1 + m 2 ) and (y − nx ) 2 = a 2 (1 + n 2 ) pair of lines, then prove that the area of the
form a rhombus. 2| c |
parallelogram enclosed by them is .
39. Prove that the equation m ( x 3 − 3xy 2 ) + y 3 − 3x 2 y = 0 (h 2 − ab )
represents three straight lines equally inclined to each 43. Prove that lines ax 2 + 2hxy + by 2 + 2gx + 2 fy + c = 0
other.
are equidistant from the origin, if
40. Show that the straight lines f 4 − g 4 = c (bf 2 − ag 2 ). Also, find the product of
( A 2 − 3B 2 )x 2 + 8ABxy + ( B 2 − 3A 2 ) y 2 = 0 form with their distances from the origin.
the line Ax + By + C = 0 an equilateral triangle whose
44. Prove that if two of the lines represented by
C2
area is ax 4 + bx 3 y + cx 2 y 2 + dxy 3 + ay 4 = 0
3( A 2 + B 2 ) bisects the angle between the other two, then
41. Find the equations of the diagonals of the parallelogram c + 6a = 0 and b + d = 0.
formed by the lines L 2 − aL = 0 and L ′ 2 − aL ′ = 0, where
L = x cos θ + y sin θ − p and L ′ = x cos θ′ + y sin θ′ − p ′.
Chap 03 Pair of Straight Lines 231

#L Pair of Straight Lines Exercise 8 :


Question Asked in Previous 13 Years Exams
n This section contains questions asked in IIT-JEE, AIEEE, 46. If one of the lines of my 2 + (1 − m 2 )xy − mx 2 = 0 is a
JEE Main & JEE Advanced from year 2005 to 2017. bisector of the angle between the lines xy = 0, then m is
45. If the pair of lines ax 2 + 2(a + b )xy + by 2 = 0 lie long [AIEEE 2007, 3M]
1
diameters of a circle and divide the circle into four (a) − (b) −2
2
sectors such that the area of one of the sector is thrice (c) 1 (d) 2
the area of the another sector, then [AIEEE 2005, 3M]
(a) 3a 2 + 2ab + 3b 2 = 0 (b) 3a 2 + 10ab + 3b 2 = 0
(c) 3a 2 − 2ab + 3b 2 = 0 (d) 3a 2 − 10ab + 3b 2 = 0

Answers
Exercise for Session 1 Exercise for Session 5
1. (c) 2. (a) 3. (b) 4. (b) 5. (c, d) 1. (c) 2. (d) 3. (d) 4. (a) 5. (b)
7.  ,
8 8 6. 9x2 + 10xy − 9 y2 = 0 7. ± 1 10. x2 − xy − 2 y2 = 0
6. bx + ay = 0, ax − by = 0 
 3 3
9. (ab′ − a′ b) = 4 (ah′ − a′ h) (h′ b − hb′ )
2
Chapter Exercises
1. (b) 2. (c) 3. (d) 4. (c) 5. (b) 6. (b)
Exercise for Session 2 7. (c) 8. (a) 9. (c) 10. (c) 11. (d) 12. (c)
1. (b) 2. (c, d) 3. (a) 4. (c) 5. (b, c) 13. (a, c) 14. (a, b, d) 15. (c, d) 16. (a, d) 17. (a, b) 18. (a, d)
−1  1 −1
(± p )
1
6. tan   8. sec 9. 3 sq units 19. (b) 20. (c) 21. (d) 22. (b) 23. (a) 24. (b)
 3 6
25. (a) 26. (b) 27. (c) 28. (7) 29. (1) 30. (2)
31. (7) 32. (2)
Exercise for Session 3
33. (A) → (p, r); (B) → (r); (C) → (p, q, s); (D) → (q, r)
1. (b) 2. (a, c) 3. (a, c) 4. (b) 9. ± 2
34. (b) 35. (b) 36. (a) 37. (b)
Exercise for Session 4  x(cosθ − cosθ′ ) + y (sin θ − sin θ′ ) − p + p′ = 0
41. 
1. (b) 2. (d) 3. (a, d) 4. (b) 5. (a) and x(cosθ + cosθ′ ) + y(sin θ + sin θ′ ) − p − p′ − a = 0
| g 2 − ac | | c|
6. 1 sq unit 7. x − y = 0 9. 43.
| a | h − ab
2 (a − b ) 2 + 4 h 2

10. x2 − 5xy + 4 y2 + 3x − 3 y = 0 45. (a) 46. (c)


7. Q ( x - 2y ) 2 + l( x - 2y ) = 0

Solutions Þ ( x - 2y ) ( x - 2y + l) = 0
\ Lines are x - 2y = 0 and x - 2y + l = 0
Distance between lines = 3
Þ
| l - 0|
=3
(given)

(1 + 4 )
- 2l 4
1. Qm1 + m2 = m1m2 Þ = \ | l | =3 5
-7 -7
\ l = -2 8. \ 12 x + 7 xy - 12y 2 = 0
2
...(i)
2. For perpendicular lines Þ (3 x + 4y ) ( 4 x - 3y ) = 0
3a + (a 2 - 2 ) = 0 \Lines represented by Eq. (i) are
3 x + 4y = 0 and 4 x - 3y = 0
- 3 ± (9 + 8 ) - 3 ± 17
Þ a= = and 12 x 2 + 7 xy - 12y 2 - x + 7y - 1 = 0 ...(ii)
6 6
3. For mirror image with respect to y = 0 replace y by - y, then Þ (3 x + 4y - 1 ) ( 4 x - 3y + 1 ) = 0
image of the pair of lines ax 2 + 2hxy + by 2 = 0 is \ Lines represented by Eq. (ii) are
3 x + 4y - 1 = 0 and 4 x - 3y + 1 = 0
ax 2 + 2hx( - y ) + b( - y ) 2 = 0
Distance between parallel lines 3 x + 4y = 0 and
or ax 2 - 2hxy + by 2 = 0 1
3 x + 4y - 1 = 0 is .
4. Q 15 x 2 + 56 xy + 48y 2 = 0 ...(i) 5
And distance between parallel lines 4 x - 3y = 0 and
Þ (3 x + 4y ) (5 x + 12y ) = 0 1
Equation of lines represented by Eq. (i) are 4 x - 3y + 1 = 0 is .
5
3 x + 4y = 0 and 5 x + 12y = 0
Hence, all sides along a square.
æ - 7l -2 ö
Let any point on 7 x + 4y + 2 = 0 is ç l, ÷. 9. Given,
è 4 ø
4 x 2 + 20 xy + 25y 2 + 2 x + 5y - 12 = 0 ...(i)
According to questions,
2
æ - 7l - 2 ö æ - 7l - 2 ö Þ (2 x + 5y ) + (2 x + 5y ) - 12 = 0
3l + 4ç ÷ 5 l + 12 ç ÷
è 4 ø è 4 ø Þ (2 x + 5y + 4 ) (2 x + 5y - 3 ) = 0
=
5 13 \ Lines represented by Eq. (i) are
2 x + 5y + 4 = 0 and 2 x + 5y - 3 = 0
1 1 | 4 - (- 3) | 7
Þ | - 4 l - 2 | = | - 16 l - 6 | Hence, distance between parallel lines = = .
5 13 4 + 25 29

Þ
| 2l + 1 | | 8l + 3 |
= 10. Let f ( x, y ) º 2x 2 - 5xy + 2y 2 + 3x + 3y + 1 = 0
5 13 ¶f ¶f
Þ 13(2 l + 1 ) = ± 5 (8 l + 3 ) \ = 4 x - 5y + 3 and = - 5 x + 4y + 3
¶x ¶y
1 14
\ l=- ,- ¶f ¶f
7 33 For point of intersection = 0 and =0
¶x ¶y
5. Given lines are xy - 3x - 5y + 15 = 0
Þ 4 x - 5y + 3 = 0 and - 5 x + 4y + 3 = 0, we get
Y
y=3 x = 3, y = 3
(0,3) B C (5,3)
\ Point of intersection is (3, 3 ).
3x+ 11. ax 2 + 4g xy + by 2 + 4p( x + y + 1) = 0
5y= x=5
15 represents a pair of straight lines.
X \D=0
O A(5,0)
Þ 4 abp + 16 p 2g - 4 p 2a - 4 p 2b - 16 g 2p = 0
Þ ( x - 5 ) (y - 3 ) = 0
\ x = 5 and y = 3 Þ (16 p ) g 2 - 16 p 2g + 4 p( pa + pb - ab) = 0
Hence, orthocentre is (5, 3). \ B 2 - 4 AC ³ 0
6. 3x 3 + 3x 2y - 3xy 2 + dy 3 = ( x 2 + pxy - y 2 ) (3x - dy ) Þ (16 p 2 ) 2- 4 × (16 p ) × 4 p ( pa + pb - ab) ³ 0
2 2
On comparing coefficients of x y and xy , we get Þ p 2( p 2 - pa - pb + ab) ³ 0
3p - d = 3 Þ ( p - a) ( p - b) ³ 0
and - 3 - pd = - 3 \ p £ a or p ³ b [Q a < b]
\ p = 0, d = -3
Chap 03 Pair of Straight Lines 233

12. Equation of first line is y - 1 = tan q( x - 1) 18. The separate equations of the sides are x + y = 0, x - y = 0 and
Equation of second line is y - 1 = cot q( x - 1 ) x y
+ = 1.
So, their joint equation is 3 2
[(y - 1 ) - tan q ( x - 1 )] [(y - 1 ) - cot q ( x - 1 )] = 0 x=–2
Y

2x+
Þ(y - 1 ) 2 - ( x - 1 ) (y - 1 ) (tan q + cot q) + ( x - 1 ) 2 = 0

3y–
x 2 - (tan q + cot q)xy + y 2 + (tan q B

6=
x+

0
+ cot q - 2 ) ( x + y - 1 ) = 0
=0

y=
y
On comparing with the given equation, we get x–

0
tan q + cot q = a + 2 A
1 y=1
or =a + 2 X′ X
sin q cos q O
2
or sin 2 q = Y′
(a + 2 )
13. For image w.r.t., Y -axis replace x by - x, then required image Intersection of x = - 2 with y = - x and 2 x + 3y - 6 = 0 gives
of lines is the range of values of a.
y = | - x - 1 | or y = | x + 1 | 10
\ 2 <a <
and on squaring both sides, then 3
y 2 = x 2 + 2x + 1 and intersection of y = 1 with y = x and 2 x + 3y - 6 = 0 given
the range of values of b
Þ x 2 - y 2 + 2x + 1 = 0 -1 <b <1 [Q(b, 1 ) is exterior point]
14. Given equation is
ax 2 + by 2 + cx + cy = 0 ...(i) Sol. (Q. Nos. 19 to 21)
Eq. (i) represents a pair of straight lines. Given equation is x 2 - 3 xy + ly 2 + 3 x - 5y + 2 = 0
\ D=0 Here, a = 1, b = l , c = 2,
c2 c2 5 3 3
Þ 0 + 2× 0 - a ´ -b ´ =0 f = - , g = and h = - .
4 4 2 2 2
or c 2(a + b ) = 0 19. D = 0
\ c = 0 or a + b = 0 5 3 3 25 9 9
1 ´ l ´2 + 2 ´ - ´ ´ - -1 ´ - l ´ -2 ´ = 0
15. Given, equation is 2 2 2 4 4 4
x 2 + ay 2 + 2by - a 2 = 0 ...(i) 45 25 9 l 9
Þ 2l + - - - =0
4 4 4 2
Equation (i) represents a pair of perpendicular straight lines
l 1
\ D = 0 and coefficient of x 2 + coefficient of y 2 = 0 Þ - + =0
4 2
Þ - a 2a - b 2 = 0 and 1 + a = 0 \ l =2
2 2
\ a = - 1 and b = ± a 20. Let f ( x, y ) = x - 3xy + 2y + 3x - 5y + 2 [Q l = 2 ]
16. Given pair of lines is ¶f ¶f
\ = 2 x - 3y + 3, = - 3 x + 4y - 5
ax 2 + 2hxy + by 2 + 2 gx + 2 fy + c = 0 ...(i) ¶x ¶y
Q Point of intersection of lines represented by (i) is ¶f ¶f
For point of intersection = 0 and = 0, we get
æ hf - bg gh - af ö æ æ f 2 - bc ö æ g 2 - ac ö ö ¶x ¶y
ç , ÷ or ç ç ÷, ç ÷÷
è ab - h 2 ab - h 2 ø çè è h 2 - ab ø è h 2 - ab ø ÷ø x = - 3, y = - 1
\ a = - 3 and b = - 1
For Y -axis put x = 0
\ The value of a 2 + b 2 is 10.
\ hf = bg, f 2 = bc and h 2 ¹ ab
æ9 ö
Also, D = 0 2 ç - 2÷
2 (h 2 - ab ) è4 1 ø
\ abc + 2 fgh - af 2 - bg 2 - ch 2 = 0 21. tan q = = =
(a + b ) 1+2 3
For f 2 = bc, 2 fgh = bg 2 + ch 2 1
1-
17. Let y = mx be the common line, then m 2 + m - 12 = 0 and 1 - tan 2 q 9 =4
\ cos 2 q = 2
=
bm 2 + 2hm + a = 0, then from first equation m = -4, 3 1 + tan q 1 + 1 5
Substitute m = -4 in second equation, then a = 8(h - 2b ) and 9
substitute m = 3 in second equation, then a = - 3(2h + 3b ).
234 Textbook of Coordinate Geometry

Solutions (Q. Nos. 22 to 24) Also, intersection on X -axis put y = 0.


22. Q f1( x, y ) º ax 2 + 2hxy + by 2 = 0 ...(i) C
bisectors of f1( x, y ) is f 2( x, y )
x 2 - y 2 xy
\ Equation of bisectors of (i) is =
a -b h (x2,0)B A(x1,0)
2 2 M
Þ hx - (a - b ) xy - hy = 0
11
\ f 2( x, y ) º hx 2 - (a - b ) xy - hy 2 = 0 ...(ii) \ 2 x 2 - 11 x + 2 = 0 Þ x1 + x 2 =
2
Now, equation of bisectors of (ii) is æ x + x2 ö
\ Circumcentre of DABC Þ M ç 1 , 0÷
x2 - y 2 xy è 2 ø
=
h - ( - h ) - (a - b ) æ 11 ö
2 \ M º ç , 0÷
è4 ø
2 2
x -y 2 xy æ 1 19 ö
Þ =- and orthocentre is C ºç , ÷
2h (a - b ) è5 5 ø
Þ (a - b ) x 2 + 4hxy - (a - b )y 2 = 0 2 2
æ 11 1 ö æ 19 ö
2 2 \Required distance = ç - ÷ + ç 0 - ÷
\ f 3( x, y ) º (a - b ) x + 4hxy - (a - b )y = 0 è 4 5ø è 5ø
23. f 2( x, y ) º bx 2 - 2hxy + ay 2= 0 8377
\ l= = 20.94 = 4.57
and f 3( x, y ) º ax 2 + 2hxy + by 2 = 0 400
is same as f1( x, y ) = 0 [ l ] = [ 4.57 ] = 4
24. For all n ³ 2 28. Given, 4y - xy - 10x 2y + lx 3 = 0
3 2

3 2
fn + 2( x, y ) = fn ( x, y ) æy ö æy ö æy ö
Þ 4 ç ÷ - ç ÷ - 10 ç ÷ + l = 0
fn + 2( x, y ) èxø èxø èxø
\ =1
fn ( x, y ) Let y = mx, then
5 fn + 2( x, y ) 5
4m 3 - m 2 - 10m + l = 0
Now, å fn ( x, y )
= å1 = 1 + 1 + 1 + 1 = 4 and let m1, m2, m3 be the slopes of three lines, then
n=2 n=2
1
Sol. (Q. Nos. 25 to 27) m1 + m2 + m3 = ...(i)
4
For perpendicular straight lines, then
coefficient of x 2 + coefficient of y 2 = 0 10
m1m2 + m2m3 + m3m1 = - ...(ii)
4
Þ 2+b=0
l
\ b = -2 m1m2m3 = - ...(iii)
2 2
4
25. 2x + 3xy - 2y - 11x + 13y + c = 0
Since, two lines are equally inclined with y = x, so let m2m3 = 1
represents a pair of straight lines, then we have l
13 11 3 169 121 9 From Eq. (iii), m1 = -
2 ´ -2 ´ c + 2 ´ ´- ´ -2 ´ +2 ´ -c ´ = 0 4
2 2 2 4 4 4 l+1
429 169 121 9c From Eq. (i), m2 + m3 = ...(iv)
Þ - 4c - - + - =0 4
4 2 2 4 æ lö æ l + 1ö 10
and from Eq. (ii), 1 + ç - ÷ ç ÷=-
Þ c = -2 è 4ø è 4 ø 4
26. | b + 2c | = | - 2 - 4 | = 6 Þ 16 - l2 - l = - 40
2 2
27. Now, pair of lines is 2x + 3xy - 2y - 11x + 13y - 2 = 0 Þ l2 + l - 56 = 0
Let f ( x, y ) º 2 x 2 + 3 xy - 2y 2 - 11 x + 13y - 2 = 0 Þ (l + 8) (l - 7) = 0
¶f ¶f l = - 8, 7
\ = 4 x + 3y - 11 and = 3 x - 4y + 13
¶x ¶y Hence, l =7 [Q l > 0]
For point of intersection,
¶f
= 0 and
¶f
=0 29. Qy 2 - 5xy + 6x 2 + 3x - y = 0
dx ¶y Þ (3 x - y ) (2 x - y + 1 ) = 0
Þ 4 x + 3y - 11 = 0 and 3 x - 4y + 13 = 0 and y 2 - 5 xy + 6 x 2 + 2 x - y = 0
æ 1 19 ö Þ (3 x - y + 1 ) (2 x - y ) = 0
\ C ºç , ÷
è5 5 ø
Chap 03 Pair of Straight Lines 235

2x – y+1=0 33. (A) ® (p, r); (B) ® (r); (C) ® (p, q, s); (D) ® (q, r)
D C 2 x + 2y
(A) Given =1 ...(i)

0
=0
-l

+1 =
3x–y

and 9 x 2 + 16y 2 = 144 ...(ii)

3x–y
A B
Homogenising Eq. (ii) with Eq. (i), we get
2x–y=0 2
æ 2 x + 2y ö
9 x 2 + 16y 2 = 144 ç ÷
(1 - 0 ) (1 - 0 ) è -l ø
\Required Area = = 1 sq unit
3 -1 Þ (9 l2 - 576 ) x 2 - 1152 xy + (16 l2 - 576 )y 2 = 0
2 -1 The lines are coincident, then
\ l =1 ( - 576 ) 2 = (9 l2 - 576 ) (16 l2 - 576 )
30. Given pairs are Þ 144 l4 - 576 l2(25 ) = 0
x 2 + 2 lxy + 2y 2 = 0 ...(i)
or l2 = 100
and ( l + 1 ) x 2 - 8 xy + y 2 = 0 ...(ii) \ | l | = 10
Q Equations of angle bisectors of (i) and (ii) are equal 4 x + 3y
(B) Given, =1 ...(i)
x 2 - y 2 xy x2 - y 2 xy 24
Þ = and = must be same
1 -2 l (l + 1) - 1 - 4 and ( x - 3 ) 2 + (y - 4 ) 2 = l2
l 4 or x 2 + y 2 - (6 x + 8y ) + 25 - l2 = 0 ...(ii)
Þ =- Þ l2 = 4 Þ l = ± 2
-1 l Homogenising Eq. (ii) with Eq. (i), we get
2
\ | l| =2 ( 4 x + 3y ) æ 4 x + 3y ö
x 2 + y 2 - (6 x + 8y ) + (25 - l2 ) ç ÷ =0
y - nx 24 è 24 ø
31. Given, =1 ...(i)
2 Þ 16 (25 - l2 ) x 2 + 9(25 - l2 )y 2 + ( - 600 - 24 l2 ) xy = 0
and x2 + y 2 = 1 ...(ii)
The lines will be at right angles, when
Homogenising Eq. (ii) with Eq. (i), we get 16(25 - l2 ) + 9(25 - l2 ) = 0
2
æ y - nx ö Þ 625 - 25 l2 = 0
x2 + y 2 = ç ÷
è 2 ø
Þ l2 = 25
æ n2 ö æ1 ö
Þ ç - 1 ÷ x 2 - nxy + ç - 1 ÷y 2 = 0 \ | l| =5
è 4 ø è4 ø
(C) The lines pairs are (y - 1 ) (y - 3 ) = 0 i.e. y = 1 and y = 3
or (n 2 - 4 ) x 2 - 4 nxy - 3y 2 = 0 the other line pair is x 2 + 4 xy + 4y 2 - 5 x - 10y + 4 = 0
The lines will be at right angles, when i.e. ( x + 2y - 4 ) ( x + 2y - 1 ) = 0
(n 2 - 4 ) + ( - 3 ) = 0 Þ n 2 = 7 or x + 2y - 4 = 0 and x + 2y - 1 = 0
y=3
32. Q x 2 - y 2 + 2y = 1 D C
2 2 2
Þ x = (y - 2y + 1 ) = (y - 1 )
x+2
x+2

y–4=
y

Þ x = ± (y - 1 )
–1=

i.e. x - y + 1 = 0 and x + y - 1 = 0
0
0

\ Bisectors are y = 1 and x = 0 A B


y=1
Y
3 (3 - 1 ) ( 4 - 1 )
Required area = = 6 sq units
1 2
0 1
1 (2,1) \ l =6
(D) Equation of the bisectors of 3 x 2 - 2 pxy - 3y 2 = 0 is
o X
x2 - y 2 xy
= Þ px 2 + 6 xy - py 2 = 0
3 - (- 3) - p
So, area between x = 0, y = 1 and x + y = 3 is given by
1 Which is same as 5 x 2 - 2qxy - 5y 2 = 0
´ 2 ´ 2 = 2 sq units
2 p 6 -p
i.e. = = Þ pq = - 15
\ l =2 5 - 2q - 5
\ l = | pq | = | - 15 | = 15
236 Textbook of Coordinate Geometry

34. Q Lines represented by 6x 2 + 5xy - 6y 2 = 0 are l1 : 2x + 3y = 0, 38. From (y - mx ) 2 = a 2 (1 + m 2 )


l 2 : 3 x - 2y = 0
Þ y - mx = ± a (1 + m 2 )
and lines represented by
6 x 2 + 5 xy - 6y 2 - x + 5y - 1 = 0 \ Lines y - mx = a 1 + m 2 …(i)
are l 3 : 2 x + 3y - 1 = 0 2
and y - mx = - a 1 + m …(ii)
l 4 : 3 x - 2y + 1 = 0
Q l1 ^ l 2, l 2 ^ l 3, l 3 ^ l 4 and l 4 ^ l1. and from (y - nx )2 = a 2 (1 + n 2 )
Also, l1, l 2 intersect at ( 0, 0 ) and ( 0, 0 ) is equidistant from l 3 and
Þ y - nx = ± a (1 + n 2 )
l 4 . These lines form sides of square.
\Statement I is true. \ Lines y - nx = a 1 + n 2 …(iii)
Statement II is true but Statement II is not a correct 2
explanation for Statement I. and y - nx = - a 1 + n …(iv)
35. dx 3 + cx 2y + bxy 2 + ay 3 = ( x 2 + pxy - y 2 ) (dx - ay ) Since, lines (i) and (ii) are parallel, the distance between
On comparing, we get lines (i) and (ii) is
c+a
c = - a + pd Þ p = ...(i) |a 1 + m 2 + a 1 + m 2 |
d = | 2a |
-b -d 1 + m2
and b = - ap - d Þ p = ...(ii)
a Similarly, (iii) and (iv) are parallel lines and the distance
From Eq. (i) and Eq. (ii), between them = | 2a |. Therefore, distance between parallel
c + a -b -d lines are same.
=
d a Hence, lines (i), (ii), (iii) and (iv) form a rhombus.
Þ d 2 + bd + ac + a 2 = 0 39. Dividing by x 3, the given equation can be written as
æ 2 3
\ Statement I is true. æy ö ö æy ö æy ö
m çç1 - 3ç ÷ ÷÷ + ç ÷ - 3ç ÷ = 0
Hence, both statements are true but Statement II is not correct è x ø è x ø èx ø
è ø
explanation for Statement I.
36. Pair of bisectors between the lines x 2 - 2axy - y 2 = 0 is y
writing = tan q, we have
x
x2 - y 2 xy Y
=
1 - (- 1) - a 2′
2 2
Þ ax + 2 xy - ay = 0 ...(i) 3
2
Comparing it with y + 2bxy - x = 0 , then2 2π/3
π/3 1
a 1 l π/3
- = = φ/3
1 b -1 X′ X
O 2π/3
1′ 2π/3
Þ ab = - 1
2 3′
Hence, both statements are true and Statement II is correct Y′
explanation for Statement I.
m(1 - 3 tan 2 q ) + tan 3 q - 3 tan q = 0
37. Put 2h = - (a + b ) in ax 2 + 2hxy + by 2 = 0
3 tan q - tan 3 q
Þ ax 2 - (a + b ) xy + by 2 = 0 \ m= = tan 3q
1 - 3 tan 2 q
Þ ( x - y ) (ax - by ) = 0
Þ One of the line bisects the angle between coordinate axes Let m = tan f,
in positive quadrant. then tan 3q = tan f
Also, put b = - 2h - a in ax - by , we have \ 3q = f, f + p , f + 2p
ax - by = ax - ( - 2h - a )y f f p f 2p
= ax + (2h + a )y Þ q= , + , +
3, 3 3 3 3
Hence, ax + (2h + a )y is a factor of
Thus, the given equation represents three lines through the
ax 2 + 2hyx + by 2 = 0 f f p f 2p
origin and they are inclined at angles , + , + with
\ Both statements are true but Statement II is not correct 3 3 3 3 3
explanation for Statement I. the X-axis. Clearly, they are equally inclined to each other.
Chap 03 Pair of Straight Lines 237

40. Given pair of lines is -3A 2 + 3 A 2 B - 3AB 2 + 3B 3


2 2 2 2 2 2 = tan -1
( A - 3B )x + 8ABxy + ( B - 3A ) y = 0 …(i) - 3A 3 + A 2 B - 3AB 2 + B 3
Given line is Ax + By + C = 0 …(ii) = tan -1 ( 3 ) = 60°
2 2
Comparing Eq. (i) with ax + 2hxy + by = 0
then remaining third angle is 180° - (60° + 60° ) = 60°.
a = A 2 - 3B 2 , h = 4 AB, b = B 2 - 3A 2 \ The DAOB is equilateral.
If q be the acute angle between the lines represented by In DOAB,
Eq. (i), then 0+0+C
Length of ^ from O on AB = p = OM =
2 (h 2 - ab ) 2 16A 2 B 2 - ( A 2 - 3B 2 )( B 2 - 3A 2 ) (A 2 + B2 )
tan q = =
|a + b | |( A 2 - 3B 2 + B 2 - 3A 2 )|
C
p=
2 ( 3A 4 + 3B 4 + 6A 2 B 2 ) 3( A 2 + B 2 ) (A + B2 )
2
= 2 2
= 2 2
= 3
| -2( A + B )| (A + B )
and in DOAM,
\ q = 60° p 2p
Y sin60° = Þ OA =
B OA 3
Ax
+

2| C |
By

60° \ OA =
+

M
3 (A 2 + B2 )
C=
0

p
3 3
60°
A
\ Area of the equilateral triangle = (side)2 = (OA )2
60° 4 4
(0,0) 2
X 3 4C C2
O = ´ =
2 2
4 3 ´ (A + B ) 3(A 2 + B 2 )
Eq. (i) can be written as 41. Lines of parallelogram are
2
æy ö æy ö L = 0, L = a
( B 2 - 3A 2 )ç ÷ + 8AB ç ÷ + ( A 2 - 3B 2 ) = 0
èx ø èx ø L¢ = 0, L ¢ = a
C
Solving for y/x, we have L=a

y -8AB ± 64 A 2 B 2 - 4( B 2 - 3A 2 )( A 2 - 3B 2 ) D
=
x 2( B 2 - 3A 2 ) L′=a

- 4 AB ± 16A 2 B 2 - ( A 2 B 2 - 3B 4 - 3A 4 + 9 A 2 B 2 ) L′=0
= 2 2 B
( B - 3A )
L=0
y -4 AB ± 3( A 2 + B 2 ) A
Þ =
x ( B 2 - 3A 2 ) where, L = x cos q + y sin q - p
L ¢ = x cos q ¢ + y sin q ¢ - p ¢
ì -4 AB ± 3 ( A 2 + B 2 ) ü
Þ y =í ýx Equation of the line AC, through the point of intersection of
î B 2 - 3A 2 þ L = 0 and L ¢ = 0 is
Taking the positive sign, slope of one of the lines L + lL ¢ = 0
3 ( A 2 + B 2 ) - 4 AB It also passes through
= 2 2
B - 3A L = a and L ¢ = a, …(i)
and slope of the line Ax + By + C = 0 is – A/B. then, a + la = 0
Acute angle between these lines \ l = -1
Hence, diagonal of AC is L - L ¢ = 0
3( A 2 + B 2 ) - 4 AB A
2 2
+ i.e. x (cos q - cos q ¢ ) + y (sin q - sin q ¢ ) - p + p ¢ = 0
B - 3A B
= tan -1 Equation of the line BD through point of intersection of
2 2
æ A ö é 3( A + B ) - 4 AB ù L = 0 and L ¢ = a is
1 - ç ÷ê ú
è B øë B 2 - 3A 2 úû L + l( L ¢ - a ) = 0
238 Textbook of Coordinate Geometry

it also passes through L = a and L ¢ = 0. Since, the lines lx + my + n = 0 and l1x + m1y + n1 = 0 are
\ a + l(0 - a ) = 0 equidistant from the origin, then
\ l =1 |n | | n1 |
=
2 2
Equation of diagonal BD is L + L ¢ - a = 0 l +m l1 + m12
2

Þ x (cos q + cos q¢ ) + y (sin q + sin q¢ ) - p - p ¢ - a = 0 Þ n 2l12 + n 2m12 = n12l 2 + n12m 2


42. Let the first equation represents lines
Þ n 2l12 - n12l 2 = n12m 2 - n 2m12
(lx + my + n ) (l1x + m1y + n1 ) = 0
then the second equation represents lines Þ (nl1 - n1l ) (nl1 + n1l ) = (n1m - nm1 ) (n1m + nm1 )
(lx + my - n )(l1x + m1y - n1 ) = 0 On squaring both sides, we get
On comparing coefficients, ll1 = a , mm1 = b, nn1 = c (nl1 - n1l )2 (nl1 + n1l )2 = (n1m - nm1 )2 (n1m + nm1 )2
mn1 + m1n = 2 f , nl1 + n1l = 2g , lm1 + l1 m = 2h . Þ [(nl1 + n1l )2 - 4ll1nn1 ] [nl1 + n1l ]2
Let the angle between two non-parallel lines be q, then = [(n1m + nm1 )2 - 4mm1nn1 ] [n1m + nm1 ]2
2
2 (h - ab ) Þ ( 4 g 2 - 4ac ) ( 4 g 2 ) = ( 4 f 2
- 4bc ) ( 4 f 2 )
tan q =
|a + b |
[using the six relations]
2 (h 2 - ab ) Þ 2
( g - ac )g = ( f 2 2
- bc ) f 2
[from Eq. (i)]
\ sin q =
2 2
( a - b ) + 4h Þ f 4 4
- g = c (bf 2
- ag ) 2

p 1p 2 Also, product of distances from origin to the lines


\ Area of parallelogram = ,
sinq represented by
where, p 1 and p 2 are distance between two parallel sides. ax 2 + 2hxy + by 2 + 2gx + 2 fy + c = 0
| n - ( - n )| | n - ( -n1 )|
\ p1 = and p 2 = 1 |n | | n1|
2 2 2 is .
l +m l1 + m12 2 2
(l + m ) (l12 + m12 )
| 4nn1|
\ p 1p 2 = | nn1|
(l 2l12 + l 2m12 + m 2l12 + m 2m12 ) =
(l 2l12 +l 2
m12 + m 2l12 + m 2m12 )
4| nn1|
= | nn1|
{l 2l12 + m 2m12 + (lm1 + l1m )2 - 4l l1mm1 } =
(l 2l12 +l 2
m12 + m 2l12 + m 2m12 )
4 |c | 4 |c |
= = | nn1|
a 2 + b 2 + 4h 2 - 2ab ( a - b ) 2 + 4h 2 =
(l 2l12 +m 2
m12 + (lm1 + ml1 )2 - 2ll1mm1 )
pp
\ Area = 1 2 |c |
sinq = [from Eq. (i)]
4| c | (a + b + 4h 2 - 2ab )
2 2
=
æ 2 h 2 - ab ö |c |
( ( a - b ) 2 + 4h 2 ) ç ÷ =
ç ( a - b ) 2 + 4h 2 ÷ (a - b ) 2 + 4h 2
è ø
2|c | 44. We know that bisectors are mutually perpendicular to each
= other, then
2
(h - ab )
ax 4 + bx 3y + cx 2y 2 + dxy 3 + ay 4 = 0
43. Given, equation
represents two pairs of mutually perpendicular lines.
ax 2 + 2hxy + by 2 + 2gx + 2 fy + c = 0
Let ax 4 + bx 3y + cx 2y 2 + dxy 3 + ay 4
represents two lines
= (ax 2 + pxy - ay 2 ) ( x 2 + qxy - y 2 )
lx + my + n = 0 and l1x + m1y + n1 = 0
where, p and q are constants.
then, ax 2 + 2hxy + by 2 + 2gx + 2 fy + c
Comparing the coefficients of similar terms, we get
= (lx + my + n ) (l1x + m1y + n1 )
b = aq + p …(i)
Comparing the coefficients of similar terms, we get
c = pq - 2a …(ii)
ll1 = a , mm1 = b, nn1 = c ü
ý …(i) d = - p - aq …(iii)
lm1 + ml1 = 2h , n1l + nl1 = 2g ,nm1 + n1m = 2 f þ
Chap 03 Pair of Straight Lines 239

On adding Eqs. (i) and (iii), we have \ A + 3 A + A + 3 A = pr 2


b+d =0 p
Þ A = r2
From Eq. (ii), pq = c + 2a …(iv) 8
But given that bisectors of one pair are given by the other, 1 2 p 2 1
Þ r q= r [Q area of sector = r 2q]
i.e. 2 8 2
x2 - y2 xy \ q=
p
=
a - ( -a ) ( p / 2 ) 4
\ Angle between lines represented by
x2 - y2 a p
Þ = …(v) ax 2 + 2(a + b ) xy + by 2 = 0 is .
4 xy p 4
is the same as the other pair. 2
p 2 (a + b ) - ab
2 2 Þ tan =
x + qxy - y = 0 4 |a + b|
x2 - y2 q 2 (a + b ) 2 - ab
or =- …(vi) Þ 1=
4 xy 4 |a + b|
a q Þ (a + b ) 2 = 4(a + b ) 2 - 4ab
From Eqs. (v) and (vi), =-
p 4 \ 3a + 2ab + 3b 2 = 0
2

\ pq = - 4a
46. The equation of the bisectors of the lines xy = 0 are y = ± x.
Again from Eq. (iv), -4a = c + 2a
Putting y = ± x in
\ c + 6a = 0 my 2 + (1 - m 2 ) xy - mx 2 = 0, we get
45. Let A be the area of small sector, then area of major sector is 3A. ± (1 - m 2 ) x 2 = 0
r Þ m2 = 1
θ
O r \

r
θ r

You might also like